Labor

Published on June 2016 | Categories: Documents | Downloads: 69 | Comments: 0 | Views: 1508
of x
Download PDF   Embed   Report

Comments

Content

PRIVATE SECTOR L-M RELATIONS BOOK V – LABOR CODE PART ONE LABOR RELATIONS POLICY A. CONSTITUTION – Art. III, Sec. 8; Art. XIII, Sec. 3
1987 Constitution, Article III, Sec. 8. The right of the people, including those employed in the public and private sectors, to form unions, associations, or societies for purposes not contrary to law shall not be abridged. 1987 Constitution, Article III, Sec. 3. The State shall afford full protection to labor, local and overseas, organized and unorganized, and promote full employment and equality of employment opportunities for all. It shall guarantee the rights of all workers to self-organization, collective bargaining and negotiations, and peaceful concerted activities, including the right to strike in accordance with law. They shall be entitled to security of tenure, humane conditions of work, and a living wage. They shall also participate in policy and decision-making processes affecting their rights and benefits as may be provided by law. The State shall promote the principle of shared responsibility between workers and employers and the preferential use of voluntary modes in settling disputes, including conciliation, and shall enforce their mutual compliance therewith to foster industrial peace. The State shall regulate the relations between workers and employers, recognizing the right of labor to its just share in the fruits of production and the right of enterprises to reasonable returns to investments, and to expansion and growth.

B. STATUTORY 1. Re: Methods of Dispute Settlement – Art. XIII, Sec 3, Constitution; 211(a), (b); 263 (g); 124 (5 th par.); 299; 257
1987 Constitution, Article III, Sec. 3. (pls. refer above) Art. 211. Declaration of Policy. 1. It is the policy of the State: (a) To promote and emphasize the primacy of free collective bargaining and negotiations, including voluntary arbitration, mediation and conciliation, as modes of settling labor or industrial disputes; (b) To promote free trade unionism as an instrument for the enhancement of democracy and the promotion of social justice and development; Art. 263. Strikes, picketing and lockouts. (g) When, in his opinion, there exists a labor dispute causing or likely to cause a strike or lockout in an industry indispensable to the national interest, the Secretary of Labor and Employment may assume jurisdiction over the dispute and decide it or certify the same to the Commission for compulsory arbitration. Such assumption or certification shall have the effect of automatically enjoining the intended or impending strike or lockout as specified in the assumption or certification order. If one has already taken place at the time of assumption or certification, all striking or locked out employees shall immediately return-to-work and the employer shall immediately resume operations and readmit all workers under the same terms and conditions prevailing before the strike or lockout. The Secretary of Labor and Employment or the Commission may seek the assistance of law enforcement agencies to ensure compliance with this provision as well as with such orders as he may issue to enforce the same. In line with the national concern for and the highest respect accorded to the right of patients to life and health, strikes and lockouts in hospitals, clinics and similar medical institutions shall, to every extent possible, be avoided, and all serious efforts, not only by labor and management but government as well, be exhausted to substantially minimize, if not prevent, their adverse effects on such life and health, through the exercise, however legitimate, by labor of its right to strike and by management to lockout. In labor disputes adversely affecting the continued operation of such hospitals, clinics or medical institutions, it shall be the duty of the striking union or locking-out employer to provide and maintain an effective skeletal workforce of medical and other health personnel, whose movement and services shall be unhampered and unrestricted, as are necessary to insure the proper and adequate protection of the life and health of its patients, most especially emergency cases, for the duration of the strike or lockout. In such cases, therefore, the Secretary of Labor and Employment may immediately assume, within twenty four (24) hours from knowledge of the occurrence of such a strike or lockout, jurisdiction over the same or certify it to the Commission for compulsory arbitration. For this purpose, the contending parties are strictly enjoined to comply with such orders, prohibitions and/or injunctions as are issued by the Secretary of Labor and Employment or the Commission, under pain of immediate disciplinary action, including dismissal or loss of employment status or payment by the locking-out employer of backwages, damages and other affirmative relief, even criminal prosecution against either or both of them. 1|Page

The foregoing notwithstanding, the President of the Philippines shall not be precluded from determining the industries that, in his opinion, are indispensable to the national interest, and from intervening at any time and assuming jurisdiction over any such labor dispute in order to settle or terminate the same. Art. 124. Standards/Criteria for minimum wage fixing. In cases where there are no collective agreements or recognized labor unions, the employers and workers shall endeavor to correct such distortions. Any dispute arising therefrom shall be settled through the National Conciliation and Mediation Board and, if it remains unresolved after ten (10) calendar days of conciliation, shall be referred to the appropriate branch of the National Labor Relations Commission (NLRC). It shall be mandatory for the NLRC to conduct continuous hearings and decide the dispute within twenty (20) calendar days from the time said dispute is submitted for compulsory arbitration. Art. 257. Petitions in unorganized establishments. In any establishment where there is no certified bargaining agent, a certification election shall automatically be conducted by the Med-Arbiter upon the filing of a petition by a legitimate labor organization. (As amended by Section 24, Republic Act No. 6715, March 21, 1989) Art. 299. Disposition of pending cases. All cases pending before the Court of Industrial Relations and the National Labor Relations Commission established under Presidential Decree No. 21 on the date of effectivity of this Code shall be transferred to and processed by the corresponding labor relations divisions or the National Labor Relations Commission created under this Code having cognizance of the same in accordance with the procedure laid down herein and its implementing rules and regulations. Cases on labor relations on appeal with the Secretary of Labor or the Office of the President of the Philippines as of the date of effectivity of this Code shall remain under their respective jurisdictions and shall be decided in accordance with the rules and regulations in force at the time of appeal. All workmen’s compensation cases pending before the Workmen’s Compensation Units in the regional offices of the Department of Labor and those pending before the Workmen’s Compensation Commission as of March 31, 1975, shall be processed and adjudicated in accordance with the law, rules and procedure existing prior to the effectivity of the Employees Compensation and State Insurance Fund.

Kiok Loy v. NLRC, 141 SCRA 179 (83)

CUEVAS: January 22, 1986 NATURE: Petition for certiorari to annul the decision of the National Labor Relations Commission FACTS: In a certification election held on October 3, 1978, the Pambansang Kilusang Paggawa (Union for short) was subsequently certified in a resolution dated November 29, 1978 by the Bureau of Labor Relations as the sole and exclusive bargaining agent of the rank-and-file employees of Sweden Ice Cream Plant (Company for short). The Company's motion for reconsideration of the said resolution was denied on January 25, 1978. December 7, 1978, the Union furnished the Company with two copies of its proposed collective bargaining agreement. It also requested the Company for its counter proposals. Both requests were ignored and remained unacted upon by the Company. The Union, on February 14, 1979, filed a "Notice of Strike", with the Bureau of Labor Relations (BLR) on ground of unresolved economic issues in collective bargaining. Conciliation proceedings then followed during the thirty-day statutory cooling-off period. The Bureau of Labor Relations to certify the case to the National Labor Relations Commission for compulsory arbitration. The labor arbiter set the initial hearing for April 29, 1979. For failure however, of the parties to submit their respective position papers as required, the said hearing was cancelled and reset to another date. The Union submitted its position paper. On July 20, 1979, the National Labor Relations Commission rendered its decision declaring the respondent guilty of unjustified refusal to bargain Petitioner contends that the National Labor Relations Commission acted without or in excess of its jurisdiction or with grave abuse of discretion amounting to lack of jurisdiction in rendering the challenged decision. Petitioner further contends that the National Labor Relations Commission's finding of unfair labor practice for refusal to bargain is not supported by law ISSUE/S: WON the respondent is guilty of unjustified refusal to bargain HELD: YES

2|Page

unfair labor practice is committed when it is shown that the respondent employer, after having been served with a written bargaining proposal by the petitioning Union, did not even bother to submit an answer or reply to the said proposal Ratio Unfair labor practice is committed when it is shown that the respondent employer, after having been served with a written bargaining proposal by the petitioning Union, did not even bother to submit an answer or reply to the said proposal Reasoning Collective bargaining which is defined as negotiations towards a collective agreement, is one of the democratic frameworks under the New Labor Code, designed to stabilize the relation between labor and management and to create a climate of sound and stable industrial peace. It is a mutual responsibility of the employer and the Union and is characterized as a legal obligation. So much so that Article 249, par. (g) of the Labor Code makes it an unfair labor practice for an employer to refuse "to meet and convene promptly and expeditiously in good faith for the purpose of negotiating an agreement with respect to wages, hours of work, and all other terms and conditions of employment including proposals for adjusting any grievance or question arising under such an agreement and executing a contract incorporating such agreement, if requested by either party. While it is a mutual obligation of the parties to bargain, the employer, however, is not under any legal duty to initiate contract negotiation. The mechanics of collective bargaining is set in motion only when the following jurisdictional preconditions are present, namely, (1) possession of the status of majority representation of the employees' representative in accordance with any of the means of selection or designation provided for by the Labor Code; (2) proof of majority representation; and (3) a demand to bargain under Article 251, par. (a) of the New Labor Code . - From the over-all conduct of petitioner company in relation to the task of negotiation, there can be no doubt that the Union has a valid cause to complain against its (Company's) attitude, the totality of which is indicative of the latter's disregard of, and failure to live up to, what is enjoined by the Labor Code to bargain in good faith. DISPOSITION: - Petition dismissed

Manila Diamond Hotel Employees Union v. Court of Appeals, 447 SCRA 97 (05)
AZCUNA; December 16, 2004 Nature Petition for review on certiorari of a decision of the Court of Appeals Facts -Union filed a petition for certification election to be declared the exclusive bargaining representative of the Hotel’s employees. This petition was dismissed by DOLE for lack of legal requirements. -after a few months, Union sent a letter to Hotel informing it of its desire to negotiate for a collective bargaining agreement. This was rejected by the Hotel stating that the Union was not the employee’s bargaining agent as their petition for cetification election was denied. -Union filed a Notice of Strike with the NCMB alleging the Hotel’ refusal to bargain and for acts of unfair labor practices. NCMB summoned both parties and held series of dialogues. Union however suddenly went on strike -Secretary of DOLE assumed jurisdiction and ordered compulsory arbitration pursuant to art. 263 (g) of LC. And Union members were directed to return to work and for Hotel to accept them back. Hotel refused to accept the employees return. The order was modified (by a different Secretary) such that reinstatement was to be done only in the payroll. -Union filed for certiorari alleging grave abuse of discretion. Case was referred to the CA. CA affirmed that the “payroll reinstatement” was not a grave abuse of discretion. ISSUE WON the CA commit grave abuse of discretion in affirming the validity of “payroll reinstatement” HELD Yes
3|Page

-CA based its decision on the UST v NLRC case which affirms validity of payroll reinstatement. However, the UST case was made in light of a very important fact- the teachers could not be given back their academic assignments because the semester was already halfway. In the present case, there is no showing that the facts called for payroll reinstatement as an ALTERNATIVE remedy. -as to the nature of art.263(g), the State encourages an environment wherein employers and employees themselves must deal with their problems in a manner that mutually suits them best (as embodied in Art 3, Sec 3 of the Constitution). Hence a voluntary instead of compulsory mode of dispute settlement is the general rule. -Art. 263 (g) is an exception to this rule by allowing the Secretary of the DOLE to assume jurisdiction over a dispute involving an industry indispensable to the national interest. And under this rule, the law uses the phrase “under the same terms and conditions” which contemplates only ACTUAL REINSTATEMENT.

II. Re: Trade Unionism – 211 (b) (c)

Art. 211. Declaration of Policy. 1. It is the policy of the State: (b) To promote free trade unionism as an instrument for the enhancement of democracy and the promotion of social justice and development; (c) To foster the free and voluntary organization of a strong and united labor movement;

III. Re: Worker Enlightenment – 211 (d); 277 (a); 241 (p)

Art. 211. Declaration of Policy. 1. It is the policy of the State: (d) To promote the enlightenment of workers concerning their rights and obligations as union members and as employees; Art. 277. Miscellaneous provisions. a. All unions are authorized to collect reasonable membership fees, union dues, assessments and fines and other contributions for labor education and research, mutual death and hospitalization benefits, welfare fund, strike fund and credit and cooperative undertakings. (As amended by Section 33, Republic Act No. 6715, March 21, 1989) Art. 241. Rights and conditions of membership in a labor organization. The following are the rights and conditions of membership in a labor organization: (p) It shall be the duty of any labor organization and its officers to inform its members on the provisions of its constitution and by-laws, collective bargaining agreement, the prevailing labor relations system and all their rights and obligations under existing labor laws.

Victoria v. Inciong, 157 SCRA 339 (88) NATURE Review of Order of Labor Secretary reversing decision of NLRC FACTS - Complainant Saturno Victoria is the president of the Far East Broadcasting Company Employees Union. On September 8, 1972, the said union declared a strike against respondent company for refusal to recognize them. On September 11, 1972, respondent filed with the Court of First Instance of Bulacan, Civil Case No. 750-V, for the issuance of an injunction and a prayer that the strike be declared illegal. - Strikers filed case with NLRC for reinstatement. Reinstatement granted without prejudice to outcome of Civil Case 750-V. - Strike was declared illegal because company was a non-profit organization. Company dismissed complainant. - Complainant filed for illegal dismissal. NLRC granted. Sec. Inciong reversed. ISSUE/S 1. WON company should obtain clearance under Art. 267 before dismissing complainant. HELD 1. NO The purpose in requiring a prior clearance from the Secretary of Labor in cases of shutdown or dismissal of employees, is to afford the Secretary ample opportunity to examine and determine the reasonableness of the request. Consequently,

4|Page

private respondent acted in good faith when it terminated the employment of petitioner upon a declaration of illegality of the strike. Ratio This is a matter of responsibility and of answerability. Petitioner as a union leader, must see to it that the policies and activities of the union in the conduct of labor relations are within the precepts of law and any deviation from the legal boundaries shall be imputable to the leader. He bears the responsibility of guiding the union along the path of law and to cause the union to demand what is not legally demandable, would foment anarchy which is a prelude to chaos. Reasoning As a strike is an economic weapon at war with the policy of the Constitution and the law at that time, a resort thereto by laborers shall be deemed to be a choice of remedy peculiarly their own and outside of the statute, and as such, the strikers must accept all the risks attendant upon their choice. If they succeed and the employer succumbs, the law will not stand in their way in the enjoyment of the lawful fruits of their victory. But if they fail, they cannot thereafter invoke the protection of the law for the consequences of their conduct unless the right they wished vindicated is one which the law will, by all means, protect and enforce. Disposition WHEREFORE, the petition is dismissed. The decision of the acting Secretary of Labor is AFFIRMED in toto. IV. Re: Machinery Dispute Settlement – 211 (e); 213-22; 226; 260-262; 277 (i) 1987 Administrative code of the Philippines, Title VII, Labor and Employment Chapter, Sec. 24
Art. 211. Declaration of Policy. 1. It is the policy of the State: (e) To provide an adequate administrative machinery for the expeditious settlement of labor or industrial disputes; Art. 213. National Labor Relations Commission. There shall be a National Labor Relations Commission which shall be attached to the Department of Labor and Employment for program and policy coordination only, composed of a Chairman and fourteen (14) Members. Five (5) members each shall be chosen from among the nominees of the workers and employers organizations, respectively. The Chairman and the four (4) remaining members shall come from the public sector, with the latter to be chosen from among the recommendees of the Secretary of Labor and Employment. Upon assumption into office, the members nominated by the workers and employers organizations shall divest themselves of any affiliation with or interest in the federation or association to which they belong. The Commission may sit en banc or in five (5) divisions, each composed of three (3) members. Subject to the penultimate sentence of this paragraph, the Commission shall sit en banc only for purposes of promulgating rules and regulations governing the hearing and disposition of cases before any of its divisions and regional branches, and formulating policies affecting its administration and operations. The Commission shall exercise its adjudicatory and all other powers, functions, and duties through its divisions. Of the five (5) divisions, the first, second and third divisions shall handle cases coming from the National Capital Region and the parts of Luzon; and the fourth and fifth divisions, cases from the Visayas and Mindanao, respectively; Provided that the Commission sitting en banc may, on temporary or emergency basis, allow cases within the jurisdiction of any division to be heard and decided by any other division whose docket allows the additional workload and such transfer will not expose litigants to unnecessary additional expense. The divisions of the Commission shall have exclusive appellate jurisdiction over cases within their respective territorial jurisdictions. [As amended by Republic Act No. 7700]. The concurrence of two (2) Commissioners of a division shall be necessary for the pronouncement of judgment or resolution. Whenever the required membership in a division is not complete and the concurrence of two (2) commissioners to arrive at a judgment or resolution cannot be obtained, the Chairman shall designate such number of additional Commissioners from the other divisions as may be necessary. The conclusions of a division on any case submitted to it for decision shall be reached in consultation before the case is assigned to a member for the writing of the opinion. It shall be mandatory for the division to meet for purposes of the consultation ordained herein. A certification to this effect signed by the Presiding Commissioner of the division shall be issued and a copy thereof attached to the record of the case and served upon the parties. The Chairman shall be the Presiding Commissioner of the first division and the four (4) other members from the public sector shall be the Presiding Commissioners of the second, third, fourth and fifth divisions, respectively. In case of the effective absence or incapacity of the Chairman, the Presiding Commissioner of the second division shall be the Acting Chairman. The Chairman, aided by the Executive Clerk of the Commission, shall have administrative supervision over the Commission and its regional branches and all its personnel, including the Executive Labor Arbiters and Labor Arbiters.

5|Page

The Commission, when sitting en banc shall be assisted by the same Executive Clerk and, when acting thru its Divisions, by said Executive Clerks for the second, third, fourth and fifth Divisions, respectively, in the performance of such similar or equivalent functions and duties as are discharged by the Clerk of Court and Deputy Clerks of Court of the Court of Appeals. (As amended by Section 5, Republic Act No. 6715, March 21, 1989) Art. 214. Headquarters, Branches and Provincial Extension Units. The Commission and its First, Second and Third divisions shall have their main offices in Metropolitan Manila, and the Fourth and Fifth divisions in the Cities of Cebu and Cagayan de Oro, respectively. The Commission shall establish as many regional branches as there are regional offices of the Department of Labor and Employment, sub-regional branches or provincial extension units. There shall be as many Labor Arbiters as may be necessary for the effective and efficient operation of the Commission. Each regional branch shall be headed by an Executive Labor Arbiter. (As amended by Section 6, Republic Act No. 6715, March 21, 1989) Art. 215. Appointment and Qualifications. The Chairman and other Commissioners shall be members of the Philippine Bar and must have engaged in the practice of law in the Philippines for at least fifteen (15) years, with at least five (5) years experience or exposure in the field of labor-management relations, and shall preferably be residents of the region where they are to hold office. The Executive Labor Arbiters and Labor Arbiters shall likewise be members of the Philippine Bar and must have been engaged in the practice of law in the Philippines for at least seven (7) years, with at least three (3) years experience or exposure in the field of labor-management relations: Provided, However, that incumbent Executive Labor Arbiters and Labor Arbiters who have been engaged in the practice of law for at least five (5) years may be considered as already qualified for purposes of reappointment as such under this Act. The Chairman and the other Commissioners, the Executive Labor Arbiters and Labor Arbiters shall hold office during good behavior until they reach the age of sixty-five years, unless sooner removed for cause as provided by law or become incapacitated to discharge the duties of their office. The Chairman, the division Presiding Commissioners and other Commissioners shall be appointed by the President, subject to confirmation by the Commission on Appointments. Appointment to any vacancy shall come from the nominees of the sector which nominated the predecessor. The Executive Labor Arbiters and Labor Arbiters shall also be appointed by the President, upon recommendation of the Secretary of Labor and Employment and shall be subject to the Civil Service Law, rules and regulations. The Secretary of Labor and Employment shall, in consultation with the Chairman of the Commission, appoint the staff and employees of the Commission and its regional branches as the needs of the service may require, subject to the Civil Service Law, rules and regulations, and upgrade their current salaries, benefits and other emoluments in accordance with law. (As amended by Section 7, Republic Act No. 6715, March 21, 1989) Art. 216. Salaries, benefits and other emoluments. The Chairman and members of the Commission shall receive an annual salary at least equivalent to, and be entitled to the same allowances and benefits as those of the Presiding Justice and Associate Justices of the Court of Appeals, respectively. The Executive Labor Arbiters shall receive an annual salary at least equivalent to that of an Assistant Regional Director of the Department of Labor and Employment and shall be entitled to the same allowances and benefits as that of a Regional Director of said Department. The Labor Arbiters shall receive an annual salary at least equivalent to, and be entitled to the same allowances and benefits as that of an Assistant Regional Director of the Department of Labor and Employment. In no case, however, shall the provision of this Article result in the diminution of existing salaries, allowances and benefits of the aforementioned officials. (As amended by Section 8, Republic Act No. 6715, March 21, 1989) Chapter II POWERS AND DUTIES Art. 217. Jurisdiction of the Labor Arbiters and the Commission. 1. Except as otherwise provided under this Code, the Labor Arbiters shall have original and exclusive jurisdiction to hear and decide, within thirty (30) calendar days after the submission of the case by the parties for decision without extension, even in the absence of stenographic notes, the following cases involving all workers, whether agricultural or non-agricultural: 1. Unfair labor practice cases; 2. Termination disputes; 3. If accompanied with a claim for reinstatement, those cases that workers may file involving wages, rates of pay, hours of work and other terms and conditions of employment; 4. Claims for actual, moral, exemplary and other forms of damages arising from the employer-employee relations; 5. Cases arising from any violation of Article 264 of this Code, including questions involving the legality of strikes and lockouts; and 6. Except claims for Employees Compensation, Social Security, Medicare and maternity benefits, all other claims arising from employer-employee relations, including those of persons in domestic or household service, involving an amount exceeding five thousand pesos (P5,000.00) regardless of whether accompanied with a claim for reinstatement. 2. The Commission shall have exclusive appellate jurisdiction over all cases decided by Labor Arbiters. 3. Cases arising from the interpretation or implementation of collective bargaining agreements and those arising from the interpretation or enforcement of company personnel policies shall be disposed of by the Labor Arbiter by referring the same to the grievance machinery and voluntary arbitration as may be provided in said agreements. (As amended by Section 9, Republic Act No. 6715, March 21, 1989) Art. 218. Powers of the Commission. The Commission shall have the power and authority.

6|Page

1. To promulgate rules and regulations governing the hearing and disposition of cases before it and its regional branches, as well as those pertaining to its internal functions and such rules and regulations as may be necessary to carry out the purposes of this Code; (As amended by Section 10, Republic Act No. 6715, March 21, 1989) 2. To administer oaths, summon the parties to a controversy, issue subpoenas requiring the attendance and testimony of witnesses or the production of such books, papers, contracts, records, statement of accounts, agreements, and others as may be material to a just determination of the matter under investigation, and to testify in any investigation or hearing conducted in pursuance of this Code; 3. To conduct investigation for the determination of a question, matter or controversy within its jurisdiction, proceed to hear and determine the disputes in the absence of any party thereto who has been summoned or served with notice to appear, conduct its proceedings or any part thereof in public or in private, adjourn its hearings to any time and place, refer technical matters or accounts to an expert and to accept his report as evidence after hearing of the parties upon due notice, direct parties to be joined in or excluded from the proceedings, correct, amend, or waive any error, defect or irregularity whether in substance or in form, give all such directions as it may deem necessary or expedient in the determination of the dispute before it, and dismiss any matter or refrain from further hearing or from determining the dispute or part thereof, where it is trivial or where further proceedings by the Commission are not necessary or desirable; and 4. To hold any person in contempt directly or indirectly and impose appropriate penalties therefor in accordance with law. A person guilty of misbehavior in the presence of or so near the Chairman or any member of the Commission or any Labor Arbiter as to obstruct or interrupt the proceedings before the same, including disrespect toward said officials, offensive personalities toward others, or refusal to be sworn, or to answer as a witness or to subscribe an affidavit or deposition when lawfully required to do so, may be summarily adjudged in direct contempt by said officials and punished by fine not exceeding five hundred pesos (P500) or imprisonment not exceeding five (5) days, or both, if it be the Commission, or a member thereof, or by a fine not exceeding one hundred pesos (P100) or imprisonment not exceeding one (1) day, or both, if it be a Labor Arbiter. The person adjudged in direct contempt by a Labor Arbiter may appeal to the Commission and the execution of the judgment shall be suspended pending the resolution of the appeal upon the filing by such person of a bond on condition that he will abide by and perform the judgment of the Commission should the appeal be decided against him. Judgment of the Commission on direct contempt is immediately executory and unappealable. Indirect contempt shall be dealt with by the Commission or Labor Arbiter in the manner prescribed under Rule 71 of the Revised Rules of Court; and (As amended by Section 10, Republic Act No. 6715, March 21, 1989) 5. To enjoin or restrain any actual or threatened commission of any or all prohibited or unlawful acts or to require the performance of a particular act in any labor dispute which, if not restrained or performed forthwith, may cause grave or irreparable damage to any party or render ineffectual any decision in favor of such party: Provided, That no temporary or permanent injunction in any case involving or growing out of a labor dispute as defined in this Code shall be issued except after hearing the testimony of witnesses, with opportunity for cross-examination, in support of the allegations of a complaint made under oath, and testimony in opposition thereto, if offered, and only after a finding of fact by the Commission, to the effect: 1. That prohibited or unlawful acts have been threatened and will be committed and will be continued unless restrained, but no injunction or temporary restraining order shall be issued on account of any threat, prohibited or unlawful act, except against the person or persons, association or organization making the threat or committing the prohibited or unlawful act or actually authorizing or ratifying the same after actual knowledge thereof; 2. That substantial and irreparable injury to complainant’s property will follow; 3. That as to each item of relief to be granted, greater injury will be inflicted upon complainant by the denial of relief than will be inflicted upon defendants by the granting of relief; 4. That complainant has no adequate remedy at law; and 5. That the public officers charged with the duty to protect complainant’s property are unable or unwilling to furnish adequate protection. Such hearing shall be held after due and personal notice thereof has been served, in such manner as the Commission shall direct, to all known persons against whom relief is sought, and also to the Chief Executive and other public officials of the province or city within which the unlawful acts have been threatened or committed, charged with the duty to protect complainant’s property: Provided, however, that if a complainant shall also allege that, unless a temporary restraining order shall be issued without notice, a substantial and irreparable injury to complainant’s property will be unavoidable, such a temporary restraining order may be issued upon testimony under oath, sufficient, if sustained, to justify the Commission in issuing a temporary injunction upon hearing after notice. Such a temporary restraining order shall be effective for no longer than twenty (20) days and shall become void at the expiration of said twenty (20) days. No such temporary restraining order or temporary injunction shall be issued except on condition that complainant shall first file an undertaking with adequate security in an amount to be fixed by the Commission sufficient to recompense those enjoined for any loss, expense or damage caused by the improvident or erroneous issuance of such order or injunction, including all reasonable costs, together with a reasonable attorney’s fee, and expense of defense against the order or against the granting of any injunctive relief sought in the same proceeding and subsequently denied by the Commission. The undertaking herein mentioned shall be understood to constitute an agreement entered into by the complainant and the surety upon which an order may be rendered in the same suit or proceeding against said complainant and surety, upon a hearing to assess damages, of which hearing, complainant and surety shall have reasonable notice, the said complainant and surety submitting themselves to the jurisdiction of the Commission for that purpose. But nothing herein contained shall deprive any party having a claim or cause of action under or upon such undertaking from electing to pursue his ordinary remedy by suit at law or in equity: Provided, further, That the reception of evidence for the application of a writ of injunction may be delegated by the Commission to any of its Labor Arbiters who shall conduct such 7|Page

hearings in such places as he may determine to be accessible to the parties and their witnesses and shall submit thereafter his recommendation to the Commission. (As amended by Section 10, Republic Act No. 6715, March 21, 1989) Art. 219. Ocular inspection. The Chairman, any Commissioner, Labor Arbiter or their duly authorized representatives, may, at any time during working hours, conduct an ocular inspection on any establishment, building, ship or vessel, place or premises, including any work, material, implement, machinery, appliance or any object therein, and ask any employee, laborer, or any person, as the case may be, for any information or data concerning any matter or question relative to the object of the investigation. Art. 220. Compulsory arbitration. The Commission or any Labor Arbiter shall have the power to ask the assistance of other government officials and qualified private citizens to act as compulsory arbitrators on cases referred to them and to fix and assess the fees of such compulsory arbitrators, taking into account the nature of the case, the time consumed in hearing the case, the professional standing of the arbitrators, the financial capacity of the parties, and the fees provided in the Rules of Court.] (Repealed by Section 16, Batas Pambansa Bilang 130, August 21, 1981) Art. 221. Technical rules not binding and prior resort to amicable settlement. In any proceeding before the Commission or any of the Labor Arbiters, the rules of evidence prevailing in courts of law or equity shall not be controlling and it is the spirit and intention of this Code that the Commission and its members and the Labor Arbiters shall use every and all reasonable means to ascertain the facts in each case speedily and objectively and without regard to technicalities of law or procedure, all in the interest of due process. In any proceeding before the Commission or any Labor Arbiter, the parties may be represented by legal counsel but it shall be the duty of the Chairman, any Presiding Commissioner or Commissioner or any Labor Arbiter to exercise complete control of the proceedings at all stages. Any provision of law to the contrary notwithstanding, the Labor Arbiter shall exert all efforts towards the amicable settlement of a labor dispute within his jurisdiction on or before the first hearing. The same rule shall apply to the Commission in the exercise of its original jurisdiction. (As amended by Section 11, Republic Act No. 6715, March 21, 1989) Art. 222. Appearances and Fees. 1. Non-lawyers may appear before the Commission or any Labor Arbiter only: 1. If they represent themselves; or 2. If they represent their organization or members thereof. 2. No attorney’s fees, negotiation fees or similar charges of any kind arising from any collective bargaining agreement shall be imposed on any individual member of the contracting union: Provided, However, that attorney’s fees may be charged against union funds in an amount to be agreed upon by the parties. Any contract, agreement or arrangement of any sort to the contrary shall be null and void. (As amended by Presidential Decree No. 1691, May 1, 1980) Art. 226. Bureau of Labor Relations. The Bureau of Labor Relations and the Labor Relations Divisions in the regional offices of the Department of Labor, shall have original and exclusive authority to act, at their own initiative or upon request of either or both parties, on all inter-union and intra-union conflicts, and all disputes, grievances or problems arising from or affecting labor-management relations in all workplaces, whether agricultural or non-agricultural, except those arising from the implementation or interpretation of collective bargaining agreements which shall be the subject of grievance procedure and/or voluntary arbitration. The Bureau shall have fifteen (15) working days to act on labor cases before it, subject to extension by agreement of the parties. (As amended by Section 14, Republic Act No. 6715, March 21, 1989). Art. 260. Grievance machinery and voluntary arbitration. The parties to a Collective Bargaining Agreement shall include therein provisions that will ensure the mutual observance of its terms and conditions. They shall establish a machinery for the adjustment and resolution of grievances arising from the interpretation or implementation of their Collective Bargaining Agreement and those arising from the interpretation or enforcement of company personnel policies. All grievances submitted to the grievance machinery which are not settled within seven (7) calendar days from the date of its submission shall automatically be referred to voluntary arbitration prescribed in the Collective Bargaining Agreement. For this purpose, parties to a Collective Bargaining Agreement shall name and designate in advance a Voluntary Arbitrator or panel of Voluntary Arbitrators, or include in the agreement a procedure for the selection of such Voluntary Arbitrator or panel of Voluntary Arbitrators, preferably from the listing of qualified Voluntary Arbitrators duly accredited by the Board. In case the parties fail to select a Voluntary Arbitrator or panel of Voluntary Arbitrators, the Board shall designate the Voluntary Arbitrator or panel of Voluntary Arbitrators, as may be necessary, pursuant to the selection procedure agreed upon in the Collective Bargaining Agreement, which shall act with the same force and effect as if the Arbitrator or panel of Arbitrators has been selected by the parties as described above. Art. 261. Jurisdiction of Voluntary Arbitrators or panel of Voluntary Arbitrators. The Voluntary Arbitrator or panel of Voluntary Arbitrators shall have original and exclusive jurisdiction to hear and decide all unresolved grievances arising from the interpretation or implementation of the Collective Bargaining Agreement and those arising from the interpretation or enforcement of company personnel policies referred to in the immediately preceding article. Accordingly, violations of a Collective Bargaining Agreement, except those which are gross in character, shall no longer be treated as unfair labor practice and shall be resolved as grievances under the Collective Bargaining Agreement. For purposes of this article, gross violations of Collective Bargaining Agreement shall mean flagrant and/or malicious refusal to comply with the economic provisions of such agreement. The Commission, its Regional Offices and the Regional Directors of the Department of Labor and Employment shall not entertain disputes, grievances or matters under the exclusive and original jurisdiction of the Voluntary Arbitrator or panel of Voluntary Arbitrators and shall immediately dispose and refer the same to the Grievance Machinery or Voluntary Arbitration provided in the Collective Bargaining Agreement.

8|Page

Art. 262. Jurisdiction over other labor disputes. The Voluntary Arbitrator or panel of Voluntary Arbitrators, upon agreement of the parties, shall also hear and decide all other labor disputes including unfair labor practices and bargaining deadlocks. Art. 277. Miscellaneous provisions. (i) To ensure speedy labor justice, the periods provided in this Code within which decisions or resolutions of labor relations cases or matters should be rendered shall be mandatory. For this purpose, a case or matter shall be deemed submitted for decision or resolution upon the filing of the last pleading or memorandum required by the rules of the Commission or by the Commission itself, or the Labor Arbiter, or the Director of the Bureau of Labor Relations or Med-Arbiter, or the Regional Director. Upon expiration of the corresponding period, a certification stating why a decision or resolution has not been rendered within the said period shall be issued forthwith by the Chairman of the Commission, the Executive Labor Arbiter, or the Director of the Bureau of Labor Relations or Med-Arbiter, or the Regional Director, as the case may be, and a copy thereof served upon the parties. Despite the expiration of the applicable mandatory period, the aforesaid officials shall, without prejudice to any liability which may have been incurred as a consequence thereof, see to it that the case or matter shall be decided or resolved without any further delay. (Incorporated by Section 33, Republic Act No. 6715, March 21, 1989) 1987 Administrative code of the Philippines, Title VII, Labor and Employment Chapter, Sec. 24. Regional Offices, District Offices and Provincial Extension Units. - The Department is hereby authorized to establish, operate and maintain such Department-wide Regional Offices, District Offices and Provincial Extension Units in each of the administrative regions of the country, insofar as necessary to promote economy and efficiency in the delivery of its services. Its Regional Office shall be headed by a Regional Director who shall have supervision and control thereof. The Regional Director, whenever necessary, shall be assisted by an Assistant Regional Director. A Regional Office shall have, within its regional areas, the following functions:

V. Re: Industrial Peace – 211 (f); 273

Art. 211. Declaration of Policy. 1. It is the policy of the State: (f) To ensure a stable but dynamic and just industrial peace; Art. 273. Study of labor-management relations. The Secretary of Labor shall have the power and it shall be his duty to inquire into: (a) the existing relations between employers and employees in the Philippines; (b) the growth of associations of employees and the effect of such associations upon employer-employee relations; (c) the extent and results of the methods of collective bargaining in the determination of terms and conditions of employment; (d) the methods which have been tried by employers and associations of employees for maintaining mutually satisfactory relations; (e) desirable industrial practices which have been developed through collective bargaining and other voluntary arrangements; (f) the possible ways of increasing the usefulness and efficiency of collective bargaining for settling differences; (g) the possibilities for the adoption of practical and effective methods of labor-management cooperation; (h) any other aspects of employer-employee relations concerning the promotion of harmony and understanding between the parties; and (i) the relevance of labor laws and labor relations to national development. The Secretary of Labor shall also inquire into the causes of industrial unrest and take all the necessary steps within his power as may be prescribed by law to alleviate the same, and shall from time to time recommend the enactment of such remedial legislation as in his judgment may be desirable for the maintenance and promotion of industrial peace.

VI. Re: Worker Participation Decision Making – Art. XIII, Sec. 3, Constitution; 211 (g); 255; 277 (g), (h)

1987 Constitution, Article III, Sec. 3. They shall also participate in policy and decision-making processes affecting their rights and benefits as may be provided by law. Art. 211. Declaration of Policy. (g) To ensure the participation of workers in decision and policy-making processes affecting their rights, duties and welfare. Art. 255. Exclusive bargaining representation and workers’ participation in policy and decision-making. The labor organization designated or selected by the majority of the employees in an appropriate collective bargaining unit shall be the exclusive representative of the employees in such unit for the purpose of collective bargaining. However, an individual employee or group of employees shall have the right at any time to present grievances to their employer. Any provision of law to the contrary notwithstanding, workers shall have the right, subject to such rules and regulations as the Secretary of Labor and Employment may promulgate, to participate in policy and decision-making processes of the establishment where they are employed insofar as said processes will directly affect their rights, benefits and welfare. For this purpose, workers and employers may form labor-management councils: Provided, That the representatives of the workers in such labor-management councils shall be elected by at least the majority of all employees in said establishment. (As amended by Section 22, Republic Act No. 6715, March 21, 1989) Art. 277. Miscellaneous provisions. 9|Page

(g) The Ministry shall help promote and gradually develop, with the agreement of labor organizations and employers, labor-management cooperation programs at appropriate levels of the enterprise based on the shared responsibility and mutual respect in order to ensure industrial peace and improvement in productivity, working conditions and the quality of working life. (Incorporated by Batas Pambansa Bilang 130, August 21, 1981) (h) In establishments where no legitimate labor organization exists, labor-management committees may be formed voluntarily by workers and employers for the purpose of promoting industrial peace. The Department of Labor and Employment shall endeavor to enlighten and educate the workers and employers on their rights and responsibilities through labor education with emphasis on the policy thrusts of this Code. (As amended by Section 33, Republic Act No. 6715, March 21, 1989)

Philippine Airlines, Inc. v. NLRC, 225 SCRA 301 (93)
NATURE Petition for certiorari FACTS - On March 15, 1985, PAL completely revised its 1966 Code of Discipline. The Code was circulated among the employees and was immediately implemented, and some employees were subjected to the disciplinary measures. - The Philippine Airlines Employees Association (PALEA) filed a complaint before the NLRC contending that PAL, by its unilateral implementation of the Code, was guilty of unfair labor practice, specifically Paragraphs E and G of Art 249 and Art 253 of the Labor Code. PALEA alleged that copies of the Code had been circulated in limited numbers; that being penal in nature the Code must conform with the requirements of sufficient publication, and that the Code was arbitrary, oppressive, and prejudicial to the rights of the employees. It prayed that implementation of the Code be held in abeyance; that PAL should discuss the substance of the Code with PALEA; that employees dismissed under the Code reinstated and their cases subjected to further hearing; and that PAL be declared guilty of unfair labor practice and be ordered to pay damages. - PAL filed a MTD, asserting its prerogative as an employer to prescribe rules and regulations regarding employees' conduct in carrying out their duties and functions, and alleging that it had not violated the CBA or any provision of the Labor Code. ISSUE 1. WON the formulation of a Code of Discipline among employees is a shared responsibility of the employer and the employees HELD 1. YES. Ratio Employees have a right to participate in the deliberation of matters which may affect their rights and the formulation of policies relative thereto and one such matter is the formulation of a code of discipline. Reasoning It was only on March 2, 1989, with the approval of RA 6715, amending Art 211 of the Labor Code, that the law explicitly considered it a State policy "to ensure the participation of workers in decision and policy-making processes affecting their rights, duties and welfare." However, even in the absence of said clear provision of law, the exercise of management prerogatives was never considered boundless. Thus, in Cruz vs. Medina, it was held that management's prerogatives must be without abuse of discretion. - In San Miguel Brewery Sales Force Union vs. Ople, we upheld the company's right to implement a new system of distributing its products, but gave the following caveat: So long as a company's management prerogatives are exercised in good faith for the advancement the employer's interest and not for the purpose of defeating or circumventing the rights of the employee, under special laws or under valid agreements, this Court will uphold them. - All this points to the conclusion that the exercise of managerial prerogatives is not unlimited. It is circumscribed by limitations found in law, a CBA, or the general principles of fair play and justice. Moreover, it must be duly established that the prerogative being invoked is clearly a managerial one. - Verily, a line must be drawn between management prerogatives regarding business operations per se and those which affect the rights of the employees. In treating the latter, management should see to it that its employees are at least properly informed of its decisions or modes of action. PAL asserts that all its employees have been furnished copies of the Code, the LA and the NLRC found to the contrary, which finding, is entitled to great respect. - PALEA recognizes the right of the Company to determine matters of management policy and Company operations and to direct its manpower. Management of the Company includes the right to organize, plan, direct and control operations, to hire, assign employees to work, transfer employees from one department to another, to promote, demote, discipline, suspend or discharge employees for just
10 | P a g e

cause; to lay-off employees for valid and legal causes, to introduce new or improved methods or facilities or to change existing methods or facilities and the right to make and enforce Company rules and regulations to carry out the functions of management. The exercise by management of its prerogative shall be done in a just, reasonable, humane and/or lawful manner. - Such provision in the CBA may not be interpreted as cession of employees' rights to participate in the deliberation of matters which may affect their rights and the formulation of policies relative thereto. And one such matter is the formulation of a code of discipline. Industrial peace cannot be achieved if the employees are denied their just participation in the discussion of matters affecting their rights. Disposition Petition is DISMISSED.

Manila Electric Co. v. Quisumbing, 326 SCRA 172 (00)
NATURE Motion for Reconsideration FACTS - Members of the Meralco Employees and Workers Association (MEWA) filed a motion for reconsideration of an earlier decision of this Court directing the parties to execute a CBA incorporating the terms and conditions contained in the unaffected portions of the Secretary of Labor's orders, and prayed for certain modifications. ISSUE/S 1. WON the decision should be modified HELD 1. YES Ratio (On the requirement of consultation imposed by the Secretary in cases of contracting out for 6 months or more) A line must be drawn between management prerogatives regarding business operations per se and those which affect the rights of employees, and in treating the latter, the employer should see to it that its employees are at least properly informed of its decision or modes of action in order to attain a harmonious labor-management relationship and enlighten the workers concerning their rights. Hiring of workers is within the employer's inherent freedom to regulate and is a valid exercise of its management prerogative subject only to special laws and agreements on the matter and the fair standards of justice. Reasoning The management cannot be denied the faculty of promoting efficiency and attaining economy by a study of what units are essential for its operation. It has the ultimate determination of whether services should be performed by its personnel or contracted to outside agencies. Absent proof that management acted in a malicious or arbitrary manner, the Court will not interfere with the exercise of judgment by an employer. Obiter - On increasing the wage awards: The Court does "not seek to enumerate in this decision the factors that should affect wage determination" because collective bargaining disputes particularly those affecting the national interest and public service "requires due consideration and proper balancing of the interests of the parties to the dispute and of those who might be affected by the dispute.” - On the retroactivity of the arbitral awards: The law is silent as to the retroactivity of a CBA arbitral award or that granted not by virtue of the mutual agreement of the parties but by intervention of the government. Despite the silence of the law, the Court rules herein that CBA arbitral awards granted after six months from the expiration of the last CBA shall retroact to such time agreed upon by both employer and the employees or their union. Absent such an agreement as to retroactivity, the award shall retroact to the first day after the six-month period following the expiration of the last day of the CBA should there be one. In the absence of a CBA, the Secretary's determination of the date of retroactivity as part of his discretionary powers over arbitral awards shall control - On the grant of a housing loan but not a cooperative loan: The award of loans for housing is justified because it pertains to a basic necessity of life. It is part of a privilege recognized by the employer and allowed by law. In contrast, providing seed money for the establishment of the employee's cooperative is a matter in which the employer has no business interest or legal obligation. Disposition Petition PARTIALLY GRANTED. The arbitral award was made to retroact and the award of wages was increased from P1,900 to P2,000 for the years 1995 and 1996, subject to the monetary advances granted by petitioner to its rank-and-file employees during the pendency of this case assuming such advances had.

11 | P a g e

VII. Re: Wage Fixing – 211 (b); 263 (g); 99; 124

Art. 211. Declaration of Policy. 1. It is the policy of the State: (b) To promote free trade unionism as an instrument for the enhancement of democracy and the promotion of social justice and development; Art. 263. Strikes, picketing and lockouts. (g) When, in his opinion, there exists a labor dispute causing or likely to cause a strike or lockout in an industry indispensable to the national interest, the Secretary of Labor and Employment may assume jurisdiction over the dispute and decide it or certify the same to the Commission for compulsory arbitration. Such assumption or certification shall have the effect of automatically enjoining the intended or impending strike or lockout as specified in the assumption or certification order. If one has already taken place at the time of assumption or certification, all striking or locked out employees shall immediately return-to-work and the employer shall immediately resume operations and readmit all workers under the same terms and conditions prevailing before the strike or lockout. The Secretary of Labor and Employment or the Commission may seek the assistance of law enforcement agencies to ensure compliance with this provision as well as with such orders as he may issue to enforce the same. In line with the national concern for and the highest respect accorded to the right of patients to life and health, strikes and lockouts in hospitals, clinics and similar medical institutions shall, to every extent possible, be avoided, and all serious efforts, not only by labor and management but government as well, be exhausted to substantially minimize, if not prevent, their adverse effects on such life and health, through the exercise, however legitimate, by labor of its right to strike and by management to lockout. In labor disputes adversely affecting the continued operation of such hospitals, clinics or medical institutions, it shall be the duty of the striking union or locking-out employer to provide and maintain an effective skeletal workforce of medical and other health personnel, whose movement and services shall be unhampered and unrestricted, as are necessary to insure the proper and adequate protection of the life and health of its patients, most especially emergency cases, for the duration of the strike or lockout. In such cases, therefore, the Secretary of Labor and Employment may immediately assume, within twenty four (24) hours from knowledge of the occurrence of such a strike or lockout, jurisdiction over the same or certify it to the Commission for compulsory arbitration. For this purpose, the contending parties are strictly enjoined to comply with such orders, prohibitions and/or injunctions as are issued by the Secretary of Labor and Employment or the Commission, under pain of immediate disciplinary action, including dismissal or loss of employment status or payment by the locking-out employer of backwages, damages and other affirmative relief, even criminal prosecution against either or both of them. The foregoing notwithstanding, the President of the Philippines shall not be precluded from determining the industries that, in his opinion, are indispensable to the national interest, and from intervening at any time and assuming jurisdiction over any such labor dispute in order to settle or terminate the same. Art. 99. Regional minimum wages. The minimum wage rates for agricultural and non-agricultural employees and workers in each and every region of the country shall be those prescribed by the Regional Tripartite Wages and Productivity Boards. (As amended by Section 3, Republic Act No. 6727, June 9, 1989). Art. 124. Standards/Criteria for minimum wage fixing. The regional minimum wages to be established by the Regional Board shall be as nearly adequate as is economically feasible to maintain the minimum standards of living necessary for the health, efficiency and general well-being of the employees within the framework of the national economic and social development program. In the determination of such regional minimum wages, the Regional Board shall, among other relevant factors, consider the following: 1. The demand for living wages; 2. Wage adjustment vis-à-vis the consumer price index; 3. The cost of living and changes or increases therein; 4. The needs of workers and their families; 5. The need to induce industries to invest in the countryside; 6. Improvements in standards of living; 7. The prevailing wage levels; 8. Fair return of the capital invested and capacity to pay of employers; 9. Effects on employment generation and family income; and 10. The equitable distribution of income and wealth along the imperatives of economic and social development. The wages prescribed in accordance with the provisions of this Title shall be the standard prevailing minimum wages in every region. These wages shall include wages varying with industries, provinces or localities if in the judgment of the Regional Board, conditions make such local differentiation proper and necessary to effectuate the purpose of this Title. Any person, company, corporation, partnership or any other entity engaged in business shall file and register annually with the appropriate Regional Board, Commission and the National Statistics Office, an itemized listing of their labor component, specifying the names of their workers and employees below the managerial level, including learners, apprentices and disabled/handicapped workers who were hired under the terms prescribed in the employment contracts, and their corresponding salaries and wages. Where the application of any prescribed wage increase by virtue of a law or wage order issued by any Regional Board results in distortions of the wage structure within an establishment, the employer and the union shall negotiate to correct the distortions. Any dispute arising from wage distortions shall be resolved through the

12 | P a g e

grievance procedure under their collective bargaining agreement and, if it remains unresolved, through voluntary arbitration. Unless otherwise agreed by the parties in writing, such dispute shall be decided by the voluntary arbitrators within ten (10) calendar days from the time said dispute was referred to voluntary arbitration. In cases where there are no collective agreements or recognized labor unions, the employers and workers shall endeavor to correct such distortions. Any dispute arising therefrom shall be settled through the National Conciliation and Mediation Board and, if it remains unresolved after ten (10) calendar days of conciliation, shall be referred to the appropriate branch of the National Labor Relations Commission (NLRC). It shall be mandatory for the NLRC to conduct continuous hearings and decide the dispute within twenty (20) calendar days from the time said dispute is submitted for compulsory arbitration. The pendency of a dispute arising from a wage distortion shall not in any way delay the applicability of any increase in prescribed wage rates pursuant to the provisions of law or wage order. As used herein, a wage distortion shall mean a situation where an increase in prescribed wage rates results in the elimination or severe contraction of intentional quantitative differences in wage or salary rates between and among employee groups in an establishment as to effectively obliterate the distinctions embodied in such wage structure based on skills, length of service, or other logical bases of differentiation. All workers paid by result, including those who are paid on piecework, takay, pakyaw or task basis, shall receive not less than the prescribed wage rates per eight (8) hours of work a day, or a proportion thereof for working less than eight (8) hours. All recognized learnership and apprenticeship agreements shall be considered automatically modified insofar as their wage clauses are concerned to reflect the prescribed wage rates. (As amended by Republic Act No. 6727, June 9, 1989)

VIII. Re: Labor Injunction – 254, 218 (e); 264; Sec. 22, R.A. 8791

Art. 254. Injunction prohibited. No temporary or permanent injunction or restraining order in any case involving or growing out of labor disputes shall be issued by any court or other entity, except as otherwise provided in Articles 218 and 264 of this Code. (As amended by Batas Pambansa Bilang 227, June 1, 1982) Art. 218. Powers of the Commission. The Commission shall have the power and authority: (e) To enjoin or restrain any actual or threatened commission of any or all prohibited or unlawful acts or to require the performance of a particular act in any labor dispute which, if not restrained or performed forthwith, may cause grave or irreparable damage to any party or render ineffectual any decision in favor of such party: Provided, That no temporary or permanent injunction in any case involving or growing out of a labor dispute as defined in this Code shall be issued except after hearing the testimony of witnesses, with opportunity for cross-examination, in support of the allegations of a complaint made under oath, and testimony in opposition thereto, if offered, and only after a finding of fact by the Commission, to the effect: 1. That prohibited or unlawful acts have been threatened and will be committed and will be continued unless restrained, but no injunction or temporary restraining order shall be issued on account of any threat, prohibited or unlawful act, except against the person or persons, association or organization making the threat or committing the prohibited or unlawful act or actually authorizing or ratifying the same after actual knowledge thereof; 2. That substantial and irreparable injury to complainant’s property will follow; 3. That as to each item of relief to be granted, greater injury will be inflicted upon complainant by the denial of relief than will be inflicted upon defendants by the granting of relief; 4. That complainant has no adequate remedy at law; and 5. That the public officers charged with the duty to protect complainant’s property are unable or unwilling to furnish adequate protection. Such hearing shall be held after due and personal notice thereof has been served, in such manner as the Commission shall direct, to all known persons against whom relief is sought, and also to the Chief Executive and other public officials of the province or city within which the unlawful acts have been threatened or committed, charged with the duty to protect complainant’s property: Provided, however, that if a complainant shall also allege that, unless a temporary restraining order shall be issued without notice, a substantial and irreparable injury to complainant’s property will be unavoidable, such a temporary restraining order may be issued upon testimony under oath, sufficient, if sustained, to justify the Commission in issuing a temporary injunction upon hearing after notice. Such a temporary restraining order shall be effective for no longer than twenty (20) days and shall become void at the expiration of said twenty (20) days. No such temporary restraining order or temporary injunction shall be issued except on condition that complainant shall first file an undertaking with adequate security in an amount to be fixed by the Commission sufficient to recompense those enjoined for any loss, expense or damage caused by the improvident or erroneous issuance of such order or injunction, including all reasonable costs, together with a reasonable attorney’s fee, and expense of defense against the order or against the granting of any injunctive relief sought in the same proceeding and subsequently denied by the Commission. The undertaking herein mentioned shall be understood to constitute an agreement entered into by the complainant and the surety upon which an order may be rendered in the same suit or proceeding against said complainant and surety, upon a hearing to assess damages, of which hearing, complainant and surety shall have reasonable notice, the said complainant and surety submitting themselves to the jurisdiction of the Commission for that purpose. But nothing herein contained shall deprive any party having a claim or cause of action under or upon such undertaking from electing to pursue his ordinary remedy by suit at law or in equity: Provided, further, That the reception of evidence for the application of a writ of injunction may be delegated by the Commission to any of its Labor Arbiters who shall conduct such hearings in such places as he may determine to be accessible to the parties and their witnesses and shall submit thereafter his recommendation to the Commission. (As amended by Section 10, Republic Act No. 6715, March 21, 1989) 13 | P a g e

Art. 264. Prohibited activities. 1. No labor organization or employer shall declare a strike or lockout without first having bargained collectively in accordance with Title VII of this Book or without first having filed the notice required in the preceding Article or without the necessary strike or lockout vote first having been obtained and reported to the Ministry. No strike or lockout shall be declared after assumption of jurisdiction by the President or the Minister or after certification or submission of the dispute to compulsory or voluntary arbitration or during the pendency of cases involving the same grounds for the strike or lockout. Any worker whose employment has been terminated as a consequence of any unlawful lockout shall be entitled to reinstatement with full backwages. Any union officer who knowingly participates in an illegal strike and any worker or union officer who knowingly participates in the commission of illegal acts during a strike may be declared to have lost his employment status: Provided, That mere participation of a worker in a lawful strike shall not constitute sufficient ground for termination of his employment, even if a replacement had been hired by the employer during such lawful strike. 2. No person shall obstruct, impede, or interfere with, by force, violence, coercion, threats or intimidation, any peaceful picketing by employees during any labor controversy or in the exercise of the right to self-organization or collective bargaining, or shall aid or abet such obstruction or interference. 3. No employer shall use or employ any strike-breaker, nor shall any person be employed as a strike-breaker. 4. No public official or employee, including officers and personnel of the New Armed Forces of the Philippines or the Integrated National Police, or armed person, shall bring in, introduce or escort in any manner, any individual who seeks to replace strikers in entering or leaving the premises of a strike area, or work in place of the strikers. The police force shall keep out of the picket lines unless actual violence or other criminal acts occur therein: Provided, That nothing herein shall be interpreted to prevent any public officer from taking any measure necessary to maintain peace and order, protect life and property, and/or enforce the law and legal order. (As amended by Executive Order No. 111, December 24, 1986) 5. No person engaged in picketing shall commit any act of violence, coercion or intimidation or obstruct the free ingress to or egress from the employer’s premises for lawful purposes, or obstruct public thoroughfares. (As amended by Batas Pambansa Bilang 227, June 1, 1982) R.A. 8791, Section 22. Strikes and Lockouts. - The banking industry is hereby declared as indispensable to the national interest and, notwithstanding the provisions of any law to the contrary, any strike or lockout involving banks, if unsettled after seven (7) calendar days shall be reported by the Bangko Sentral to the secretary of Labor who may assume jurisdiction over the dispute or decide it or certify the sane to the National Labor Relations Commission for compulsory arbitration. However, the President of the Philippines may at any time intervene and assume jurisdiction over such labor dispute in order to settle or terminate the same. (6-E)

IX. Tripartism – 275

Art. 275. Tripartism and tripartite conferences. 1. Tripartism in labor relations is hereby declared a State policy. Towards this end, workers and employers shall, as far as practicable, be represented in decision and policy-making bodies of the government. 2. The Secretary of Labor and Employment or his duly authorized representatives may, from time to time, call a national, regional, or industrial tripartite conference of representatives of government, workers and employers for the consideration and adoption of voluntary codes of principles designed to promote industrial peace based on social justice or to align labor movement relations with established priorities in economic and social development. In calling such conference, the Secretary of Labor and Employment may consult with accredited representatives of workers and employers. (As amended by Section 32, Republic Act No. 6715, March 21, 1989) PART TWO WORKER RIGHT TO SELF-ORGANIZATION I. BASIS OF RIGHT S.S. Ventures International v. S.S. Ventures Labor Union, 559 SCRA 435 (08) Ventures is an export firm located in Bataan and is in the business of manufacturing rubbershoes. SS Ventures labor union (union), is a labor organization registered with DOLE. In 2000, the Union filed with Dole a petition for certification election in behalf of the rank and file employees of Ventures. There were 542 signatures, 82 f which belong to terminated Ventures employees. Ventures filed a petition to cancel the certificate of registration of the Union because: (1) The Union deliberately and maliciously included the names of more or less 82 former employees no longer connected with Ventures in its list of members who attended the organizational meeting and in the adoption/ratification of its constitution and by-laws held on January 9, 2000 in Mariveles, Bataan; and the Union forged the signatures of these 82 former employees to make it appear they took part in the organizational meeting and adoption and ratification of the constitution; (2) The Union maliciously twice entered the signatures of three persons namely: Mara Santos, Raymond Balangbang, and Karen Agunos; (3) No organizational meeting and ratification actually took place; and

14 | P a g e

(4) The Union's application for registration was not supported by at least 20% of the rank-and-file employees of Ventures, or 418 of the total 2,197- employee complement. Since more or less 82 of the 500 signatures were forged or invalid, then the remaining valid signatures would only be 418, which is very much short of the 439 minimum (2197 total employees x 20% = 439.4) required by the Labor Code. Union’s answer: (1) the organizational meeting actually took place on January 9, 2000 at the Shoe City basketball court in Mariveles; (2) the 82 employees adverted to in Ventures' petition were qualified Union members for, although they have been ordered dismissed, the one-year prescriptive period to question their dismissal had not yet lapsed; (3) it had complied with the 20%-member registration requirement since it had 542 members; and (4) the "double" signatures were inadvertent human error. Ventures showed affidavits showing that the 82 employees were coerced into signing the minutes that an organizational meeting was held. DOLE regional director found for Ventures. In a Motion for Reconsideration, the BLR, reversed DOLE decision. CA affirmed. Issue: could Ventures have the Union decertified for violating Art 239 (a) of the labor code? (misrepresentation with regard to adoption or ratification of the union’s constitution and like documents) The SC held: No. To decertify a union, it is not enough to show that the union includes ineligible employees in its membership. It must also be shown that there was misrepresentation, false statement, or fraud in connection with the application for registration and the supporting documents, such as the adoption or ratification of the constitution and bylaws or amendments thereto and the minutes of ratification of the constitution or by-laws, among other documents. the employees' withdrawal from a labor union made before the filing of the petition for certification election is presumed voluntary, while withdrawal after the filing of such petition is considered to be involuntary and does not affect the same After a labor organization has filed the necessary registration documents, it becomes mandatory for the BLR to check if the requirements under Art. 234 of the Labor Code have been sedulously complied with. If the union's application is infected by falsification and like serious irregularities, especial those appearing on the face of the application and its attachments, a union should be denied recognition as a legitimate labor organization. the issuance to the Union of Certificate of necessarily implies that its application for registration and the supporting documents thereof are prima facie free from any vitiating irregularities. The assailed inclusion of the said 82 individuals to the meeting and proceedings adverted to is not really fatal to the Union's cause for, as determined by the BLR, the allegations of falsification of signatures or misrepresentation with respect to these individuals are without basis. , the procedure for acquiring or losing union membership and the determination of who are qualified or disqualified to be members are matters internal to the union and flow from its right to self-organization. In its union records on file with this Bureau, respondent union submitted the names of [542] members x x x. This number easily complied with the 20% requirement, be it 1,928 or 2,202 employees in the establishment. Even subtracting the 82 employees from 542 leaves 460 union members, still within 440 or 20% of the maximum total of 2,202 rank-and-file employees. The issue surrounding the involvement of the 82 employees is a matter of membership or voter eligibility. It is not a ground to cancel union registration. "[F]or fraud and misrepresentation [to be grounds for] cancellation of union registration under Article 239 [of the Labor Code], the nature of the fraud and misrepresentation must be grave and compelling enough to vitiate the consent of a majority of union members." the Union points out that for almost seven (7) years following the filing of its petition, no certification election has yet been conducted among the rank-and-file employees. If this be the case, the delay has gone far enough and can no longer be allowed to continue. A certification election is exclusively the concern of employees and the employer lacks the legal personality to challenge it. WHEREFORE, the petition is DENIED A. Constitution, 1987 Constitution – Arts. III, Sec. 8, and XIII, Sec. 3 See also: 1935 Constitution – Art. III, Sec. 6; and 1973 Constitution – Art. IV, Sec. 7 1987 Constitution, Article III, Sec. 8. The right of the people, including those employed in the public and private sectors, to form unions, associations, or societies for purposes not contrary to law shall not be abridged. 1987 Constitution, Article XIII, Sec. 3. The State shall afford full protection to labor, local and overseas, organized and unorganized, and promote full employment and equality of employment opportunities for all. It shall guarantee the rights of all workers to self-organization, collective bargaining and negotiations, and peaceful concerted activities, including the right to strike in accordance with law. They shall be entitled to security of tenure, humane conditions of work, and a living wage. They shall also participate in policy and decision-making processes affecting their rights and benefits as may be provided by law. The State shall promote the principle of shared responsibility between workers and employers and the preferential use of voluntary modes in settling disputes, including conciliation, and shall enforce their mutual compliance therewith to foster industrial peace. The State shall regulate the relations between workers and employers, recognizing the right of labor to its just share in the fruits of production and the right of enterprises to reasonable returns to investments, and to expansion and growth. 1935 Constitution, Art. III, Sec. 6. The right to form associations or societies for purposes not contrary to law shall not be abridged. 1973 Constitution, Art. IV, Sec. 7. The right to form associations or societies for purposes not contrary to the law shall not be abridged.

15 | P a g e

B. Statutory – 243; 244; 245; 269; 212 (g); 212 (f) Art. 243. Coverage and employees’ right to self-organization. All persons employed in commercial, industrial and agricultural enterprises and in religious, charitable, medical, or educational institutions, whether operating for profit or not, shall have the right to self-organization and to form, join, or assist labor organizations of their own choosing for purposes of collective bargaining. Ambulant, intermittent and itinerant workers, self-employed people, rural workers and those without any definite employers may form labor organizations for their mutual aid and protection. (As amended by Batas Pambansa Bilang 70, May 1, 1980) Art. 244. Right of employees in the public service. Employees of government corporations established under the Corporation Code shall have the right to organize and to bargain collectively with their respective employers. All other employees in the civil service shall have the right to form associations for purposes not contrary to law. (As amended by Executive Order No. 111, December 24, 1986) Art. 245. Ineligibility of managerial employees to join any labor organization; right of supervisory employees. Managerial employees are not eligible to join, assist or form any labor organization. Supervisory employees shall not be eligible for membership in a labor organization of the rank-and-file employees but may join, assist or form separate labor organizations of their own. (As amended by Section 18, Republic Act No. 6715, March 21, 1989) Art. 269. Prohibition against aliens; exceptions. All aliens, natural or juridical, as well as foreign organizations are strictly prohibited from engaging directly or indirectly in all forms of trade union activities without prejudice to normal contacts between Philippine labor unions and recognized international labor centers: Provided, however, That aliens working in the country with valid permits issued by the Department of Labor and Employment, may exercise the right to self-organization and join or assist labor organizations of their own choosing for purposes of collective bargaining: Provided, further, That said aliens are nationals of a country which grants the same or similar rights to Filipino workers. (As amended by Section 29, Republic Act No. 6715, March 21, 1989) Art. 212. Definitions. (f) "Employee" includes any person in the employ of an employer. The term shall not be limited to the employees of a particular employer, unless the Code so explicitly states. It shall include any individual whose work has ceased as a result of or in connection with any current labor dispute or because of any unfair labor practice if he has not obtained any other substantially equivalent and regular employment. (g) "Labor organization" means any union or association of employees which exists in whole or in part for the purpose of collective bargaining or of dealing with employers concerning terms and conditions of employment. UST Faculty Union v. Bitonio, 318 SCRA 186 (99) NATURE Special civil action in the Supreme Court. Certiorari. FACTS - Union announced a general assembly to elect next union officers. - TRO was issued by med-arbiter enjoining them from conducting election. - UST held a general faculty assembly , attended by both union members and non-members. Here, appellants were elected as new union officers by acclamation and clapping of hands. - Appellees filed instant petition to seek injunctive relief and to nullify results of election. - Bitonio upheld med-arbiter and said election was void. He rejected contention that it was a legitimate exercise of right to self organization ISSUE/S Basis of right to self-organization (p5 of outline) / Workers with right of self-organization (p6 of outline) HELD Ratio Self-organization is a fundamental right to form, join or assist labor organizations for collective bargaining, mutual aid and protection . Whether employed for a definite period or not, employee shall be considered as such, beginning on 1 st day of service, for purposes of membership in a labor union. Corollary to this right is the prerogative not to join. Reasoning The election can’t be considered as exercise of right to self-organization because the petitioners’ frustration over the performance of the respondents could not justify the method they chose to impose their will on the union. 1. All Employees ART. 243. Coverage and employees’ right to self-organization. - All persons employed in commercial, industrial and agricultural enterprises and in religious, charitable, medical, or educational institutions, whether operating for profit or not, shall have the right to self-organization and to form, join, or assist labor organizations of their own choosing for purposes of collective bargaining. Ambulant, intermittent and itinerant workers, self-employed people, rural workers and those without any definite employers may form labor organizations for their mutual aid and protection. (As amended by Batas Pambansa Bilang 70, May 1, 1980). National Union of Bank Employees v. Minister of Labor 110 SCRA 274 (81) NATURE: MANDAMUS FACTS: -The Union filed this action to compel the public respondents to hold a certification election among the rank and file employees of Producers Bank. -The Bank agreed to recognize and negotiate with the Union as soon as the latter obtained its registration certificate as local union. -However, even if said certificate had already been secured, the Bank failed to submit its payroll of employees (which was required previously at the hearing for direct certification. ) - The Med- Arbiter issued an order for the holding of a certification election, a proceeding which was sought to be suspended by the Bank on the grounds that a prejudicial question was pending re: cancellation of the Union’s registration for allegedly engaging in prohibited and unlawful activities. 16 | P a g e

ISSUE: WON a certification election should be held despite the pending petition to cancel the Union’s certificate of Registration HELD: YES. The pendency of the petition for cancellation of the registration certificate of herein petitioner union is not a bar to the holding of a certification election. Reasoning: -The pendency of the petition for cancellation of the registration certificate of petitioner union founded on the alleged illegal strikes staged by the leaders and members of the intervenor union and petitioner union should not suspend the holding of a certification election, because there is no order directing such cancellation. - Aside from the fact that the petition for cancellation of the registration certificate of petitioner union has not yet been finally resolved, there is another fact that militates against the stand of the Bank, the liberal approach observed by this Court as to matters of certification election. - Atlas Free Workers Union (AFWU)-PSSLU Local vs. Hon. Carmelo C. Noriel, et al. : "[T]he Court resolves to grant the petition (for mandamus) in line with the liberal approach consistently adhered to by this Court in matters of certification election. The whole democratic process is geared towards the determination of representation, not only in government but in other sectors as well, by election. Thus, the Court has declared its commitment to the view that a certification election is crucial to the institution of collective bargaining, for it gives substance to the principle of majority rule as one ' of the basic concepts of a democratic policy" (National Mines and Allied Workers Union vs. Luna, 83 SCRA 610). - Scout Ramon V. Albano Memorial College vs. Noriel, et al.: ... The institution of collective bargaining is, to recall Cox, a prime manifestation of industrial democracy at work. The two parties to the relationship, labor and management, make their own rules by coming to terms. That is to govern themselves in matters that really count. As labor, however, is composed of a number of individuals, it is indispensable that they be represented by a labor organization of their choice. Thus may be discerned how crucial is a certification election. So our decisions from the earliest case of PLDT Employees Union v. PLDT Co. Free Telephone Workers Union to the latest, Philippine Communications, Electronics & Electricity Workers' Federation (PCWF) v. Court of Industrial Relations, had made clear. The same principle was again given expression in language equally emphatic in the subsequent case of Philippine Association of Free Labor Unions v. Bureau of Labor Relations: 'Petitioner thus appears to be woefully lacking in awareness of the significance of a certification election for the collective bargaining process. It is the fairest and most effective way of determining which labor organization can truly represent the working force. It is a fundamental postulate that the will of the majority, if given expression in an honest election with freedom on the part of the voters to make their choice, is controlling. No better device can assure the institution of industrial democracy with the two parties to a business enterprise, management and labor, establishing a regime of self-rule .' That is to accord respect to the policy of the Labor Code, indisputably partial to the holding of a certification election so as to arrive in a manner definitive and certain concerning the choice of the labor organization to represent the workers in a collective bargaining unit (emphasis supplied). - Cancellation of the registration certificate is not the only resultant penalty in case of any violation of the Labor Code. (See Sec. 8 Rule II Book V of Labor Code, in relation to A273) - As aptly ruled by respondent Bureau of Labor Relations Director Noriel: "The rights of workers to self-organization finds general and specific constitutional guarantees. Section 7, Article IV of the Philippine Constitution provides that the right to form associations or societies for purposes not contrary to law shall not be abridged. This right is more pronounced in the case of labor. Section 9, Article II (ibid) specifically declares that the State shall assure the rights of workers to self-organization, collective bargaining, security of tenure and just and humane conditions of work. Such constitutional guarantees should not be lightly taken much less easily nullified. A healthy respect for the freedom of association demands that acts imputable to officers or members be not easily visited with capital punishments against the association itself" Dispositive: THE WRIT OF mandamus IS GRANTED; BLR DIR ORDERED TO CALL AND DIRECT THE IMMEDIATE HOLDING OF A CERTIFICATION ELECTION. C. Universal Declaration of Human Rights – Preamble; Arts. 2; International Covenant on Economic, Social and Cultural Rights – Arts. 2 and 8 (a); International Covenant on Civil and Political Rights – Art. 22 UDHR, Preamble Whereas recognition of the inherent dignity and of the equal and inalienable rights of all members of the human family is the foundation of freedom, justice and peace in the world, Whereas disregard and contempt for human rights have resulted in barbarous acts which have outraged the conscience of mankind, and the advent of a world in which human beings shall enjoy freedom of speech and belief and freedom from fear and want has been proclaimed as the highest aspiration of the common people, Whereas it is essential, if man is not to be compelled to have recourse, as a last resort, to rebellion against tyranny and oppression, that human rights should be protected by the rule of law, Whereas it is essential to promote the development of friendly relations between nations, Whereas the peoples of the United Nations have in the Charter reaffirmed their faith in fundamental human rights, in the dignity and worth of the human person and in the equal rights of men and women and have determined to promote social progress and better standards of life in larger freedom, Whereas Member States have pledged themselves to achieve, in co-operation with the United Nations, the promotion of universal respect for and observance of human rights and fundamental freedoms, Whereas a common understanding of these rights and freedoms is of the greatest importance for the full realization of this pledge, Now, Therefore THE GENERAL ASSEMBLY proclaims THIS UNIVERSAL DECLARATION OF HUMAN RIGHTS as a common standard of achievement for all peoples and all nations, to the end that every individual and every organ of society, keeping this Declaration constantly in mind, shall strive by teaching and education to promote 17 | P a g e

respect for these rights and freedoms and by progressive measures, national and international, to secure their universal and effective recognition and observance, both among the peoples of Member States themselves and among the peoples of territories under their jurisdiction. UDHR, Article 2. Everyone is entitled to all the rights and freedoms set forth in this Declaration, without distinction of any kind, such as race, colour, sex, language, religion, political or other opinion, national or social origin, property, birth or other status. Furthermore, no distinction shall be made on the basis of the political, jurisdictional or international status of the country or territory to which a person belongs, whether it be independent, trust, non-self-governing or under any other limitation of sovereignty. ICESCR, Article 2 1. Each State Party to the present Covenant undertakes to take steps, individually and through international assistance and co-operation, especially economic and technical, to the maximum of its available resources, with a view to achieving progressively the full realization of the rights recognized in the present Covenant by all appropriate means, including particularly the adoption of legislative measures. 2. The States Parties to the present Covenant undertake to guarantee that the rights enunciated in the present Covenant will be exercised without discrimination of any kind as to race, colour, sex, language, religion, political or other opinion, national or social origin, property, birth or other status. 3. Developing countries, with due regard to human rights and their national economy, may determine to what extent they would guarantee the economic rights recognized in the present Covenant to non-nationals. Article 8 1. The States Parties to the present Covenant undertake to ensure: (a) The right of everyone to form trade unions and join the trade union of his choice, subject only to the rules of the organization concerned, for the promotion and protection of his economic and social interests. No restrictions may be placed on the exercise of this right other than those prescribed by law and which are necessary in a democratic society in the interests of national security or public order or for the protection of the rights and freedoms of others; (b) The right of trade unions to establish national federations or confederations and the right of the latter to form or join international trade-union organizations; (c) The right of trade unions to function freely subject to no limitations other than those prescribed by law and which are necessary in a democratic society in the interests of national security or public order or for the protection of the rights and freedoms of others; (d) The right to strike, provided that it is exercised in conformity with the laws of the particular country. 2. This article shall not prevent the imposition of lawful restrictions on the exercise of these rights by members of the armed forces or of the police or of the administration of the State. 3. Nothing in this article shall authorize States Parties to the International Labour Organisation Convention of 1948 concerning Freedom of Association and Protection of the Right to Organize to take legislative measures which would prejudice, or apply the law in such a manner as would prejudice, the guarantees provided for in that Convention. ICPPR, Article 22 1. Everyone shall have the right to freedom of association with others, including the right to form and join trade unions for the protection of his interests. 2. No restrictions may be placed on the exercise of this right other than those which are prescribed by law and which are necessary in a democratic society in the interests of national security or public safety, public order (ordre public), the protection of public health or morals or the protection of the rights and freedoms of others. This article shall not prevent the imposition of lawful restrictions on members of the armed forces and of the police in their exercise of this right. 3. Nothing in this article shall authorize States Parties to the International Labour Organisation Convention of 1948 concerning Freedom of Association and Protection of the Right to Organize to take legislative measures which would prejudice, or to apply the law in such a manner as to prejudice, the guarantees provided for in that Convention. D. ILO Convention No. 87, Art. 2, 11 and 98 ILO 87, Art. 2. Workers and employers, without distinction whatsoever, shall have the right to establish and, subject only to the rules of the organisation concerned, to join organisations of their own choosing without previous authorisation. ILO 87, Art. 11. Each Member of the International Labour Organisation for which this Convention is in force undertakes to take all necessary and appropriate measures to ensure that workers and employers may exercise freely the right to organise. Standard Chartered Bank Employees Union v. Confessor, 432 SCRA 308 (04) Facts: • Petitioner Union is the exclusive bargaining agent of the rank and file employees of Standard Chartered Bank (Bank). • The Union and the Bank signed a 5 year CBA in 1990 with a provision to renegotiate its terms in its 3 rd year. • In 1993, the Union initiated negotiations. It submitted its proposal to the Bank as well as a list containing the members of its negotiating panel. The Bank submitted its counter-proposal and the respective list of the members of its negotiating panel. • Before negotiations can begin, the Union suggested that the Bank’s lawyers are to be excluded from its negotiating panel. The Bank agreed. The Bank however, suggested that Jose P. Umali, Jr., the President of NUBE be excluded from the Union’s negotiating panel. However, Umali remained a member of the Union’s panel. 18 | P a g e



During the preliminary negotiations, the parties laid down the ground rules. The Bank suggested that the negotiation be kept a “family affair”. The negotiations on the revisions on the CBA began on March 12, 1993. The proposed non-economic provisions were discussed first. The parties were not able to reach at an agreement leaving some of the provisions as DEFERRED/DEADLOCKED. • Afterwards, negotiations on the economic provisions began on May 18, 1993. Like before, the Union and the Bank could not reach an agreement. Umali chided the Bank for the insufficiency of its counter-proposal and reminded the Bank how they got what they wanted in 1987 and that they were willing to resort to such means if needed. • The negotiations resumed but even after the submission of counter-proposals from both parties, the impasse remained. Exasperated, Umali asserted that it would be easier to bargain if both parties trusted each other like before. The Bank requested the Union to refrain from involving personalities and to focus on the negotiations. • On June 21, 1993, the Union declared a deadlock and filed a Notice of Strike with the NCMB. • On its part, the Bank filed a complaint for unfair labor practices against the Union with the Labor Arbiter. The Bank claimed that the Union engaged in blue sky bargaining (unrealistic or unreasonable demands in negotiations where neither concedes anything or demands the impossible) . The Bank also claimed that the Union violated the no strike-no lockout clause of the CBA. • Then Secretary of Labor Confesor assumed jurisdiction of the dispute and dismissed the ULP charges of both the Bank and Union. She also ordered the award of certain benefits. On March 22, 1994, the Bank and Union signed the CBA. • Claiming that the SOLE committed grave abuse of discretion when she dismissed the ULP charge filed by the Union, they filed a petition praying that her Order be set aside. In the petition, it was claimed that the Bank committed ULP when it asked that Umali be excluded from the panel and when it asked that the negotiations be kept a “family affair”. It also claimed that the Bank engaged in surface bargaining. For its part the bank reiterated its claims for ULP against the Union in its complaint before the Labor Arbiter. Held: • No ULP committed by the Bank or Union. • Article 248(a) considers it an ULP if the employer interferes, restrains or coerces employees in the exercise of their right to self organization or the right to form an association. • In order to show that the employer committed ULP under the Labor Code, substantial evidence is required to support the claim. Substantial evidence is such relevant evidence as a reasonable mind might accept as adequate to support a conclusion. • The facts show that the suggestion to exclude Umali Jr., was not an anti-unioon conduct from which it can be inferred that the Bank adopted to undermine the free exercise of the right to self-organization and collective bargaining of the employees especially when it was requested after the Union requested the exclusion of the Bank’s lawyer from its negotiating panel. • Further, the ULP charge was merely an afterthought as the complaint was only made after a deadlock was declared by the Union. • There was no surface bargaining on the part of the Bank. Surface bargaining is defined as going through the motions of negotiating without any legal intent to reach an agreement. Such is a question of the intent of the party in question and usually such intent can be inferred from the totality of the challenged party’s conduct both at and away from the table. • The duty to bargain does not compel either party to agree to a proposal or require the making of a concession. • There was no grave abuse of discretion on the part of the Secretary of Labor. It cannot be said that she acted in a capricious and whimsical exercise of judgment. There was no showing that the public respondent exercised her power in an arbitrary and despotic manner by reason of passion or personal hostility. • Likewise, neither is the Union guilty of ULP for engaging in blue sky bargaining. The demands of the Union were not exaggerated or unreasonably but based on the data of rank and file employees and other prevailing economic benefits received by employees in the industry. II. WORKER QUALIFICATION – 277 (c) AND REQUIRED ACTION – CONSTITUTION AND BY-LAW Art. 277. Miscellaneous provisions. (c) Any employee, whether employed for a definite period or not, shall, beginning on his first day of service, be considered as an employee for purposes of membership in any labor union. (As amended by Section 33, Republic Act No. 6715) UST Faculty Union v. Bitonio, 318 SCRA 185 (99) Facts: • Petitioner USTFU is the exclusive bargaining representative of the faculty of UST. In September 1996, the Union announced the convening of a general assembly on 05 October 1996. Among the agenda in the general assembly is the election of USTFU. The proposed election was contested by some members of the Union alleging that the preparations for the said election as demanded by the Union’s constitution and by-laws (CBL) were not satisfied. The meeting was moved earlier to 04 October 1996.

19 | P a g e



On 04 October 1996, the general faculty assembly was held. It was attended by both members of USTFU and non-members thereof. In the said general assembly a new set of USTFU’s officers was elected by acclamation and clapping of hands. • Respondents filed a petition with the med-arbiter alleging the illegality of the election conducted on 04 October 1996 as it did not conform to the requirements of the CBL. • In the meantime a new CBA was declared to be ratified purportedly by a majority of UST’s academic community. • The med-arbiter rendered a decision declaring the election done in 04 October 1996 as null and void for not having been executed conformably to the CBL of the Union. The said decision was affirmed by public respondent Bitonio Jr., hence this petition. Issue: WON the election conducted on 04 October 1996 is null and void Held: • Yes the election is null and void. Petitioners argue that the anomalies allegedly comtted by the respondents impelled them to conduct the election on 04 October 1996 and that such exercise was pursuant to their right to self-organization. • Self-organization is a fundamental right guaranteed by the workers by the Philippine Constitution and the Labor Code. Such right is better understood in the context of ILO Convention No. 87 to which the Philippines is a signatory. It provides that the incident of the assertion of the right to self-organization is reflected in the promulgation of the union’s CBL which governs the relationship between the members of the union. It is where the rights, duties and obligations, powers, functions and authority of the officers and as well as the members is defined. It determines the validity of the acts done by any officer or member of the union. • Clearly the election conducted on 04 October 1996 was tainted with irregularities as: o The assembly was not convened in accordance with the provision of the CBL o There was no commission of elections to oversee the election as mandated by Sections 1 and 2 of Article IX of the CBL o The purported election was not done thru secret balloting as provided by Section 6 Article IX of the CBL as well as Article 241 (c) of the Labor Code • The foregoing infirmities more than justifies the conclusion by the public respondents that the proceedings were rendered void due to lack of due process. As to the contention that the 04 October 1996 assembly suspended the operation of the CBL, such argument is bereft of merit. Such is in fact, an implied admission that the election held on that date could not be considered as valid under the existing USTFU constitution. III. EXTENT AND SCOPE OF RIGHT – 243, 246 Art. 243. Coverage and employees’ right to self-organization. All persons employed in commercial, industrial and agricultural enterprises and in religious, charitable, medical, or educational institutions, whether operating for profit or not, shall have the right to self-organization and to form, join, or assist labor organizations of their own choosing for purposes of collective bargaining. Ambulant, intermittent and itinerant workers, self-employed people, rural workers and those without any definite employers may form labor organizations for their mutual aid and protection. (As amended by Batas Pambansa Bilang 70, May 1, 1980) Art. 246. Non-abridgment of right to self-organization. It shall be unlawful for any person to restrain, coerce, discriminate against or unduly interfere with employees and workers in their exercise of the right to self-organization. Such right shall include the right to form, join, or assist labor organizations for the purpose of collective bargaining through representatives of their own choosing and to engage in lawful concerted activities for the same purpose for their mutual aid and protection, subject to the provisions of Article 264 of this Code. (As amended by Batas Pambansa Bilang 70, May 1, 1980) Reyes v. Trajano, 209 SCRA 484 (92) Facts: • In the certification election for the exclusive representative of the bargaining unit for the Tri-Union Industries Corporation, two unions vie for the support of the workers. • The competing unions were Tri-Union Employees Union-Organized Labor Association in Line Industries and Agriculture (TUEU-OLALIA) and Trade Union of the Philippines and Allied Services (TUPAS). 348 workers were supposed to participate in the certification election but only 240 cast their vote. • Among the 240 employees, 141 were members of the Iglesia Ni Kristo who all voted for NO UNION. Their votes were challenged on the ground that they should not be allowed to vote as they are not members of any union and refused to participate in the previous certification elections. It is also argued that their religion does not allow membership in labor unions and thus they should not be allowed to vote in the certification elections. • The INK employees filed a case before the Med-Arbiter to protest the exclusion of their votes. The Med-Arbiter dismissed the petition. On appeal to the BLR, Assistant Labor Secretary Cresenciano Trajano affirmed the Med-Arbiter’s decision as the INK employees are bereft of legal personality to protest their disenfranchisement. Issue: WON the votes of the 141 members of the INK can be validly excluded from the certification election. Held: • Guaranteed to all employees or workers is the right to self-organization and to form, join or assist labor organizations of their own choosing for purposes of collective bargaining.

20 | P a g e

• • • •

The right not to join, affiliate with or assist any union and to disaffiliate or resign from a labor organization is subsumed in the right to join affiliate with or assist any union. The said right includes the right to refuse and refrain from exercising such right. Just as anyone cannot be denied the exercise of a right granted by law, neither can they be compelled to exercise such a conferred right. The purpose of a certification election is to determine the wishers of majority of the employees in the appropriate bargaining unit as to whether to be or not to be represented by a labor organization and if yes, by what particular labor organization. If the results show that no union is desired by the workers, the minority cannot impose their will on the majority and they will have to wait for the next certification election to attempt a change. The fact that the INK members, as employees voted for NO UNION, they were simply exercising that right of self-organization albeit in its negative aspect. All bona fide employees possess such right. The fact that they did not participate in previous certification elections nor the fact that they are not members of any union does not deprive them of such right.

Pan-American World Airways, Inc. v. Pan-American Employees Association, 27 SCRA 1202 (69) FACTS: Respondent Union filed a strike with the Department of Labor against the petitioner company. On that same day, the union declared and maintained such strike. The strike, being an industrial dispute affecting the national interest, was certified by the President to CIR. Several conferences were held to settle their dispute. The position of the Union was that they would not return to work unless its officers were also included in the return-to-work order. However, petitioner company alleged that the strike that the union held was illegal. Hence, its officers should be liable for dismissal. The CIR issued an order requiring petitioner company to accept the five union officers pending resolution on the merits of the dispute involved in the strike. An MR was filed but was denied by the same court. Hence, the union filed the instant petition, alleging a grave abuse of discretion of the said court. ISSUES & HELD: 1. WON CIR has jurisdiction over the case. Yes.The case comes under the operation of CA 103 which empowers the CIR to act on cases of labor disputes in industries indispensable to the national interest. Since the court was granted authority by the president to find a solution in an industrial dispute and such solution consists in ordering of employees to return back to work, it cannot be said that the CIR does not have jurisdiction over such case. 2. WON the five union officers should be included in the return-to-work order. Yes. The record is bereft of the slightest indication that any danger is to be expected from their return to work. However, the Court said that the greater offense is to the labor movement itself, mores specifically the right to self organization. There is both a constitutional and statutory recognition that laborers have the right to form unions which would take care of their interests vis-avis their employers. Their freedom would be rendered nugatory if they could not choose their own leaders who would represent them to the management. Union of Supervisors (R.B.)-NATU v. Sec. of Labor, 109 SCRA 139 (81) FACTS: Petitioner Union filed with the NLRC a complaint against respondent Bank, charging it with unfair labor practice committed against Norberto Luna, the president of the union, for harassment, unjust suspension from his employment as a branch manager, and unlawful dismissal as administrator and secretary of the fund, all due to his militant espousal and defense of workers' rights. It was alleged that Luna uttered libelous remarks at the meeting of the Bord of Trustees of the RB Provident Fund. Afterwhich, a supplemental complaint was filed, alleging that the respondent bank terminated Luna's employment for grave misconduct and insubordination. The NLRC Arbitrator rendered a decision, finding that Luna's dismissal was without sufficient just cause and ordering the company to pay complainant separation pay. With regards to the charges of unfair labor practice, the arbitrator dismissed the said charge. On appeal, the NLRC affirmed the aforementioned decision. The petitioner thereafter appealed to the Secretary of Labor, which affirmed the decision insofar as it granted clearance for the termination of Luna's employment and discharging the unfair labor charge. HELD: The SC held that Luna's dismissal had no justifiable ground. Mr. Luna's utterance at the meeting of an official board are priveleged for it was precisely in acting out his role as the president of the union to protect the interests of the members of the Provident Fund from what he believed to be a risky venture on the part of the management, hence, his statements should therefore be regarded as falling under protected labor activity. Moreover, the other basis for dismissal – insubordination – also appears to be without justifiable ground. The “order” which was allegedly diobeyed by Luna was only a letter-request from the Provident Fund Chairman, requesting him to turnover the records of Provident Fund. When Luna explained that he felt that it would be better to keep the records, the management preferred as many charges as it could frame against Luna. IV. WORKERS WITH RIGHT OF SELF-ORGANIZATION – Art. III, Sec. 8; and Art. XIII, Sec. 3, 1987 Constitution 1987 Constitution, Article III, Sec. 8. The right of the people, including those employed in the public and private sectors, to form unions, associations, or societies for purposes not contrary to law shall not be abridged. 1987 Constitution, Article XIII, Sec. 3. It shall guarantee the rights of all workers to self-organization, collective bargaining and negotiations, and peaceful concerted activities, including the right to strike in accordance with law. A. All Employees – 243, 212 (f)

21 | P a g e

Art. 243. Coverage and employees’ right to self-organization. All persons employed in commercial, industrial and agricultural enterprises and in religious, charitable, medical, or educational institutions, whether operating for profit or not, shall have the right to self-organization and to form, join, or assist labor organizations of their own choosing for purposes of collective bargaining. Ambulant, intermittent and itinerant workers, self-employed people, rural workers and those without any definite employers may form labor organizations for their mutual aid and protection. (As amended by Batas Pambansa Bilang 70, May 1, 1980) Art. 212. Definitions. (f) "Employee" includes any person in the employ of an employer. The term shall not be limited to the employees of a particular employer, unless the Code so explicitly states. It shall include any individual whose work has ceased as a result of or in connection with any current labor dispute or because of any unfair labor practice if he has not obtained any other substantially equivalent and regular employment. 1. Non-Profit Organization FEU-Dr. Nicanor Reyes Medical Foundation, Inc. v. Trajano, 152 SCRA 725 (87) FACTS: Alliance Filipino Workers, members of which are employees of petitioner FEU-Dr. Nicanor Reyes Memorial Foundation, Inc., filed a petition for Consent/or Certification Election with the Ministry of Labor. The petitioner opposed the petition on the ground that a similar petition is pending resolution before the SC. AFW admitted that it filed a similar petition, but this petition was denied by MED Arbiter and the Secretary of Labor on appeal, on the ground that the petitioner was a nonstock, non-profit medical institution. Pursuant to Art. 244 of the Labor Code, the employees may not form, join, or organize a union. The Med Arbiter declared that a certification election be conducted to determine the exclusive bargaining representative of the company's employees. This order was affirmed by the respondent director. ISSUE: WON respondent director gravely abused his discretion in granting the petition for certification of election. HELD: No. At the time the petition for certification election was filed, Article 244 was already amended by BP Blg 70, which added the phrase whether operating for profit or not. Under the Art. 244 LC, there is no doubt that rank and file employees of non-profit medical institutions are now permitted to form, organize or join labor unions of their choice for purposes of collective bargaining. Since private respondent had complied with the requisites provided by law for calling a certification of election, it was incumbent upon respondent Director to conduct such certification election to ascertain the bargaining representative of the employees. Victoria v. Inciong, 157 SCRA 339 (88) Petitioner Saturno Victoria was employed on March 17, 1956 by private respondent Far East Broadcasting Company, Incorporated as a radio transmitter operator. Sometime in July 1971, he and his co-workers organized the Far East Broadcasting Company Employees Association. After registering their association with the then Department of Labor, they demanded recognition of said association by the company but the latter refused on the ground that being a non-profit, non-stock, noncommercial and religious corporation, it is not covered by Republic Act 875, otherwise known as the Industrial Peace Act, the labor law enforced at that time. Several conciliation meetings were held at the Department of Labor Director of Labor Rel: no right to bargain collectively Association held strike Company sued for damages and injunction which CFI granted, later company asked that strike be declared illegal Case on strike transferred to NLRC created by PD 21 LA:employees to return to work without prejudice to the pending CFI case LA affirmed by NLRC, SOLE and OP CFI: strike illegal, FEBC is a non-profit/non-stock org which does not issue dividends, cannot compel to recognize labor org because not covered by RA 875 Because of CFI decision, FEBC dismissed Victoria as an officer in the illegal strike to which Victoria filed ID case with LA (another issue altogether) SKIPPING TO THE ACTUAL RELATED ISSUE: Whether or not right to self-organization and collective bargaining exists vis. non-profit/non stock org SC: The strike staged by the union in 1972 was a futile move. The law then enforced, Republic Act 875 specifically excluded respondent company from its coverage. Even if the parties had gone to court to compel recognition, no positive relief could have been obtained since the same was not sanctioned by law. Because of this, there was no necessity on the part of private respondent to show specific acts of petitioner during the strike to justify his dismissal. This is a matter of responsibility and of answerability. Petitioner as a union leader, must see to it that the policies and activities of the union in the conduct of labor relations are within the precepts of law. Petitioner should have known and it was his duty to impart this imputed knowledge to the members of the union that employees and laborers in non- profit organizations are not covered by the provisions of the Industrial Peace Act and the Court of Industrial Relations [in the case at bar, the Court of First Instance] has no jurisdiction to entertain petitions of labor unions or organizations of said non-profit organizations for certification as the exclusive bargaining representatives of said employees and laborers. As a strike is an economic weapon at war with the policy of the Constitution and the law at that time, a resort thereto by laborers shall be deemed to be a choice of remedy peculiarly their own and outside of the statute, and as such, the strikers must accept all the risks attendant upon their choice. If they succeed and the employer succumbs, the law will not stand in their way in the enjoyment of the lawful fruits of their victory. But if they fail, they cannot thereafter invoke the protection of the law for the consequences of their conduct unless the right they wished vindicated is one which the law will, by all means, protect and enforce. 22 | P a g e

(PLEASE NOTE THAT UNDER THE CURRENT LAW, even employees of non-profit orgs have a right to self-organization) 2. Members – Religious Group Victoriano v. Elizalde Workers Union, 59 SCRA 54 (74) Benjamin Victoriano (hereinafter referred to as Appellee), a member of the religious sect known as the "Iglesia ni Cristo", had been in the employ of the Elizalde Rope Factory, Inc. (hereinafter referred to as Company) since 1958. As such employee, he was a member of the Elizalde Rope Workers' Union (hereinafter referred to as Union) which had with the Company a collective bargaining agreement containing a closed shop provision which reads as follows: Membership in the Union shall be required as a condition of employment for all permanent employees workers covered by this Agreement. The collective bargaining agreement expired on March 3, 1964 but was renewed the following day, March 4, 1964. Under Section 4(a), paragraph 4, of Republic Act No. 875, prior to its amendment by Republic Act No. 3350, the employer was not precluded "from making an agreement with a labor organization to require as a condition of employment membership therein, if such labor organization is the representative of the employees." On June 18, 1961, however, Republic Act No. 3350 was enacted, introducing an amendment to — paragraph (4) subsection (a) of section 4 of Republic Act No. 875, as follows: ... "but such agreement shall not cover members of any religious sects which prohibit affiliation of their members in any such labor organization". Victoriano was Iglesia, and presented his resignation to appellant Union in 1962, and when no action was taken thereon, he reiterated his resignation on September 3, 1974. Thereupon, the Union wrote a formal letter to the Company asking the latter to separate Appellee from the service in view of the fact that he was resigning from the Union as a member. The management of the Company in turn notified Appellee and his counsel that unless the Appellee could achieve a satisfactory arrangement with the Union, the Company would be constrained to dismiss him from the service. This prompted Appellee to file an action for injunction, docketed as Civil Case No. 58894 in the Court of First Instance of Manila to enjoin the Company and the Union from dismissing Appellee. CFI: Elizalde cannot be dismissed in light of RA 3350 exempting members of religious organizations from being dismissed due to closed shop provisions in CBA ISSUE: whether or not in light of the closed shop provision in the CBA, Elizalde should be dismissed for terminating his membership with the union. UNION ARGUMENTS: 1. RA 3350 is unconstitutional because a. it deprives members of relig orgs of their constitutional right to form or join lawful associations or organizations guaranteed by the Bill of Rights, and thus becomes obnoxious to Article III, Section 1 (6) of the 1935 Constitution b. it impairs obligations of contracts (CBA and closed shop) 2. discriminates by favoring certain relig sectors in violation of Article Ill, Section 1 (7) of the 1935 Constitution; and while said Act unduly protects certain religious sects, it leaves no rights or protection to labor organizations. 3. violates the constitutional provision that "no religious test shall be required for the exercise of a civil right," in that the laborer's exercise of his civil right to join associations for purposes not contrary to law has to be determined under the Act by his affiliation with a religious sect; that conversely, if a worker has to sever his religious connection with a sect that prohibits membership in a labor organization in order to be able to join a labor organization, said Act would violate religious freedom

23 | P a g e

1. RA 3350 is unconstitutional because a. it deprives members of relig orgs of their constitutional right to form or join lawful associations or organizations guaranteed by the Bill of Rights, and thus becomes obnoxious to Article III, Section 1 (6) of the 1935 Constitution

b.

it impairs obligations of contracts (CBA and closed shop)

a right comprehends at least two broad notions, namely: first, liberty or freedom, i.e., the absence of legal restraint, whereby an employee may act for himself without being prevented by law; and second, power, whereby an employee may, as he pleases, join or refrain from Joining an association. It is, therefore, the employee who should decide for himself whether he should join or not an association; and should he choose to join, he himself makes up his mind as to which association he would join; and even after he has joined, he still retains the liberty and the power to leave and cancel his membership with said organization at any time.;the right to join associations includes the right not to join or to resign from a labor org; the Act has given substance to such right by prohibiting the compulsion of workers to join labor organizations; What the exception provides, therefore, is that members of said religious sects cannot be compelled or coerced to join labor unions even when said unions have closed shop agreements with the employers; that in spite of any closed shop agreement, members of said religious sects cannot be refused employment or dismissed from their jobs on the sole ground that they are not members of the collective bargaining union. It is clear, therefore, that the assailed Act, far from infringing the constitutional provision on freedom of association, upholds and reinforces it. It does not prohibit the members of said religious sects from affiliating with labor unions. It still leaves to said members the liberty and the power to affiliate, or not to affiliate, with labor unions. If, notwithstanding their religious beliefs, the members of said religious sects prefer to sign up with the labor union, they can do so. If in deference and fealty to their religious faith, they refuse to sign up, they can do so; the law does not coerce them to join; neither does the law prohibit them from joining; and neither may the employer or labor union compel them to join. Republic Act No. 3350, therefore, does not violate the constitutional provision on freedom of association. agreement was already in existence at the time Republic Act No. 3350 was enacted on June 18, 1961, and it cannot, therefore, be deemed to have been incorporated into the agreement; The Act, therefore, introduced a change into the express terms of the union security clause; the Company was partly absolved by law from the contractual obligation it had with the Union of employing only Union members in permanent positions, It cannot be denied, therefore, that there was indeed an impairment of said union security clause. It should not be overlooked, however, that the prohibition to impair the obligation of contracts is not absolute and unqualified In spite of the constitutional prohibition, the State continues to possess authority to safeguard the vital interests of its people. Legislation appropriate to safeguarding said interests may modify or abrogate contracts already in effect. 25 For not only are existing laws read into contracts in order to fix the obligations as between the parties, but the reservation of essential attributes of sovereign power is also read into contracts as a postulate of the legal order. All contracts made with reference to any matter that is subject to regulation under the police power must be understood as made in reference to the possible exercise of that power This has special application to contracts regulating relations between capital and labor which are not merely contractual, and said labor contracts, for being impressed with public interest, must yield to the common good Legislation impairing the obligation of contracts can be sustained when it is enacted for the promotion of the general good of the people, and when the means adopted to secure that end are reasonable. Both the end sought and the means adopted must be legitimate, i.e., within the scope of the reserved power of the state construed in harmony with the constitutional limitation of that power the purpose sought to be achieved by Republic Act No. 3350? Its purpose was to insure freedom of belief and religion, and to promote the general welfare by preventing discrimination against those members of religious sects which prohibit their members from joining labor unions, confirming thereby their natural, statutory and constitutional right to work, the fruits of which work are usually the only means whereby they can maintain their own life and the life of their dependents. It cannot be gainsaid that said purpose is legitimate. The questioned Act also provides protection to members of said religious sects against two aggregates of group strength from which the individual needs protection. The individual employee, at various times in his working life, is confronted by two aggregates of power — collective labor, directed by a union, and collective capital, directed by management. The union, an institution developed to organize labor into a collective force and thus protect the individual employee from the power of collective capital, is, paradoxically, both the champion of employee rights, and a new source of their frustration. 24 | P a g e the means adopted by the Act to achieve that purpose — exempting the members of said religious sects from coverage of union security agreements — is reasonable.

violates the "equal protection of laws" clause of the Constitution, it being a discriminately legislation, inasmuch as by exempting from the operation of closed shop agreement the members of the "Iglesia ni Cristo", it has granted said members undue advantages over their fellow workers, for while the Act exempts them from union obligation and liability, it nevertheless entitles them at the same time to the enjoyment of all concessions, benefits and other emoluments that the union might secure from the employer 5. violates the constitutional provision regarding the promotion of social justice 6. trade unionism in this country would be wiped out as employers would prefer to hire or employ members of the Iglesia ni Cristo in order to do away with labor organizations SC: No. CFI is right. Freedom of religion is ousts freedom to self-organization (simple answer) PRINCIPLES: (complicated answer) Vis AF: Union denies right to AF for Victoriano because of protection under Section 24 of Republic Act No. 875: No suit, action or other proceedings shall be maintainable in any court against a labor organization or any officer or member thereof for any act done by or on behalf of such organization in furtherance of an industrial dispute to which it is a party, on the ground only that such act induces some other person to break a contract of employment or that it is in restraint of trade or interferes with the trade, business or employment of some other person or with the right of some other person to dispose of his capital or labor SC: The mere fact that appellant is a labor union does not necessarily mean that all its acts are in furtherance of an industrial dispute. 75 Appellant Union, therefore, cannot invoke in its favor Section 24 of Republic Act No. 875. This case is not intertwined with any unfair labor practice case existing at the time when Appellee filed his complaint before the lower court. Vis defense against AF using 2208 of NCC: The article provides that attorney's fees and expenses of litigation may be awarded "when the defendant's act or omission has compelled the plaintiff ... to incur expenses to protect his interest "; and "in any other case where the court deems it just and equitable that attorney's fees and expenses of litigation should be recovered". In the instant case, it cannot be gainsaid that appellant Union's act in demanding Appellee's dismissal caused Appellee to incur expenses to prevent his being dismissed from his job. SC: AF allowed Kapatiran sa Meat and Canning Division v. Calleja, 162 SCRA 367 (88) From 1984 to 1987 TUPAS was the sole and exclusive collective bargaining representative of the workers in the Meat and Canning Division of the Universal Robina Corporation, with a 3-year collective bargaining agreement (CBA) which was to expire on November 15, 1987. On October 8, 1987, the NEW ULO, composed mostly of workers belonging to the IGLESIA NI KRISTO sect, registered as a labor union. On October 12, 1987, the TUPAS staged a strike. ROBINA obtained an injunction against the strike, resulting in an agreement to return to work and for the parties to negotiate a new CBA. The next day, October 13, 1987, NEW ULO, claiming that it has "the majority of the daily wage rank and file employees numbering 191," filed a petition for a certification election at the Bureau of Labor Relations. TUPAS moved to dismiss the petition for being defective in form and that the members of the NEW ULO were mostly members of the Iglesia ni Kristo sect which three (3) years previous refused to affiliate with any labor union. It also accused the company of using the NEW ULO to defeat TUPAS' bargaining rights On November 17, 1987, the Med-Arbiter ordered the holding of a certification election within 20 days TUPAS appealed to the Bureau of Labor Relations BLR. In the meantime, it was able to negotiate a new 3-year CBA with ROBINA, which was signed on December 3, 1987 and to expire on November 15, 1990. MA, affirmed by BLR, ordered cert election I: whether or not Iglesia members can form a labor org which can be a legitimate labor organization Whether or not cert election barred by newly signed CBA between TUPAS and Robina Meat factory SC: the right of members of the IGLESIA NI KRISTO sect not to join a labor union for being contrary to their religious beliefs, does not bar the members of that sect from forming their own union The fact that TUPAS was able to negotiate a new CBA with ROBINA within the 60-day freedom period of the existing CBA, does not foreclose the right of the rival union, NEW ULO, to challenge TUPAS' claim to majority status, by filing a timely petition for certification election on October 13, 1987 before TUPAS' old CBA expired on November 15, 1987 and before it signed a new CBA with the company on December 3, 1987. a "certification election is the best forum in ascertaining the majority status of the contending unions wherein the workers themselves can freely choose their bargaining representative thru secret ballot. C. Government Corporation Employees – 244 Art. 244. Right of employees in the public service. Employees of government corporations established under the Corporation Code shall have the right to organize and to bargain collectively with their respective employers. All other employees in the civil service shall have the right to form associations for purposes not contrary to law. (As amended by Executive Order No. 111, December 24, 1986) 25 | P a g e

4.

D. Supervisors – 245; 212 (m) Art. 245. Ineligibility of managerial employees to join any labor organization; right of supervisory employees. Managerial employees are not eligible to join, assist or form any labor organization. Supervisory employees shall not be eligible for membership in a labor organization of the rank-and-file employees but may join, assist or form separate labor organizations of their own. (As amended by Section 18, Republic Act No. 6715, March 21, 1989) Art. 212. Definitions. (m) "Managerial employee" is one who is vested with the powers or prerogatives to lay down and execute management policies and/or to hire, transfer, suspend, lay-off, recall, discharge, assign or discipline employees. Supervisory employees are those who, in the interest of the employer, effectively recommend such managerial actions if the exercise of such authority is not merely routinary or clerical in nature but requires the use of independent judgment. All employees not falling within any of the above definitions are considered rank-and-file employees for purposes of this Book. 1. Test Paper Industries Corporation v. Laguesma, 330 SCRA 295 (00) FACTS: The company (PICOP) is engaged in manufacture of paper and timber products in Bislig Surigao with 9000 employees, 944 of whom are supervisory employees and out of which, 487 are members of PICOP-Bislig Supervisory and Technical Staff Employees Union (PBSTSEU). PBSTSEU instituted a Certification of Election to determine the sole and exclusive bargaining agent of supervisory and technical staff employees of PICOP for CBA purposes. Med-Arbiter set the holding of the elections with PBSTSEU as one of the choices for union. During the pre-election conference, PICOP questioned the inclusion of some section heads and supervisors in the list of voters whose positions, it claimed were reclassified as managerial employees under the Revised Organizational Structure of the PICOP and argued that as managerial employees, they were ineligible to form or join any labor organization. Med-Arbiter held that the supervisors and section heads are managerial employees and should be excluded. Upon appeal, Usec Laguesma set aside the Med-Arbiter order and held that the subject employees are supervisory employees and are eligible to vote. Laguesma denied appeal, thus the current petition arguing that the subject employees has the power to hire and fire and were designated as Section Managers and Unit Managers. Issue: WON the subject employees are managerial employees and as such are ineligible to form a union. Held: No. Subject employees are supervisory employees. As held in United Pepsi-Cola Supervisory Union v Laguesma, “managers” are divided into 3: top-level managers, middle managers, and first-line managers. Top and Middle Managers have the authority to devise, implement and control strategic and operational policies while the task of First-Line Managers is simply to ensure that such policies are carried out by the rank-and-file employees of an organization. Under this distinction, “managerial employees” therefore fall in two (2) categories, namely: 1.The “managers” per se composed of Top and Middle Managers, and; 2. the “supervisors” composed of First-Line Managers. The mere fact that an employee is designated as a “manager” does not make him one. Designations should be reconciled with the actual job description. A dissection of the job description of the subject employees show that they are not managerial as they do not company policies. Moreover, their alleged power to hire and fire was just recommendatory in character, subject to review of the higher executives. Samson v. NLRC, 330 SCRA 460 (00) FACTS: Rufino Samson is a District Sales Manager of Schering-Plough Corporation. In a letter sent by the corporation through one Mr. Estingor (HR Manager) calling his attention on his misconduct on two occasions, one happening during their informal Christmas party where he uttered obscene and malicious words (“bullshit yang si EDT”, where EDT is the president and gen manager) and made lewd gestures (the finger) in front of his co-workers, and the other happening on a subsequent and different date where he threatened to disrupt or otherwise create violence during the company’s forthcoming National Sales Conference (saying that its going to be bloody). In the same letter, Samson was given 2 days to explain why no disciplinary action should be given and in the meantime placing him under preventive suspension. Samson sent a reply denying the acts imputed, and apologizing for whatever words he uttered and saying that it was prompted by his feelings on the management decision of a certain Cua Lim case and not towards a specific person. At the same time, Samson filed a complaint for illegal suspension. On a subsequent date, he sent another letter to the HR Manager asking for reinstatement. Instead of reinstating to former position, Samson received a subsequent letter from the company advising him of the termination of his employment. Thus, Samson amended his complaint to illegal dismissal. Labor Arbiter held that termination was illegal as Samson’s conduct was not so serious as to warrant dismissal and that under company rules and regulations, the penalty for his offense is only verbal reminder. NLRC however reversed L. A. and held that there was just cause for dismissal, i.e. gross misconduct. It further held that Samson is a managerial employee, and as such, the employer is allowed wider latitude of discretion than in the case of rank-and-file employees. Issue: WON NLRC committed grave abuse of discretion in reversing the L.A. (vis. WON he was unjustly dismissed) Held: Yes. Misconduct is improper or wrong conduct; it must be of such serious and aggravated character and not merely trivial. In the current case, the misconduct of petitioner, when viewed in the context of the environment is not of such serious and grave character as to warrant dismissal. Considering the environment in which the event transpired (i.e. it was an informal party where liquors were being served), employees are most likely to loosen their tongue and express their grievance against their employer. Furthermore, the utterances were not directed to attack the person of the president as he was not in the present when the words were uttered. Further, being a first offense, the penalties sanctioned by the company rules is just verbal reminder, which he was already given when he was admonished in a meeting after the event. Neither is his dismissal justified on ground of loss of confidence as this ground is restricted to managerial employees, which he is not. His job description does not 26 | P a g e

contain the power to lay down policies not to hire, transfer, suspend, lay off, recall, discharge, assign, or discipline employees. Instead, his function includes only efficient planning, auditing and control, and “management functions” translated to directing the activities of Med Reps. Tagaytay Highlands v. Tagaytay Highlands, 395 SCRA 699 (03) FACTS: On Oct 1997, the TAGAYTAY HIGHLANDS EMPLOYEES UNION-PTGWO (THEU- PTGWO), a legitimate labor organization said to represent majority of the rankand-file employees of Tagaytay Highlands Gold Club (THIGCI) filed a petition for certification election before the DOLE Med-Arb unit. THIGCI opposed the petition on the ground that the list of union members submitted was flawed as it includes supervisors, resigned employees, AWOL employees and non-employees of THIGCI. DOLE Med-Arbiter ordered holding of a certification election on the ground that the union is a legitimate labor federation, and that the issues on the members should be properly raised in exclusion-inclusion proceedings in the pre-election conference, THIGCI appealed to the DOLE Secretary which set aside the Med-Arbiter decision on the ground of clear absence of community of mutuality of interests” as THEU sought to represent separate bargaining units (supervisory emp and rank-and-file emp). Upon MR by THEU, DOLE Usec, by authority of the Secretary set aside the resolution dismissing the petition for certification election and held that rather than disregarding the legitimate status of THEU, the names of alleged supervisory employees should be simply removed form the roster of membership. THUGCI filed an MR but was denied. CA likewise denied. Issue: WON the inclusion of supervisory employees in the rank-and-file union affects the legitimacy of the union Held: No. While Article 245 expressly prohibits supervisory employees from joining a rank-and-file union, it does not provide what would be the effect if a rank-and-file union counts supervisory employees as members. After a certificate of registration is issued to a union, its legal personality cannot be subject to collateral attack. It may be questioned only in an independent petition for cancellation in accordance with Section 5 of Rule V, Book IV of the "Rules to Implement the Labor Code" (Implementing Rules) which section reads: Sec. 5. Effect of registration. The labor organization or workers' association shall be deemed registered and vested with legal personality on the date of issuance of its certificate of registration. Such legal personality cannot thereafter be subject to collateral attack, but may be questioned only in an independent petition for cancellation in accordance with these Rules. The inclusion in a union of disqualified employees is not among the grounds for cancellation, unless such inclusion is due to misrepresentation, false statement or fraud under the circumstances enumerated in Sections (a) and (c) of Article 239 of above-quoted Article 239 of the Labor Code. As for the lack of mutuality of interest, it does not lie, given the company’s failure to present substantial evidence that the assailed employees are actually occupying supervisory positions. While petitioner submitted a list of its employees with their corresponding job titles and ranks, there is nothing mentioned about the supervisor’s respective duties, powers, and prerogatives that would show that they can effectively recommend managerial actions with the required use of the independent judgment. 2. Right Filoil Refinery Corporation v. Filoil Supervisory and Confidential Employees Association, 46 SCRA 512 (72) NATURE Appeal from the orders of the Court of Industrial Relations FACTS - Filoil Refinery Corporation executed a collective bargaining agreement with the Filoil Employees & Workers Association (FEWA), a labor association composed of the corporation's rank-and-file employees . This collective bargaining agreement expressly excluded from its coverage petitioner's supervisory and confidential employees, who in turn organized their own labor association, respondent herein. - The Corporation filed a motion to dismiss the petition for certification of the respondent association as the sole and exclusive collective bargaining agent of all petitioner’s supervisory and confidential employees working at its refinery in Rosario, Cavite. Their reason being, since they are part of the management, they do not have the right to bargain collectively although they may organize an organization of their own. ISSUE WON supervisors 'shall have the right to self-organization, and to form, join or assist labor organizations of their own choosing for the purpose of collective bargaining HELD YES. Reasoning As stated for the Court by the now Chief Justice in AG & P Co. of Manila, Inc. vs. C.I.R., 8 section 3 of the Industrial Peace Act "explicitly provides that 'employees' and this term includes supervisors 'shall have the right to self-organization, and to form, join or assist labor organizations of their own choosing for the purpose of collective bargaining through representations of their own choosing and to engage in concerted activities for the purpose of collective bargaining and other mutual aid or protection' and that 'individuals employed as supervisors . . . may form separate organizations of their own'. Indeed, it is well settled that 'in relation to his employer,' a foreman or supervisor 'is an employee within the meaning of the Act' . . . For this reason, supervisors are entitled to engage in union activities and any discrimination against them by reason thereof constitutes an unfair labor practice." Supervisors and confidential employees, even though they may exercise the prerogatives of management as regards the rank and file employees are indeed employees in relation to their employer, the company which is owned by the stockholders and bondholders (capital) and should therefore be entitled under the law to bargain collectively with the top management with respect to their terms and conditions of employment. \ DISPOSITION Petition DISMISSED. 27 | P a g e

Santa Rosa Coca-Cola Plant Employees Union v. Coca-Cola Bottlers Philippines, Inc, 312 SCRA 437 (07) FACTS: The Sta. Rosa Coca-Cola Plant Employees Union (Union) is the sole and exclusive bargaining representative of the regular daily paid workers and the monthly paid non-commission-earning employees of the Coca-Cola Bottlers Philippines, Inc. (Company) in its Sta. Rosa, Laguna plant. The individual petitioners are Union officers, directors, and shop stewards. On August 30, 1999, the Union, its officers, directors and six shop stewards filed a "Notice of Strike" with the National Conciliation and Mediation Board (NCMB) Regional Office in Southern Tagalog, Imus, Cavite. The petitioners relied on two grounds: (a) deadlock on CBA ground rules; and (b) unfair labor practice arising from the company’s refusal to bargain. The Union then participated in a picket, which was later on declared by the Labor Arbiter to be an illegal strike. According to the Labor Arbiter, the strike conducted by the Union was illegal since there was no showing that the Union conducted a strike vote, observed the prescribed cooling-off period, much less, submitted a strike vote to the DOLE within the required time. Consequently, for knowingly participating in the illegal strike, the individual petitioners (who are union officers) were considered to have lost their employment status. NLRC affirmed. Appeal to the CA was dismissed. ISSUE: 1. Whether the union staged a strike 2. WON the strike was legal 3. whether the individual officers and shop stewards of petitioner Union should be dismissed from their employment. HELD: 1. YES. That there was a labor dispute between the parties, in this case, is not an issue. Petitioners notified the respondent of their intention to stage a strike, and not merely to picket. Petitioners’ insistence to stage a strike is evident in the fact that an amended notice to strike was filed even as respondent moved to dismiss the first notice. The basic elements of a strike are present in this case: 106 members of petitioner Union, whose respective applications for leave of absence on September 21, 1999 were disapproved, opted not to report for work on said date, and gathered in front of the company premises to hold a mass protest action. Petitioners deliberately absented themselves and instead wore red ribbons, carried placards with slogans such as: "YES KAMI SA STRIKE," "PROTESTA KAMI," "SAHOD, KARAPATAN NG MANGGAGAWA IPAGLABAN," "CBA-‘WAG BABOYIN," "STOP UNION BUSTING." They marched to and fro in front of the company’s premises during working hours. Thus, petitioners engaged in a concerted activity which already affected the company’s operations. The mass concerted activity constituted a strike. 2. NO. In the present case, there is no evidence on record to show that respondents had complied with the above mandatory requirements of law for a valid strike. Particularly, there is no showing that respondents had observed the prescribed cooling-off period, conducted a strike vote, much less submitted a strike vote report to the Department of Labor within the required time. This being the case, respondents’ strike on September 21, 1999 is illegal. In the recent case of CCBPI Postmix Workers Union vs. NLRC, 2999 (sic) SCRA 410, the Supreme Court had said: "It bears stressing that the strike requirements under Article 264 and 265 of the Labor Code are mandatory requisites, without which, the strike will be considered illegal. The evidence (sic) intention of the law in requiring the strike notice and strike-vote report as mandatory requirements is to reasonably regulate the right to strike which is essential to the attainment of legitimate policy objectives embodied in the law. Verily, substantial compliance with a mandatory provision will not suffice. Strict adherence to the mandate of the law is required. 3.YES. Union officers are duty-bound to guide their members to respect the law. If instead of doing so, the officers urge the members to violate the law and defy the duly constituted authorities, their dismissal from the service is just penalty or sanction for their unlawful acts. The officers’ responsibility is greater than that of the members. As to the shop-stewards who contend that they are merely union members and not union officers, the SC said that they are union officers. It is quite clear that the jurisdiction of shop stewards and the supervisors includes the determination of the issues arising from the interpretation or even implementation of a provision of the CBA, or from any order or memorandum, circular or assignments issued by the appropriate authority in the establishment. 1awphi1.net In fine, they are part and parcel of the continuous process of grievance resolution designed to preserve and maintain peace among the employees and their employer. They occupy positions of trust and laden with awesome responsibilities. In this case, instead of playing the role of "peacemakers" and grievance solvers, the petitioners-shop stewards participated in the strike. Thus, like the officers and directors of petitioner Union who joined the strike, petitioners-shop stewards also deserve the penalty of dismissal from their employment. United Pepsi Cola v. Laguesma, 288 SCRA 15 (98) FACTS: Petitioner is a union of supervisory employees. It appears that on March 20, 1995 the union filed a petition for certification election on behalf of the route managers at Pepsi-Cola Products Philippines, Inc. However, its petition was denied by the med-arbiter and, on appeal, by the Secretary of Labor and Employment, on the ground that the route managers are managerial employees and, therefore, ineligible for union membership under the first sentence of Art. 245 of the Labor Code, which provides: Ineligibility of managerial employees to join any labor organization; right of supervisory employees. – Managerial employees are not eligible to join, assist or form any labor organization. Supervisory employees shall not be eligible for membership in a labor organization of the rank-and-file employees but may join, assist or form separate labor organizations of their own. Petitioner brought this suit challenging the validity of the order dated August 31, 1995, as reiterated in the order dated September 22, 1995, of the Secretary of Labor and Employment. Its petition was dismissed by the Third Division for lack of showing that respondent committed grave abuse of discretion. But petitioner filed a motion for reconsideration, pressing for resolution its contention that the first sentence of Art. 245 of the Labor Code, so far as it declares managerial employees to be ineligible to form, assist or join unions, contravenes Art. III § 8 of the Constitution which provides: 28 | P a g e

The right of the people, including those employed in the public and private sectors, to form unions, associations, or societies for the purposes not contrary to law shall not be abridged. ISSUES: 1) whether the route managers at Pepsi-Cola Products Philippines, Inc. are managerial employees and (2) whether Art. 245, insofar as it prohibits managerial employees from forming, joining or assisting labor unions, violates Art. III, § 8 of the Constitution. HELD: 1. YES. Earlier in this opinion, reference was made to the distinction between managers per se (top managers and middle managers) and supervisors (first-line managers). That distinction is evident in the work of the route managers which sets them apart from supervisors in general. Unlike supervisors who basically merely direct operating employees in line with set tasks assigned to them, route managers are responsible for the success of the company's main line of business through management of their respective sales teams. Such management necessarily involves the planning, direction, operation and evaluation of their individual teams and areas which the work of supervisors does not entail. The route managers cannot thus possibly be classified as mere supervisors because their work does not only involve, but goes far beyond, the simple direction or supervision of operating employees to accomplish objectives set by those above them. They are not mere functionaries with simple oversight functions but business administrators in their own right. An idea of the role of route managers as managers per se can be gotten from a memo sent by the director of metro sales operations of respondent company to one of the route managers. 2. NO. When read in relation to this definition in Art. 212(m), it will be seen that Art. 245 faithfully carries out the intent of the Constitutional Commission in framing Art. III, §8 of the fundamental law. Nor is the guarantee of organizational right in Art. III, §8 infringed by a ban against managerial employees forming a union. The right guaranteed in Art. III, §8 is subject to the condition that its exercise should be for purposes " not contrary to law." In the case of Art. 245, there is a rational basis for prohibiting managerial employees from forming or joining labor organizations. As Justice Davide, Jr., himself a constitutional commissioner, said " the rationale for this inhibition has been stated to be, because if these managerial employees would belong to or be affiliated with a Union, 1. The latter might not be assured of their loyalty to the Union in view of evident conflict of interests. 2. The Union can also become company-dominated with the presence of managerial employees in Union membership." In Bulletin Publishing Co., Inc. v. Hon. Augusto Sanchez, this Court elaborated on this rationale, thus: ". . . The rationale for this inhibition has been stated to be, because if these managerial employees would belong to or be affiliated with a Union, the latter might not be assured of their loyalty to the Union in view of evident conflict of interests. The Union can also become company-dominated with the presence of managerial employees in Union membership." NOTE: Art. 245 prohibits managerial employees to organize for purpose of collective bargaining. BUT it does not prevent them for organizing for other purpose such as for mutual aid and protection. The basis of this is the fiduciary and confidential relationship between manager and employer. E. Alien – 269 Art. 269. Prohibition against aliens; exceptions. All aliens, natural or juridical, as well as foreign organizations are strictly prohibited from engaging directly or indirectly in all forms of trade union activities without prejudice to normal contacts between Philippine labor unions and recognized international labor centers: Provided, however, That aliens working in the country with valid permits issued by the Department of Labor and Employment, may exercise the right to self-organization and join or assist labor organizations of their own choosing for purposes of collective bargaining: Provided, further, That said aliens are nationals of a country which grants the same or similar rights to Filipino workers. (As amended by Section 29, Republic Act No. 6715, March 21, 1989) F. Security Guards Manila Electric Co. v. Secretary of Labor, 197 SCRA 275 (91) Facts: MERALCO Employees and Workers Association (MEWA) is an existing union who represents the rank and file employees in Pay Grades 1-6 of MERALCO. It holds a valid CBA. Meanwhile, the Staff and Technical Employees Association of MERALCO (STEAM-PCWF), a labor organization, filed a petition (consolidated with a similar petition of FLAMES or First Line Association of MERALCO Supervisory Employees) for certification election , seeking to represent regular employees of MERALCO who are a) non-managerial employees with Pay Grades 7 and up; b) employees of the Patrol Division, Treasury Security Services Section who are automatically removed from the bargaining unit; and c) employees within the rank and file unit who are disqualified from becoming members of the same bargaining unit. MERALCO opposed said petition on the grounds that the employees represented a) are managerial employees; b) security services personnel who are prohibited from joining unions; c) did not give their consent; and d) already represented by MEWA. Med-Arbiter ruled IFO STEAM-PCWF; Labor Sec. Drilon affirmed with modification insofar as rank and file employees already represented in the existing CBA. With the passage of RA 6715 and its IRR, which disqualify supervisory employees and security guards from membership in a labor organization of the rank and file, STEAM-PCWF later withdrew its petition with respect to employees of the Patrol Division, Treasury Security Services Section and rank-and-file employees in Pay Grades 1-6. Issue: 1) WON the Resolution of the Labor Sec is valid, considering the passage of RA 6715. (focus on part about security employees) Held: Resolution affirmed with modification

29 | P a g e

1) Yes. The Resolution is based on Art 212 of the 1988 Labor Code which defines managerial and rank and file employees, the only groups of employees recognized at that time. This is why Drilon limited his classification of employees belonging to Pay Grades 7 and up to the said two groups. As to alleged failure to establish a demarcation line between managerial and supervisory, Art 212-m of RA 6715 itself provides a definition: managerial employees are those who lay down or execute managerial policies; while supervisory employees merely recommend them. As for security employees, the implementation rules insofar as it disqualifies security employees from joining labor organizations of the rank and file, is null and void, for not being germane to the purposes of EO 111 (annulled the provision from Art. 245 of old Labor Code, which was erroneously “carried over” in RA 6715’s IRR) and RA 6715. Now they may join either the union of the rank and file or of the supervisors, depending on their rank. (Rationale of previous exclusion: Potential conflict of interest, divided loyalty). MERALCO had admitted that employees within Pay Grades 7 and up are supervisory. The authority of the Labor Sec for the establishment of two labor organizations for the rank and file will have to be disregarded since SC upheld certification elections only for supervisory employees, with STEAM-PCWF and FLAMES as choices. Those employees already covered in the CBA shall remain in the existing bargaining unit. V. WORKERS WITH NO RIGHT OF SELF-ORGANIZATION A. Managerial and Confidential Employees – 212 (m); 82 Art. 212. Definitions. (m) "Managerial employee" is one who is vested with the powers or prerogatives to lay down and execute management policies and/or to hire, transfer, suspend, lay-off, recall, discharge, assign or discipline employees. Supervisory employees are those who, in the interest of the employer, effectively recommend such managerial actions if the exercise of such authority is not merely routinary or clerical in nature but requires the use of independent judgment. All employees not falling within any of the above definitions are considered rank-and-file employees for purposes of this Book. Art. 82. Coverage. The provisions of this Title shall apply to employees in all establishments and undertakings whether for profit or not, but not to government employees, managerial employees, field personnel, members of the family of the employer who are dependent on him for support, domestic helpers, persons in the personal service of another, and workers who are paid by results as determined by the Secretary of Labor in appropriate regulations. As used herein, "managerial employees" refer to those whose primary duty consists of the management of the establishment in which they are employed or of a department or subdivision thereof, and to other officers or members of the managerial staff. "Field personnel" shall refer to non-agricultural employees who regularly perform their duties away from the principal place of business or branch office of the employer and whose actual hours of work in the field cannot be determined with reasonable certainty. 1. Test Standard Chartered, etc. v. Standard Chartered Bank, 552 SCRA 284 (08) San Miguel Corporation Supervisors v. Laguesma, 277 SCRA 370 (97) Facts: Petitioner Union filed before DOLE a petition for certification election among supervisors and exempt employees of the SMC Magnolia Poultry Products Plants of Cabuyao, San Fernando and Otis. Labor USec issued an Order ordering the conduct of the said election, with the employees of the three plants considered as one bargaining unit. SMC appealed, assailing the grouping together of the three plants, and the inclusion of supervisory levels 3 and above whose positions are allegedly confidential. An Order was issued directing the conduct of separate certification elections of Supervisors ranked 1-4 in the three plants. Issue: 1) WON Supervisors 3 and 4 are considered confidential employees. 2) WON the employees of the three plants constitute an appropriate single bargaining unit. Held: 1) No. Confidential employees are those who a) assist or act in a confidential capacity; AND b) those who formulate, determine and effectuate management policies in the field of labor relations (i.e. he must have necessary access to labor relations information, and said access must not merely be incidental to his duties). The broad rationale behind their exclusion from bargaining units is that employees should not be placed in position involving potential conflict of interests. The criteria have not been met. While it is true that Supervisors ranked 3 and 4 answered in the affirmative when asked “Do you handle confidential data or documents?” in the Position Questionnaires submitted by the Union, the same questionnaire stated that the confidential information refer to products formula and standards, not to labor relations. 2) Yes. The fact that the employees belong to three different plants is immaterial. Geographical locations can be completely disregarded so long as their mutual and communal interests are not sacrificed. A bargaining unit is “a group of employees of a given employer, comprised of all or less than all of the entire body of employees, which the collective interest of all the employees consistent with equity to the employer, indicate to be best suited to serve the reciprocal rights and duties of the parties under the collective bargaining provisions of the law.” A unit must have substantial, mutual interests in wages, hours and other working conditions. If the three groups in this case were to be considered as separate bargaining entities, their bargaining leverage will be diminished. Any concerted activity of one will not have much impact on the operations of the employer. Sugbuanon Rural Bank Inc, v. Laguesma, 325 SCRA 425 (00) NATURE: Special civil action for certiorari and prohibition 30 | P a g e

FACTS: Petitioner Sugbuanon Rural Bank, Inc., (SRBI, for brevity) is a duly-registered banking institution with principal office in Cebu City and a branch in Mandaue City. Private respondent SRBI Association of Professional, Supervisory, Office, and Technical Employees Union (APSOTEU) is a legitimate labor organization affiliated with the Trade Unions Congress of the Philippines (TUCP). On October 8, 1993, the DOLE Regional Office in Cebu City granted Certificate of Registration to APSOTEU-TUCP, hereafter referred to as the union. On October 26, 1993, the union filed a petition for certification election of the supervisory employees of SRBI. It alleged, among others, that: (1) APSOTEUTUCP was a labor organization duly-registered with the Labor Department; (2) SRBI employed 5 or more supervisory employees; (3) a majority of these employees supported the petition: (4) there was no existing collective bargaining agreement (CBA) between any union and SRBI; and (5) no certification election had been held in SRBI during the past 12 months prior to the petition. On November 12, 1993, SRBI filed a motion to dismiss the union's petition. It sought to prevent the holding of a certification election on two grounds. First, that the members of APSOTEU-TUCP were in fact managerial or confidential employees. Second, the Association of Labor Unions-Trade Unions Congress of the Philippines or ALU-TUCP was representing the union. The union filed its opposition to the motion to dismiss on December 1, 1993. It argued that its members were not managerial employees but merely supervisory employees. On December 9, 1993, the Med-Arbiter denied petitioner's motion to dismiss. SRBI appealed the Med-Arbiter's decision to the Secretary of Labor and Employment. The appeal was denied for lack of merit. The certification election was ordered. On June 16, 1994, the Med-Arbiter scheduled the holding of the certification election for June 29, 1994. On June 17, 1994, SRBI filed with the Med-Arbiter an urgent motion to suspend proceedings. The Med-Arbiter denied the same. SRBI then filed a motion for reconsideration. Two days later, the Med-Arbiter cancelled the certification election scheduled for June 29, 1994 in order to address the motion for reconsideration. The Med-Arbiter later denied petitioner's motion for reconsideration SRBI appealed the order of denial to the DOLE Secretary Petitioner proceeded to file a petition with the DOLE Regional Office seeking the cancellation of the respondent union's registration. It averred that the APSOTEU-TUCP members were actually managerial employees who were prohibited by law from joining or organizing unions. DOLE Undersecretary denied SRBI's appeal for lack of merit. He ruled that APSOTEU-TUCP was a legitimate labor organization. It was fully entitled to all the rights and privileges granted by law to a legitimate labor organization, including the right to file a petition for certification election. He also held that until and unless a final order is issued cancelling APSOTEU-TUCP's registration certificate, it had the legal right to represent its members for collective bargaining purposes. SRBI moved for reconsideration of the Undersecretary's decision ISSUE/S: (1) WON the members of the respondent union are managerial employees and/or highly-placed confidential employees, hence prohibited by law from joining labor organizations and engaging in union activities (2) WON the Med-Arbiter may validly order the holding of a certification election HELD: 1. NO Reasoning - Article 212 (m) of the Labor Code defines the terms "managerial employee" and "supervisory employees" as follows: Art. 212. Definitions (m) "Managerial employee" is one who is vested with powers or prerogatives to lay down and execute management policies and/or hire, transfer, suspend, lay-off, recall, discharge, assign or discipline employees. Supervisory employees are those who, in the interest of the employer, effectively recommend such managerial actions if the exercise of such authority is not merely routinary or clerical in nature but requires the use of independent judgment. All employees not falling within any of the above definitions are considered rank-and-file employees for purposes of - Petitioner submitted detailed job descriptions to support its contention that the union members are managerial employees and/or confidential employees proscribed from engaging in labor activities. In the present case, however, petitioner failed to show that the employees in question were vested with managerial powers. At best they only had recommendatory powers subject to evaluation, review, and final decision by the bank's management. The job description forms submitted by petitioner clearly show that the union members in question may not transfer, suspend, lay-off, recall, discharge, assign, or discipline employees. Moreover, the forms also do not show that the Cashiers, Accountants, and Acting Chiefs of the Loans Department formulate and execute management policies which are normally expected of management officers. 2. YES Reasoning One of the rights of a legitimate labor organization under Article 242(b) of the Labor Code is the right to be certified as the exclusive representative of all employees in an appropriate bargaining unit for purposes of collective bargaining. Having complied with the requirements of Art. 234, it is our view that respondent union is a legitimate labor union. Article 257 of the Labor Code mandates that a certification election shall automatically be conducted by the MedArbiter upon the filing of a petition by a legitimate labor organization. 31 | P a g e

DISPOSITIVE: Petition dismissed. Samson v. NLRC, 330 SCRA 460 (00) NATURE Special civil action of certiorari FACTS - Rufino Norberto F. Samson was dismissed from the Company for uttering what was considered by the company as obscene, insulting, and offensive words and for making malicious and lewd gestures directed at the President and General Manager of the company during an informal Sales and Marketing gathering in relation to the decision of the Management Committee on a dispute with another employee. He was also accused of threatening to disrupt or create violence in a forthcoming National Sales Conference. - The Labor Arbiter found that Samson was illegally dismissed but the decision was reversed by NLRC. ISSUE/S WON the dismissal on the ground of loss of confidence is valid HELD No. As a ground for dismissal, the term “trust and confidence” is restricted to managerial employees. And before one may be properly considered a managerial employee, three conditions must be met: a. Their primary duty consists of the management of the establishment in which they are employed or of a department or subdivision thereof: b. They customarily and regularly direct the work of two or more employees therein; c. They have the authority to hire or fire other employees of lower rank; or their suggestions and recommendations as to hiring an firing and as to the promotion or any other change of status of other employees are given particular weight. -In this case the job description of Samson does not mention that petitioner possesses the power “to lay down policies nor to hire, transfer, suspend, lay off, recall. discharge, assign or discipline employees”. Absent this crucial element, petitioner cannot be considered a managerial employee despite his designation as District Sales Manager. Disposition Petition is granted. NLRC decision is reversed and set aside. Samson is reinstated to his position without loss of seniority rights and is awarded payment of his full backwages. Paper Industries Corporation of the Philippines v. Laguesma, 330 SCRA 295 (00) Petitioner Paper Industries Corporation of the Philippines (PICOP) has over 9,000 employees, 944 of whom are supervisory and technical staff employees. PBSTSEU instituted a Petition for Certification Election to determine the sole and exclusive bargaining agent of the supervisory and technical staff employees of PICOP for collective bargaining agreement (CBA) purposes. Meanwhile, private respondents Federation of Free Workers (FFW) and Associated Labor Union (ALU) filed their respective petitions for intervention. Med Arbiter granted the petitions for interventions of the FFW and ALU. It held that the supervisors and section heads are managerial employees and therefore excluded from the list of voters for purposes of certification election. DOLE Undersecretary Laguesma reversed Med Arbiter; hence this appeal by PICOP. PICOP questioned and objected to the inclusion of some section heads and supervisors in the list of voters whose positions it averred were reclassified as managerial employees in the light of the reorganization effected by it. Under the Revised Organizational Structure of the PICOP, the company was divided into 4) main business groups, namely: Paper Products Business, Timber Products Business, Forest Resource Business and Support Services Business. A vice- president or assistant vicepresident heads each of these business groups. A division manager heads the divisions comprising each business group. A department manager heads the departments comprising each division. Section heads and supervisors, now called section managers and unit managers , head the sections and independent units, respectively, comprising each department . PICOP advanced the view that considering the alleged present authority of these section managers and unit managers to hire and fire, they are classified as managerial employees, and hence, ineligible to form or join any labor organization. Issue: WON section managers and unit managers are considered managerial employees; hence, ineligible to form or join any labor organization. SC: No. A thorough dissection of the job description of the concerned supervisory employees and section heads indisputably show that they are not actually managerial but only supervisory employees since they do not lay down company policies. PICOP's contention that the subject section heads and unit managers exercise the authority to hire and fire is ambiguous and quite misleading for the reason that any authority they exercise is not supreme but merely advisory in character. Theirs is not a final determination of the company policies inasmuch as any action taken by them on matters relative to hiring, promotion, transfer, suspension and termination of employees is still subject to confirmation and approval by their respective superior. Thus, where such power, which is in effect recommendatory in character, is subject to evaluation, review and final action by the department heads and other higher executives of the company, the same, although present, is not effective and not an exercise of independent judgment as required by law. The fact that PICOP voiced out its objection to the holding of certification election, despite numerous opportunities to ventilate the same, only after respondent Undersecretary of Labor affirmed the holding thereof, simply bolstered the conclusion that PICOP raised the issue merely to prevent and thwart the concerned section heads and supervisory employees from exercising a right granted them by law. Needless to stress, no obstacle must be placed to the holding of certification elections, for it is a statutory policy that should not be circumvented. Cainta v. Cainta, 489 SCRA 468 (06) 32 | P a g e

Facts: CBA between the school and the Union provided stipulation in a CBA that allows management to retire an employee in its employ for a predetermined lengthy period but who has not yet reached the minimum compulsory retirement age Union held an election of officers, with Mrs. Llagas, Dean, being elected as President; Ms. Javier, subject coordinator, as VP. The School retired Llagas and Javier, who had rendered more than 20 years of continuous service, pursuant to the CBA. Union filed a notice of strike and picketed the School’s entrances. Union filed a complaint for unfair labor practice before the NLRC. The NLRC ruled that the retirement of Llagas and Javier is legal as the School was merely exercising an option given to it under the CBA. The NLRC dismissed the unfair labor practice charge against the School for insufficiency of evidence. Furthermore, it was found that the strike declared by the Union is illegal, thereby declaring all union officers to have lost their employment status. CA annulled and set aside the resolutions of the NLRC; hence, this appeal. Issues: (1) WON the retirement of Llagas and Javier is legal; (2) WON the School is guilty of unfair labor practice; and (3) WON the strike is legal. SC: ifo School. Affirmed CA. (1) Retirement is legal, arising as it did from a management prerogative granted by the mutually-negotiated CBA between the School and the Union. The retirement of Rosalinda Llagas has become inevitable because , being a managerial employee by reason of her position as Dean of Student Affairs, she accepted the Union presidency. She lost the trust and confidence on her by the SCHOOL as she occupied a managerial position as Dean Being also the union president, she has allowed her loyalties to be divided between the administration and the union. As to Paz Javier, her retirement was decided upon after an evaluation shows that she was not performing well as her students were complaining about her brusque attitude and bad language, aside from being habitually absent and late . By their acceptance of the CBA, the Union and its members are obliged to abide by the commitments and limitations they had agreed to cede to management. The questioned retirement provisions cannot be deemed as an imposition foisted on the Union, which very well had the right to have refused to agree to allowing management to retire employees with at least 20 years of service. (2) The law and this Court frowns upon unfair labor practices by management, including so-called union-busting but the exercise by the employer of a valid and duly established prerogative to retire an employee does not constitute unfair labor practice. (3) The functions of the Dean of Student Affairs, as occupied by Llagas, are enumerated in the Faculty Manual, are managerial in nature, thereby classifying Llagas as a managerial employee. Javier was occupying the position of Subject Area Coordinator. Her duties and responsibilities include various recommendations to the principal’s consideration the appointment of faculty members in the department, their promotion, discipline and even termination. She was therefore a supervisory employee. Both are therefore proscribed from joining a labor union, more so being elected as union officer. In the case of Javier, a supervisory employee, she may join a labor union composed only of supervisory employees. Finding both union officers to be employees not belonging to the rank-and-file, their membership in the Union has become questionable, rendering the Union inutile to represent their cause; hence, strike is considered illegal. 2. Prohibition and Rationale Metrolab Industries, Inc. v. Roldan-Confessor, 254 SCRA 182 (96) Facts: Metro Drug Corporation Employees Association-Federation of Free Workers (Union) is a labor organization representing the rank and file employees of petitioner Metrolab Industries, Inc. (Metro Drug, Inc.). CBA expired. The negotiations for a new CBA, however, ended in a deadlock. Union filed a notice of strike against Metrolab and Metro Drug Inc. SOLE Torres issued an order resolving all the disputed items in the CBA and ordered the parties involved to execute a new CBA. On the basis of its management prerogative, Metrolab laid off 94 of its rank and file employees, including executive secretaries. Acting Labor Secretary Nieves Confesor issued a resolution declaring the layoff of Metrolab's 94 rank and file workers illegal and ordered their reinstatement. Contention of Metrolab: The executive secretaries of the President, Executive Vice-President, Vice-President, Vice-President for Sales, Personnel manager, and Director for Corporate Planning who may have access to vital labor relations information or who may otherwise act in a confidential capacity to persons who determine or formulate management policies. This being the case, they could not be made members of a labor organization. Issue: Can the executive secretaries be given the benefit of whatever the Union avails by way of its petitions. Stated differently, can they be members of a labor organization? SC: No, as they are considered confidential employees. Although Article 245 of the Labor Code limits the ineligibility to join, form and assist any labor organization to managerial employees, jurisprudence has extended this prohibition to confidential employees or those who by reason of their positions or nature of work are required to assist or act in a fiduciary manner to managerial employees and hence, are likewise privy to sensitive and highly confidential records. By the very nature of their functions, they assist and act in a confidential capacity to, or have access to confidential matters of, persons who exercise managerial functions in the field of labor relations. As such, the rationale behind the ineligibility of managerial employees to form, assist or join a labor union equally applies to them.

33 | P a g e

Moreover, unionization of confidential employees for the purpose of collective bargaining would mean the extension of the law to persons or individuals who are supposed to act "in the interest of the employers. It is not farfetched that in the course of collective bargaining, they might jeopardize that interest which they are dutybound to protect. Finally, confidential employees cannot be classified as rank and file. The nature of employment of confidential employees is quite distinct from the rank and file, thus, warranting a separate category. Excluding confidential employees from the rank and file bargaining unit, therefore, is not tantamount to discrimination. Pepsi Cola Products, v. Secretary of Labor, 312 SCRA 104 (99) Facts: Consolidation of 2 cases. 1. The union Pepsi-Cola Employees Organization-UOEF filed a petition for certification election. Med-Arbiter granted the petition, explicitly stating that it was an affiliate of the federation Union de Obreros Estivadores de Filipinas, together with the rank and file unions of Pepsi employees. Pepsi filed with the Bureau of Labor Relations a petition to cancel and/or revoke the charter affiliation of the union, saying that the union's members were managers and that a supervisors' union can not affiliate with a federation whose members include the rank and file union of the same company. 2. The Med-Arbiter issued an order to conduct a certification election to be participated by and among the supervisory workers of Pepsi. Pepsi appealed the order. The Labor Secretary modified by referring the cases to the Regional Director, but the call for certification election was sustained. The Bureau of Labor Relations later issued a registration certificate in favor of the union. Issues: 1. WON a supervisors' union can affiliate with the same federation of which two rank and file unions are also members, 2. WON the petition to cancel/revoke the union's registration is a prejudicial question to the petition for certification election, 3. WON confidential employees can join labor unions of rank and file employees. Held: 1. In view of the supervisors' union's withdrawal from the federation, the issue has become moot and academic, and was dismissed. But to guide the parties and others similarly situated, the Court resolved it. Article 245 only states that managerial employees are not eligible to join, assist or form any labor organization, and that supervisory employees are not eligible to join rank and file unions but may join, assist or form their own organizations. While the wording of the law does not cover federations of unions of rank and file employees, the intent of the law must be upheld, i.e. to avoid a situation where supervisors would co-mingle with those employees whom they directly supervise and thus represent conflicting interests, especially if the federation takes active part in union activities in the same company. In a battery of cases the Court has said that the prohibition extends to a supervisors' union applying for membership in a federation which has rank and file unions for its members. 2. No, it is not a prejudicial question. Certification proceedings are investigations which are non-adversarial and fact-finding in character. Thus, technical rules of evidence do not apply. A union still has the legal personality to petition for certification election absent an order directing the cancellation. 3. No, they cannot. Under the doctrine of necessary implication, Article 245 disqualifies confidential employees from becoming members of rank and file unions, even though the provision singles out managerial employees.The reasons for such disqualification of managerial employees are 1) the union will not be so sure of the managers' loyalty because managers are supposed to be on the side of the employer, and 2) the union may become company-dominated, making collective bargaining a one-sided affair. The same reasons hold for confidential employees, especially in view of the fact that they have access to the company's confidential matters. B. Worker/Member of Cooperative Benguet Electric Cooperative, Inc. v. Calleja, 180 SCRA 740 (89) Facts: BWLU-ADLO filed a petition for direct certification as the sole and exclusive bargaining representative of Benguet Electric Cooperative (BENECO). BELU opposed, saying that it was the sole and exclusive bargaining representative of BENECO's workers, and in fact had filed two cases against BENECO for bargaining deadlock and ULP. BENECO on the other hand filed a motion to dismiss the petition claiming that it is a non-profit electric cooperative, and the employees sought to be represented by BWLU-ADLO are not eligible to form, join or assist labor unions because they are members and joint owners of the cooperative. The Med-Arbiter gave due course to the petition for certification election, but limiting it to 37 employees who are not members and without any involvement in the actual ownership of the cooperative. BELU and BENECO appealed, but their appeals were dismissed. The election was held with BELU emerging as the winner (49 out of 83 votes cast). BENECO through counsel protested, submitting a certification that only 4 employees were not BENECO members and were the only ones eligible to vote. The Med-arbiter dismissed the protest. BLU director Ferrer-Calleja affirmed. BENECO filed a petition for certiorari with the SC. Issue: WON the BLU director committed grave abuse of discretion in certifying BELU as the bargaining representative of BENECO's rank and file employees. Held: Yes. The right to collective bargaining and self-organization is not available to an employee who is also a member and co-owner of the same cooperative. The fact that the members-employees do not participate in the actual management does not make them eligible to join, form or assist labor organizations. It is the fact of ownership of the cooperative and not involvement in its management which disqualifies them. BELU contends that "there would be no hindrance for employers to grant on a scheme of generous profit sharing stock bonuses to their employees and thereafter claim that since their employees are not stockholders, they are now also co-owners and thus disqualified to form unions". The SC said that membership in cooperatives is different from being a stockholder of a corporation. The owners and members of a cooperative are the ones who run and operate the business. Thus, an employee who is also a member and co-owner of the same cooperative cannot invoke the right to collective bargaining because he cannot bargain with himself or his co-owners. Even if there were 37 employees who were originally non-members of the cooperative but were forced to join it, and therefore must be allowed to vote, the election is 34 | P a g e

still void because those who were already members of the cooperative at the time of the issuance of the Med-arbiter's order were allowed to vote. The director thus committed grave abuse of discretion in affirming the med-arbiter's order. Central Negros Electric Cooperative, Inc, v. Secretary of DOLE, 201 SCRA 584 (91) NATURE Special civil action for certiorari FACTS - Petitioner Central Negros Electric Cooperative, Inc. (CENECO) seeks to annul the order issued by then Acting Secretary of Labor Laguesma declaring the projected certification election unnecessary and directing petitioner CENECO to continue recognizing private respondent CENECO Union of Rational Employees (CURE) as the sole and exclusive bargaining representative of all the rank-and-file employees of petitioner's electric cooperative. - Previous events: Their CBA was valid for a term of 3 years; CURE then wrote CENECO proposing that negotiations be conducted for a new agreement. CENECO denied CURE's request on the ground that, under applicable decisions of the Supreme Court, employees who at the same time are members of an electric cooperative are not entitled to form or join a union. - CURE filed a petition for direct recognition or for certification election; CENECO filed a motion to dismiss on the ground of legal restraints. - Some employees of CENECO then filed for withdrawal of membership in the cooperative but CENECO contended that this cannot be allowed. ISSUE/S 1. WON the employees were allowed to withdraw membership from the cooperative so as to entitle them to form or join CURE for purposes of the negotiations for a collective bargaining agreement HELD 1. YES Ratio Membership in the cooperative is on a voluntary basis. Hence, withdrawal therefrom cannot be restricted unnecessarily. The right to join an organization necessarily includes the equivalent right not to join the same . No Right of Self-Organization: WORKER/MEMBER OF COOPERATIVE - It was held in Batangas I Electric Cooperative Labor Union vs. Romeo A. Young that "employees who at the same time are members of an electric cooperative are not entitled to form or join unions for purposes of collective bargaining agreement, for certainly an owner cannot bargain with himself or his co-owners." - However, nowhere in said case is it stated that member-employees are prohibited from withdrawing their membership in the cooperative in order to join a labor union. Disposition The questioned order is hereby ANNULLED and SET ASIDE. The med-arbiter is hereby ordered to conduct a certification election among the rank-and- file employees of CENECO with CURE and No Union as the choices therein. Republic etc. v. Asia Pro Cooperative, 538 SCRA 659 (07) PARTIES: RP through the Social Security Commission, Social Security System (RA 1161 as amended by 8282) (p) v. Asiapro Cooperative (RA 6938) (r) FACTS: Asiapro is a cooperative composed of 2 types of owners-members: 1. Regular members – entitled to all the rights and privileges of membership 2. Associate members – who has no right to vote and be voted upon and entitled only to the rights and privileges provided in its by-laws. Its objectives are to provide savings and credit facilities and to develop other livelihood services for its members. It entered into several Service Contracts with Stanfilco, a division of DOLE Phils, based in Bukidnon. The members earn their income from the service surplus generated by the quality and amount of services they rendered which is determined by the BOD of the cooperative. The members who were assigned to Stanfilco requested the services of Asiapro to register them with SSS as self-employed and to remit their contributions. They pay the same premium contribution as that paid by both an employer and employee. SSS sent Asiapro a letter ordering it to register as an employer of the members because it is actually a manpower contractor supplying employees to Stanfilco. Asiapro declined asserting that the owners-members are the cooperative itself hence, it cannot be its own employer. SSS filed a petition with the SSC against Asiapro and Stanfilco praying that Asiapro or, in the alternative, Stanfilco, be directed to register as an employer and to report the members as employees under compulsory coverage of SSS and to remit the necessary contributions in accordance with the SSS Law of 1997. Asiapro reasoned that there was no employer-employee relationship between itself and the members and since only the NLRC can determine the existence of such relationship, the SSC therefore has no jurisdiction. SSC granted the petition of the SSS. Upon appeal to the CA, it also reasoned that it is a multipurpose cooperative created under RA 6938 and composed of owners-members whose rights and obligations are derived from their Membership Agreement, the Cooperative By-Laws, and RA 6938 and not from any contract of employment or from the Labor Laws. CA granted Asiapro’s petition. ISSUE: 1. (ISSUE RELATED TO THE CASE) WON the existence of an employer-employee relationship between the cooperative and its owners-members entitle them to the right to collective bargaining. 2. WON the SSC has jurisdiction over the issue 3. WON there is an employer-employee relationship between the cooperative and its owners-members. 1. As to the right of an owner-member of a cooperative to collective bargaining, the SC cited its ruling in Cooperative Rural Bank of Davao City, Inc. v. Ferrer-Calleja that a cooperative is by nature, different from an ordinary business concern, being run either by persons, partnerships, or corporations and tts owners and/or members are the ones who run and operate the business while the others are its employees. An employee therefore of such a cooperative who is a member and co-owner of it cannot invoke the right to collective bargaining for certainly an owner cannot bargain with himself or his co-owners. The employees of cooperatives who are themselves members of the cooperative have no right to form or join labor organizations for purposes of collective bargaining for being themselves co-owners of the cooperative. However, as to cooperatives whose employees are not members or co-owners, such employees are entitled to exercise the rights of all workers to organization, 35 | P a g e

collective bargaining, negotiations, and others enshrined in the Constitution and existing laws of the country. Thus, even if there is the existence of an employeremployee relationship between a cooperative and its owners-members, they do not have the right to collective bargaining. 2. YES, SSC has jurisdiction. Sec. 5 of RA 8282 and Sec 1, Rule 3, of the Revised Rules of Procedure of the SSC is clear that any issue regarding the compulsory coverage of the SSS is within the exclusive domain of the SSC although mandatory coverage under the SSS Law is premised on the existence of an employer-employee relationship, except in self-employed cases. Jurisdiction over an issue is dependent upon the allegations in a complaint and once jurisdiction is acquired, it remains until the termination of the case. Since, the petition filed by the SSS in the SSC alleges that the members are subject to compulsory coverage because they are employees of the cooperative and thus, it has to register as their employer and report the members as covered members of the SSS, then it is clear that this case clearly falls within the SSC’s jurisdiction. Challenging the jurisdiction of the SSC on an alleged lack of employer-employee relationship in a Motion to Dismiss is not enough to deprive the SSC of its jurisdiction. But the SSC still has to determine WON there is an Er-Ee relationship since the compulsory coverage of the SSS Law is predicated on the existence of such relationship and it has jurisdiction to do so because the NLRC is not the only governmental body that has exclusive jurisdiction since Social Security claims are exceptions to the NLRC jurisdiction and these claims are undeniably rooted in the coverage by the system. Thus, the question of the existence of an Er-Ee relationship for the purpose of determining the coverage of the SSS is explicitly excluded from the jurisdiction of the NLRC and falls within the jurisdiction of the SSC which is primarily charged with the duty of settling disputes arising under the SS Law of 1997. 3. As to WON there exists an Er-Ee relationship between the cooperative and its owners-members, the SC held that there is an Er-Ee relationship because the 4 elements are present. 1. Based on the Service Contracts, Asiapro has the exclusive discretion in the selection and engagement of the owners-members as well as the team members who will be assigned at Stanfilco. 2. The weekly stipends or shares in the service surplus given by Asiapro were in reality wages since they were equivalent to the prevailing rates of wages and were given as compensation in rendering services to Asiapro’s client, Stanfilco. 3. The Service Contracts expressly state that Asiapro has the power to investigate, discipline, and remove the owners-members and the team leaders rendering services to Stanfilco. 4. Asiapro has control over the means and manner of performing the services under the Service Contracts as well as the means and methods of work and is solely and entirely responsible for its owner-members, team leaders, and other representatives at Stanfilco. All these prove that there is indeed an Er-Ee relationship. The express provision stated in the Service Contracts that there is no Er-Ee relationship must be struck down as being contrary to law and public policy since it is being used by the cooperative to circumvent the compulsory coverage of its employees. C. Non-Employees – 243 Art. 243. Coverage and employees’ right to self-organization. All persons employed in commercial, industrial and agricultural enterprises and in religious, charitable, medical, or educational institutions, whether operating for profit or not, shall have the right to self-organization and to form, join, or assist labor organizations of their own choosing for purposes of collective bargaining. Ambulant, intermittent and itinerant workers, self-employed people, rural workers and those without any definite employers may form labor organizations for their mutual aid and protection. (As amended by Batas Pambansa Bilang 70, May 1, 1980) Republic Planters Bank v. Laguesma, 264 SCRA 637 (96) NATURE Petition for certiorari FACTS -on January 21, 1991, Republic Planters Bank General Services Employees Union-National Association of Trade Unions (petitioner) filed a petition for certification election to determine the sole and exclusive bargaining representative of all regular employees outside the bargaining unit of Republic Planters Bank. -The proposed bargaining unit is composed of clerks, messengers, janitors, plumbers, telex operators, mailing and printing personnel, drivers, mechanics and computer personnel. Allegedly, these employees are regular employees but are considered as contractual employees by the bank and are excluded from the existing collective bargaining agreement. -The bank filed moved to dismiss the petition for certification election on the contentions that they are employed on contractual basis, that there was already an existing bargaining unit, and that the petition failed to state the number of employees in the proposed bargaining unit. - The petition was dismissed by the Med-Arbiter. On Dec 21, 1992, Undersecretary Bienvenido Laguesma reversed the Order of the Med-arbiter. Petitioner filed a Motion for Reconsideration. Usec Laguesma modified the December 21, 1992 Resolution, finding 6 employees as regular and included in the existing rank and file unit. - Both parties moved for reconsideration. Petitioner sought a ruling that the other workers in the proposed bargaining unit should also be considered regular employees. On Feb 24, 1995, Usec Laguesma issued another Order reinstating the Resolution dated December 21, 1992. ISSUE WON petitioners have the right to self-organization, and thus be allowed to file a petition for certification election. HELD NO. - In the case of Singer Sewing Machine Company vs . Drilon, et al., it was rules that if the union members are not employees, no right to organize for purposes of bargaining, nor to be certified as bargaining agent can be recognized. Since the persons involved are not employees of the company, they are not entitled to the constitutional right to join or form a labor organization for purposes of collective bargaining. Disposition Petition is dismissed VI. PARTY PROTECTED Mactan Workers Union v. Aboitiz, 45 SCRA 577 (72) 36 | P a g e

NATURE Appeal from CFI Cebu by intervenor Associated Labor Union FACTS - The employees and laborers of Cebu Shipyard and Eng’g Works belong to two rival unions: the Mactan Workers Union and Associated Labor Union. - Associated Labor Union (ALU), as exclusive bargaining representative of the workers, entered into a CBA with Cebu Shipyard where they agreed on a profit-sharing bonus scheme. It was agreed that Cebu Shipyard will release the money to ALU then ALU will deliver it to the members. Any unclaimed money will be returned to the company after 60 days. -The 72 members of Mactan Workers Union failed to get their money (bec they did not like to go to ALU’s office). Their money was returned to the company and ALU advised the company not to deliver the amount to the members of the Mactan Workers Union unless ordered by the Court, otherwise the ALU will take steps to protect the interest of its members. -So Cebu Shipyard did not pay to the plaintiffs, but instead, deposited the said amount with the Labor Administrator. For the recovery of their money, Mactan Workers filed money claims against Cebu Shipyard. -The lower court declared the Cebu Shipyard to deliver the money to ALU, and for ALU to distribute it immediately to Mactan Workers members. - ALU appealed, alleging lack of a cause of action, of jurisdiction of the City Court of Lapulapu and of personality of the Mactan Workers Union to represent its members. ISSUES WON CFI Cebu was correct in ordering ALU to deliver Mactan Workers Union’s share HELD YES -The lower court just required literal compliance with the terms of a collective bargaining contract -The terms and conditions of a collective bargaining contract constitute the law between the parties. Those who are entitled to its benefits can invoke its provisions. In the event that an obligation therein imposed is not fulfilled, the aggrieved party has the right to go to court for redress. - The benefits of a collective bargaining agreement extend to the laborers and employees in the collective bargaining unit, including those who do not belong to the chosen bargaining labor organization. … The labor union that gets the majority vote as the exclusive bargaining representative does not act for its members alone. It represents all the employees in such a bargaining unit. - The raison d’etre of labor unions: it is not to be forgotten that what is entitled to constitutional protection is labor, or more specifically the working men and women, not labor organizations. The latter are merely the instrumentalities through which their welfare may be promoted and fostered. That is the raison d'etre of labor unions. Disposition CFI Decision affirmed 2.06 NON-ABRIDGMENT OF RIGHT ART. 246. Non-abridgment of right to self-organization. - It shall be unlawful for any person to restrain, coerce, discriminate against or unduly interfere with employees and workers in their exercise of the right to self-organization. Such right shall include the right to form, join, or assist labor organizations for the purpose of collective bargaining through representatives of their own choosing and to engage in lawful concerted activities for the same purpose for their mutual aid and protection, subject to the provisions of Article 264 of this Code. (As amended by Batas Pambansa Bilang 70, May 1, 1980). ART. 248. Unfair labor practices of employers. - It shall be unlawful for an employer to commit any of the following unfair labor practice: (a) To interfere with, restrain or coerce employees in the exercise of their right to self-organization; (b) To require as a condition of employment that a person or an employee shall not join a labor organization or shall withdraw from one to which he belongs; (c) To contract out services or functions being performed by union members when such will interfere with, restrain or coerce employees in the exercise of their rights to self-organization; (d) To initiate, dominate, assist or otherwise interfere with the formation or administration of any labor organization, including the giving of financial or other support to it or its organizers or supporters; (e) To discriminate in regard to wages, hours of work and other terms and conditions of employment in order to encourage or discourage membership in any labor organization. Nothing in this Code or in any other law shall stop the parties from requiring membership in a recognized collective bargaining agent as a condition for employment, except those employees who are already members of another union at the time of the signing of the collective bargaining agreement. Employees of an appropriate bargaining unit who are not members of the recognized collective bargaining agent may be assessed a reasonable fee equivalent to the dues and other fees paid by members of the recognized collective bargaining agent, if such non-union members accept the benefits under the collective bargaining agreement: Provided, that the individual authorization required under Article 242, paragraph (o) of this Code shall not apply to the non-members of the recognized collective bargaining agent; (f) To dismiss, discharge or otherwise prejudice or discriminate against an employee for having given or being about to give testimony under this Code; (g) To violate the duty to bargain collectively as prescribed by this Code; (h) To pay negotiation or attorney’s fees to the union or its officers or agents as part of the settlement of any issue in collective bargaining or any other dispute; or (i) To violate a collective bargaining agreement. The provisions of the preceding paragraph notwithstanding, only the officers and agents of corporations, associations or partnerships who have actually participated in, authorized or ratified unfair labor practices shall be held criminally liable. (As amended by Batas Pambansa Bilang 130, August 21, 1981). ART. 249. Unfair labor practices of labor organizations. - It shall be unfair labor practice for a labor organization, its officers, agents or representatives: (a) To restrain or coerce employees in the exercise of their right to self-organization. However, a labor organization shall have the right to prescribe its own rules with respect to the acquisition or retention of membership; 37 | P a g e

(b) To cause or attempt to cause an employer to discriminate against an employee, including discrimination against an employee with respect to whom membership in such organization has been denied or to terminate an employee on any ground other than the usual terms and conditions under which membership or continuation of membership is made available to other members; (c) To violate the duty, or refuse to bargain collectively with the employer, provided it is the representative of the employees; (d) To cause or attempt to cause an employer to pay or deliver or agree to pay or deliver any money or other things of value, in the nature of an exaction, for services which are not performed or not to be performed, including the demand for fee for union negotiations; (e) To ask for or accept negotiation or attorney’s fees from employers as part of the settlement of any issue in collective bargaining or any other dispute; or (f) To violate a collective bargaining agreement. The provisions of the preceding paragraph notwithstanding, only the officers, members of governing boards, representatives or agents or members of labor associations or organizations who have actually participated in, authorized or ratified unfair labor practices shall be held criminally liable. (As amended by Batas Pambansa Bilang 130, August 21, 1981). VII. NON-ABRIDGMENT OF RIGHT – 246; 248 (a); 249 (a) Art. 246. Non-abridgment of right to self-organization. It shall be unlawful for any person to restrain, coerce, discriminate against or unduly interfere with employees and workers in their exercise of the right to self-organization. Such right shall include the right to form, join, or assist labor organizations for the purpose of collective bargaining through representatives of their own choosing and to engage in lawful concerted activities for the same purpose for their mutual aid and protection, subject to the provisions of Article 264 of this Code. (As amended by Batas Pambansa Bilang 70, May 1, 1980) Art. 248. Unfair labor practices of employers. It shall be unlawful for an employer to commit any of the following unfair labor practice: a. To interfere with, restrain or coerce employees in the exercise of their right to self-organization; Art. 249. Unfair labor practices of labor organizations. It shall be unfair labor practice for a labor organization, its officers, agents or representatives: a. To restrain or coerce employees in the exercise of their right to self-organization. However, a labor organization shall have the right to prescribe its own rules with respect to the acquisition or retention of membership; 7. Union Information – 241 (p) Art. 241. Rights and conditions of membership in a labor organization. The following are the rights and conditions of membership in a labor organization: p. It shall be the duty of any labor organization and its officers to inform its members on the provisions of its constitution and by-laws, collective bargaining agreement, the prevailing labor relations system and all their rights and obligations under existing labor laws. Union Officer – Obligation Continental Cement Corporation Labor Union v. Continental Cement, 189 SCRA 134 (90) The NLRC issued an arbitration award resolving certain demands of the Union respecting the working terms and conditions that should be observed in the establishment of respondent company. However, due to disagreement on the interpretation of the provisions of the award, compliance therewith was delayed. To compel the immediate implementation of the award, the union staged a strike which was later lifted after the company agreed to pay the employee’s vacation and sick leaves in 3 installments. The company sought clarification from the LA on whether a group of 91 workers who were unable to complete 300 days of work within a 12-month period was entitled to the payment of the said leaves. The LA ruled that the award required the company to make the payment in favor of the workers. The company negotiated with petitioners for staggered payment due to its financial difficulties and planned shutdown. The Union agreed to instalment payments but gave warning that payments must be made not later than the set deadlines therefore. The company requested for an extension for payment but this was rejected by the union. The latter staged a strike and picketed in the entrance of the premises of the company. The Minister of Labor issued an order directing striking workers to resume work under the terms and conditions prevailing prior the work stoppage. Only 11 out of 120 workers reported back for work. The union filed a motion for reconsideration of the return to work order or its suspension pending compliance of company with the NLRC award. Picketing was resumed despite the return to work order. The Minister of Labor certified the dispute to the NLRC for compulsory arbitration but 110 workers still did not return to work. The company filed with the Department of Labor reports on the dismissal of those who failed to comply with the order. The union president and 7 other officers requested admission to work but were informed that their employment had been terminated. The NLRC rendered judgment and ordered that the union officers with its BOD be considered separated from service and from their positions as officers of the union, and that the union members who participated in the strike be considered under suspension. Union appealed the NLRC decision to the Minister of Labor who affirmed the former. Issues: (1) WON the strike was illegal and (2) WON the penalties meted out to the union officers and members are warranted by the circumstances and the law. Held: 1. Strike was illegal. Under PD 823, all form of strikes, picketing and lockouts are hereby prohibited in vital industries. Meanwhile, under LOI 368, among those considered as vital industries are companies or firms engaged in the manufacture or processing cement. 38 | P a g e

The company was engaged in the manufacture of cement which is a vital industry. Assuming that the company is not a vital industry, the strike was still illegal. It was not in connection with any unresolved economic issue in collective bargaining which is the only ground for which a lawful strike can be held. The issue between the parties is concerned merely on the implementation of the arbitration award. The union had a remedy by applying for a writ of execution to enforce that award. Its resort to strike is without lawful basis. The strikers did not only violate the no-strike policy in regard to vital industries, they also defied the orders of the Director of Labor Relations and the Ministry of Labor. 2. The officers of the union had the duty to guide their members to respect the law. Instead they urged them to violate the law and defy the duly constituted authorities. Their responsibility is greater than that of the members. Their dismissal from service is a just penalty for their unlawful acts [Art. 242 (p)]. The officers of the union misinformed the members and led them into striking an illegal strike. The removal of undesirable labor leaders must be effected. 8. Enforcement and Remedies – Procedure and Sanctions – 241 last par., 226 Art. 241. Rights and conditions of membership in a labor organization. For this purpose, registered labor organizations may assess reasonable dues to finance labor relations seminars and other labor education activities. Any violation of the above rights and conditions of membership shall be a ground for cancellation of union registration or expulsion of officers from office, whichever is appropriate. At least thirty percent (30%) of the members of a union or any member or members specially concerned may report such violation to the Bureau. The Bureau shall have the power to hear and decide any reported violation to mete the appropriate penalty. Criminal and civil liabilities arising from violations of above rights and conditions of membership shall continue to be under the jurisdiction of ordinary courts. Art. 226. Bureau of Labor Relations. The Bureau of Labor Relations and the Labor Relations Divisions in the regional offices of the Department of Labor, shall have original and exclusive authority to act, at their own initiative or upon request of either or both parties, on all inter-union and intra-union conflicts, and all disputes, grievances or problems arising from or affecting labor-management relations in all workplaces, whether agricultural or non-agricultural, except those arising from the implementation or interpretation of collective bargaining agreements which shall be the subject of grievance procedure and/or voluntary arbitration. The Bureau shall have fifteen (15) working days to act on labor cases before it, subject to extension by agreement of the parties. (As amended by Section 14, Republic Act No. 6715, March 21, 1989). Jurisdiction – Exhaustion Internal Remedies Requirements: Diokno v. Cacdac, 526 SCRA 440 (07) FLAMES is the supervisory union of MERALCO. Its Executive Board created the COMELEC for its union elections on May 7, 2003. The petitioners also sought the disqualification of respondents Edgardo Daya et al. They alleged that the latter allowed themselves to be assisted by non-union members and committed acts of disloyalty when they colluded with the officers of MESALA and MEMABA and exerted undue influence on the members of FLAMES. The COMELEC issued a decision declaring Daya et al. officially disqualified to run and resolved to exclude their names from the list of candidates in the polls.Daya et al.filed with the MedArb a petition to nullify the order of disqualification, the election proceedings and counting of votes, declare failure of election and declare the holding of new election. The MedArb reversed the disqualification of respondents. He found that the COMELEC erred in relying on Art. IV of Sec 4(a), (6) of the CBL as basis for the disqualification. The said provision refers to the dismissal and/or expulsion of a member from FLAMES but not to the disqualification of a member as a candidate in a union election. He also held that there was denial of due process because the COMELEC failed to receive respondents MR of the order of disqualification. The Director of the BLR affirmed in toto the MedArb. He also held that the failure of respondents to file a petition before the COMELEC to nullify its order is an exception to the rule on exhaustion of administrative remedies. A subsequent election was held on June 30 which was participated and won by Daya et al. The CA upheld the validity of the said elections and the declaration of Daya at al. as the duly elected winners. The appellate court found the petitioners’ appeal to be bereft of merit. It applied the exception to the rule on exhaustion of admin remedies on the ground that resort to such remedy would have been futile, illusory or vain. Respondents’ efforts to have their disqualification reconsidered were rebuffed by the COMELEC and they were left with no choice but to seek intervention of the BLR. ISSUE: WON the CA abused its discretion when it upheld the ruling of the MedArb and BLR nullifying the order of disqualification of respondents HELD: No. The CA properly upheld the application by the BLR director of the exception to the rule of exhaustion of admin remedies. As a general rule, before a party is allowed to seek the intervention of the court, it is a precondition that he should have availed of all the means of admin processed afforded him. The premature invocation of court intervention is fatal to one’s cause of action. However, there are exceptions to the applicability of this doctrine which include violation of due process. Respondents were prejudiced by the disqualification order. They endeavoured to seek reconsideration but the COMELEC failed to act thereon. The COMELEC also refused to receive their letter protest. Respondents were deprived of due process and it became incumbent upon them to seek the aid of BLR. To insist otherwise is to render their exhaustion of remedies within the union illusory and vain. The respondents were also wrongfully disqualified as there was a blatant misapplication by the COMELEC of the union CBL. The provision relied upon by the COMELEC applies on the expulsion of members from the union. The disqualification made by the COMELEC was a disregard of due process. The procedural requirements for 39 | P a g e

expulsion from the union or disqualification from elections must be observed. The erroneous disqualification of the respondents constituted a case of disenfranchisement on the part of the members-voters. By wrongfully excluding them from the elections, the options afforded the members were clipped. Petition is denied and the decision of the CA is affirmed. Saparol v. Salarga, 538 SCRA 324 (07) Petitioners Sarapat, Durian and Castillo were president, treasurer and director of the Phil Veterans Bank Employees Union (union) while Respondents Sarapat and Silapan are members of said union. In 1985, the Bank went bankrupt and was placed under receivership/liquidation by the CB. When the bank re-opened in 1992, the employees were not re-hired. The union filed a notice of strike and cases of ULP against the bank. Later on, the bank and the union entered into a compromise agreement for amicable settlement of the cases where a financial settlement of P35M was granted to the union, 5% thereof as special assessment fee to be deducted from the settlement amount of each member to defray litigation expenses. Respondents filed a petition with the DOLE NCR for an audit of finances of the union and a separate and full accounting of the 5% special assessment fee. Despite notices and directives upon petitioners to submit documents for audit, they failed to do so. The DOLE issued an order directing the latter to open the union books of account and hold a general meeting to explain the financial status of the union to the members. Petitioners filed an appeal with the BLR questioning the order calling for the conduct of a general meeting. They admitted that the general meeting is not feasible because of the dissolution of the union and the termination of the members’ employment in the bank. BLR gave the petitioners a final chance to submit documents in support of the alleged litigation expenses Sarapat stated that the litigation expense was not limited to the NLRC but included the SC, the office of the Secretary of Labor and the NCR Arbitration Branch of the NLRC. The BLR issued a resolution declaring the statement of receipts and disbursement as insufficient to prove actual litigation expenses since no official receipts, dv, checks and acknowledgement receipts were submitted. It held the petitioners solidarily liable to restitute union members the amount of the partial remittance of check-off for the special assessment. It also ordered the bank to refrain from deducting anymore 5% special assessment fee from any member’s settlement amount. Petitioners appealed to the CA but the latter dismissed their petition. ISSUE: WON the petitioners were denied due process when BLR ruled motu proprio on the propriety of the litigation expenses and ordered restitution without holding a hearing therefore and that despite the fact that the propriety of the said expenses was not an issue on appeal. HELD: NO. Petition is bereft of merit. Petitioners cannot claim denial of due process. The essence of due process is the opportunity to be heard or to explain one’s side or to seek a reconsideration of the action or ruling complained of. Not all cases require a trial-type hearing. Due process is satisfied when the parties are given the opportunity to submit their position papers to which they are to attach supporting documentary evidence to prove their claims. Petitioners were given every opportunity to present their side and submit union books of accounts and other documents to justify the litigation expenses but they failed to do so. When they filed MR assailing the ruling on the propriety of the said expenses, such action satisfies due process. Petitioners are also estopped from assailing the jurisdiction of the BLR to rule of the propriety of the litigation expenses. They filed a compliance with the BLR’s directive to submit documents in support of the expenses without raising an objection. The BLR is justified in taking cognizance of the case to resolve the issue of the expenses. Since the case was limited to audit and accounting of the litigation expenses, such matter was open to evaluation. Technical rules of procedure are not applicable in labor cases but may apply only by analogy or in suppletory character when there is a need to attain substantial justice and expeditious, practical and convenient solution to a labor problem. The 5% special assessment fee was not intended to form part of the general fund of the union but for expenses incurred in the prosecution of the case. The order of restitution was proper since the petitioners failed to prove the justification for deducting said amount from the members’ settlement amount. Diamonon v. DOLE, 327 SCRA 282 (00) Petition for certiorari Petitioner Diamonon filed 2 complaints before the DOLE after being removed from his capacity as vice president of the unions NACUSIP and PACIWU. In his first complaint, he questioned the validity of his removal from office and in the second, he accused both the national treasurer and national president of NACUSIP and PACIWU (private respondents) for wanton violation of the Constitution and By-Laws of both orgs, unauthorized and illegal disbursement of union funds and also, abuse of authority. The 1st complaint was decided in his favor, his removal was declared null and void. The 2 nd complaint was dismissed for lack of personality in view of his removal from the offices he held. Diamonon appealed but public respondent Laguesma, as undersecretary of DOLE, held that petitioner’s failure to show, in his complaint, that the administrative remedies provided for in the constitution and by-laws of both unions have been exhausted or such remedies are not available, was fatal. With his MFR denied, Diamonon filed this case. ISSUE: WON respondent Laguesma acted with grave abuse of discretion in dismissing the appeal for non-exhaustion of administrative remedies. HELD: No. Ratio When the Constitution and By-Laws of Unions dictate the remedy for internal dispute, they should be resorted to before recourse can be made to the appropriate administrative or judicial body.

40 | P a g e

- A party with an administrative remedy must not merely initiate the prescribed administrative procedure to obtain relief, but also pursue it to its appropriate conclusion before seeking judicial intervention. Reasoning Not only did petitioner fail to comply with Section 2, Rule VIII, Book V of the Implementing Rules of the Labor Code but also the record reveals that neither did he exhaust the remedies set forth by the Constitution and by-laws of both unions. In the National Convention of PACIWU and NACUSIP, nothing was heard of petitioner’s complaint against private respondents on the latter’s alleged unauthorized and illegal disbursement of union funds. His failure to seek recourse before the National convention on his complaint against private respondents taints his action with prematurity. Disposition Petition DISMISSED. Villar v. Inciong, 121 SCRA 444 (83) Facts: Petitioners were members of the Amigo Employees Union-PAFLU, a duly registered labor organization which was the existing bargaining agent of the employees in private respondent Amigo Manufacturing, Inc. They then signed a joint resolution which stated that they were disaffiliating themselves as members of the PAFLU, and were not further authorizing PAFLU to represent them in any CBA. During a special meeting of the Union, a Resolution was approved which called for the investigation of all the petitioners for "continuously maligning, libelling and slandering not only the incumbent officers but even the union itself and the federation" and for causing divisiveness. The union security clause was reincorporated in the new CBA: “any member who shall resign, be expelled, or shall in any manner cease to be a member of the UNION, shall be dismissed from his employment upon written request of the UNION to the Company.” Petitioners contend that their acts do not constitute disloyalty as these are in the exercise of their constitutional right to self-organization. PAFLU President then rendered a decision finding them guilty of the charges and expelling them from the Union and as a consequence the Management of the employer is hereby requested to terminate them from their employment in conformity with the security clause. Issues: WON PAFLU had the authority to investigate the petitioners and, thereafter, expel them from the roll of membership of the Amigo Employees Union-PAFLU SC: YES. That PAFLU had the authority to investigate petitioners on the charges filed by their co-employees in the local union and after finding them guilty as charged, to expel them from the roll of membership of the Amigo Employees Union-PAFLU is clear under the constitution of the PAFLU to which the local union was affiliated. And pursuant to the security clause of the new CBA, reiterating the same clause in the old CBA, PAFLU was justified in applying said security clause. Inherent in every labor union, or any organization for that matter, is the right of self-preservation. When members of a labor union, therefore, sow the seeds of dissension and strife within the union; when they seek the disintegration and destruction of the very union to which they belong, they thereby forfeit their rights to remain as members of the union which they seek to destroy. Prudence and equity, as well as the dictates of law and justice, therefore, compelling mandate the adoption by the labor union of such corrective and remedial measures in keeping with its laws and regulations, for its preservation and continued existence; lest by its folly and inaction, the labor union crumble and fall. Issue: WON the petitioners were entitled to disaffiliate from the Union. SC: Yes. Although they are entitled to disaffiliate from their union and form a new organization of their own, they must, however, suffer the consequences of their separation from the union under the security clause of the CBA. Disaffiliation from a labor union is not open to legal objection. It is implicit in the freedom of association ordained by the Constitution. But this Court has laid down the ruling that a closed shop is a valid form of union security, and such provision in a collective bargaining agreement is not a restriction of the right of freedom of association guaranteed by the Constitution. Remedy Kapisanan ng Manggagawang Pinagyakap (KMP) v. Trajano, 134 SCRA 236 (85) FACTS: In 1981, private respondents Silvestre and 13 other union members filed a request for accounts examination of the financial status of the KMP, the existing labor union at Franklin Baker Company. DOLE Union Account examiner conducted an investigation and submitted a report with the findings: total of P1,278 as disallowed expenditures; that KMP failed to maintain records of union accounts for 1977-1979; failed to segregate disbursement receipts; and that the constitution and bylaws were not ratified by the general membership, and hence illegal. Based on the foregoing revelations, private respondents filed with the DOLE Regional Office a petition for the expulsion of the union officers on the ground that they committed gross violation of the Labor Code (Art 242) and the union’s constitution and by-laws. In their Answer, the union officers contend that: the disallowed expenditures were made in good faith and used for the benefit of the members; they should not be held accountable for the non-production of the books of accounts for the years 1977-1979 because they were not the officers then; that the non-ratification of the constitution and by-laws and the non-segregation of the union funds occurred before they became officers and that they have already been correcting the same. Instead of ordering expulsion, Med-Arbiter ordered the holding of a referendum to decide on the issue of whether to expel or suspend the union officers from their respective positions. BLR affirmed. In the meantime, the same officers were re-elected in a general election of the union. ISSUES: 1) WON the officers should be expelled. HELD: No. Petition granted. 41 | P a g e

1) First of all: If the union officers were guilty of the alleged acts imputed against them, said public respondent pursuant to Article 242 of the New Labor Code and in the light of the ruling in Duyag vs. Inciong, should have meted out the appropriate penalty, i.e., to expel them from the Union, as prayed for, and not call for a referendum to decide the issue. But in this case, the alleged falsification and misrepresentation of the union officers were not supported by substantial evidence. The expenditures appeared to have been made in good faith and for the union members’ benefit. Furthermore, their re-election is a convincing manifestation and demonstration of the union membership's faith in the herein officers' leadership. Effect of re-election: The holding of the referendum in question has become moot and academic. As held in Pascual vs. Provincial Board of Nueva Ecija, “The Court should never remove a public officer for acts done prior to his present term of office. To do otherwise would be to deprive the people of their right to elect their officers. When the people have elected a man to office, it must be assumed that they did this with knowledge of his life and character, and that they disregarded or forgave his faults or misconduct, if he had been guilty of any. It is not for the court, by reason of such faults or misconduct to practically overrule the will of the people.” V. UNION AFFILIATION: LOCAL AND PARENT UNION RELATIONS – INTERNATIONAL COVENANT ECONOMIC, SOCIAL AND CULTURAL RIGHTS (Art. 8 (b), ILO Convention No. 87, Art. 5) ICESCR, Article 8 1. The States Parties to the present Covenant undertake to ensure: (b) The right of trade unions to establish national federations or confederations and the right of the latter to form or join international trade-union organizations; ILO Convention No. 87, Article 5 Workers' and employers' organisations shall have the right to establish and join federations and confederations and any such organisation, federation or confederation shall have the right to affiliate with international organisations of workers and employers. A. Affiliation: Purpose of: Nature of Relations – 211 (c) Art. 211. Declaration of Policy. A. It is the policy of the State: c. To foster the free and voluntary organization of a strong and united labor movement; 1. Purpose Philippine Skylanders v. NLRC, 375 SCRA 369 (02) Philippine Skylanders Employees Association (PSEA), a local labor union affiliated with the Philippine Association of Free Labor Unions (PAFLU) September (PAFLU), won in the certification election conducted among the rank and file employees of Philippine Skylanders, Inc. (PSI). Its rival union, Philippine Skylanders Employees Association-WATU (PSEA-WATU) immediately protested the result of the election before the Secretary of Labor. Several months later, pending settlement of the controversy, PSEA sent PAFLU a notice of disaffiliation citing as reason PAFLU's supposed deliberate and habitual dereliction of duty toward its members. Attached to the notice was a copy of the resolution adopted and signed by the officers and members of PSEA authorizing their local union to disaffiliate from its mother federation. PSEA subsequently affiliated itself with the National Congress of Workers (NCW), changed its name to Philippine Skylanders Employees Association - National Congress of Workers (PSEA-NCW), and to maintain continuity within the organization, allowed the former officers of PSEA-PAFLU to continue occupying their positions as elected officers in the newly-forged PSEA-NCW. PSEA-NCW entered into a collective bargaining agreement with PSI which was immediately registered with the Department of Labor and Employment. PSI through its personnel manager Francisco Dakila denied the request citing as reason PSEA's disaffiliation from PAFLU and its subsequent affiliation with NCW. Agitated by PSI's recognition of PSEA-NCW, PAFLU through Serafin Ayroso filed a complaint for unfair labor practice against PSI. PAFLU alleged that aside from PSI’s refusal to bargain collectively with its workers, the company through its president and personnel manager, was also liable for interfering with its employees' union activities. ISSUE: The fundamental issue tapers down to the legitimacy of PSEA's disaffiliation: May PSEA, which is an independent and separate local union, validly disaffiliate from PAFLU pending the settlement of an election protest questioning its status as the sole and exclusive bargaining agent of PSI's rank and file employees? HELD: YES. At the outset, let it be noted that the issue of disaffiliation is an inter-union conflict the jurisdiction of which properly lies with the Bureau of Labor Relations (BLR) and not with the Labor Arbiter. Nonetheless, with due recognition of this fact, we deem it proper to settle the controversy at this instance since to remand the case to the BLR would only mean intolerable delay for the parties. The right of a local union to disaffiliate from its mother federation is not a novel thesis unillumined by case law. In the landmark case of Liberty Cotton Mills Workers Union vs. Liberty Cotton Mills, Inc. [16] we upheld the right of local unions to separate from their mother federation on the ground that as separate and voluntary associations, local unions do not owe their creation and existence to the national federation to which they are affiliated but, instead, to the will of their members. The sole essence of affiliation is to increase, by collective action, the common bargaining power of local unions for the effective enhancement and protection of their

42 | P a g e

interests. Admittedly, there are times when without succor and support local unions may find it hard, unaided by other support groups, to secure justice for themselves. Yet the local unions remain the basic units of association, free to serve their own interests subject to the restraints imposed by the constitution and by-laws of the national federation, and free also to renounce the affiliation upon the terms laid down in the agreement which brought such affiliation into existence.There is nothing shown in the records nor is it claimed by PAFLU that the local union was expressly forbidden to disaffiliate from the federation nor were there any conditions imposed for a valid breakaway. As such, the pendency of an election protest involving both the mother federation and the local union did not constitute a bar to a valid disaffiliation. The mere act of disaffiliation did not divest PSEA of its own personality; neither did it give PAFLU the license to act independently of the local union. Recreant to its mission, PAFLU cannot simply ignore the demands of the local chapter and decide for its welfare. PAFLU might have forgotten that as an agent it could only act in representation of and in accordance with the interests of the local union. 2. Nature Relationship Filipino Pipe and Foundry Corporation, 318 SCRA 68 (99) Respondent National Labor Union-Trade Union Congress of the Philippines (NLU-TUCP), a national federation of labor unions, filed with the then Ministry of Labor and Employment, in behalf of its local chapter, the Filipino Pipe Workers Union-National Labor Union (FPWU-NLU, hereinafter referred to as Union), a notice of strike signed by its national president, Atty. Eulogio R. Lerum, against the petitioner, Filipino Pipe and Foundry Corporation, alleging as grounds therefor union busting and nonimplementation of the Collective Bargaining Agreement. The initial conciliation conference was set on February 24, 1986 but due to lack of notice thereof to petitioner company, as well as the failure of FPWU-NLU to furnish the latter a copy of the notice of strike, the initial conciliation conference was re-set to March 3, 1986. In the early morning of March 3, 1986, however, without waiting for the outcome of the conciliation conference scheduled on said date, the FPWU-NLU staged the strike in question which lasted until June 13, 1986, when a return to work agreement was reached by the union and petitioner company. Section 1. Grounds for strike and lockout . A strike or lockout may be declared in cases of bargaining deadlocks and unfair labor practices. Violations of collective bargaining agreements, except flagrant and/or malicious refusal to comply with its economic provisions, shall not be considered unfair labor practice and shall not be strikeable. No strike or lockout may be declared on grounds involving inter-union and intra-union disputes or on issues brought to voluntary or compulsory arbitration. Section 3. Notice of strike or lockout.- In cases of bargaining deadlocks, a notice of strike or lockout shall be filed withthe regional branch of the Board at least thirty (30) days before the intended date thereof, a copy of said notice having been served on the other party concerned. "Section 6. Conciliation. - Upon receipt of the notice, the regional branch of the Board shall exert all efforts at mediation and conciliation to enable the parties to settle the dispute amicably. The regional branch of the Board may, upon consultation, recommend to the parties to submit the dispute to voluntary arbitration. During the proceedings, the parties shall not do any act which may disrupt or impede the early settlement of the dispute. They are obliged as part of the duty to bargain collectively in good faith, to participate fully and promptly in the conciliation meetings called by the regional branch of the board. The regional branch of the Board shall have the power to issue subpoenas requiring the attendance of the parties to the meetings. Applying the aforecited provision of law in point to the case under consideration, the Court is of the finding and conclusion that the strike staged by FPWU-NLU was illegal for want of any legal basis. Then too, the failure of the union to serve petitioner company a copy of the notice of strike is a clear violation of Section 3 of the aforestated Rules. The same strike blatantly disregarded the prohibition on the doing of any act which may impede or disrupt the conciliation proceedings, when the union staged the strike in the early morning of the very same day the conciliation conference was scheduled. ISSUE: WON the local union or the mother union is responsible for the illegal strike. HELD: Local Union. Evidently, direct and primary responsibility for the damages allegedly caused by the illegal strike sued upon fall on the local union FPWU, being the principal, and not on respondent NLU-TUCP, a mere agent of FPWU-NLU which assisted the latter in filing the notice of strike. Being just an agent, the notice of strike filed by Atty. Eulogio Lerum, the national president of NLU-TUCP, is deemed to have been filed by its principal, the FPWU-NLU. Having thus dismissed the claim for damages against the principal, FPWU-NLU, the action for damages against its agent, respondent NLU-TUCP. It is important to clarify the relationship between the mother union and the local union. In the case of Liberty Cotton Mills Workers Union v. Liberty Cotton Mills, Inc., the Court held that the mother union, acting for and in behalf of its affiliate, had the status of an agent while the local union remained the basic unit of the association, free to serve the common interest of all its members subject only to the restraints imposed by the constitution and by-laws of the association. 3. Effect-Legal Personality Adamson and Adamson Inc. v. CIR, 127 SCRA 268 (84) Adamson and Adamson, Inc., filed this petition to set aside orders of the respondent Court of Industrial Relations (CIR) holding that the Adamson and Adamson, Inc. supervisory Union (FFW) can legally represent supervisors of the petitioner corporation notwithstanding the affiliation of the rank and file union of the same company with the same labor federation, the Federation of Free Workers.

43 | P a g e

The Adamson and Adamson, Inc. Supervisory Union (FFW) informed the petitioner about its having organized on the same date that the Adamson and Adamson, Inc. Salesmen Association (FFW) advised the petitioner that the rank and file salesmen had formed their own union. Subsequently and during the pendency of the present petition, the rank and file employees formed their own union, naming it Adamson and Adamson Independent Workers (FFW). The petitioner submits that should affiliation be allowed, this would violate the requirement of separateness of bar units under Section 12 of the Act because only one union will in fact represent both supervisors and rank-and-file employees of the petitioner. ISSUE: Whether or not a supervisor's union may affiliate with a federation with which unions of rank and-file employees of the same employer are also affiliated. HELD: YES The Court finds without merit the contentions of petitioner that if affiliation will be allowed, only one union will in fact represent both supervisors and rank-and-file employees of the petitioner; that there would be an indirect affiliation of supervisors and rank-and-file employees with one labor organization; that there would be emerging of two bargaining units ; and that the respondent union will lose its independence because it becomes an alter ego of the federation. The Adamson and Adamson Supervisory Union and the Adamson and Adamson, Inc., Salesmen Association (FFW), have their own respective constitutions and by-laws. They are separately and independently registered of each other. Both sent their separate proposals for collective bar agreements with their employer. There could be no employer influence on rank-and-file organizational activities nor there could be any rank and file influence on the supervisory function of the supervisors because of the representation sought to be proscribed. The supervisors referred to in here are supervisors who do not directly manage the rank-and-file employees who formed the union. In interpreting the protection to labor and social justice provisions of the Constitution and the labor laws or rules and regulations implementing the constitutional mandates, the court have always adopted the liberal approach which favors the exercise of labor rights. 4. Supervisor – Rank and File Union Affiliation = 245; 243, 245-A Art. 245. Ineligibility of managerial employees to join any labor organization; right of supervisory employees. Managerial employees are not eligible to join, assist or form any labor organization. Supervisory employees shall not be eligible for membership in a labor organization of the rank-and-file employees but may join, assist or form separate labor organizations of their own. (As amended by Section 18, Republic Act No. 6715, March 21, 1989) Art. 243. Coverage and employees’ right to self-organization. All persons employed in commercial, industrial and agricultural enterprises and in religious, charitable, medical, or educational institutions, whether operating for profit or not, shall have the right to self-organization and to form, join, or assist labor organizations of their own choosing for purposes of collective bargaining. Ambulant, intermittent and itinerant workers, self-employed people, rural workers and those without any definite employers may form labor organizations for their mutual aid and protection. (As amended by Batas Pambansa Bilang 70, May 1, 1980) Rule – Affiliation – Compare: 245-A and Cases Adamson and Adamson, Inc. v. CIR, 127 SCRA 268 (84) Adamson and Adamson, Inc., filed this petition to set aside orders of the respondent Court of Industrial Relations (CIR) holding that the Adamson and Adamson, Inc. supervisory Union (FFW) can legally represent supervisors of the petitioner corporation notwithstanding the affiliation of the rank and file union of the same company with the same labor federation, the Federation of Free Workers. The Adamson and Adamson, Inc. Supervisory Union (FFW) informed the petitioner about its having organized on the same date that the Adamson and Adamson, Inc. Salesmen Association (FFW) advised the petitioner that the rank and file salesmen had formed their own union. Subsequently and during the pendency of the present petition, the rank and file employees formed their own union, naming it Adamson and Adamson Independent Workers (FFW). The petitioner submits that should affiliation be allowed, this would violate the requirement of separateness of bar units under Section 12 of the Act because only one union will in fact represent both supervisors and rank-and-file employees of the petitioner. ISSUE: Whether or not a supervisor's union may affiliate with a federation with which unions of rank and-file employees of the same employer are also affiliated. HELD: YES The Court finds without merit the contentions of petitioner that if affiliation will be allowed, only one union will in fact represent both supervisors and rank-and-file employees of the petitioner; that there would be an indirect affiliation of supervisors and rank-and-file employees with one labor organization; that there would be emerging of two bargaining units ; and that the respondent union will lose its independence because it becomes an alter ego of the federation. The Adamson and Adamson Supervisory Union and the Adamson and Adamson, Inc., Salesmen Association (FFW), have their own respective constitutions and by-laws. They are separately and independently registered of each other. Both sent their separate proposals for collective bar agreements with their employer. There could be no employer influence on rank-and-file organizational activities nor there could be any rank and file influence on the supervisory function of the supervisors because of the representation sought to be proscribed. The supervisors referred to in here are supervisors who do not directly manage the rank-and-file employees who formed the union. In interpreting the protection to labor and social justice provisions of the Constitution and the labor laws or rules and regulations implementing the constitutional mandates, the court have always adopted the liberal approach which favors the exercise of labor rights. 44 | P a g e

Atlas Lithographic Services, Inc. v. Laguesma, 205 SCRA 12 (92) The supervisory, administrative personnel, production, accounting and confidential employees of the petitioner Atlas Lithographic Services, Inc. (ALSI) is affiliated with private respondent Kaisahan ng Manggagawang Pilipino, a national labor organization. The local union adopted the name Atlas Lithographic Services, Inc. Supervisory, Administrative, Personnel, Production, Accounting and Confidential Employees Association or ALSI-SAPPACEA-KAMPIL in short. Private respondent Kampil-Katipunan filed on behalf of the "supervisors" union a petition for certification election so that it could be the sole and exclusive bargaining agent of the supervisory employees. The petitioners opposed the private respondent's petition claiming that under Article 245 of the Labor Code the private respondent cannot represent the supervisory employees for collective bargaining purposeless because the private respondent also represents the rank-and-file employees' union. ISSUE: Whether or not, under Article 245 of the Labor Code, a local union of supervisory employees may be allowed to affiliate with a national federation of labor organizations of rank-and-file employees. HELD: NO. The petitioner argues that KAMPIL-KATIPUNAN already represents its rank-and-file employees and, therefore, to allow the supervisors of those employees to affiliate with the private respondent is tantamount to allowing the circumvention of the principle of the separation of unions under Article 245 of the Labor Code. It further argues that the intent of the law is to prevent a single labor organization from representing different classes of employees with conflicting interests. The rationale for the amendment is the government's recognition of the right of supervisors to organize with the qualification that they shall not join or assist in the organization of rank-and-file employees. The functions of supervisors, being recommendatory in nature, are more identified with the interests of the employer. The performance of those functions may, thus, run counter to the interests of the rank-and-file. More important, the factual issues in the Adamson case are different from the present case. First, the rank-and-file employees in the Adamson case are not directly under the supervisors who comprise the supervisors' union. In the case at bar, the rank-and file employees are directly under the supervisors organized by one and the same federation. Second, the national union in the Adamson case did not actively represent its local chapters. In the present case, the local union is actively represented by the national federation. In fact, it was the national federation, the KAMPIL-KATIPUNAN, which initially filed a petition for certification in behalf of the respondent union. The prohibition against a supervisors' union joining a local union of rank-and-file is replete with jurisprudence. The Court emphasizes that the limitation is not confined to a case of supervisors wanting to join a rank-and-file local union. The prohibition extends to a supervisors' local union applying for membership in a national federation the members of which include local unions of rank-and-file employees. The intent of the law is clear especially where, as in the case at bar, the supervisors will be co-mingling with those employees whom they directly supervise in their own bargaining unit. Finally, the respondent contends that the law prohibits the employer from interfering with the employees' right to self-organization. There is no question about this intendment of the law. There is, however, in the present case, no violation of such a guarantee to the employee. Supervisors are not prohibited from forming their own union. What the law prohibits is their membership in a labor organization of rank-and-file employees (Art. 245, Labor Code) or their joining a national federation of rank-and-file employees that includes the very local union which they are not allowed to directly join. De La Salle University Medical Center v. Laguesma, 294 SCRA (98) Facts: DLSUMCCM is a labor organization composed of the supervisory employees of petitioner DLSUMCCM. FFW recognized the Supervisor’s Union as its local chapter. FFW filed on behalf of the Supervisory union a petition for certification election among the supervisory employees of Company. Contention of Company: Several employees who signed the petition for certification election were managerial employees and that the FFW-DLSUMCCMSUC was composed of both supervisory and rank-and-file employees in the company. MA granted CE. SOLE affirmed MA. Issue: WON unions formed independently by supervisory and rank-and-file employees of a company may validly affiliate with the same national federation. SC: Supervisory employees have the right to self-organization as do other classes of employees save only managerial ones. As we recently held in United Pepsi-Cola Supervisory Union v. Loguesma, the framers of the Constitution intended, by this provision, to restore the right of supervisory employees to self-organization which had been withdrawn from them during the period of martial law. Art. 245 now provides for the right of supervisory employees to self-organization, subject to the limitation that they cannot join an organization of rank-and-file employees. The reason for the segregation of supervisory and rank-and-file employees of a company with respect to the exercise of the right to self-organization is the difference in their interests. These consequences can obtain not only in cases where supervisory and rank-and-file employees in the same company belong to a single union but also where unions formed independently by supervisory and rank-and-file employees of a company are allowed to affiliate with the same national federation. Atlas, Qualified (Requisties) First, R&F employees are directly under the authority of supervisory employees. Second, the national federation is actively involved in union activities in the company. Indeed, it is the presence of these two conditions which distinguished Atlas Lithographic Services, Inc. v. Laguesma from Adamson & Adamson, Inc. v. CIR where a different conclusion was reached.

45 | P a g e

The affiliation of two local unions in a company with the same national federation is not by itself a negation of their independence since in relation to the employer, the local unions are considered as the principals, while the federation is deemed to be merely their agent. This conclusion is in accord with the policy that any limitation on the exercise by employees of the right to self-organization guaranteed in the Constitution must be construed strictly Although private respondent FFW-DLSUMCCMSUC and another union composed of rank-and-file employees of petitioner DLSUMCCM are indeed affiliated with the same national federation, the FFW, petitioner DLSUMCCM has not presented any evidence showing that the rank-and-file employees composing the other union are directly under the authority of the supervisory employees . As held in Adamson & Adamson, Inc. v. CIR, the fact that the two groups of workers are employed by the same company and the fact that they are affiliated with a common national federation are not sufficient to justify the conclusion that their organizations are actually just one. Mention has already been made of the fact that the petition for certification election in this case was filed by the FFW on behalf of the local union. This circumstance, while showing active involvement by the FFW in union activities at the company, is by itself insufficient to justify a finding of violation of Art. 245 since there is no proof that the supervisors who compose the local union have direct authority over the rank-and-file employees composing the other local union which is also affiliated with the FFW. It follows Adamson should be applied. B. Local Union Disaffiliation 1. Nature Right Disaffiliation Volkschel Labor Union v. Bureau of Labor Relations, 137 SCRA 42 (85) Facts: Union was once affiliated with the Associated Labor Union for Metal Workers, ALUMETAL for short. Both unions jointly entered into a CBA with their corresponding companies. One of the subjects dealt with is the payment of union dues. A majority of Volkschel Union decided to disaffiliate from ALUMETAL in order to operate on its own as an independent labor group pursuant to Article 241. Incumbent affiliates of existing federations or national unions may disaffiliate only for the purpose of joining a federation or national union in the industry or region in which it properly belongs or for the purpose of operating as an independent labor group. Confronted with the predicament of whether or not to continue deducting from employees' wages and remitting union dues to respondent, ALUMETAL companies sought the legal opinion of the respondent Bureau which in effect found the disaffiliation legal but at the same time gave the opinion that, members should continue paying their dues to ALUMETAL in the concept of agency fees. Contention of Volkschel: That Volkschel Union’s members remained obligated to pay dues to respondent ALUMETAL was inconsistent with the dispositive finding that petitioner's disaffiliation from ALUMETAL was valid; hence, Bureau recognized "the continued affiliation of Volkschel Labor Union with the Associated Labor Union for Metal Workers. SOLE issued a of execution to enforce judgment. Issue: (1) Is petitioner union's disaffiliation from respondent federation valid? SC: Yes. Valid. The right of a local union to disaffiliate from its mother union is well-settled. This right is consistent with the Constitutional guarantee of freedom of association. Disaffiliation was prompted by the federation's deliberate and habitual dereliction of duties as mother federation towards petitioner union. While a disaffiliation does not disturb the enforceability and administration of a collective agreement; it is nowhere in the record does it appear that the contract entered into by the Volkschel Union and ALUMETAL prohibits the withdrawal of the former from the latter. Issue: (2) Is ALUMETAL entitled to union dues payments from Volkschel’s union's members notwithstanding their disaffiliation from said federation? SC: No. Under Section 3, Article I, of the CBA, the obligation of the companies to deduct and remit dues to ALUMETAL is conditioned on the individual check-off authorization of petitioner's members, In other words, ALUMETAL is entitled to receive the dues from companies as long as petitioner union is affiliated with it and respondent companies are authorized by their employees (members of petitioner union) to deduct union dues. The obligation of an employee to pay union dues is coterminous with his affiliation or membership. The employees' check-off authorization, even if declared irrevocable, is good only as long as they remain members of the union concerned. A contract between an employer and the parent organization as bargaining agent for the employees is terminated by the disaffiliation of the local of which the employees are members. Philippine Labor Alliance Council v. BLR, 75 SCRA 162 (77) Facts: Renewal of CBA with a union shop clause between PLAC and company. CBA was ratified by a unanimous vote. FFW-ORION, setting forth that its members represent more than 60% out of 1,500 members, more or less, rank-and-file employees of respondent company, sought a certification election. 1 Contention of PLAC: PLAC opposed such a move as in its view the CBA had been certified. NLRC=SOLE = PLAC: Dismiss petition for CE on the ground of the existence of a certified collective bargaining agreement. Contention of FFW in BLR case: 848 employees disaffiliated from petitioner union and affiliated with it. BLR ordered CE. CBA was not ratified by the majority of the employees within the bargaining unit. Issue: WON BLR was correct in ordering CE. SC: There is nothing in the exhaustive memorandum of petitioner either that would justify the imputation that respondent Bureau, in ordering decertification of the collective bargaining agreement with petitioner to be followed by a certification election, committed a transgression of the present Labor Code 46 | P a g e

Controversy would not have arisen if there were no mass disaffiliation from petitioning Union. It is implicit in the freedom of association explicitly ordained by the Constitution. Once the fact of disaffiliation has been demonstrated beyond doubt, as in this case, a certification election is the most expeditious way of determining which labor organization is to be the exclusive bargaining representative. It is as simple as that. A different conclusion could have been reached had there been no decertification. The contract-bar rule could then be invoked. A principle in labor law that a collective bargaining agreement of reasonable duration is, in the interest of the stability of industrial relations, a bar to certification elections. In the instant case, it is not disputed that the CBA certified by the NLRC was not ratified by the majority of the employees within the bargaining unit . This is defective. It is blatant non-observance of the basic requirement necessary to certification. To allow it to remain uncorrected would allow circumvention of what the law specifically ordained. With respect to the complaint of the confirmation of disaffiliation of the members of PALC counsel the same should be resolved in the most expedient and simple method of determining the exclusive bargaining representative — the holding of a certification election." A certification election can still be held even if the collective agreement were certified, considering the peculiar facts of the case. Good policy and equity demand that when an agreement is renegotiated before the appointed 60-day period, its certification must still give way to any representation issue that may be raised within 60-day period so that the right of employees to choose a bargaining unit agent and the right, of unions to be chosen shall be preserved. Malayang Samahan ng mga Manggagawa sa Greenfield v. Ramos, 326 SCRA 428 (00) Facts: Samahan,the "local union", is an affiliate of ULGWP, referred to as the "federation". CBA between Samahan and Company included Union Security; Dismissal any such employee who fails to maintain his membership in the UNION shall be dismissed Samahan election was held under the auspices of the ULGWP wherein Villanueva, and the other union officers were proclaimed as winners. A Petition for Impeachment was filed with the federation by the defeated candidates in the aforementioned election. Federation conducted an audit of the local union funds. The investigation did not yield any unfavorable result and the local union officers were cleared. MA = Villanueva. Failure to allege fact on the part of losing candidates. Samahan held a general membership meeting where several union members failed to attend the meeting. Samahan sought to deduct the union fines from the wages/salaries of those union members who failed to attend the general membership meeting. Company refused and suggested that the union refer the matter to the proper government office. The imposition of P50.00 fine became the subject of bitter disagreement between the Federation and the local union. In retaliation, the federation asked company to stop the remittance of the local union's share in the funds, this was opposed by the Samahan. The company was thus constrained to file a Complaint for Interpleader. Held therein the Company shall remit fund to the ULGWP The federation wrote to company informing the latter of its designation of a certain Alfredo Kalingking as local union president and "disauthorizing" the incumbent union officers from representing the employees. Villanueva replied advising that Samahan did not commit any act of disloyalty as it has remained an affiliate of ULGWP. On the same day, the federation advised company of the expulsion of the 30 union officers and demanded their separation from employment pursuant to the Union Security Clause in their collective bargaining agreement. Company terminated the 30 union officers from employment. SOLE affirmed this action. 78 union shop stewards were placed under preventive suspension by company. This prompted the union members to again stage a walk-out and resulted in the official declaration of strike. The employees who participated in the strike and allegedly figured in the violent incident were placed under preventive suspension by company by virtue of request of the federation in line with the union security clause. Only 261 employees were eventually accepted back to work. Complaint for ULP filed. LA held that dismissal was valid in compliance with the union security clause. NLRC affirmed LA. Issue: WON company was justified in dismissing Villanueva et al merely upon the labor federation's demand for the enforcement of the union security clause embodied in their collective bargaining agreement. SC: No. The reason behind the enforcement of union security clauses which is the sanctity and inviolability of contractscannot override one's right to due process. Carino case - maintenance of membership provision of the CBA is bound to dismiss any employee expelled by the union for disloyalty upon its written request, this undertaking should not be done hastily and summarily. The company acts in bad faith in dismissing a worker without giving him the benefit of a hearing. Villanueva et al were expelled by the federation for allegedly committing acts of disloyalty and/or inimical to the interest of ULGWP and in violation of its Constitution and By-laws. Upon demand of the federation, the company terminated the petitioners without conducting a separate and independent investigation. Respondent company did not inquire into the cause of the expulsion and whether or not the federation had sufficient grounds to effect the same. An employee is entitled to be protected not only from a company which disregards his rights but also from his own union the leadership of which could yield to the temptation of swift and arbitrary expulsion from membership and mere dismissal from his job. The enforcement of union security clauses is authorized by law provided such enforcement is not characterized by arbitrariness, and always with due process. Even on the assumption that the federation had valid grounds to expel the union officers, due process requires that these union officers be accorded a separate hearing by respondent company.

47 | P a g e

While it is true that the issue of expulsion of the local union officers is originally between the local union and the federation, hence, intra-union in character, the issue was later on converted into a termination dispute when the company dismissed the petitioners from work without the benefit of a separate notice and hearing. Thus, notwithstanding the fact that the dismissal was at the instance of the federation and that it undertook to hold the company free from any liability resulting from such a dismissal, the company may still be held liable if it was remiss in its duty to accord the would-be dismissed employees their right to be heard on the matter. Issue: WON the federation was a principal party to the CBA between the company and the union SC: Yes. It is undisputed that ULGWP is the certified sole and exclusive collective bargaining agent of all the regular rank-and-file workers of the company. Issue: WON Samahan declared that the act of disaffiliation and declaration of autonomy by the local union was part of its plan to take over the federation. SC: No. This is purely conjecture and totally unsupported by the evidence. A local union has the right to disaffiliate from its mother union or declare its autonomy. A local union, being a separate and voluntary association, is free to serve the interests of all its members including the freedom to disaffiliate or declare its autonomy from the federation to which it belongs when circumstances warrant, in accordance with the constitutional guarantee of freedom of association. 2 The purpose of affiliation by a local union with a mother union or a federation. Thus, a local union which has affiliated itself with a federation is free to sever such affiliation anytime and such disaffiliation cannot be considered disloyalty. In the absence of specific provisions in the federation's constitution prohibiting disaffiliation or the declaration of autonomy of a local union, a local may dissociate with its parent union.The evidence on hand does not show that there is such a provision in ULGWP's constitution. There is no disloyalty to speak of, neither is there any violation of the federation's constitution because there is nothing in the said constitution which specifically prohibits disaffiliation or declaration of autonomy. The act of establishing a different federation, entirely separate from the federation which expelled them, is but a normal retaliatory reaction to their expulsion. Issue: WON strikers should be dismissed. SC: No. There was no abandonment of work. (1) that the employee must have failed to report for work or must have been absent without valid or justifiable reason; and (2) that there must have been a clear intention to sever the employer-employee relationship manifested by some overt acts. Deliberate and unjustified refusal on the part of the employee to go back to his work post and resume his employment must be established. Absence must be accompanied by overt acts unerringly pointing to the fact that the employee simply does not want to work anymore. And the burden of proof to show that there was unjustified refusal to go back to work rests on the employer. 2. Rule – Legality Act – Disaffiliation Villar v. Inciong, 121 SCRA 444 (83) Facts: Petitioners were members of the Amigo Employees Union-PAFLU, a duly registered labor organization which was the existing bargaining agent of the employees in private respondent Amigo Manufacturing, Inc. They then signed a joint resolution which stated that they were disaffiliating themselves as members of the PAFLU, and were not further authorizing PAFLU to represent them in any CBA. During a special meeting of the Union, a Resolution was approved which called for the investigation of all the petitioners for "continuously maligning, libelling and slandering not only the incumbent officers but even the union itself and the federation" and for causing divisiveness. The union security clause was reincorporated in the new CBA: “any member who shall resign, be expelled, or shall in any manner cease to be a member of the UNION, shall be dismissed from his employment upon written request of the UNION to the Company.” Petitioners contend that their acts do not constitute disloyalty as these are in the exercise of their constitutional right to self-organization. PAFLU President then rendered a decision finding them guilty of the charges and expelling them from the Union and as a consequence the Management of the employer is hereby requested to terminate them from their employment in conformity with the security clause. Issues: WON PAFLU had the authority to investigate the petitioners and, thereafter, expel them from the roll of membership of the Amigo Employees Union-PAFLU SC: YES. That PAFLU had the authority to investigate petitioners on the charges filed by their co-employees in the local union and after finding them guilty as charged, to expel them from the roll of membership of the Amigo Employees Union-PAFLU is clear under the constitution of the PAFLU to which the local union was affiliated. And pursuant to the security clause of the new CBA, reiterating the same clause in the old CBA, PAFLU was justified in applying said security clause. Inherent in every labor union, or any organization for that matter, is the right of self-preservation. When members of a labor union, therefore, sow the seeds of dissension and strife within the union; when they seek the disintegration and destruction of the very union to which they belong, they thereby forfeit their rights to remain as members of the union which they seek to destroy. Prudence and equity, as well as the dictates of law and justice, therefore, compelling mandate the adoption by the labor union of such corrective and remedial measures in keeping with its laws and regulations, for its preservation and continued existence; lest by its folly and inaction, the labor union crumble and fall. Issue: WON the petitioners were entitled to disaffiliate from the Union. SC: Yes. Although they are entitled to disaffiliate from their union and form a new organization of their own, they must, however, suffer the consequences of their separation from the union under the security clause of the CBA. Disaffiliation from a labor union is not open to legal objection. It is implicit in the freedom of association 48 | P a g e

ordained by the Constitution. But this Court has laid down the ruling that a closed shop is a valid form of union security, and such provision in a collective bargaining agreement is not a restriction of the right of freedom of association guaranteed by the Constitution. Tropical Hut Employees Union v. Tropical Hut Food Market, Inc., 181 SCRA 173 (90) FACTS: Nov 1993, the Philippine Skylanders Employees Association (PSEA), a local labor union affiliated with the Philippine Association of Free Labor Unions (PAFLU) September (PAFLU), won in the certification election conducted among the rank and file employees of Philippine Skylanders, Inc. (PSI). Its rival union, Philippine Skylanders Employees Association-WATU (PSEA-WATU) immediately protested the result of the election before the Secretary of Labor. Several months later, pending settlement of the controversy, PSEA sent PAFLU a notice of disaffiliation citing as reason PAFLU's supposed deliberate and habitual dereliction of duty toward its members. PSEA subsequently affiliated itself with the National Congress of Workers (NCW), changed its name to Philippine Skylanders Employees Association - National Congress of Workers (PSEA-NCW), and allowed the former officers of PSEA-PAFLU to continue occupying their positions as elected officers PSEA-NCW. On 17 March 1994 PSEA-NCW entered into a collective bargaining agreement with PSI which was immediately registered with DOLE. PAFLU Secretary General Serafin Ayroso wrote Mariles C. Romulo requesting a copy of PSI's audited financial statement. Ayroso explained that with the dismissal of PSEA-WATU's election protest the time was ripe for the parties to enter into a collective bargaining agreement. PSI through its personnel manager Francisco Dakila denied the request citing as reason PSEA's disaffiliation from PAFLU and its subsequent affiliation with NCW. PAFLU through Serafin Ayroso filed a complaint for unfair labor practice against PSI, its president Mariles Romulo and personnel manager Francisco Dakila. PAFLU amended its complaint by including the elected officers of PSEA-PAFLU as additional party respondents. Labor Arbiter declared PSEA's disaffiliation from PAFLU invalid and held PSI, PSEA-PAFLU and their respective officers guilty of unfair labor practice. NLRC upheld the Decision of the Labor Arbiter and conjectured that since an election protest questioning PSEA-PAFLU's certification as the sole and exclusive bargaining agent was pending resolution before the Secretary of Labor, PSEA could not validly separate from PAFLU, join another national federation and subsequently enter into a collective bargaining agreement with its employer-company ISSUE/S: WON PSEA, which is an independent and separate local union, may validly disaffiliate from PAFLU pending the settlement of an election protest questioning its status as the sole and exclusive bargaining agent of PSI's rank and file employees HELD YES. The pendency of an election protest involving both the mother federation and the local union did not constitute a bar to a valid disaffiliation. Reasoning In Liberty Cotton Mills Workers Union vs. Liberty Cotton Mills, Inc. the SC upheld the right of local unions to separate from their mother federation on the ground that as separate and voluntary associations, local unions do not owe their creation and existence to the national federation to which they are affiliated but, instead, to the will of their members. The sole essence of affiliation is to increase, by collective action, the common bargaining power of local unions for the effective enhancement and protection of their interests. Yet the local unions remain the basic units of association, free to serve their own interests subject to the restraints imposed by the constitution and by-laws of the national federation, and free also to renounce the affiliation upon the terms laid down in the agreement which brought such affiliation into existence. Policy considerations dictate that in weighing the claims of a local union as against those of a national federation, those of the former must be preferred. Parenthetically though, the desires of the mother federation to protect its locals are not altogether to be shunned. It will however be to err greatly against the Constitution if the desires of the federation would be favored over those of its members. If it were otherwise, instead of protection, there would be disregard and neglect of the lowly workingmen. Disposition Petition is granted. Alex Ferrer v. NLRC, 224 SCRA 410 (93) Facts: • Petitioners are piece workers employed in Occidental Foundry Corporation (OFC). They were also members of Samahang Manggagawa ng Occidental Foundry Corporation – FFW (SAMAHAN). • SAMAHAN and OFC entered into a collective bargaining agreement which provided for a union security clause that requires employees of OFC to remain as members of SAMAHAN for the duration of their employment with OFC. Failure to do so will be a ground for termination. • Alex Ferrer and SAMAHAN filed with the DOLE a complaint for expulsion of Genaro Capitle, then President of SAMAHAN and other officers thereof on the ground of the alleged inattentiveness of the said officers to the economic demands of the workers. • Petitioners then conducted a special election of SAMAHAN’s officers. Such special election was opposed by FFW. Petitioners also tried to organize a strike but were disowned by the members of SAMAHAN. • On September 11, 1989, a union resolution was issued by Capitle and other officers of SAMAHAN expelling petitioners from the union. The following day, they wrote OFC, through its management, requesting for the dismissal of the petitioners. • On the date OFC received the letter, the petitioners were dismissed from OFC. • As such, they volunteered to be admitted as members of FEDLU and sought its representation before the DOLE questioning their illegal dismissal. • The Labor Arbiter dismissed their complaint ruling that their dismissal was validly done pursuant to the closed shop provision in the CBA. The NLRC affirmed. 49 | P a g e

Issues: 1. Whether or not the dismissal of the petitioners pursuant to the closed shop provision of the CBA was valid. 2. Whether or not the petitioners’ act of seeking assistance from FEDLU is an act of disloyalty against the union. Held: 1st Issue: NO • A CBA is the law between the company and the union and compliance therewith is mandated by the express policy to give protection to labor. • A CBA provision for a closed shop is a valid form of union security and is not a restriction on the right or freedom of association guaranteed by the Constitution. However, in implementation of the provisions of the CBA, both parties should see to it that no right is violated or impaired. • While the CBA between OFC and SAMAHAN provided for a right to dismiss an employee pursuant to a closed shop provision, such right must be subjected to the demands of procedural due process. • In the Constitution and By-laws of the SAMAHAN, a procedure for temporary or permanent expulsion was provided. In the said procedure, the conduct of a hearing was necessary for expulsion to be effective. No hearing was conducted by it when it caused the expulsion of Capitle and his supporters. • The need for a company investigation is founded on the consistent ruling of the SC of the twin requirements of notice and hearing which are the essential elements of due process in termination cases. 2nd issue: NO • While petitioners’ act of holding a special election to oust Capitle et al. may be considered as an act of sowing disunity among the SAMAHAN members and perhaps to its officers which may be the subject of a disciplinary matter, it cannot be construed as an act of disloyalty against the union. Faced with a SAMAHAN leadership which they tried to remove as officials, it was but a natural act of self-preservation to seek the assistance of FEDLU. • Neither may it be inferred that the petitioners sought disaffiliation for petitioners had not formed a union distinct from SAMAHAN. • In any case, the right of a local union to disaffiliate from a federation in the absence of any provision in the federation’s constitution preventing disaffiliation of a local union is legal. Such right is consistent with the constitutional guarantee of freedom of association. Philippine Skylanders, Inc. v. NLRC, 375 SCRA 369 (02) Philippine Skylanders Employees Association (PSEA), a local labor union affiliated with the Philippine Association of Free Labor Unions (PAFLU) September (PAFLU), won in the certification election conducted among the rank and file employees of Philippine Skylanders, Inc. (PSI). Its rival union, Philippine Skylanders Employees Association-WATU (PSEA-WATU) immediately protested the result of the election before the Secretary of Labor. Several months later, pending settlement of the controversy, PSEA sent PAFLU a notice of disaffiliation citing as reason PAFLU's supposed deliberate and habitual dereliction of duty toward its members. Attached to the notice was a copy of the resolution adopted and signed by the officers and members of PSEA authorizing their local union to disaffiliate from its mother federation. PSEA subsequently affiliated itself with the National Congress of Workers (NCW), changed its name to Philippine Skylanders Employees Association - National Congress of Workers (PSEA-NCW), and to maintain continuity within the organization, allowed the former officers of PSEA-PAFLU to continue occupying their positions as elected officers in the newly-forged PSEA-NCW. PSEA-NCW entered into a collective bargaining agreement with PSI which was immediately registered with the Department of Labor and Employment. PSI through its personnel manager Francisco Dakila denied the request citing as reason PSEA's disaffiliation from PAFLU and its subsequent affiliation with NCW. Agitated by PSI's recognition of PSEA-NCW, PAFLU through Serafin Ayroso filed a complaint for unfair labor practice against PSI. PAFLU alleged that aside from PSI’s refusal to bargain collectively with its workers, the company through its president and personnel manager, was also liable for interfering with its employees' union activities. ISSUE: The fundamental issue tapers down to the legitimacy of PSEA's disaffiliation: May PSEA, which is an independent and separate local union, validly disaffiliate from PAFLU pending the settlement of an election protest questioning its status as the sole and exclusive bargaining agent of PSI's rank and file employees? HELD: YES. At the outset, let it be noted that the issue of disaffiliation is an inter-union conflict the jurisdiction of which properly lies with the Bureau of Labor Relations (BLR) and not with the Labor Arbiter. Nonetheless, with due recognition of this fact, we deem it proper to settle the controversy at this instance since to remand the case to the BLR would only mean intolerable delay for the parties. The right of a local union to disaffiliate from its mother federation is not a novel thesis unillumined by case law. In the landmark case of Liberty Cotton Mills Workers Union vs. Liberty Cotton Mills, Inc. [16] we upheld the right of local unions to separate from their mother federation on the ground that as separate and voluntary associations, local unions do not owe their creation and existence to the national federation to which they are affiliated but, instead, to the will of their members. The sole essence of affiliation is to increase, by collective action, the common bargaining power of local unions for the effective enhancement and protection of their interests. Admittedly, there are times when without succor and support local unions may find it hard, unaided by other support groups, to secure justice for themselves. Yet the local unions remain the basic units of association, free to serve their own interests subject to the restraints imposed by the constitution and by-laws of the national federation, and free also to renounce the affiliation upon the terms laid down in the agreement which brought such affiliation into existence.There is nothing 50 | P a g e

shown in the records nor is it claimed by PAFLU that the local union was expressly forbidden to disaffiliate from the federation nor were there any conditions imposed for a valid breakaway. As such, the pendency of an election protest involving both the mother federation and the local union did not constitute a bar to a valid disaffiliation. The mere act of disaffiliation did not divest PSEA of its own personality; neither did it give PAFLU the license to act independently of the local union. Recreant to its mission, PAFLU cannot simply ignore the demands of the local chapter and decide for its welfare. PAFLU might have forgotten that as an agent it could only act in representation of and in accordance with the interests of the local union. VI. UNION SECURITY A. STATUTORY BASIS – 248 (e) Art. 248. Unfair labor practices of employers. It shall be unlawful for an employer to commit any of the following unfair labor practice: e. To discriminate in regard to wages, hours of work and other terms and conditions of employment in order to encourage or discourage membership in any labor organization. Nothing in this Code or in any other law shall stop the parties from requiring membership in a recognized collective bargaining agent as a condition for employment, except those employees who are already members of another union at the time of the signing of the collective bargaining agreement. Employees of an appropriate bargaining unit who are not members of the recognized collective bargaining agent may be assessed a reasonable fee equivalent to the dues and other fees paid by members of the recognized collective bargaining agent, if such non-union members accept the benefits under the collective bargaining agreement: Provided, that the individual authorization required under Article 242, paragraph (o) of this Code shall not apply to the non-members of the recognized collective bargaining agent; B. RATIONALE – 211 (c) Art. 211. Declaration of Policy. A. It is the policy of the State c. To foster the free and voluntary organization of a strong and united labor movement; Alabang Country Club, Inc. v. NLRC, 545 SCRA 351 (08) Facts: • Alabang Country Club, Inc. (Club) is a non-profit corporation while Alabang Country Club Independent Employees Union (Union) is the exclusive bargaining agent of the Club’s rank and file employees. • In 1996, Pizarro, Braza, and Castueras were elected as President, Vice-President and Treasurer of the Union respectively. • On June 21, 1999, the Club and Union entered into a CBA providing for a closed shop provision. Further, the CBA provided that the Union may request the termination of a rank and file employee for certain grounds, among which is malversation of union funds. • In July 2001, an election was held a new set of officers elected. An audit of the union funds was conducted and several unliquidated expenses, discrepancies and irregularities were discovered. Pizarro, Braza, and Castueras were notified of the results and were asked to explain the same. • Despite their explanations, the Union decided to expel for malversation of funds. They furnished them with a document calling for their expulsion signed by 37 of the 63 members of the union as well as a resolution of the Board of Directors effecting the expulsion. • The Union then requested the Club to terminate the said employees due to their expulsion as provided for by the union security clause of the CBA. The Club for its part called Pizarro, Braza, and Castueras to an informal conference to allow them to explain their side. However, despite their verbal and written explanation, they were terminated by the Club. • Consequently, they filed a case before the Labor Arbiter assailing their dismissal from the Club as illegal. The LA dismissed the complaint but was reversed by the NLRC. The CA affirmed the NLRC. Issues: 3. Whether or not the dismissal of Pizarro, Braza, and Castueras was justified under the circumstances. Held: 1st Issue: YES • Enforcement of a union security clause in the CBA is another valid ground for termination of employment. This practice strengthens the union and prevents disunity in the bargaining unit within the duration of the CBA with the threat of expulsion from the union and consequent termination of employment, the union gains more members and strengthens its position against other unions who may wish to claim majority representation. • In this case, the CBA provided for a union shop and maintenance of membership shop. There is union shop when all new regular employees are required to join the union within a certain period as a condition for their continued employment. There is maintenance of membership shop when employees who are union members as of the effective date of the agreement, or who thereafter become members, must maintain union membership as a condition for continued employment until they are promoted or transferred out of the bargaining unit or the agreement is terminated.

51 | P a g e



In terminating the employment of an employee by enforcing the union security clause, the employer needs only to determine and prove that: (1) the union security clause is applicable; (2) the union is requesting for the enforcement of the union security provision in the CBA; and (3) there is sufficient evidence to support the union’s decision to expel the employee from the union. These requisites constitute just cause for terminating an employee based on the CBA’s union security provision. Per the CBA, malversation of union funds is a valid cause of termination of employment pursuant to the union security clause contained therein. In this case, the respondents were expelled from the union following the acts of dishonesty and malversation of union funds by them as found by the investigation committed by the Union. When the Union, wrote to the Club to terminate the respondents from employment, the Club conducted its own investigation and afforded the respondents an opportunity to explain their side and to controvert the allegations imputed against them by the Union. The Supreme Court held that the investigation conducted by the Club satisfied the standard of procedural due process required by law for dismissal cases. Caltex Refining Employees Association v. Ballontoc Guijarno v. CIR, 52 SCRA 307 (73) Facts: • Petitioners filed a case against Central Santos Lopez Co., Inc. (Company) and United Sugar Workers Union-ILO (Union) questioning their allegedly illegal dismissal pursuant to a closed shop provision provided in the existing CBA. • Petitioners were those expelled from the Union and the Company assumed that it had to dismiss them pursuant to the closed shop provision in the CBA upon the instance of the Union. • The Company argues in its defense that it did not commit an unfair labor practice for the dismissal was sought upon the instance of the Union from which the petitioners were expelled; that it merely complied with the terms of the CBA and that a closed shop agreement is allowed under the law. Issue: Whether or not the dismissal of the petitioners was valid. Held: NO • A closed shop provision in a CBA must not be given a retroactive effect so as to preclude its being applied to employees in the service. Such was the ruling in the case of Confederated Sons of Labor v. Anakan Lumber Co., et al. (107 Phil. 915). • As was held in that case, an undertaking of this nature is so harsh that it must be strictly construed and doubts must be resolved against the existence of a ‘closed shop’. • A closed shop agreement that is allowed under the Industrial Peace Act should apply to persons to be hired or to employees who are not yet members of any labor organization. • It is inapplicable to those already in the service who are members of another union. To hold otherwise would render minority unions at a grave disadvantage and force their members to disaffiliate with them to join the majority union and render nugatory the right of employees to self-organization. • Further, it is well settled that a closed shop provision may not justify the termination by the employer of an employee who is arbitrarily refused membership to the union without any reasonable ground. • The Supreme Court emphasized that such provisions are looked upon with great disfavor whenever they are being utilized as an excuse for termination of employment. The very rationale of a closed shop provision such as this is to strengthen the effectivity labor organizations. However, such provision may be used as an instrumentality by the many to stifle the voices of the few in dissent. This is the precise reason as to why the doctrine in Anakan was formulated. C. VALIDITY AGREEMENT AND EFFECT ON FREEDOM OF CHOICE Tanduay Distillery Labor Union v. NLRC, 149 SCRA 470 (87) Facts: Employees and members of the Tanduay Distillery Labor Union TDLU, the exclusive bargaining agent of the rank and file employees of the Company. CBA has union security clause. All workers who are or may during the effectivity of this contract become members of the union in accordance with its constitution and by-laws shall as a condition of their continued employment, maintain membership in good standing in the union for the duration of the agreement . Santos et al joined another union, KAMPIL and organized its local chapter in TDI. KAMPIL filed a petition for CE, which development compelled TDI to file a grievance with TDLU. Santos et al to explain why TDLU should not take disciplinary action against them for, among other things. TDLU created a committee to investigate its erring members. Despite due notice, Santos et al were absent during the investigation. TDLU's Board of Directors, expelled Santos et al for disloyalty to the Union. TDLU notified company that private respondents had been expelled from TDLU and demanded that TDI terminate the employment. TDI filed with the MOLE its application for clearance to terminate the employment of Santos et al.

52 | P a g e

Meanwhile, MA granted the KAMPIL’s petition calling for a certification election among the rank and file employees of TDI. The existence of an uncertified CBA cannot be availed of as a bar to the holding of a certification election . BLR reversed the order for the holding of a CE. Contention of TDLU: Uncertified CBA was duly filed and submitted on 29 October 1980, to last until June 30, 1982. Indeed, said CBA is certifiable for having complied with all the necessary requirements for certification. Consistent with the intent and spirit of P.D. 1391 and its implementing rules, the contract bar rule should have been applied in this case. The representation issue cannot be entertained except within the last sixty (60) days of the collective agreement. Contention of KAMPIL: The CBA in question is not certifiable and, hence, the contract bar rule cannot properly apply in this case. BLR: It should be noted that the alleged CBA has now expired. Its expiry date being 30 June 1982. Consequently; there appears to be no more obstacle in allowing a certification election to be conducted among the rank and file of respondent. The contract bar rule will no longer apply in view of the supervening event, that is, the expiration of the contract. LA rendered a decision denying TDI's application to terminate Santos et al. NLRC affirmed. MR by TDI and TDLU: NLRC failed to consider the fact that at the time the Santos et al were expelled by TDLU and consequently terminated by TDI, the union security clause of the CBA was in full force and effect, binding TDI and TDLU. Issues: (a) WON TDI was justified in terminating Santos et al on the basis of TDLU's demand for the enforcement of the Union Security Clause of the CBA between TDI and TDLU; and (b) WON TDI is guilty of ULP in complying with TDLU's demand for the dismissal of private respondents. SC: The closed shop, the union shop, the maintenance of membership shop, the preferential shop, the maintenance of treasury shop, and check-off provisions are valid forms of union security and strength. They do not constitute ULP nor are they violations of the freedom of association clause of the Constitution. PP: principle of sanctity and inviolability of contracts guaranteed by the Constitution. Having ratified that CBA and being then members of the TDLU, the Santos et al owe fealty and are required under the Union Security Clause to maintain their membership in good standing with it during the term thereof, a requirement which ceases to be binding only during the 60-day freedom period immediately preceding the expiration of the CBA. When the private respondents organized and joined the KAMPIL Chapter in TDI and filed the corresponding petition for certification election in November 1980, there was no freedom period to speak of yet. For under Presidential Decree No. 1391, promulgated May 29, 1978, contract bar rule is still applicable. No certification election shall be entertained if a Collective Bargaining Agreement which has been submitted in accordance with Article 231 of the Code exists between the employer and a legitimate labor organization except within sixty (60) days prior to the expiration of the life of such collective agreement . The validity of the CBA is not here assailed by Santos et al. They admitted having organized the local chapter of KAMPIL at TDI, although it is claimed that this was done when there was no certified CBA between TDI and TDLU that would constitute a bar to the certification election. While the BLR ordered the holding of a certification election because the CBA in question had already expired, its expiry date being June 30, 1982. Consequently, there appears to be no more obstacle in allowing a certification election. "... [T]he contract bar rule will not apply in view of the supervening event, that is, the expiration of the CBA, the ineluctable fact is that private respondents committed acts of disloyalty against TDLU while the CBA was in force and existing for which they have to face the necessary sanctions lawfully imposed by TDLU. The private respondents cannot, therefore, escape the effects of the security clause of their own applicable collective bargaining agreement. Alabang Country Club, Inc. v. NLRC, 545 SCRA 351 (08) Facts: • Alabang Country Club, Inc. (Club) is a non-profit corporation while Alabang Country Club Independent Employees Union (Union) is the exclusive bargaining agent of the Club’s rank and file employees. • In 1996, Pizarro, Braza, and Castueras were elected as President, Vice-President and Treasurer of the Union respectively. • On June 21, 1999, the Club and Union entered into a CBA providing for a closed shop provision. Further, the CBA provided that the Union may request the termination of a rank and file employee for certain grounds, among which is malversation of union funds. • In July 2001, an election was held a new set of officers elected. An audit of the union funds was conducted and several unliquidated expenses, discrepancies and irregularities were discovered. Pizarro, Braza, and Castueras were notified of the results and were asked to explain the same. • Despite their explanations, the Union decided to expel for malversation of funds. They furnished them with a document calling for their expulsion signed by 37 of the 63 members of the union as well as a resolution of the Board of Directors effecting the expulsion. • The Union then requested the Club to terminate the said employees due to their expulsion as provided for by the union security clause of the CBA. The Club for its part called Pizarro, Braza, and Castueras to an informal conference to allow them to explain their side. However, despite their verbal and written explanation, they were terminated by the Club. • Consequently, they filed a case before the Labor Arbiter assailing their dismissal from the Club as illegal. The LA dismissed the complaint but was reversed by the NLRC. The CA affirmed the NLRC. Issues: 4. Whether or not the dismissal of Pizarro, Braza, and Castueras was justified under the circumstances. Held: 53 | P a g e

1st Issue: YES • Enforcement of a union security clause in the CBA is another valid ground for termination of employment. This practice strengthens the union and prevents disunity in the bargaining unit within the duration of the CBA with the threat of expulsion from the union and consequent termination of employment, the union gains more members and strengthens its position against other unions who may wish to claim majority representation. • In this case, the CBA provided for a union shop and maintenance of membership shop. There is union shop when all new regular employees are required to join the union within a certain period as a condition for their continued employment. There is maintenance of membership shop when employees who are union members as of the effective date of the agreement, or who thereafter become members, must maintain union membership as a condition for continued employment until they are promoted or transferred out of the bargaining unit or the agreement is terminated. • In terminating the employment of an employee by enforcing the union security clause, the employer needs only to determine and prove that: (1) the union security clause is applicable; (2) the union is requesting for the enforcement of the union security provision in the CBA; and (3) there is sufficient evidence to support the union’s decision to expel the employee from the union. These requisites constitute just cause for terminating an employee based on the CBA’s union security provision. Per the CBA, malversation of union funds is a valid cause of termination of employment pursuant to the union security clause contained therein. In this case, the respondents were expelled from the union following the acts of dishonesty and malversation of union funds by them as found by the investigation committed by the Union. When the Union, wrote to the Club to terminate the respondents from employment, the Club conducted its own investigation and afforded the respondents an opportunity to explain their side and to controvert the allegations imputed against them by the Union. The Supreme Court held that the investigation conducted by the Club satisfied the standard of procedural due process required by law for dismissal cases. D. TYPES OF UNION SECURITY PROVISIONS Alabang Country Club, Inc. v. NLRC, 545 SCRA 351 (08) Facts: • Alabang Country Club, Inc. (Club) is a non-profit corporation while Alabang Country Club Independent Employees Union (Union) is the exclusive bargaining agent of the Club’s rank and file employees. • In 1996, Pizarro, Braza, and Castueras were elected as President, Vice-President and Treasurer of the Union respectively. • On June 21, 1999, the Club and Union entered into a CBA providing for a closed shop provision. Further, the CBA provided that the Union may request the termination of a rank and file employee for certain grounds, among which is malversation of union funds. • In July 2001, an election was held a new set of officers elected. An audit of the union funds was conducted and several unliquidated expenses, discrepancies and irregularities were discovered. Pizarro, Braza, and Castueras were notified of the results and were asked to explain the same. • Despite their explanations, the Union decided to expel for malversation of funds. They furnished them with a document calling for their expulsion signed by 37 of the 63 members of the union as well as a resolution of the Board of Directors effecting the expulsion. • The Union then requested the Club to terminate the said employees due to their expulsion as provided for by the union security clause of the CBA. The Club for its part called Pizarro, Braza, and Castueras to an informal conference to allow them to explain their side. However, despite their verbal and written explanation, they were terminated by the Club. • Consequently, they filed a case before the Labor Arbiter assailing their dismissal from the Club as illegal. The LA dismissed the complaint but was reversed by the NLRC. The CA affirmed the NLRC. Issues: 5. Whether or not the dismissal of Pizarro, Braza, and Castueras was justified under the circumstances. Held: 1st Issue: YES • Enforcement of a union security clause in the CBA is another valid ground for termination of employment. This practice strengthens the union and prevents disunity in the bargaining unit within the duration of the CBA with the threat of expulsion from the union and consequent termination of employment, the union gains more members and strengthens its position against other unions who may wish to claim majority representation. • In this case, the CBA provided for a union shop and maintenance of membership shop. There is union shop when all new regular employees are required to join the union within a certain period as a condition for their continued employment. There is maintenance of membership shop when employees who are union members as of the effective date of the agreement, or who thereafter become members, must maintain union membership as a condition for continued employment until they are promoted or transferred out of the bargaining unit or the agreement is terminated. • In terminating the employment of an employee by enforcing the union security clause, the employer needs only to determine and prove that: (1) the union security clause is applicable; (2) the union is requesting for the enforcement of the union security provision in the CBA; and (3) there is sufficient evidence to support the union’s decision to expel the employee from the union. These requisites constitute just cause for terminating an employee based on the CBA’s union security provision. 54 | P a g e

Per the CBA, malversation of union funds is a valid cause of termination of employment pursuant to the union security clause contained therein. In this case, the respondents were expelled from the union following the acts of dishonesty and malversation of union funds by them as found by the investigation committed by the Union. When the Union, wrote to the Club to terminate the respondents from employment, the Club conducted its own investigation and afforded the respondents an opportunity to explain their side and to controvert the allegations imputed against them by the Union. The Supreme Court held that the investigation conducted by the Club satisfied the standard of procedural due process required by law for dismissal cases. National Union of Workers in Hotels, etc. v. NLRC, 567 SCRA 291 (08) Facts: Hotel entered into a CBA with UNION, the exclusive bargaining agent of the rank-and-file employees of the Hotel. During the 60-day freedom period which preceded the expiration of the CBA, the Hotel and Union negotiated the extension of the provisions of the existing CBA. Another Union, NUWHRAIN was accorded the status of a legitimate labor organization. Thereafter, NUWHRAIN exercised the right to challenge the majority status of the incumbent union, by filing a pertition for CE. After the lapse of the 60-day freedom period, but pending the disposition of the Petition for CE filed by NUWHRAIN, the Union served the Hotel with a written demand for the dismissal of 36 employees for alleged acts of disloyalty. Hotel issued Disciplinary Action Notices, which directed the 36 employees to submit a written explanation for their alleged acts of disloyalty and violation of the union security clause for which Union sought their dismissal. During the reconciliatory conference, Hotel’s Vice President, Norma Azores, stated her preference to deal with HIMPHLU, while blaming NUWHRAIN for the labor problems of the Hotel. NLRC pronounced that the Hotel was not guilty of ULP. Hotel had not acted improperly as it did not wrongfully terminate any of the 36 employees. The NLRC concluded that these statements did not constitute ULP for they could not have coerced or influenced either of the contending unions, both of whom did not agree in the suggested course of action or to any other manner of settling the dispute. NUWHRAIN filed a Motion for Reconsideration of the foregoing NLRC Resolution. It was denied by the NLRC in another Resolution dated 30 June 2006. CA: It declared that the Hotel had acted prudently when it issued the Notices to the 36 employees after HIMPHLU demanded their dismissal. It clarified that these Notices did not amount to the termination of the employees concerned but merely sought their explanation on why the union security clause should not be applied to them. NUWHRAIN’s Motion for Reconsideration of the aforementioned Decision of the Court of Appeals was denied. Certification Election was held, which HIMPHLU won. Issue: WON Hotel’s action/response to NUWHRAIN’s request proper. SC: Yes. "Union security" is a generic term which is applied to and comprehends "closed shop," "union shop," "maintenance of membership" or any other form of agreement which imposes upon employees the obligation to acquire or retain union membership as a condition affecting employment. 25 Article 248(e) of the Labor Code recognizes the effectivity of a union shop clause. By thus promoting unionism, workers are able to negotiate with management on an even playing field and with more persuasiveness than if they were to individually and separately bargain with the employer. In the present case, the Collective Bargaining Agreement includes a union security provision. 28 To avoid the clear possibility of liability for breaching the union security clause of the Collective Bargaining Agreement and to protect its own interests, the only sensible option left to the Hotel, upon its receipt of the demand of HIMPHLU for the dismissal of the 36 employees, was to conduct its own inquiry so as to make its own findings on whether there was sufficient ground to dismiss the said employees who defected from HIMPHLU. The issuance by the respondent of the Notices requiring the 36 employees to submit their explanations to the charges against them was the reasonable and logical first step in a fair investigation. It is important to note that the Hotel did not take further steps to terminate the 36 employees. Instead, it arranged for reconciliatory conferences between the contending unions in order to avert the possibility of dismissing the 36 employees for violation of the union security clause of the Collective Bargaining Agreement. General Rule: The dismissal of an employee by the company pursuant to a labor union’s demand in accordance with a union security agreement does not constitute unfair labor practice. An employer is not considered guilty of unfair labor practice if it merely complied in good faith with the request of the certified union for the dismissal of employees expelled from the union pursuant to the union security clause in the Collective Bargaining Agreement . In the case at bar, there is even less possibility of sustaining a finding of guilt for unfair labor practice where Hotel did not dismiss the 36 employees, despite the insistence of HIMPHLU, the sole bargaining agent for the rank and file employees of the Hotel, on the basis of the union security clause of the Collective Bargaining Agreement. The only act attributed to the Hotel is its issuance of the Notices which, contrary to being an unfair labor practice, even afforded the employees involved a chance to be heard. Still, NUWHRAIN asserts that the sworn testimony signed by its six union members that the officers of the respondent and the Hotel did utter the offending statements deserve more credence than the unsworn denial of respondent. NUWHRAIN has the burden of proving its allegation that Norma Azores did make the statements being attributed to them. The burden of proof rests upon the party who asserts the affirmative of an issue. And in labor cases, the quantum of proof necessary is substantial evidence, or such amount of relevant evidence which a reasonable mind might accept as adequate to justify a conclusion, which NUWHRAIN failed to discharge in the present case.

55 | P a g e

In the case at bar, the NLRC found, and the Court of Appeals affirmed, that the officers of the respondent and the Hotel did not make statements that would have constituted unfair labor practice . E. CONTRACT DRAFTING AND INTERPRETATION OF PROVISIONS – UNION SECURITY Rizal Labor Union v. Rizal Cement Co., Inc, 17 SCRA 858 (55) FACTS: • Santos and 14 other employees of respondent company, whole still members of Binangonan Labor Union, formed and organized Rizal Labor Union. • Thereafter, Carlos Santos and Teofines Minguillan, president and secretary, of the newly-organized union, received letters from the Binangonan Labor Union, requiring them to explain why they should not be expelled for disloyalty. • Although Santos and Minguillan requested for the convocation of a general meeting of the members of the Binangonan Labor Union to explain their side, the organizers of RLU were expelled from BLU and were thereafter dismissed from employment. • The dismissed employees filed a complain for unfair labor practices against the company and BLU. • The respondent denied the allegation, stating that there exists a closed-shop proviso in their CBA. • After the trial, the lower court rendered a decision, stating that the supposed closed-shop proviso, while valid, was inadequate to justify the dismissal of complainants from employment. • The lower court's decision was reversed by the court en banc, finding that the dismissal of complainants was found to be justified by the closed shop proviso of the collective bargaining agreement. ISSUE: WON the dismissal of the complaining employees was justified. HELD: No. Respondent Union and Company are guilty of ULP. The resolution of the question hinges on the validity of the supposed closed-shop proviso of the CBA between the respondent company and petitioner union. The two provisions are: The EMPLOYER agrees in all its branches, units, plants, quarries, warehouses, docks, etc. The UNION agrees to furnish at all time the laborers, employees and all technical helps that the EMPLOYER may require. EMPLOYER, however, reserves its right to accept or reject where they fail to meet its requirements. (Article 1, Sec. 5.) The EMPLOYER agrees not to have in its employ nor to hire any new employee or laborer unless he is a member of good standing of the UNION, and a bona fide holder of a UNION (NWB) card, provided such new employee or laborer meets the qualifications required by the EMPLOYER. (Article VII, Sec. 1-d). The first provision should be construed as applicable to those already on the job at the time the agreement was entered into in 1954, while the second provision applicable to those getting employment thereafter. However, such aforequoted pertinent provision of the collective bargaining agreement does not prescribe the period within which the employees must remain as members of good standing of the union, and therefore the dismissal of the complainants after they were expelled from the union was unjustified. The union security clause in the CBA merely provided for a limited closed shop which did not justify the dismissal. Manila Cordage Co. v. CIR, 78 SCRA 398 (77) Facts: • Manila Cordage Company (MCC) is a corporation duly organized and existing under Philippine law. Manila Cordage Workers Union (MCWU) and Manco Labor Union (Manco) are legitimate labor organizations. Both unions are composed of employees of MCC. • In 1957, Manco Labor Union, acting as exclusive bargaining representative of the employees of MCC, executed a collective bargaining agreement (CBA) with MCC which provided a Maintenance of Membership clause that provided: “Both parties agree that all employees of the COMPANY who are already members of the UNION at the time of the signing of this AGREEMENT shall continue to remain members of the UNION for the duration of this AGREEMENT”. • In 1959, during the effectivity of the CBA, several members of Manco resigned from membership in the union to form MCWU. Invoking the CBA, Manco requested MCC to dismiss from its employ the employees who resigned from Manco invoking the Maintenance of Membership clause in the CBA. MCC complied. • In 1961, MCWU filed a case for unfair labor practice before the Court of Industrial Relations (CIR) against MCC and Manco. • CIR upheld MCWU adjudging MCC and Manco as guilty of unfair labor practices. It also declared the reinstatement of the employees who were dismissed at the instance of Manco. Issue: Whether or not MCC was justified in dismissing the employees who resigned from Manco pursuant to the Maintenance of Membership clause found in the CBA. Held: NO. • The Maintenance of Membership clause in the CBA does not clearly state that maintenance of membership in Manco is a condition for continuous employment in MCC. • As held in Confederated Sons of Labor v. Anakan Lumber Co., et al. (107 Phil. 915), a stipulation to this effect must be so clear as to leave no room for doubt thereon. • An undertaking of this nature is so harsh that it must be strictly construed and doubts must be resolved against the existence of the right to dismiss. 56 | P a g e



Neither does a reading of the provision in connection with the Whereas clause of the CBA justify the existence of a right to dismiss. The said Whereas clause provides: “WHEREAS, the parties hereto have decided to enter into an agreement relating to the terms and conditions of employment and reference to those employees to whom the provisions of this AGREEMENT apply.” • The Whereas clause neither refers to tenure or duration of employment but only to terms and conditions of employment such as working hours, wages, other benefits and privileges clearly specified therein. Even assuming the said Whereas clause includes tenure or duration of employment, its ambiguity should not adversely affect the complainants. To construe the foregoing provisions as imposing as a condition to continued employment in MCC, the maintenance of membership in Manco is to violate the natural and constitutional right of the laborer to organize freely. Such interpretation is inconsistent to the constitutional mandate that the State shall afford protection to labor. F. COVERAGE – WORKER INCLUSION AND EXCLUSION, 248 (e) Art. 248. Unfair labor practices of employers. It shall be unlawful for an employer to commit any of the following unfair labor practice: e. To discriminate in regard to wages, hours of work and other terms and conditions of employment in order to encourage or discourage membership in any labor organization. Nothing in this Code or in any other law shall stop the parties from requiring membership in a recognized collective bargaining agent as a condition for employment, except those employees who are already members of another union at the time of the signing of the collective bargaining agreement. Employees of an appropriate bargaining unit who are not members of the recognized collective bargaining agent may be assessed a reasonable fee equivalent to the dues and other fees paid by members of the recognized collective bargaining agent, if such non-union members accept the benefits under the collective bargaining agreement: Provided, that the individual authorization required under Article 242, paragraph (o) of this Code shall not apply to the non-members of the recognized collective bargaining agent; Guijarno v. CIR, 52 SCRA 307 (73) Facts: • Petitioners filed a case against Central Santos Lopez Co., Inc. (Company) and United Sugar Workers Union-ILO (Union) questioning their allegedly illegal dismissal pursuant to a closed shop provision provided in the existing CBA. • Petitioners were those expelled from the Union and the Company assumed that it had to dismiss them pursuant to the closed shop provision in the CBA upon the instance of the Union. • The Company argues in its defense that it did not commit an unfair labor practice for the dismissal was sought upon the instance of the Union from which the petitioners were expelled; that it merely complied with the terms of the CBA and that a closed shop agreement is allowed under the law. Issue: Whether or not the dismissal of the petitioners was valid. Held: NO • A closed shop provision in a CBA must not be given a retroactive effect so as to preclude its being applied to employees in the service. Such was the ruling in the case of Confederated Sons of Labor v. Anakan Lumber Co., et al. (107 Phil. 915). • As was held in that case, an undertaking of this nature is so harsh that it must be strictly construed and doubts must be resolved against the existence of a ‘closed shop’. • A closed shop agreement that is allowed under the Industrial Peace Act should apply to persons to be hired or to employees who are not yet members of any labor organization. • It is inapplicable to those already in the service who are members of another union. To hold otherwise would render minority unions at a grave disadvantage and force their members to disaffiliate with them to join the majority union and render nugatory the right of employees to self-organization. • Further, it is well settled that a closed shop provision may not justify the termination by the employer of an employee who is arbitrarily refused membership to the union without any reasonable ground. • The Supreme Court emphasized that such provisions are looked upon with great disfavor whenever they are being utilized as an excuse for termination of employment. The very rationale of a closed shop provision such as this is to strengthen the effectivity labor organizations. However, such provision may be used as an instrumentality by the many to stifle the voices of the few in dissent. This is the precise reason as to why the doctrine in Anakan was formulated. G. IMPLEMENTATION – OBLIGATION AND LIABILITIES National Union of Workers in Hotels, etc. v. NLRC, 567 SCRA 291 (08) Facts: Hotel entered into a CBA with UNION, the exclusive bargaining agent of the rank-and-file employees of the Hotel. During the 60-day freedom period which preceded the expiration of the CBA, the Hotel and Union negotiated the extension of the provisions of the existing CBA. Another Union, NUWHRAIN was accorded the status of a legitimate labor organization. Thereafter, NUWHRAIN exercised the right to challenge the majority status of the incumbent union, by filing a pertition for CE. 57 | P a g e

After the lapse of the 60-day freedom period, but pending the disposition of the Petition for CE filed by NUWHRAIN, the Union served the Hotel with a written demand for the dismissal of 36 employees for alleged acts of disloyalty. Hotel issued Disciplinary Action Notices, which directed the 36 employees to submit a written explanation for their alleged acts of disloyalty and violation of the union security clause for which Union sought their dismissal. During the reconciliatory conference, Hotel’s Vice President, Norma Azores, stated her preference to deal with HIMPHLU, while blaming NUWHRAIN for the labor problems of the Hotel. NLRC pronounced that the Hotel was not guilty of ULP. Hotel had not acted improperly as it did not wrongfully terminate any of the 36 employees. The NLRC concluded that these statements did not constitute ULP for they could not have coerced or influenced either of the contending unions, both of whom did not agree in the suggested course of action or to any other manner of settling the dispute. NUWHRAIN filed a Motion for Reconsideration of the foregoing NLRC Resolution. It was denied by the NLRC in another Resolution dated 30 June 2006. CA: It declared that the Hotel had acted prudently when it issued the Notices to the 36 employees after HIMPHLU demanded their dismissal. It clarified that these Notices did not amount to the termination of the employees concerned but merely sought their explanation on why the union security clause should not be applied to them. NUWHRAIN’s Motion for Reconsideration of the aforementioned Decision of the Court of Appeals was denied. Certification Election was held, which HIMPHLU won. Issue: WON Hotel’s action/response to NUWHRAIN’s request proper. SC: Yes. "Union security" is a generic term which is applied to and comprehends "closed shop," "union shop," "maintenance of membership" or any other form of agreement which imposes upon employees the obligation to acquire or retain union membership as a condition affecting employment. 25 Article 248(e) of the Labor Code recognizes the effectivity of a union shop clause. By thus promoting unionism, workers are able to negotiate with management on an even playing field and with more persuasiveness than if they were to individually and separately bargain with the employer. In the present case, the Collective Bargaining Agreement includes a union security provision. 28 To avoid the clear possibility of liability for breaching the union security clause of the Collective Bargaining Agreement and to protect its own interests, the only sensible option left to the Hotel, upon its receipt of the demand of HIMPHLU for the dismissal of the 36 employees, was to conduct its own inquiry so as to make its own findings on whether there was sufficient ground to dismiss the said employees who defected from HIMPHLU. The issuance by the respondent of the Notices requiring the 36 employees to submit their explanations to the charges against them was the reasonable and logical first step in a fair investigation. It is important to note that the Hotel did not take further steps to terminate the 36 employees. Instead, it arranged for reconciliatory conferences between the contending unions in order to avert the possibility of dismissing the 36 employees for violation of the union security clause of the Collective Bargaining Agreement. General Rule: The dismissal of an employee by the company pursuant to a labor union’s demand in accordance with a union security agreement does not constitute unfair labor practice. An employer is not considered guilty of unfair labor practice if it merely complied in good faith with the request of the certified union for the dismissal of employees expelled from the union pursuant to the union security clause in the Collective Bargaining Agreement . In the case at bar, there is even less possibility of sustaining a finding of guilt for unfair labor practice where Hotel did not dismiss the 36 employees, despite the insistence of HIMPHLU, the sole bargaining agent for the rank and file employees of the Hotel, on the basis of the union security clause of the Collective Bargaining Agreement. The only act attributed to the Hotel is its issuance of the Notices which, contrary to being an unfair labor practice, even afforded the employees involved a chance to be heard. Still, NUWHRAIN asserts that the sworn testimony signed by its six union members that the officers of the respondent and the Hotel did utter the offending statements deserve more credence than the unsworn denial of respondent. NUWHRAIN has the burden of proving its allegation that Norma Azores did make the statements being attributed to them. The burden of proof rests upon the party who asserts the affirmative of an issue. And in labor cases, the quantum of proof necessary is substantial evidence, or such amount of relevant evidence which a reasonable mind might accept as adequate to justify a conclusion, which NUWHRAIN failed to discharge in the present case. In the case at bar, the NLRC found, and the Court of Appeals affirmed, that the officers of the respondent and the Hotel did not make statements that would have constituted unfair labor practice . Olvido v. CA, 536 SCRA 73 (07) Olvido, et. al. were 11 regular employees of Sicaltek Manufacturing. Olvido, Suico and Macasinag were the President, VP, & Sergeant-at-Arms, of Sicaltek Employees Union-ADFLO (SEU-ADFLO) while the other petitioners were founding members. Olvido with the 2 other officers filed a complaint for illegal lay-off, illegal deductions, non-payment of overtime pay, premium pay for holiday, service incentive leave pay, 13th month pay, and night shift differential pay. Meanwhile, SEU-ADFLO filed a petition for certification election on August 28, 1992.During the certification proceedings, ADFLO and Sicaltek agreed that SEU-ADFLO will withdraw the labor case in exchange for the company’s voluntary recognition of SEU-ADFLO as the sole bargaining agent of its employees. Subsequently, SEU-ADFLO was certified by the Med-Arbiter as the sole bargaining agent of Sicaltek’s rank-and-file employees. ADFLO prepared a Motion to Dismiss the labor case but petitioners refused to sign it. The petitioners were barred from attending and participating in the initial negotiations of the new CBA and because of this, they disaffiliated from SEU-ADFLO and they formed another union, the Sicaltek Workers Union (SWU), and filed a petition for 58 | P a g e

certification election which was dismissed because there was already the earlier certification of SEU-ADFLO. SWU appealed to the DOLE Secretary which was denied. Sicaltek and SEU-ADFLO concluded their new CBA. SEU-ADFLO thereafter demanded the dismissal of petitioners as provided in the Modified Union Shop Provision in the CBA due to falsification and disloyalty. The petitioners were subsequently required by Sicaltek to explain in writing why they should not be dismissed to which they replied that the demand for their dismissal was pure harassment since the certification issue between the two unions was still pending at the moment with the DOLE Sec. while the falsification charge has no basis. The petitioners were dismissed and they filed a complaint for ULP, illegal dismissal, damages, and attorney’s fees. LA dismissed the complaint while the NLRC reversed it ordering the reinstatement of the petitioners. They both filed a complaint to the SC which referred the complaint to the CA. CA ruled that petitioners were not covered by the Modified Union Shop Provision in the CBA because the provision requires all new employees to become union members after sometime but does not require present employees to join the union. Since the petitioners were already regular employees and were already members of SWU during the signing of the CBA, they could not be obliged to become members of SEU-ADFLO after the signing of the CBA under pain of being dismissed from employment. But because the dismissal wasn’t attended by bad faith, since there was no evidence that Sicaltek and SEU-ADFLO rushed the execution of the CBA to prevent SWU from being certified as the new cb agent, the Company cannot be held liable. There was nothing sinister about the company’s act of settling amicably the labor case with ADFLO. Sicaltek cannot also be faulted for complying with the demand of SEU-ADFLO to dismiss the petitioners since it was only protecting itself and it was shown that company gave the petitioners due process. Petitioners then appeals to the SC claiming that they are entitled to backwages. ISSUE: WON petitioners are entitled to backwages. SC: Ruling of the CA that petitioners are not covered by the Modified Union Shop provision in the CBA is upheld. But as to the contention that the petitioners dismissal was effected in bad faith entitling them not only to reinstatement but also backwages, SC held that Sicaltek is not obliged to pay backwages because it merely complied in good faith with the request of the certified union for the dismissal of employees expelled from the union pursuant to the union security clause in the CBA. Hence, the company may not be ordered to grant either backwages or financial assistance in the form of separation pay as a form of penalty. But since this doctrine is inconsistent with Article 279 of the LC as amended by RA 6715, which provides that an employee who is unjustly dismissed from work shall be entitled to reinstatement without loss of seniority rights and other privileges and to his/her full backwages computed from the time his/her compensation was withheld up to the time of actual reinstatement. Thus, where reinstatement is adjudged, backwages and other benefits must be awarded from the date of the LA’s decision up to the time the order is actually carried out. Petition granted. Malayang Samahan v. M. Greenfield v. Ramos, 326 SCRA 428 (00) Facts: Samahan,the "local union", is an affiliate of ULGWP, referred to as the "federation". CBA between Samahan and Company included Union Security; Dismissal any such employee who fails to maintain his membership in the UNION shall be dismissed Samahan election was held under the auspices of the ULGWP wherein Villanueva, and the other union officers were proclaimed as winners. A Petition for Impeachment was filed with the federation by the defeated candidates in the aforementioned election. Federation conducted an audit of the local union funds. The investigation did not yield any unfavorable result and the local union officers were cleared. MA = Villanueva. Failure to allege fact on the part of losing candidates. Samahan held a general membership meeting where several union members failed to attend the meeting. Samahan sought to deduct the union fines from the wages/salaries of those union members who failed to attend the general membership meeting. Company refused and suggested that the union refer the matter to the proper government office. The imposition of P50.00 fine became the subject of bitter disagreement between the Federation and the local union. In retaliation, the federation asked company to stop the remittance of the local union's share in the funds, this was opposed by the Samahan. The company was thus constrained to file a Complaint for Interpleader. Held therein the Company shall remit fund to the ULGWP The federation wrote to company informing the latter of its designation of a certain Alfredo Kalingking as local union president and "disauthorizing" the incumbent union officers from representing the employees. Villanueva replied advising that Samahan did not commit any act of disloyalty as it has remained an affiliate of ULGWP. On the same day, the federation advised company of the expulsion of the 30 union officers and demanded their separation from employment pursuant to the Union Security Clause in their collective bargaining agreement. Company terminated the 30 union officers from employment. SOLE affirmed this action. 78 union shop stewards were placed under preventive suspension by company. This prompted the union members to again stage a walk-out and resulted in the official declaration of strike. The employees who participated in the strike and allegedly figured in the violent incident were placed under preventive suspension by company by virtue of request of the federation in line with the union security clause. Only 261 employees were eventually accepted back to work. Complaint for ULP filed. LA held that dismissal was valid in compliance with the union security clause. NLRC affirmed LA. Issue: WON company was justified in dismissing Villanueva et al merely upon the labor federation's demand for the enforcement of the union security clause embodied in their collective bargaining agreement. SC: No. The reason behind the enforcement of union security clauses which is the sanctity and inviolability of contractscannot override one's right to due process. Carino case - maintenance of membership provision of the CBA is bound to dismiss any employee expelled by the union for disloyalty upon its written request, this undertaking should not be done hastily and summarily. The company acts in bad faith in dismissing a worker without giving him the benefit of a hearing. 59 | P a g e

Villanueva et al were expelled by the federation for allegedly committing acts of disloyalty and/or inimical to the interest of ULGWP and in violation of its Constitution and By-laws. Upon demand of the federation, the company terminated the petitioners without conducting a separate and independent investigation. Respondent company did not inquire into the cause of the expulsion and whether or not the federation had sufficient grounds to effect the same. An employee is entitled to be protected not only from a company which disregards his rights but also from his own union the leadership of which could yield to the temptation of swift and arbitrary expulsion from membership and mere dismissal from his job. The enforcement of union security clauses is authorized by law provided such enforcement is not characterized by arbitrariness, and always with due process. Even on the assumption that the federation had valid grounds to expel the union officers, due process requires that these union officers be accorded a separate hearing by respondent company. While it is true that the issue of expulsion of the local union officers is originally between the local union and the federation, hence, intra-union in character, the issue was later on converted into a termination dispute when the company dismissed the petitioners from work without the benefit of a separate notice and hearing. Thus, notwithstanding the fact that the dismissal was at the instance of the federation and that it undertook to hold the company free from any liability resulting from such a dismissal, the company may still be held liable if it was remiss in its duty to accord the would-be dismissed employees their right to be heard on the matter. Issue: WON the federation was a principal party to the CBA between the company and the union SC: Yes. It is undisputed that ULGWP is the certified sole and exclusive collective bargaining agent of all the regular rank-and-file workers of the company. Issue: WON Samahan declared that the act of disaffiliation and declaration of autonomy by the local union was part of its plan to take over the federation. SC: No. This is purely conjecture and totally unsupported by the evidence. A local union has the right to disaffiliate from its mother union or declare its autonomy. A local union, being a separate and voluntary association, is free to serve the interests of all its members including the freedom to disaffiliate or declare its autonomy from the federation to which it belongs when circumstances warrant, in accordance with the constitutional guarantee of freedom of association. 2 The purpose of affiliation by a local union with a mother union or a federation. Thus, a local union which has affiliated itself with a federation is free to sever such affiliation anytime and such disaffiliation cannot be considered disloyalty. In the absence of specific provisions in the federation's constitution prohibiting disaffiliation or the declaration of autonomy of a local union, a local may dissociate with its parent union.The evidence on hand does not show that there is such a provision in ULGWP's constitution. There is no disloyalty to speak of, neither is there any violation of the federation's constitution because there is nothing in the said constitution which specifically prohibits disaffiliation or declaration of autonomy. The act of establishing a different federation, entirely separate from the federation which expelled them, is but a normal retaliatory reaction to their expulsion. Issue: WON strikers should be dismissed. SC: No. There was no abandonment of work. (1) that the employee must have failed to report for work or must have been absent without valid or justifiable reason; and (2) that there must have been a clear intention to sever the employer-employee relationship manifested by some overt acts. Deliberate and unjustified refusal on the part of the employee to go back to his work post and resume his employment must be established. Absence must be accompanied by overt acts unerringly pointing to the fact that the employee simply does not want to work anymore. And the burden of proof to show that there was unjustified refusal to go back to work rests on the employer. Alabang Country Club, Inc., v. NLRC, 545 SCRA 351 (08) Facts: • Alabang Country Club, Inc. (Club) is a non-profit corporation while Alabang Country Club Independent Employees Union (Union) is the exclusive bargaining agent of the Club’s rank and file employees. • In 1996, Pizarro, Braza, and Castueras were elected as President, Vice-President and Treasurer of the Union respectively. • On June 21, 1999, the Club and Union entered into a CBA providing for a closed shop provision. Further, the CBA provided that the Union may request the termination of a rank and file employee for certain grounds, among which is malversation of union funds. • In July 2001, an election was held a new set of officers elected. An audit of the union funds was conducted and several unliquidated expenses, discrepancies and irregularities were discovered. Pizarro, Braza, and Castueras were notified of the results and were asked to explain the same. • Despite their explanations, the Union decided to expel for malversation of funds. They furnished them with a document calling for their expulsion signed by 37 of the 63 members of the union as well as a resolution of the Board of Directors effecting the expulsion. • The Union then requested the Club to terminate the said employees due to their expulsion as provided for by the union security clause of the CBA. The Club for its part called Pizarro, Braza, and Castueras to an informal conference to allow them to explain their side. However, despite their verbal and written explanation, they were terminated by the Club. • Consequently, they filed a case before the Labor Arbiter assailing their dismissal from the Club as illegal. The LA dismissed the complaint but was reversed by the NLRC. The CA affirmed the NLRC. 60 | P a g e

Issues: 6. Whether or not the dismissal of Pizarro, Braza, and Castueras was justified under the circumstances. Held: 1st Issue: YES • Enforcement of a union security clause in the CBA is another valid ground for termination of employment. This practice strengthens the union and prevents disunity in the bargaining unit within the duration of the CBA with the threat of expulsion from the union and consequent termination of employment, the union gains more members and strengthens its position against other unions who may wish to claim majority representation. • In this case, the CBA provided for a union shop and maintenance of membership shop. There is union shop when all new regular employees are required to join the union within a certain period as a condition for their continued employment. There is maintenance of membership shop when employees who are union members as of the effective date of the agreement, or who thereafter become members, must maintain union membership as a condition for continued employment until they are promoted or transferred out of the bargaining unit or the agreement is terminated. • In terminating the employment of an employee by enforcing the union security clause, the employer needs only to determine and prove that: (1) the union security clause is applicable; (2) the union is requesting for the enforcement of the union security provision in the CBA; and (3) there is sufficient evidence to support the union’s decision to expel the employee from the union. These requisites constitute just cause for terminating an employee based on the CBA’s union security provision. Per the CBA, malversation of union funds is a valid cause of termination of employment pursuant to the union security clause contained therein. In this case, the respondents were expelled from the union following the acts of dishonesty and malversation of union funds by them as found by the investigation committed by the Union. When the Union, wrote to the Club to terminate the respondents from employment, the Club conducted its own investigation and afforded the respondents an opportunity to explain their side and to controvert the allegations imputed against them by the Union. The Supreme Court held that the investigation conducted by the Club satisfied the standard of procedural due process required by law for dismissal cases. H. FINANCIAL SECURITY – Agency Shop – 248 (e); Check-off 113 (b) Art. 248. Unfair labor practices of employers. It shall be unlawful for an employer to commit any of the following unfair labor practice: a. To interfere with, restrain or coerce employees in the exercise of their right to self-organization; b. To require as a condition of employment that a person or an employee shall not join a labor organization or shall withdraw from one to which he belongs; c. To contract out services or functions being performed by union members when such will interfere with, restrain or coerce employees in the exercise of their rights to self-organization; d. To initiate, dominate, assist or otherwise interfere with the formation or administration of any labor organization, including the giving of financial or other support to it or its organizers or supporters; e. To discriminate in regard to wages, hours of work and other terms and conditions of employment in order to encourage or discourage membership in any labor organization. Nothing in this Code or in any other law shall stop the parties from requiring membership in a recognized collective bargaining agent as a condition for employment, except those employees who are already members of another union at the time of the signing of the collective bargaining agreement. Employees of an appropriate bargaining unit who are not members of the recognized collective bargaining agent may be assessed a reasonable fee equivalent to the dues and other fees paid by members of the recognized collective bargaining agent, if such non-union members accept the benefits under the collective bargaining agreement: Provided, that the individual authorization required under Article 242, paragraph (o) of this Code shall not apply to the non-members of the recognized collective bargaining agent; f. To dismiss, discharge or otherwise prejudice or discriminate against an employee for having given or being about to give testimony under this Code; g. To violate the duty to bargain collectively as prescribed by this Code; h. To pay negotiation or attorney’s fees to the union or its officers or agents as part of the settlement of any issue in collective bargaining or any other dispute; or i. To violate a collective bargaining agreement. The provisions of the preceding paragraph notwithstanding, only the officers and agents of corporations, associations or partnerships who have actually participated in, authorized or ratified unfair labor practices shall be held criminally liable. (As amended by Batas Pambansa Bilang 130, August 21, 1981) Art. 113. Wage deduction. No employer, in his own behalf or in behalf of any person, shall make any deduction from the wages of his employees, except: b. For union dues, in cases where the right of the worker or his union to check-off has been recognized by the employer or authorized in writing by the individual worker concerned; and National Brewery v. San Miguel Brewery, Inc., 8 SCRA 805 (63) Facts: National Brewery and Allied Industries Labor Union of the Philippines (PAFLU) filed a complaint against San Miguel Brewery 1962 CBA Section 7, Article VII of which provides: The Company agrees to pay the basic daily rates of those workers within the bargaining unit who may participate in the Labor Day parade held on May 1st of every year; that about 600 members of the union joined and participated in said parade whose total basic daily wage amounts to P3,900.00; 61 | P a g e

Union participated in the parade and so the union demanded the payment. The company refused to honor its obligation in bad faith. The company in its answer set up special and affirmative defenses. Contention of Company: (a) With respect to the first ground, the company contends that the union is not the real party in interest but the individual members whose right to recover the one day's wage is personal to them. (b) As regards the question of jurisdiction, the company argues that not one of the employees to whom the cause of action belongs receives a daily wage of more than P5,000.00, and hence the jurisdictionof the case is determinable on the basis of the total claim of each employee, which does not lie with the court of first instance. The company in turn moved for outright dismissal of the complaint on the plea that lack of cause of action is not correctible by amendment. Lower Court sided with the Company. What is alleged to have been violated by defendant is the contract of defendant with each and everyone of its employees, individually. That being the case, the real party in interest is the employee who has the right to receive the salary. Contention of Union: The basis of its complaint is not the individual contracts of employment which its members had entered into with the company but the CBA. Issue: What was the source of the COA. SC: CBA. The complaint filed by the union comes under the jurisdiction. Before the conclusion of said agreement, the members of the union, and for that matter any employee of the company, did not enjoy the benefit of payment of their basic daily wage even if they should attend or participate in a Labor Day parade held on Labor Day, since this right was only recognized when that agreement was concluded. The basis of the right which is sought to be enforced is the agreement itself and not the wages to be collected. The situation would be different if the purpose of the action were merely to collect wages that ordinarily accrue to members of the union because of work or services rendered in connection with their employment. Not so when the wages accrue mainly on the strength of an agreement entered into between the union and the company, as is the instant case. The action then may be brought in the name of the union that has obliged itself to secure those wages for this members. In this respect, we find pertinent Section 3, Rule 3 of our ROC, wherein it is provided, among others, that a party with whom or in whose name a contract has been made for the benefit of another may sue or be sued without joining the party for whose benefit the action is presented or defended even if the court may at its discretion order such beneficiary to be made also a party. This provision fittingly applies to this case. The union is the party with whom or in whose name the collective bargaining agreement in question has been entered into for the benefit of its members and, in line with the above rule, the union may sue thereon without joining the members for whose benefit the action has been presented. Del Pilar Academy v. Del Pilar Academy Employees Union, 553 SCRA 590 (08) G.R. No. 170112, April 30, 2008 NACHURA, J.: Facts: 1. On September 15, 1994, the UNION and DEL PILAR entered into a Collective Bargaining Agreement (CBA)[3] granting salary increase and other benefits to the teaching and non-teaching staff 2. The UNION then assessed agency fees from non-union employees, and requested DEL PILAR to deduct said assessment from the employees salaries and wages. DEL PILAR, however, refused to effect deductions claiming that the non-union employees were not amenable to it. 3. In September 1997, the UNION negotiated for the renewal of the CBA. DEL PILAR, however, refused to renew the same unless the provision regarding entitlement to two (2) months summer vacation leave with pay will be amended by limiting the same to teachers, who have rendered at least three (3) consecutive academic years of satisfactory service 4. The UNION objected to the proposal claiming diminution of benefits. DEL PILAR refused to sign the CBA, resulting in a deadlock 5. The UNION requested DEL PILAR to submit the case for voluntary arbitration, but the latter allegedly refused, prompting the UNION to file a case for unfair labor practice with the Labor Arbiter against DEL PILAR; Eduardo Espejo, its president; and Eliseo Ocampo, Jr., chairman of the Board of Trustees 6. DEL PILAR justified the non-deduction of the agency fees by the absence of individual check off authorization from the non-union employees. As regards the proposal to amend the provision on summer vacation leave with pay, DEL PILAR alleged that the proposal cannot be considered unfair for it was done to make the provision of the CBA conformable to the DECS Manual of Regulations for Private Schools. 7. LA ruled in favor of the UNION, as to the its right to agency fee, but did not consider DEL PILARs failure to deduct the same an unfair labor practice. ISSUE: WON UNION is entitled to collect agency fees from non-union members, and if so, whether an individual written authorization is necessary for a valid check off. HELD: YES 1. The collection of agency fees in an amount equivalent to union dues and fees, from employees who are not union members, is recognized by Article 248(e) of the Labor Code, thus: Employees of an appropriate collective bargaining unit who are not members of the recognized collective bargaining agent may be assessed reasonable fees equivalent to the dues and other fees paid by the recognized collective bargaining agent, if such non-union members accept the benefits under the collective bargaining agreement. Provided, That the individual authorization required under Article 241, paragraph (o) of this Code shall not apply to the non-members of recognized collective bargaining agent.

62 | P a g e

When so stipulated in a collective bargaining agreement or authorized in writing by the employees concerned, the Labor Code and its Implementing Rules recognize it to be the duty of the employer to deduct the sum equivalent to the amount of union dues, as agency fees, from the employees' wages for direct remittance to the union. The system is referred to as check off a. No requirement of written authorization from the non-union employees is necessary if the non-union employees accept the benefits resulting from the CBA Contrary to what DEL PILAR wants to portray, the grant of annual salary increase is not the only provision in the CBA that benefited the non-union employees. The UNION negotiated for other benefits, namely, limitations on teaching assignments to 23 hours per week, additional compensation for overload units or teaching assignments in excess of the 23 hour per week limit, and payment of longevity pay. It also negotiated for entitlement to summer vacation leave with pay for two (2) months for teaching staff who have rendered six (6) consecutive semesters of service. For the non-teaching personnel, the UNION worked for their entitlement to fifteen (15) days leave with pay.These provisions in the CBA surely benefited the non-union employees, justifying the collection of, and the UNIONs entitlement to, agency fees. VIII. INTERNATIONAL ACTIVITIES OF UNIONS = 269-271 Art. 269. Prohibition against aliens; exceptions. All aliens, natural or juridical, as well as foreign organizations are strictly prohibited from engaging directly or indirectly in all forms of trade union activities without prejudice to normal contacts between Philippine labor unions and recognized international labor centers: Provided, however, That aliens working in the country with valid permits issued by the Department of Labor and Employment, may exercise the right to self-organization and join or assist labor organizations of their own choosing for purposes of collective bargaining: Provided, further, That said aliens are nationals of a country which grants the same or similar rights to Filipino workers. (As amended by Section 29, Republic Act No. 6715, March 21, 1989) Art. 270. Regulation of foreign assistance. 1. No foreign individual, organization or entity may give any donations, grants or other forms of assistance, in cash or in kind, directly or indirectly, to any labor organization, group of workers or any auxiliary thereof, such as cooperatives, credit unions and institutions engaged in research, education or communication, in relation to trade union activities, without prior permission by the Secretary of Labor. "Trade union activities" shall mean: 1. organization, formation and administration of labor organization; 2. negotiation and administration of collective bargaining agreements; 3. all forms of concerted union action; 4. organizing, managing, or assisting union conventions, meetings, rallies, referenda, teach-ins, seminars, conferences and institutes; 5. any form of participation or involvement in representation proceedings, representation elections, consent elections, union elections; and 6. other activities or actions analogous to the foregoing. 2. This prohibition shall equally apply to foreign donations, grants or other forms of assistance, in cash or in kind, given directly or indirectly to any employer or employer’s organization to support any activity or activities affecting trade unions. 3. The Secretary of Labor shall promulgate rules and regulations to regulate and control the giving and receiving of such donations, grants, or other forms of assistance, including the mandatory reporting of the amounts of the donations or grants, the specific recipients thereof, the projects or activities proposed to be supported, and their duration. Art. 271. Applicability to farm tenants and rural workers. The provisions of this Title pertaining to foreign organizations and activities shall be deemed applicable likewise to all organizations of farm tenants, rural workers, and the like: Provided, That in appropriate cases, the Secretary of Agrarian Reform shall exercise the powers and responsibilities vested by this Title in the Secretary of Labor. PART FOUR PRE-CONDITIONS TO COLLECTIVE BARGAINING, APPROPRIATE BARGAINING UNIT, AND UNION MAJORITY STATUS A. APPROPRIATE BARGAINING UNIT 1. Definition and Role in Law – 225; 256 Art. 225. Contempt powers of the Secretary of Labor. In the exercise of his powers under this Code, the Secretary of Labor may hold any person in direct or indirect contempt and impose the appropriate penalties therefor. Art. 256. Representation issue in organized establishments. In organized establishments, when a verified petition questioning the majority status of the incumbent bargaining agent is filed before the Department of Labor and Employment within the sixty-day period before the expiration of the collective bargaining agreement, the Med-Arbiter shall automatically order an election by secret ballot when the verified petition is supported by the written consent of at least twenty-five percent (25%) of all the employees in the bargaining unit to ascertain the will of the employees in the appropriate bargaining unit. To have a valid election, at least a majority of all eligible voters in the unit must have cast their votes. The labor union receiving the majority of the valid votes cast shall be certified as the exclusive 63 | P a g e

2.

bargaining agent of all the workers in the unit. When an election which provides for three or more choices results in no choice receiving a majority of the valid votes cast, a run-off election shall be conducted between the labor unions receiving the two highest number of votes: Provided, that the total number of votes for all contending unions is at least fifty percent (50%) of the number of votes cast. At the expiration of the freedom period, the employer shall continue to recognize the majority status of the incumbent bargaining agent where no petition for certification election is filed. (As amended by Section 23, Republic Act No. 6715, March 21, 1989) Belyca Corporation v. Calleja, 168 SCRA 184 (88) NATURE Petition for certiorari and prohibition with preliminary injunction. FACTS - In the instant case, respondent ALU seeks direct certification as the sole and exclusive bargaining agent of all the rank-and-file workers of the livestock and agro division of petitioner BELYCA Corporation engaged in piggery, poultry raising and the planting of agricultural crops such as corn, coffee and various vegetables. But petitioner contends that the bargaining unit must include all the workers in its integrated business concerns ranging from piggery, poultry, to supermarts and cinemas so as not to split an otherwise single bargaining unit into fragmented bargaining units. ISSUE: WON the proposed bargaining unit by Belyca is an appropriate bargaining unit. HELD: No. - According to Rothenberg, a proper bargaining unit maybe said to be a group of employees of a given employer, comprised of all or less than all of the entire body of employees, which the collective interests of all the employees, consistent with equity to the employer, indicate to be best suited to serve reciprocal rights and duties of the parties under the collective bargaining provisions of the law. - This Court has already taken cognizance of the crucial issue of determining the proper constituency of a collective bargaining unit. - Among the factors considered in Democratic Labor Association v. Cebu Stevedoring Co. Inc. (103 Phil 1103 [1958]) are: "(1) will of employees (Glove Doctrine); (2) affinity and unity of employee's interest, such as substantial similarity of work and duties or similarity of compensation and working conditions; (3) prior collective bargaining history; and (4) employment status, such as temporary, seasonal and probationary employees". - Under the circumstances of that case, the Court stressed the importance of the fourth factor and sustained the trial court's conclusion that two separate bargaining units should be formed in dealing with respondent company, one consisting of regular and permanent employees and another consisting of casual laborers or stevedores. Otherwise stated, temporary employees should be treated separately from permanent employees. But more importantly, this Court laid down the test of proper grouping, which is community and mutuality of interest. Reasoning It is beyond question that the employees of the livestock and agro division of petitioner corporation perform work entirely different from those performed by employees in the supermarts and cinema. Among others, the noted difference are: their working conditions, hours of work, rates of pay, including the categories of their positions and employment status. To lump all the employees of petitioner in its integrated business concerns cannot result in an efficacious bargaining unit comprised of constituents enjoying a community or mutuality of interest. DISPOSITION The petition is DISMISSED for lack of merit. 2. Determination of Appropriate Bargaining Unit A. Factors – Unit Determination 1. In General – Standard Test UP v. Ferrer-Calleja, 211 SCRA 451 (92) Facts: ONAPUP, Claiming to have a membership of 3,236 members — comprising more than 33% of the 9,617 persons constituting the non-academic personnel of UPDiliman, Los Baños, Manila, and Visayas, it sought the holding of a certification election among all said non-academic employees of the University of the Philippines. Another registered labor union, the "All UP Workers' Union," intervened, alleging that its membership covers both academic and non-academic personnel, and that it aims to unite all UP rank-and-file employees in one union, Contention of UP: There should be two (2) unions: one for academic, the other for non-academic or administrative, personnel considering the dichotomy of interests, conditions and rules governing these employee groups. SOLE: the appropriate organizational unit should embrace all the regular rank-and-file employees, teaching and non-teaching, of the University of the Philippines, including all its branches. Basis Section 9 of Executive Order No. 180 - The appropriate organizational unit shall be the employer unit consisting of rank-and-file employees, unless circumstances otherwise require. ONAPUP = UP: distinction. All UP Workers' Union = SOLE. Issue: WON professors, associate professors and assistant professors are included in the definition of high-level employee(s)" in light of Rule I, Section (1) of the Implementing Guidelines of Executive Order No. 180, defining "high level employee" University Code. 64 | P a g e

SC: No. Professors cannot be considered as exercising such managerial or highly confidential functions as would justify their being categorized as "high-level employees" of the institution. Recommendatory powers, as in the case at bar, are subject to evaluation, review and final action by the department heads and other higher executives of the company, the same, although present, are not effective and not an exercise of independent judgment as required by law. It is readily apparent that the policy-determining functions of the University Council are subject to review, evaluation and final approval by the Board of Regents. Even assuming arguendo that UP professors discharge policy-determining functions through the University Council, still such exercise would not qualify them as highlevel employees within the context of E.O. 180. As correctly observed by private respondent, "Executive Order No. 180 is a law concerning public sector unionism. It must therefore be construed within that context. Issue (TOPIC): WON, they, and other employees performing academic functions, should comprise a collective bargaining unit distinct and different from that consisting of the non-academic employees of the University considering the dichotomy of interests, conditions and rules existing between them. SC: No. Not much reflection is needed to perceive that the community or mutuality of interests which justifies the formation of a single collective bargaining unit is wanting between the academic and non-academic personnel of the university. A "bargaining unit" has been defined as a group of employees of a given employer, comprised of all or less than all of the entire body of employees, which the collective interest of all the employees, consistent with equity to the employer, indicate to be the best suited to serve the reciprocal rights and duties of the parties under the collective bargaining provisions of the law. Our labor laws do not however provide the criteria for determining the proper collective bargaining unit. AMERICAN PRECEDENTS: No one particular factor is itself decisive of the determination. Rothenberg enumerated factors: (1) will of the employees (Globe Doctrine); (2) affinity and unit of employees' interest, such as substantial similarity of work and duties, or similarity of compensation and working conditions; (3) prior collective bargaining history; and (4) employment status, such as temporary, seasonal probationary employees. Apply "community or mutuality of interests" test has provided the standard in determining the proper constituency of a collective bargaining unit. It would seem obvious that teachers would find very little in common with the University clerks and other non-academic employees as regards responsibilities and functions, working conditions, compensation rates, social life and interests, skills and intellectual pursuits, cultural activities, etc. On the contrary, the dichotomy of interests, the dissimilarity in the nature of the work and duties as well as in the compensation and working conditions of the academic and nonacademic personnel dictate the separation of these two categories of employees for purposes of collective bargaining. The formation of two separate bargaining units, the first consisting of the rank-and-file non-academic personnel, and the second, of the rank-and-file academic employees, is the setup that will best assure to all the employees the exercise of their collective bargaining rights. 2. History San Miguel Corporation v. Laguesma, 236 SCRA 595 (94) G.R. No. 100485 September 21, 1994 PUNO, J.: Facts: 1. On June 4, 1990, the North Luzon Magnolia Sales Labor Union (respondent union for brevity) filed with the Department of Labor a petition for certification election among all the regular sales personnel of Magnolia Dairy Products in the North Luzon Sales Area 2. SMC opposed the petition and questioned the appropriateness of the bargaining unit sought to be represented by respondent union. It claimed that its bargaining history in its sales offices, plants and warehouses is to have a separate bargaining unit for each sales office. 3. Subsequently, petitioner withdrew opposition, through Atty. Batalia, a substitute lawyer, to a certification election and agreed to consider all the sales offices in northern Luzon as one bargaining unit. At the pre-election conference, the parties agreed inter alia, on the date, time and place of the consent election. 4. Respondent union won the election held on November 24, 1990 which Mediator-Arbiter Benalfre J. Galang certified respondent union as the sole and exclusive bargaining agent for all the regular sales personnel in all the sales offices of Magnolia Dairy Products in the North Luzon Sales Area 5. Petitioner appealed to the Secretary of Labor claiming that Atty. Batalla was only authorized to agree to the holding of certification elections subject to the following conditions: (1) there would only be one general election; (2) in this general election, the individual sales offices shall still comprise separate bargaining units. 6. Public respondent, by authority of the Secretary of Labor, denied SMC's appeal and affirmed the Order of the Med- Arbiter. Issue 1: WON public respondent disregarded its collective bargaining history which is to have a separate bargaining unit for each sales office HELD: No. The existence of a prior collective bargaining history is neither decisive nor conclusive in the determination of what constitutes an appropriate bargaining unit.

65 | P a g e

The test of grouping is mutuality or commonality of interests. The employees sought to be represented by the collective bargaining agent must have substantial mutual interests in terms of employment and working conditions as evinced by the type of work they perform. a. CAB: respondent union sought to represent the sales personnel in the various Magnolia sales offices in northern Luzon. There is similarity of employment status for only the regular sales personnel in the north Luzon area are covered. They have the same duties and responsibilities and substantially similar compensation and working conditions. The commonality of interest among he sales personnel in the north Luzon sales area cannot be gainsaid. In fact, in the certification election held on November 24, 1990, the employees concerned accepted respondent union as their exclusive bargaining agent. Clearly, they have expressed their desire to be one. b. What greatly militates against this position is the meager number of sales personnel in each of the Magnolia sales office in northern Luzon. Even the bargaining unit sought to be represented by respondent union in the entire north Luzon sales area consists only of approximately fifty-five (55) employees. 9 Surely, it would not be for the best interest of these employees if they would further be fractionalized. The adage "there is strength in number" is the very rationale underlying the formation of a labor union. 2. OBITER: Definition of a bargaining unit: A bargaining unit is a "group of employees of a given employer, comprised of all or less than all of the entire body of employees, consistent with equity to the employer, indicate to be the best suited to serve the reciprocal rights and duties of the parties under the collective bargaining provisions of the law. a. The fundamental factors in determining the appropriate collective bargaining unit are: (1) the will of the employees (Globe Doctrine); 6 (2) affinity and unity of the employees' interest, such as substantial similarity of work and duties, or similarity of compensation and working conditions (Substantial Mutual Interests Rule); (3) prior collective bargaining history; and (4) similarity of employment status. WON petitioner is bound by its lawyer's act of agreeing to consider the sales personnel in the north Luzon sales area as one bargaining unit due to the latter’s unfamiliarity of the CBA history. HELD: Yes. 1. The collective bargaining history of a company is not decisive of what should comprise the collective bargaining unit. Insofar as the alleged "mistake" of the substitute lawyer is concerned, we find that this mistake was the direct result of the negligence of petitioner's lawyers. The lawyers allegedly actively involved in SMC's labor case should have adequately and sufficiently briefed the substitute lawyer with respect to the matters involved in the case and the specific limits of his authority. 3. Geography – Location Benguet Consolidated, Inc. and Balatoc Mining Co. v. Bobok Lumberjack Association, 103 P 1150 (58) 23 May 1958 (This is the whole resolution. Nothing online either.) - Appeal from the order of the Court of Industrial Relations holding that it finds no valid reason to change the status of the petitioners' five camps as separate bargaining units, and from the order denying the motion for reconsideration. The court below is correct in concluding that the system of having one collective bragaining unit for each camp should be maintained and continued, for the following reasons: Such system had operated satisfactorily. The prime element in determining whether a given group of employees constitutes a proper bargaining unit is whether it will, without inequity to the employer, best serve all employees in the exercise of their bargaining rights. In the present case the separation between the camps and the different kinds of work in each all militate in favor of the present system of separate bargaining units since the problems and interest of the workers are peculiar in each camp or department. The orders appealed from are affirmed, with costs against appellants. 4. Size – Composition Philippine Diamond Hotel and Resort, Inc. v. Manila Diamond Hotel Employees Union, 494 SCRA 195 (04) FACTS: - 11 November 1996: Manila Diamond Hotel Employee's Union, which was registered on 19 August 1996 before the DOLE, filed a Petition for Certification Election before the DOLE-NCR seeking, which was latter DENIED as it failed to comply with legal requirements, specifically Section 2, Rule V, Book V of the Rules and Regulations Implementing the Labor Code, and was seen to fragment the employees of petitioner. - Francis Mendoza, one of the Hotel's outlet cashiers, was discovered to have failed to remit P71,692,50. Mendoza claimed that after accomplishing his daily cash report, the union president Jose Leonardo B. Kimpo, also an outlet cashier, who signed the same and dropped his remittances. Mendoza was subsequently suspended for one week. In the meantime, he was reassigned to the Hotel's Cost Control Department. - Kimpo, notified petitioner of its intention to negotiate a CBA. - The hotel advised the union that since it was not certified by the DOLE as the exclusive bargaining agent, it could not be recognized as such. The union clarified that it sought to bargain "for its members only," and declared that the Hotel's refusal to bargain would prompt the union to engage in concerted activities to protect and assert its rights under the Labor Code. By Notice to its member, the union anounced that its executive officers, as well as its directors decided to go on strike.

1.

66 | P a g e

- The hotel issued a final reminder and warning against continuing misinformation campaign and activities which confused the Hotel employees and disturbed their work performance. - Conciliation conferences were conducted. In the meantime, Kimpo filed before the Arbitration Branch a complain for ULP against petitioner. - In the conference held on 20 November 1997, the union demanded the holding of a consent election to which the Hotel interposed no objection, provided the union followed the procedure under the law, requesting that it be held in January 1998. - 29 November 1997, the union suddenly went on strike. The National Union of Workers in the Hotel, Restaurant and Allied Industries (NUWHRAIN) joined the strike and openly extended its support to the union. Hotel supervisors Vicente Agustin, Rowena Junio and Mary Grace de Leon were seen participating and supporting the strike. - Petitioner thus filed a petition for injunction with the NLRC. It terminated Agustin, Junio and de Leon's services, drawing them to file complaints for illegal dismissal. NLRC issued a TRO directing the strikers to immediately cease and desist from obstructing the free ingress and egress from the Hotel premises. - Strikers refused to heed the TRO, prompting the Hotel's security guards to dismantle the tents, during which a number from each side suffered physical injuries. - Petitioner filed a petition to declare the strike illegal. DOLE Sec Trajano issed an order certifying the dispute to the NLRC for compulsory arbitration, directing the striking officers and members to return to work withing 24 hours and the Hotel to accept them back under the same terms and conditions prevailing before the strike. - Acting on petitioner's motion for reconsideration, then DOLE acting secretary Espanol modified the order of Sec Trajano, directing the hotel to just reinstate the strikers to its payroll. - NLRC declared the strike illegal, union officers and members who were reinstated to the payroll were deemed to have lost their employment status. The complaints filed by de Leon, Agustin, Junio and Kimpo were dismissed. - CA affirmed NLRC, but modified by ordering the reinstatement with backwages of union members. (RELEVANT) ISSUE: W/N the strike was illegal HELD: - YES. Article 255 of the Labor Code declares that only the labor organization designated or selected by the majority of the employees in an appropriate collective bargaining unit is the exclusive representative of the employees in such unit for the purpose of collective bargaining. The union is ADMITTEDLY NOT THE EXCLUSIVE REPRESENTATIVE OF THE MAJORITY OF THE EMPLOYEES OF PETITIONER, hence, it could not demand from petitioner the right to bargain collectively in their behalf. Article 242 (a) must be read in relation to Article 255. - On respondent's contention that it was bargaining in behalf of its members, the CA, affirming the NLRC's observation that the same would only "fragment the employees" of petitioner, held that "what respondent will be achieveing is to divide the employees, more particularly, the rank-and-file employees...the other workers who are not memebers are at a serious disadvantage, because of the same shall be allowed, employees who are non-union memebrs will be economically impaired and will not be able to negotiate their terms and conditions of work, thus defeating the very essence and reason of collective bargaining, which is an effective safeguard agains the evil schemes of employers in terms and conditions of work." This Court finds the observation well-taken. - The goal of the DOLE is geared towards "a single employer-wide unit which is more to the broader and greater benefit of the employees working force." The philosophy is to avoid fragmentation fo the bargaining unit so as to strengthen teh employees' bargaining power with the management. 3. Corporate Entities Diatogon Labor Federation v. Ople, 101 SCRA 534 (80) Facts: Lianga Bay Logging Co., Inc. and Georgia Pacific International Corporation is a Delaware corporation. Diatagon Labor Federation of ULGWP had a 3-year CBA with the Lianga Bay logging Co., Inc. including 236 employees working at the veneer plant and electrical department of Georgia Pacific International Corporation in Lianga. Those 236 employees were formerly employees of Lianga Bay Logging Co., Inc. 236 employees continued to use in the pay envelopes and Identification cards of their former employer. A rival union, Mindanao Association of Trade Unions (MATU) filed for CE. MA granted CE, in which MATU lost. MATU filed protest. Contention of MATU: CBA was negotiated between Georgia Pacific International Corporation and the Diatagon Labor Federation in order to frustrate the petition for certification election at Lianga Bay Logging Co., Inc. Both companies have common management and lawyers; hence, 1 bargaining unit should be had. All employees should vote, including the 236, in the CE. BLR: 236 employees should be allowed to vote in the certification election at Lianga Bay Logging Co., Inc. because they used the company's pay envelopes and Identification cards. There existed no distinction between the employees of the two companies and. consequently, they should belong to only one bargaining unit. Issue: WON two companies should be regarded as a single collective bargaining unit. SC: reverse BLR. The two companies are indubitably distinct entities with separate juridical personalities. The fact that their businesses are related and that the 236 employees of Georgia Pacific International Corporation were originally employees of Lianga Bay Logging Co., Inc. is not a justification for disregarding their separate personalities. Hence, the 236 employees, who are now attached to Georgia Pacific International Corporation,

67 | P a g e

should not be allowed to vote in the certification election at the Lianga Bay Logging Co., Inc. They should vote at a separate certification election to determine the collective bargaining representative of the employees of Georgia Pacific International Corporation. A new certification election should be held at Lianga Bay Logging Co., Inc. but the 236 employees should not be allowed to vote in that election. Indophil Textile Mills Workers Union v. Calica, 205 SCRA 697 (92) Facts: Workers Union-PTGWO is a legitimate labor organization of Indophil Textile Mills, Inc. CBA. Indophil Acrylic Manufacturing Corporation was formed and registered with the SEC. Acrylic became operational and hired workers according to its own criteria and standards. Workers of Acrylic unionized and a CBA was executed. Contention of INDO-PTGWO: Union claimed that the plant facilities built and set up by Acrylic should be considered as an extension or expansion of the facilities of Indo textile. Acrylic is part of the Indophil bargaining unit. AOI has same kind of business. Same incorporators. Acrylic is 70% owned by Indophil. Contention of Indophil: it is a juridical entity separate and distinct from Acrylic. The existing impasse led the petitioner and private respondent to enter into a submission agreement on September 6, 1990. The parties jointly requested the public respondent to act as voluntary arbitrator in the resolution of the pending labor dispute pertaining to the proper interpretation of the CBA provision. VA: Separate and distinct. Issue: WON the operations in Acrylic are an extension or expansion of Indophil; hence, WON the rank-and-file employees working at Acrylic should be recognized as part of, and/or within the scope of the bargaining unit. SC: Indophil and Acrylic are separate and distinct. Doctrine of piercing the veil of corporate fiction should not be applied. Aside: Teodorico Calica, a professor of the U.P. Asian Labor Education Center, now the Institute for Industrial Relations, found that the existing law and jurisprudence on the matter, supported the private Under the doctrine of piercing the veil of corporate entity , when valid grounds therefore exist, the legal fiction that a corporation is an entity with a juridical personality separate and distinct from its members or stockholders may be disregarded. In such cases, the corporation will be considered as a mere association of persons. The members or stockholders of the corporation will be considered as the corporation, that is liability will attach directly to the officers and stockholders. The doctrine applies when the corporate fiction is used to defeat public convenience, justify wrong, protect fraud, or defend crime, or when it is made as a shield to confuse the legitimate issues, or where a corporation is the mere alter ego or business conduit of a person, or where the corporation is so organized and controlled and its affairs are so conducted as to make it merely an instrumentality, agency, conduit or adjunct of another corporation. ( While we do not discount the possibility of the similarities of the businesses of private respondent and Acrylic, neither are we inclined to apply the doctrine invoked by petitioner in granting the relief sought. The fact that the businesses of private respondent and Acrylic are related, that some of the employees of the private respondent are the same persons manning and providing for auxilliary services to the units of Acrylic, and that the physical plants, offices and facilities are situated in the same compound, it is our considered opinion that these facts are not sufficient to justify the piercing of the corporate veil of Acrylic. In the instant case, petitioner does not seek to impose a claim against the members of the Acrylic. Diatagon Labor Federation Local 110 of the ULGWP v . Ople : it is grave abuse of discretion to treat two companies as a single bargaining unit when these companies are indubitably distinct entities with separate juridical personalities. Philippine Scouts Veterans Security and Investigating Agency v. Torres, 224 SCRA 682 (93) Facts: PGA Brotherhood Association - Union of Filipino Workers (UFW) ( "the Union ") filed a single petition for CE among the rank and file employees of Philippine Scout Veterans Security and Investigation Agency (PSVSIA), GVM Security and Investigations Agency, Inc. ( GVM). and Abaquin Security and Detective Agency, Inc. (ASDA). These three agencies were collectively referred to by private respondent Union as the "PGA Security Agency," which is actually the first letters of the corporate names of the agencies. Contention of PGA: Three security agencies have separate and distinct corporate personalities while PGA Security Agency is not a business or corporate entity and does not possess any personality whatsoever. Motion to Dismiss on the grounds that the 721 supporting signatures do not meet the 20% minimum requirement for certification election as the number of employees totals 2374. Three companies should not be treated as one in a single bargaining unit in one petition for certification elections resulting in a violation of the right to due process of each corporation as no notice of hearing and other legal processes were served on each of said corporations. Contention of Union: Separate personalities of the three agencies were used merely to circumvent the prohibition in that a security agency must not have more than 1,000 guards in its employ; that management and operations are so intertwined that they can be deemed to be a single entity. MA = SOLE = Union, PGA should be deemed as a single entity and bargaining unit. Consequently, no jurisdiction was acquired on them by the Department of Labor and Employment. Issue: WON a single petition for CE can validly or legally be filed by a labor union in three (3) corporations each of which has a separate and distinct legal personality instead of filing three (3) separate petitions.

68 | P a g e

SC: No becasue the three are same entities. The three agencies in the case at bar failed to rebut the fact that they are managed through the Utilities Management Corporation with all of their employees drawing their salaries and wages from said entity; that the agencies have c ommon and interlocking incorporators and officers; and that the PSVSIA, GVM and ASDA employees have a single Mutual Benefit System and followed a single system of compulsory retirement. Security guards of one agency could easily transfer from one agency to another and then back again by simply filling-up a common pro forma slip called "Request for Transfer". Accordingly, the veil of corporate fiction of the three agencies should be lifted for the purpose of allowing the employees of the three agencies to form a single labor union. As a single bargaining unit, the employees therein need not file three separate petitions for certification election. All of these could be covered in a single petition. When a duly organized union files a petition for certification election, the Med-Arbiter has the duty to automatically conduct an election. He has no discretion on the matter. This is clearly the mandate of Article 257 of the Labor Code, as amended by Section 24 of R.A. 6715, which now reads: Art. 257. Petitions in unorganized establishments. — In any establishment where there is no certified bargaining agent, a certification election shall automatically be conducted by the Med-Arbiter upon the filing of a petition by a legitimate labor organization. Even if there was improper service of summons by the Med-Arbiter, the three (3) security agencies voluntarily submitted themselves to the jurisdiction of the labor authorities. The summons were clearly sent to and received by their lawyer who filed motions and pleadings on behalf of the three security agencies. Company's role in a certification election has aptly been described in T as that of a mere by-stander. It has no legal standing in a certification election as it cannot oppose the petition or appeal the Med-Arbiter's orders related thereto. An employer that involves itself in a certification election lends suspicion to the fact that it wants to create a company union. San Miguel Corporation Employees Union v. Confesor, 262 SCRA 81 (96) Facts: • 28 June 1990: SMCEU-PTGWO entered into a CBA with SMC to take effect upon the expiration of the previous CBA or on 30 June 1989, for which the duration shall be for a term of 3 years or until 30 June 1992. • The terms of the CBA also provided that insofar as the representation aspect is concerned, the term of the CBA shall be for five years: from 1 July 1989 to 30 June 1994. • 13 August 1991: In a letter, SMC management informed its employees that the company would undergo a restructuring • 1 October 1991 – the Magnolia and the Feeds & Livestocks Divisions were spun-off and became two separate and distinct corporations: Magnolia Corporation and San Miguel Foods, Inc. • The CBA was renegotiated after 30 June 1992. • SMCEU-PTGWO (petitioner-union) insisted that the bargaining unit of SMC should still include the employees of the spun-off corporations, and that the renegotiated terms of the CBA shall be effective only for the remaining period of two years or until 30 June 1994. • SMC contended that the members/employees who had moved to Magnolia and SMFI automatically ceased to be part of the bargaining unit at the SMC, and the renegotiated terms should be effective for three years in accordance with Article 253-A of the Labor Code. − Petitioner-union declared a deadlock on 29 September 1990. On 2 October 1992, a Notice of Strike was filed against SMC. The NCMB conducted preventive mediation upon the request of SMC, but no settlement was arrived at. A strike vote was conducted which resulted in a “yes vote.” The Secretary of Labor assumed jurisdiction over the labor dispute, after which several conciliation meetings were held, but still no settlement was reached. − Secretary of Labor issued the assailed order directing that the renegotiated terms of the CBA shall be effective for a period of three years from 30 June 1992, and that such CBA shall over only the employees of SMC and not of Magnolia and SMFI. Issues: − W/N the duration of the renegotiated terms of the CBA is to be effective for three years or for only two years − W/N the bargaining unit of SMC includes also the employees of the Magnolia and SMFI. (RE CORPORATE ENTITIES) Held: − The duration of the renegotiated terms of the CBS shall be effective for three years, based on Article 253-A of the Labor Code. − Under this provision, a CBA has a term of five years as far as the representation aspect is concerned, and all other provisions of the CBA shall be negotiated not later than three years after its execution. − Representation aspect: refers to the identity and majority status of the union that negotiated the CBA as the exclusive bargaining representative of the appropriate bargaining unit concerned. − The framers of the law wanted to maintain industrial peace and stability, and thus no outside union could enter the establishment within five years and challenge the status of the incumbent union as the exclusive bargaining agent.

69 | P a g e

This way, the last year of the RENEGOTIATED terms,which is technically the year after the five-year period of the representation aspect - ASSUMING THE BARGAINING AGENT IS CHANGED AFTER THE FIVE-YEAR PERIOD – becomes a sort of adjustment period of “industrial peace” so as to let the management and the new agent to get to know each other, negotiate, etc. - gist of the quoted deliberations. :) − No, the bargaining unit of SMC excludes the employees of Magnolia and SMFI. − The transformation of the companies was a management prerogative and business judgments which the courts cannot look into unless it is contrary to law, public policy or morals. − Magnolia and SMFI became distinct entities with separate juridical personalities − In determining an appropriate bargaining unit, the test of grouping is mutuality or commonality of interests. The employees sought to be represented must have substantial mutual interests in terms of employment and working conditions as evinced by the type of work they performed. Considering the spin-offs, the companies would consequently have their respective and distinctive concerns in terms of the nature of work, wages, hours of work and other conditions of employment. The nature of their products and scales of business may require different skills which must necessarily be commensurated by different compensation packages. PETITION WAS DISMISSED. Complex Electronics v. NLRC, 310 SCRA 403 (99) Facts: Complex was actually a subcontractor of electronic products where its customers gave their job orders, sent their own materials and consigned their equipment to it. The rank and file workers of Complex were organized into a union known as the Complex Electronics Employees Association (UNION) Complex received a facsimile message from Lite-On Philippines Electronics Co ., requiring it to lower its price by 10%. Complex regretfully informed the employees that it was left with no alternativae but to close down the operations of the Lite-On Line. The company, however, promised that proper compensation shall be given. Union, on the other hand, pushed for a retrenchment pay equivalent to one (1) month salary for every year of service, which Complex refused. Complex were pulled-out from the company premises and transferred to the premises of Ionics. Union filed a notice of strike. ULP case filed where Ionics was impleaded as a party defendant. Contention of Ionics: Ionics contended that it was an entity separate and distinct from Complex. Like Complex, it was also engaged in the semi-conductor business where the machinery, equipment and materials were consigned to them by their customers. The transfer of the machinery, equipment and materials from Complex was the decision of the owners who were common customers of Complex and Ionics. Contention of Union: President of both companies - Lawrence Qua. Ease in transferring of assets. ISO Certification for both companies published in a newspaper. Complex was the majority shareholder of Ionics. LA = Ionex. All parties appealed. All were denied; hence, this consolidated case. Issue: Won Complex and Ionics are separate and distinct; hence, Ionics was just a runaway shop. SC: Yes. Ionics was not just a runaway shop. A "runaway shop" is defined as an industrial plant moved by its owners from one location to another to escape union labor regulations or state laws, but the term is also used to describe a plant removed to a new location in order to discriminate against employees at the old plant because of their union activities. A "runaway shop" in this sense, is a relocation motivated by anti-union animus rather than for business reasons. In this case, however, Ionics was not set up merely for the purpose of transferring the business of Complex. At the time the labor dispute arose at Complex, Ionics was already existing as an independent company. As earlier mentioned, it has been in existence since July 5, 1984. It cannot, therefore, be said that the temporary closure in Complex and its subsequent transfer of business to Ionics was for anti-union purposes. The mere fact that one or more corporations are owned or controlled by the same or single stockholder is not a sufficient ground for disregarding separate corporate personalities justify the piercing of the corporate veil. This fiction of corporate entity can only be disregarded in certain cases such as when it is used to defeat public convenience, justify wrong, protect fraud, or defend crime. The photo/newspaper clipping merely showed that some plants of Ionics were recertified to ISO 9002 and does not show that there is a relation between Complex and Ionics except for the fact that Lawrence Qua was also the president of Ionics. However, as we have stated above, the mere fact that both of the corporations have the same president is not in itself sufficient to pierce the veil of corporate fiction of the two corporations. 4. Unit Severance and Globe Doctrine Industrial/Craft Union



70 | P a g e

Kapisanan ng mga Manggagawa sa Manila Railroad Co. v. Yard Crew Union Railroad Engineering Department Union, Manila Railroad Co., 109 Phil. 1143 (60) Petition for review by certiorari Facts: − 7 March 1955: The Kapisanan ng mga Manggagawa filed a petition praying that it be certified as the exclusive bargaining agent in the Manila Railroad Company. − The respondent court found (affirmed by the SC) three unions appropriate for purposes of collective bargaining: − the unit of locomotive drivers, firemen, assistant firemen and motormen – otherwise known as the engine crew unit − the unit of conductors, assistant conductors, unit agents, assistant route agents and train posters, otherwise known as the train crew unit, − and the unit of all the rest of the company personnel, except the supervisors, temporary employees, the members of the Auditing Department, the members of the security guard and professional and technical employees, referred to by the respondent court as the unit of the rest of the employees − To these three units, the following unions were respectively certified as the exclusive bargaining agents: − The Union de Maqunistas, Fogoneros, Ayudantes y Motormen − Union de Empleados de Trenes (conductors) − the Kapisanan ng mga Manggagawa sa Manila Railroad Company. − After the decision had become final, the Manila Railroad Yard Crew Union, the Station Employees' Union and the Railroad Engineering Department Union each filed petitions praying that it be defined as a separate bargaining unit. They were legitimate labor organizations with certificates of registration in the Department of Labor. − The Kapisanan and the Company opposed the separation of the three units. − The respondent court ordered a plebiscite to be conducted among the employees in the three proposed groups (the Engineering Department, the Station Employees and the Yard Crew Personnel), directing the employees to vote, in a secret ballot to be conducted by the Court, on the question of whether or not they desire to be separated from the unit of the rest of the employees being represented by the Kapisanan. − The respondent court also declared that the collective bargaining agreement could not be a bar to another certification election because one of its signatories, the Kapisanan President Vicente Olazo, was a supervisor. Issues: − W/N the appeal orders are interlocutory in nature − W/N the order of the respondent court, granting groups of employees to choose whether or not they desire to be separated from the certified unit to which they belong, during the existence of a valid bargaining contract entered into by a union close to the heels of its certification, is contrary to law − W/N it was error for the respondent court to hold that the bargaining agreement in question does not bar certification proceedings, only because one of the signatories for the union was adjudged by the majority of such court to be a supervisor Held: − Because of the modern complexity of the relation between both employer and union structure, it becomes difficult to determine from the evidence alone which of the several claimant groups forms a proper bargaining unit; that it becomes necessary to give consideration to the express will or desire of the employees – a practice designated as the “Globe doctrine,” which sanctions the holding of a series of elections, not for the purpose of allowing the group receiving and overall majority of votes to represent all employees, but for the specific purpose of permitting the employees in each of the several categories to select the group which each chooses as a bargaining unit. − The respondent court was simply interested “in the verification of the evidence already placed on record and submitted wherein the workers have signed manifestations and resolutions of their desire to be separated from Kapisanan.” Certainly, no one would deny the respondent court's right of full investigation in arriving at a correct and conclusive finding of fact in order to deny or grant the petitions for certification election. And one way of determining the will or desire of the employees is what the respondent court had suggested: a plebiscite – carried by secret ballot. A plebiscite and not the certification election itself. − Having in view the avowed purpose of the orders in question, one should not stretch his imagination far to see that they are clearly interlocutory, as they leave something more to be done in the trial court and do not decide one way or the other the petitions of the respondent unions. − Having reached the conclusion that the orders are not appealable, the consideration of the third issue is premature. PETITIONS WERE DISMISSED. Mechanical Department Labor Union v. CIR, 24 SCRA 925 Facts: − The Samahan ng mga Manggagawa sa Caloocan Shops filed a petition to be recognized as a separate bargaining unit. 71 | P a g e



There were three unions in the Caloocan shops of the Philippine National Railways: the Samahan, the Kapisanan ng Manggagawa sa Manila Railroad Company and the Mechanical Department Labor Union. No certification elections had been held in the last 12 months in the Caloocan shops. Both the Samahan and Mechanical had submitted different labor demands upon the management. − The petition was opposed by the management and by the Mechanical Department Labor Union. − The trial court directed the holding of a plebiscite election to determine whether the employees at the Caloocan shops desire the respondent union Samahan to be separated from Mechanical, applying the “Globe doctrine” which sanctions the holding of a series of elections, not for the purpose of allowing the group receiving and overall majority of votes to represent all employees, but for the specific purpose of permitting the employees in each of the several categories to select the group which each chooses as a bargaining unit. − Mechanical contends that the application of the “Globe doctrine” is not warranted because the workers of the Caloocan shops do not require different skills from the rest of the workers in the Mechanical Department of the Railroad Company. Issue: − W/N the “Globe doctrine” is applicable Held: − The appeal is premature, since the result of the ordered plebiscite among the workers of the Caloocan shops may be adverse to the formation of a separate unit, in which event, as stated in the appealed order, all questions raised in this case would be rendered moot and academic. − The question is primarily one of facts. The Industrial Court has found that there is a basic difference, in that those in the Caloocan shops not only have a community of interest and working conditions but perform major repairs of railway rolling stock, using heavy equipment and machineries found in said shops, while the others only perform minor repairs. It is easy to understand, therefore, that the workers in the Caloocan shops require special skill in the use of heavy equipment and machinery sufficient to set them apart from the rest of the workers. − THE ORDER APPEALED FROM IS AFFIRMED. 5. Effect Prior Agreement General Rubber and Footwear Corporation v. Bureau of Labor Relations, 155 SCRA 283 (87) Petitioner is in the business of manufacturing rubber sandals. In 1982 it executed a CBA with General Rubber Workers Union. In 1985 the daily paid rank and file employees seeking to negotiate a new CBA formed another union ANGLO. The monthly paid employees also formed their own collective bargaining unit NATU. NATU filed a petition for direct certification with the BLR. The petition was approved by the BLR. This petition and ruling of the BLR was opposed by the petitioner because it created 2 bargaining units within the petitioner corporation. The only reason that the monthly paid employees formed their own union was because they were excluded from the 1982 CBA which provided that managerial employees, and monthly paid employees are excluded from the bargaining unit. The issues are 1. whether monthly paid employees are managerial employees and 2. whether one corporation may have 2 bargaining units. Held. The monthly paid employees are not managerial. They merely recommend. They are not prohibited from forming a union. There may be 2 bargaining units in a corporation. Although proliferation of unions is discouraged, the right to self organization for purposes of collective bargaining cannot be abridged. It is the fault of the corporation-petitioner that historically the monthly paid employees have been excluded from the bargaining unit. The petitioner corporation has only itself to blame. Previous agreements preventing the monthly paid employees from joining the rank and file union or from forming their own union can not bind subsequent unions from forming their bargaining units because it is a curtailment of their right to organize. The monthly paid rank and file employees should be allowed to join the union of the daily paid rank and file employees or to form their own rank and file union. De la Salle University v. De La Salle University Employees Association, 330 SCRA 363 (00) After the expiration of the CBA formed in 1986, the union negotiated with the university in 1989 for a new CBA. One of the unresolved issues was the scope of the bargaining unit specifically WON computer operators and discipline officers were included in the CBA and WON employees of the college of st. benilde may be included in the bargaining unit. Express exclusion of the computer operators and discipline officers from the bargaining unit in 1986 does not bar any renegotiation for the future inclusion. Held. Computer operators and discipline officers are not confidential employees. These jobs are mainly clerical and non confidential. They should be included in the bargaining unit. Employees of St. Benilde belong to a different corporation/institution and hence should not be included in the bargaining unit. Previous agreements do not bar any renegotiation for any future CBA. The parties may renew or modify preexisting agreements. Other issues discussed where the ff. The inclusion of the union shop provision in addition to the existing maintenance of membership clause is affirmed. The legal protection granted to the right to refrain from joining is withdrawn by operation of law where the labor union and the employer have agreed to a closed shop. The policy of last in - first out in terminating or transferring employees is not acceptable. The university has the right to adopt valid grounds as basis of terminating or transferring employees. It is a management prerogative. With regard to the issue of salary increases, the issue is remanded to the voluntary arbitrator to be decided upon on the basis of the externally audited financial statements. 72 | P a g e

C. Changes – Composition San Miguel Corporation, etc. v. San Miguel, etc. 467 SCRA 107 (05) Facts -CA affirmes DOLE Undersecretary for Labor Relations, Rosalinda Dimapilis-Baldoz, ordering the immediate conduct of a certification election among the petitioner’s rank-and-file employees. - Federation of Free Workers (FFW/ respondent) filed a petition for certification election with the DOLE Regional Office No. VII. It sought to be certified and to represent the permanent rank-and-file monthly paid employees of the petitioner. The following documents were attached to the petition: (1) a Charter Certificate certifying that respondent as of that date was duly certified as a local or chapter of FFW; (2) a copy of the constitution of respondent prepared by its Secretary, Noel T. Bathan and attested by its President, Wilfred V. Sagun; (3) a list of respondent’s officers and their respective addresses, again prepared by Bathan and attested by Sagun; (4) a certification signifying that respondent had just been organized and no amount had yet been collected from its members, signed by respondent’s treasurer Chita D. Rodriguez and attested by Sagun; and (5) a list of all the rank-and-file monthly paid employees of the Mandaue Packaging Products Plants and Mandaue Glass Plant prepared by Bathan and attested by Sagun. -SMC (Petitioner) filed a motion to dismiss the petition for certification election on the sole ground that herein respondent is not listed or included in the roster of legitimate labor organizations based on the certification issued by the Officer-In-Charge, Regional Director of the DOLE Regional Office No. VII, Atty. Jesus B. Gabor. -Respondent submitted to the Bureau of Labor Relations the same documents earlier attached to its petition for certification. The accompanying letter, signed by respondent’s president Sagun, stated that such documents were submitted in compliance with the requirements for the creation of a local/chapter pursuant to the Labor Code and its Implementing Rules; and it was hoped that the submissions would facilitate the listing of respondent under the roster of legitimate labor organizations.The Chief of Labor Relations Division of DOLE Regional Office No. VII issued a Certificate of Creation of Local/Chapter No. ITD. I-ARFBT-058/98, certifying that from 30 July 1998, respondent has acquired legal personality as a labor organization/worker’s association, it having submitted all the required documents. Issues 1. WON the inclusion of the two alleged supervisory employees in appellee union’s membership amounts to fraud, misrepresentation, or false statement within the meaning of Article 239(a) and (c) of the Labor Code. 2. WON subsequent developments change the disposition of the case Held: 1. No, it does not. - Under the law, a managerial employee is “one who is vested with powers or prerogatives to lay down and execute management policies and/or to hire, transfer, suspend, layoff, recall, discharge, assign or discipline employees.” A supervisory employee is “one who, in the interest of the employer, effectively recommends managerial actions if the exercise of such recommendatory authority is not merely routinary or clerical in nature but requires the use of independent judgment.’” Finally, “all employees not falling within the definition of managerial or supervisory employee are considered rank-and-file employees”. It is also well-settled that the actual functions of an employee, not merely his job title, are determinative in classifying such employee as managerial, supervisory or rank and file. Good faith is presumed in all representations, an essential element of fraud, false statement and misrepresentation in order for these to be actionable is intent to mislead by the party making the representation. In this case, there is no proof to show that Bathan, or appellee union for that matter, intended to mislead anyone. If this was appellee union’s intention, it would have refrained from using a more precise description of the organization instead of declaring that the organization is composed of ‘rank and file monthlies’. Hence, the charge of fraud, false statement or misrepresentation cannot be sustained. - Even if they are supervisory employees, no action can be done that emasculates the right to self-organization and the promotion of free trade unionism. We take administrative notice of the realities in union organizing, during which the organizers must take their chances, oftentimes unaware of the fine distinctions between managerial, supervisory and rank and file employees. The grounds for cancellation of union registration are not meant to be applied automatically, but indeed with utmost discretion. Where a remedy short of cancellation is available, that remedy should be preferred. In this case, no party will be prejudiced if Bathan were to be excluded from membership in the union. The vacancy he will thus create can then be easily filled up through the succession provision of appellee union’s constitution and by-laws. What is important is that there is an unmistakeable intent of the members of appellee union to exercise their right to organize. We cannot impose rigorous restraints on such right if we are to give meaning to the protection to labor and social justice clauses of the Constitution. 2. No, it does not affect the case. The allegation that the bargaining unit that respondent sought to represent is no longer the same because of the dynamic nature of petitioner’s business, a lot of changes having occurred in the work environment, and that four of respondent’s officers are no longer connected with petitioner have no effect on the Court’s ruling that a certification election should be immediately conducted with respondent as one of the available choices. Petitioner’s bare manifestations adduce no reason why the certification election should not be conducted forthwith. If there are matters that have arisen since the filing of the petition that serve to delay or cancel the election, these can be threshed out during the pre-election conferences. Neither is the fact that some of respondent’s officers have since resigned from petitioner of any moment. The local/chapter retains a separate legal personality from that of its officers or members that remains viable notwithstanding any turnover in its officers or members. DISPOSITION WHEREFORE, the Petition is DENIED. 6. Determining Agency – 232 73 | P a g e

Art. 232. Prohibition on certification election. The Bureau shall not entertain any petition for certification election or any other action which may disturb the administration of duly registered existing collective bargaining agreements affecting the parties except under Articles 253, 253-A and 256 of this Code. (As amended by Section 15, Republic Act No. 6715, March 21, 1989) Agency and Finality Order Filoil Refinery Corporation v. Filoil Supervisory and Confidential Employees Association, 46 SCRA 512 (72) Respondent association is a duly registered labor organization . It is composed exclusively of the supervisory and confidential employees of petitioner corporation. It filed on February 18, 1965 with the industrial court its petition for certification as the sole and exclusive collective bargaining agent of all of petitioner's supervisory and confidential employees working at its refinery in Rosario, Cavite. Filoil filed a motion to dismiss the petition on the grounds of lack of cause of action and of respondent court's lack of jurisdiction over the subject-matter, under its claim that supervisors are not employees within the meaning of Republic Act 875, the Industrial Peace Act, and that since they are part of management, they do not have the right to bargain collectively although they may organize an organization of their own. ISSUE: The right of supervisors and confidential employees to organize a labor association and to bargain collectively with their employer. HELD: Since the confidential employees are very few in number and are by practice and tradition identified with the supervisors in their role as representatives of management vis-à-vis the rank and file employees, such identity of interest has allowed their inclusion in the bargaining of supervisors for purposes of collective bargaining in turn as employees in relation to the company as their employer. This identity of interest logically calls for their inclusion in the same bargaining unit and at the same time fulfills the law’s objective of insuring to them the full benefit of their right to self-organization and to collective bargaining, which could hardly be accomplished if the respondent association’s membership were to be broken up into five separate ineffective tiny units. Creating fragmentary units would not serve the interest of industrial peace. The breaking up of bargaining units into tiny units will greatly impair their organizational value. Since the confidential employees are very few and are identified with the supervisors in their role as representatives of management vis-a-vis the rank and file employees, such identity of interest has allowed their inclusion in the bargaining unit of supervisors-managers for purposes of collective bargaining. Industrial court enjoys a wide discretion in determining the procedure necessary to insure the fair and free choice of bargaining representations by employees. Action in deciding upon an appropriate unit for collective bargaining purposes is discretionary. Its judgment is entitled to finality, unless its action is arbitrary or capricious. B. UNION REPRESENTATION: ESTABLISHING UNION MAJORITY STATUS 1. PRE-CONDITION – EMPLOYER-EMPLOYEE RELATIONSHIP Election – Pre-requisite Allied Free Workers Union v. Cia Maritima, 19 SCRA 258 (67) NATURE Petitions for review by certiorari of CIR decision FACTS - This is a consolidation of 3 cases involving both parties - Respondent Compania Maritima (MARITIMA), a local corp. engaged in shipping entered into a contract for lease of services with petitioner Allied Free Workers’ Union (AFWU), a duly registered legitimate labor union. In the contract, it was stipulated that AFWU will do and perform all the work of stevedoring and arrastre services of all vessels or boats of MARITIMA in Iligan City; that the contract is good and valid for 1 month starting Aug.12, 1952, but may be renewed by agreement of the parties with the reservation that MARITIMA has the right to revoke said contract even before the expiration of the term, if and when AFWU fails to render good service. - Towards the end of 1953, MARITIMA complained to AFWU of unsatisfactory and inefficient service. To remedy the situation, MARITIMA was forced to hire extra laborers from among “stand-by” workers not affiliated to any union. - On July 1954, AFWU sent a written proposal to MARITIMA for a CBA, but the latter did not reply. Thereafter, AFWU instituted an action in the CIR praying that it be certified as the sole and exclusive bargaining unit composed of all the laborers doing arrastre and stevedoring work for MARITIMA, to which action MARITIMA answered, alleging lack of ER-EE relationship. On Aug.1954, MARITIMA informed AFWU of the termination of the contract because of the inefficient service rendered by the latter which had adversely affected its business. The termination was to take effect as of Sept.1, 1954. MARITIMA then contracted with the Iligan Stevedoring Union for the arrastre and stevedoring work. The latter agreed to perform the work subject to the same terms and conditions of the contract with AFWU. The new agreement was to be carried out on Sept.1, 1954. - On Aug.26, 1954, AFWU charged MARITIMA of unfair labor practices (ULPs) before the CIR. MARITIMA answered, again denying the ER-EE relationship between the parties. On Sept.9, 1954, MARITIMA filed an action to rescind the contract, enjoin AFWU members from doing arrastre and stevedoring work in connection with its vessels, and for recovery of damages against AFWU and its officers. The CFI ordered the rescission of the contract and permanently enjoined AFWU members from performing work in connection with MARITIMA's vessels. AFWU was later able to secure a writ of preliminary injunction ordering the maintenance of the status quo prior to Jan.6, 1961. Thus, after Jan.18, 1961, AFWU laborers were again back doing the same work as before. 74 | P a g e

- On Nov.4, 1963, after almost 10 years, the CFI finally rendered its decision: In pursuance of the provisions of Sec.12 of R.A. 875 and the Rules of this court on certification election, the Honorable Secretary of Labor or any of his authorized representative is hereby requested to conduct certification election among all the workers and/or stevedores working in the wharf of Iligan City who are performing stevedoring and arrastre service aboard Compania Maritima vessels docking at Iligan City port in order to determine their representative for collective bargaining with the employer, whether these desire to be represented by the petitioner Allied Free Workers Union or neither; and upon termination of the said election, the result thereof shall forthwith be submitted to this court for further consideration. From this ruling, both parties appealed, AFWU claiming that it should be declared outright as the majority union while MARITIMA contends that said court could not even have correctly ordered a certification election considering that there was an absence of ER-EE relationship between it and said laborers. ISSUE WON the order of a certification election by the CIR was proper. (WON there was an ER-EE relationship between AFWU and MARITIMA) HELD NO. Before a certification election can be held, there must exist an ER-EE relationship between the ER and the petitioner union. Ratio The duty to bargain collectively exists only between the “employer” and its “employees.” Where there is no duty to bargain collectively, it is not proper to hold certification elections in connection therewith. Reasoning In its findings, the CIR observed that after the rescission, the AFWU laborers continued working in accordance with the “cabo system,” which was the prevailing custom in the place. Under this system, the union was an independent contractor. The CIR also made a finding that prior to the contract between MARITIMA and AFWU, the former had an oral arrastre and stevedoring agreement with another union, the Iligan Laborers Union (ILU), which agreement was also based on the “cabo” system. After unsatisfactory service, MARITIMA cancelled this oral contract and entered into a new contract with AFWU, the terms and conditions of which were similar to the oral contract with ILU. The written contract between AFWU and MARITIMA was signed under the assurance by AFWU that the same arrangement previously had with the former union regarding performance and execution of arrastre and stevedoring contract be followed in accordance with the custom of such kind of work in Iligan. Thus, petitioner union operated as a labor contractor under the so-called “cabo” system. - From these findings, Insofar as the working agreement was concerned, there was no real difference between the contract and the prior oral agreement. Both were based on the “cabo” system. Hence, since the parties observed the "cabo" system after the rescission of the contract, and since the characteristics of said system show that the contracting union was an independent contractor, it is reasonable to assume that AFWU continued being an independent contractor of MARITIMA. And, being an independent contractor, it could not qualify as an "employee". With more reason would this be true with respect to the laborers. Moreover, there is no evidence at all regarding the characteristics of the working arrangement between AFWU and MARITIMA after the termination of the CONTRACT. All we have to go on is the court a quo's finding that the “cabo” system was observed-a system that negatives employment relationship. - Since the only function of a certification election is to determine, with judicial sanction, which union shall be the official representative or spokesman of the “employees” will be, there being no ER-EE relationship between the parties disputants, it follows that there is neither a duty to bargain collectively. Thus, the order for certification election in question cannot be sustained. Disposition appealed decision of the CIR is AFFIRMED insofar as it dismissed the charge of ULP, but REVERSED and SET ASIDE insofar as it ordered the holding of a certification election. The petition for certification election should be DISMISSED. Dunlop v. Secretary, 300 SCRA 120 (98) FACTS: Respondent union filed a Petition for Certification Election among the supervisory, office and technical employees of the petitioner company before DOLE Regional Office No. III, San Fernando, Pampanga. Petitioner company filed its Answer with Motion to Dismiss based on three (3) grounds, namely: (1) that the respondent union is comprised of supervisory and rank-and-file employees and cannot act as bargaining agent for the proposed unit; (2) that a single certification election cannot be conducted jointly among supervisory and rank-and-file employees; and (3) that the respondent union lacks legal standing since it failed to submit its books of accounts. In its Reply, the respondent union alleged that its members are supervisors and not rank-and-file employees, paid monthly by the petitioner company. Mediator Arbiter Ma. Carmen A. Espinosa granted the petition for certification election. Respondent Secretary of Labor and Employment affirmed the Arbiter's decision and denied the petitioner’s Motion for Reconsideration; hence this petition. ISSUE: whether or not the respondent union can file a petition for certification election to represent the supervisory employees of the petitioner company HELD, RATIO: Order and resolution of public respondent are annulled and set aside. The respondent union has no legal right to file a certification election to represent a bargaining unit composed of supervisors for so long as it counts rank-and-file employees among its members. Supervisors can be an appropriate bargaining unit. This is in accord with our repeated ruling that "[a]n appropriate bargaining unit is a group of employees of a given employer, composed of all or less than the entire body of employees, which the collective interests of all the employees, consistent with equity to the employer, indicate to be best suited to serve reciprocal rights and duties of the parties under the collective bargaining provisions of law. A unit to be appropriate must effect a grouping of employees who have substantial, mutual interests in wages, hours, working conditions and other subjects of collective bargaining."

75 | P a g e

The test of supervisory status as repeatedly ruled is whether an employee possesses authority to act in the interest of his employer, which authority should not be merely routinary or clerical in nature but requires the use of independent judgment. Corrollarily, what determines the nature of employment is not the employee's title, but his job description. The list of monthly paid employees submitted by the petitioner company contains the names of about twenty seven (27) supervisory employees, six (6) managerial employees, one (1) confidential employee and twenty six (26) office and technical employees holding various positions. The list reveals that the positions occupied by the twenty six (26) office and technical employees are in fact rank-and-file positions, i.e., A/C mechanic, draftsmen, storemen, etc. These positions cannot be considered as supervisory positions for they do not carry the authority to act in the interest of the employer or to recommend managerial actions. It is not decisive that these employees are monthly paid employees. Their mode of compensation is usually a matter of convenience and does not necessarily determine the nature and character of their job. Based Art. 245, a labor organization composed of both rank-and-file and supervisory employees is no labor organization at all. It cannot, for any guise or purpose, be a legitimate labor organization. Not being one, an organization which carries a mixture of rank-and-file and supervisory employees cannot possess any of the rights of a legitimate labor organization, including the right to file a petition for certification election for the purpose of collective bargaining. 2. METHODS OF ESTABLISHING MAJORITY STATUS A. Purpose Port Workers Union of the Philippines v. Laguesma, 207 SCRA 329 (92) FACTS: - The collective bargaining agreement of the International Container Terminal Services, Inc. (ICTSI) with private respondents Associate Port Checkers and Workers Union (APCWU), the incumbent union, was due to expire on April 14, 1990. Other unions were seeking to represent the laborers in the negotiation of the next CBA and were already plotting their moves. - on March 14, 1990, when the Sandigan ng Manggagawa sa Daungan (SAMADA) filed a petition for certification election. The consent signatures of at least 25% of the employees in the bargaining unit were submitted on March 26, 1990, or eleven days after the petition. - On April 2, 1990, herein petitioner Port Workers Union of the Philippines (PWUP) filed a petition for intervention. - another petition for certification election was filed by the Port Employees Association and Labor Union (PEALU), on April 6, 1990. The consent signatures were submitted on May 11, 1990, or thirty-five days after the filing of the petition. - On April 26, 1990, APCWU filed a motion to dismiss them on the ground that they did not comply with the requirement set forth in Section 6, Rule V, Book V of the Implementing Rules, quoted in part as follows: In a petition involving an organized establishment or enterprise where the majority status of the incumbent collective bargaining union is questioned through a verified petition by a legitimate labor organization, the Med-Arbiter shall immediately order the certification election by secret ballot if the petition is filed during the last sixty (60) days of the collective bargaining agreement and supported by the written consent of at least twenty-five percent (25%) of all the employees in the bargaining unit. Any petition filed before or after the sixty-day freedom period shall be dismissed outright. The twenty-five percent (25%) requirement shall be satisfied upon the filing of the petition, otherwise the petition shall be dismissed . (Emphasis supplied.) - the Med-Arbiter dismissed the consolidated petitions. PWUP appealed to the Secretary of Labor, arguing that Article 256 of the Labor Code did not require the written consent to be submitted simultaneously with the petition for certification election. DOLE Undersecretary Laguesma affirmed the order of the Med-Arbiter and dismissed PWUP's appeal. - ICTSI and APCWU resumed negotiations for a new collective bargaining agreement, which was concluded on September 28, 1990. This was ratified by a majority of the workers in the bargaining unit. - PWUP claims grave abuse of discretion on the part of the public respondent in the application of Article 256 of the Labor Code. The article provides in part as follows: Art. 256. Representation issue in organized establishments. ? In organized establishments, when a verified petition questioning the majority status of the incumbent bargaining agent is filed before the Department of Labor and Employment within the sixty-day period before the expiration of the collective bargaining agreement, the Med-Arbiter shall automatically order an election by secret ballot when the verified petition is supported by the written consent of at least twenty-five (25%) percent of all the employees in the bargaining unit to ascertain the will of the employees in the appropriate bargaining unit. . . . - The petitioner argues that under this article, the Med-Arbiter should automatically order election by secret ballot when the petition is supported by at least 25% of all employees in the bargaining unit. SAMADA and PEALU substantially complied with the law when they submitted the required consent signatures several days after filing the petition. The petitioner complains that the dismissal of the petitions for certification election, including its own petition for intervention, had the effect of indirectly certifying APCWU as the sole and exclusive bargaining representative of the ICTSI employees. - Private respondent ICTSI maintains that the dismissal was based on Article 256 of the Labor Code as implemented by Section 6, Rule V, Book V of the Implementing Rules, quoted above. Moreover, under Section 10, Rule V, Book V of the Implementing Rules, decisions of the Secretary in certification election cases shall be final and unappealable.

76 | P a g e

- For its part, APCWU questions PWUP's personality in these proceedings in view of the lack of consent signatures in its petition, and argues as well that the petitioner has no authority to represent SAMADA or PEALU, which had not appealed. The private respondent also invokes Tupas and maintains that the ratification of the new CBA by the majority of the workers was an affirmation of their membership in the union that negotiated that agreement. ISSUE: WON there was indeed grave abuse of discretion amounting to lack or excess of jurisdiction on the part of public respondents when they dismissed the petitions for certification election because the consent signatures had not been submitted simultaneously with the petition HELD: YES - pursuant to the constitutional provision guaranteeing workers the right to self-organization and collective bargaining, "the constant and unwavering policy of the Court" has been "to require a certification election as the best means of ascertaining which labor organization should be the collective bargaining representative." - The certification election is the most democratic and expeditious method by which the laborers can freely determine the union that shall act as their representative in their dealings with the establishment where they are working. The holding of a certification election is a statutory policy that should not be circumvented. - the administrative rule requiring the simultaneous submission of the 25% consent signatures upon the filing of petition for certification election should not be strictly applied to frustrate the determination of the legitimate representative of the workers. Significantly, the requirement in the rule is not found in Article 256, the law it seeks to implement. This is all the more reason why the regulation should at best be given only a directory effect. - It is not denied that the petition to intervene filed by PWUP did not carry the 25% consent signatures, but that the requirement is in fact not applicable to a petition in intervention. - the certification election is not litigation but a mere investigation of a non-adversary character where the rules of procedure are not strictly applied. Technical rules and objections should not hamper the correct ascertainment of the labor union that has the support of confidence of the majority of the workers and is thus entitled to represent them in their dealings with management. - Deviation from the contract-bar rule is justified only where the need for industrial stability is clearly shown to be imperative. Subject to this singular exception, contracts where the identity of the authorized representative of the workers is in doubt must be rejected in favor of a more certain indication of the will of the workers. The certification election is the best method of determining the will of the workers on the crucial question of who shall represent them in their negotiations with the management for a collective bargaining agreement that will best protect and promote their interests. It is essential that there be no collusion against this objective between an unscrupulous management and a union covertly supporting it while professing its loyalty to labor, or at least that the hopes of labor be not frustrated because of its representation by a union that does not enjoy its approval and support. It is therefore sound policy that any doubt regarding the real representation of the workers be resolved in favor of the holding of the certification election. This is preferable to the suppression of the voice of the workers through the prissy observance of technical rules that will exalt procedure over substantial justice. DISPOSITION Petition GRANTED. Reyes v. Trajano, 209 SCRA 484 (92) Facts: • In the certification election for the exclusive representative of the bargaining unit for the Tri-Union Industries Corporation, two unions vie for the support of the workers. • The competing unions were Tri-Union Employees Union-Organized Labor Association in Line Industries and Agriculture (TUEU-OLALIA) and Trade Union of the Philippines and Allied Services (TUPAS). 348 workers were supposed to participate in the certification election but only 240 cast their vote. • Among the 240 employees, 141 were members of the Iglesia Ni Kristo who all voted for NO UNION. Their votes were challenged on the ground that they should not be allowed to vote as they are not members of any union and refused to participate in the previous certification elections. It is also argued that their religion does not allow membership in labor unions and thus they should not be allowed to vote in the certification elections. • The INK employees filed a case before the Med-Arbiter to protest the exclusion of their votes. The Med-Arbiter dismissed the petition. On appeal to the BLR, Assistant Labor Secretary Cresenciano Trajano affirmed the Med-Arbiter’s decision as the INK employees are bereft of legal personality to protest their disenfranchisement. Issue: WON the votes of the 141 members of the INK can be validly excluded from the certification election. Held: • Guaranteed to all employees or workers is the right to self-organization and to form, join or assist labor organizations of their own choosing for purposes of collective bargaining. • The right not to join, affiliate with or assist any union and to disaffiliate or resign from a labor organization is subsumed in the right to join affiliate with or assist any union. The said right includes the right to refuse and refrain from exercising such right. Just as anyone cannot be denied the exercise of a right granted by law, neither can they be compelled to exercise such a conferred right. • The purpose of a certification election is to determine the wishers of majority of the employees in the appropriate bargaining unit as to whether to be or not to be represented by a labor organization and if yes, by what particular labor organization. If the results show that no union is desired by the workers, the minority cannot impose their will on the majority and they will have to wait for the next certification election to attempt a change. 77 | P a g e

• •

The fact that the INK members, as employees voted for NO UNION, they were simply exercising that right of self-organization albeit in its negative aspect. All bona fide employees possess such right. The fact that they did not participate in previous certification elections nor the fact that they are not members of any union does not deprive them of such right.

B. Elections – Certification Election; Consent Election; and Run-Off Election – 256 & 257, 258, 258-A Art. 256. Representation issue in organized establishments. In organized establishments, when a verified petition questioning the majority status of the incumbent bargaining agent is filed before the Department of Labor and Employment within the sixty-day period before the expiration of the collective bargaining agreement, the Med-Arbiter shall automatically order an election by secret ballot when the verified petition is supported by the written consent of at least twenty-five percent (25%) of all the employees in the bargaining unit to ascertain the will of the employees in the appropriate bargaining unit. To have a valid election, at least a majority of all eligible voters in the unit must have cast their votes. The labor union receiving the majority of the valid votes cast shall be certified as the exclusive bargaining agent of all the workers in the unit. When an election which provides for three or more choices results in no choice receiving a majority of the valid votes cast, a run-off election shall be conducted between the labor unions receiving the two highest number of votes: Provided, that the total number of votes for all contending unions is at least fifty percent (50%) of the number of votes cast. At the expiration of the freedom period, the employer shall continue to recognize the majority status of the incumbent bargaining agent where no petition for certification election is filed. (As amended by Section 23, Republic Act No. 6715, March 21, 1989) Art. 257. Petitions in unorganized establishments. In any establishment where there is no certified bargaining agent, a certification election shall automatically be conducted by the Med-Arbiter upon the filing of a petition by a legitimate labor organization. (As amended by Section 24, Republic Act No. 6715, March 21, 1989) Art. 258. When an employer may file petition. When requested to bargain collectively, an employer may petition the Bureau for an election. If there is no existing certified collective bargaining agreement in the unit, the Bureau shall, after hearing, order a certification election. All certification cases shall be decided within twenty (20) working days. The Bureau shall conduct a certification election within twenty (20) days in accordance with the rules and regulations prescribed by the Secretary of Labor. Distinctions – Certification and Consent Elections Warren Manufacturing Workers Union v. Bureau of Labor Relations, 159 SCRA 387 (88) Algire v. De Mesa, 237 SCRA 647 (94) NATURE Petition for certiorari to nullify and set aside a decision of the Secretary of Labor FACTS - Universal Robina Textile Monthly Salaried Employees Union (URTMSEU) filed on September 4, 1990 a petition for the holding of an election of union officers with the Arbitration Branch of the Department of Labor and Employment (DOLE). This was done through De Mesa. - DOLE's med-arbiter Rolando S. de la Cruz issued an Order dated October 19, 1990 directing that such an election be held. - In the pre-election conference, it was agreed that the election by secret ballot be conducted on November 16, 1990 between Catalino Algire, et al. (petitioner) and Regalado de Mesa, et al. (respondents) under the supervision of DOLE through its duly appointed representation officer. - In filling out the ballots, instructions were given to mark choices with either a check mark or an ‘X’ mark. There should also be no other markings on the ballot. - De Mesa and Algire both got 133 votes each. Total votes cast were 272. 6 were declared as spoiled ballots. - Algire filed a petition, alleging that one of the ballots which had two check marks was erroneously declared to be a spoiled ballot. The checks supposedly made it clear as to the choice made by the voter. - The med-arbiter (De la Cruz) issued an order in Algire’s favor and certified the latter’s group to be the unions validly elected officers. - De Mesa appealed to the DOLE secretary which was granted. Another order for a new election of officers was made by the Med-Arbiter and another pre-election conference was scheduled. - Algire’s group filed a motion for reconsideration which was denied for lack of merit. - Algire, et. al. contend that a representation officer (referring to a person duly authorized to conduct and supervise certification elections in accordance with Rule VI of the Implementing Rules and Regulations of the Labor Code) can validly rule only on on-the-spot questions arising from the conduct of the elections, but the determination of the validity of the questioned ballot is not within his competence. ISSUE WON the act of the DOLE secretary in denying Algire’s motion was in excess of its authority since the case is an intra-union activity HELD NO Ratio The certification election was an agreed one, the purpose being merely to determine the issue of majority representation of all the workers in the appropriate collective bargaining unit. It is a separate and distinct process and has nothing to do with the import and effort of a certification election. Reasoning - What is at question in this case was a consent election, not a certification election. 78 | P a g e

- If indeed petitioner's group had any opposition to the representation officer's ruling that the questioned ballot was spoiled, it should have done so seasonably during the canvass of votes. Its failure or inaction to assail such ballot's validity shall be deemed a waiver of any defect or irregularity arising from said election. Disposition Petition is DENIED and the challenged decision is hereby AFFIRMED. 1. No Direct Certification Colgate Palmolive Philippines v. Ople, 163 SCRA 323 (88) FACTS - The respondent Union filed a Notice of Strike with the Bureau of Labor Relations (BLR) on ground of unfair labor practice consisting of alleged refusal to bargain, dismissal of union officers/members; and coercing employees to retract their membership with the union and restraining non-union members from joining the union. - After efforts at amicable settlement proved unavailing, the Office of the MOLE, upon petition of petitioner assumed jurisdiction over the dispute pursuant to Article 264 (g) of the Labor Code. - Respondent Minister rendered a decision finding no merit in the Union's Complaint for unfair labor practice allegedly committed by petitioner as regards the alleged refusal of petitioner to negotiate with the Union, and the secret distribution of survey sheets allegedly intended to discourage unionism and at the same time respondent Minister directly certified the respondent Union as the collective bargaining agent for the sales force in petitioner company and ordered the reinstatement of the three salesmen to the company on the ground that the employees were first offenders. - Petitioner filed a Motion for Reconsideration which was denied by respondent ISSUE WON respondent Minister exceeded his power when he certified respondent Union as the exclusive bargaining agent of the company's salesmen since the case is not a representation proceeding as described under the Labor Code and the Union did not pray for certification but merely for a finding of unfair labor practice imputed to petitioner-company. HELD YES. - The procedure for a representation case is outlined in Arts. 257-260 of the Labor Code, in relation to the provisions on cancellation of a Union registration under Arts. 239-240 thereof, the main purpose of which is to aid in ascertaining majority representation. - The requirements under the law, specifically Secs. 2, 5, and 6 of Rule V, Book V, of the Rules Implementing the Labor Code are all calculated to ensure that the certified bargaining representative is the true choice of the employees against all contenders. The Constitutional mandate that the State shall "assure the rights of the workers to self-organization, collective bargaining, security of tenure and just and humane conditions of work," should be achieved under a system of law such as the aforementioned provisions of the pertinent statutes. - When an overzealous official by-passes the law on the pretext of retaining a laudable objective, the intendment or purpose of the law will lose its meaning as the law itself is disregarded. - When respondent Minister directly certified the Union, he in fact disregarded this procedure and its legal requirements. There was therefore failure to determine with legal certainty whether the Union indeed enjoyed majority representation. - Contrary to the respondent Minister's observation, the holding of a certification election at the proper time is not necessarily a mere formality as there was a compelling legal reason not to directly and unilaterally certify a union whose legitimacy is precisely the object of litigation in a pending cancellation case filed by certain "concerned salesmen," who also claim majority status. - Even in a case where a union has filed a petition for certification elections, the mere fact that no opposition is made does not warrant a direct certification. - More so, when the records of the suit show that the required proof was not presented in an appropriate proceeding and that the basis of the direct certification was the Union's mere allegation in its position paper that it has 87 out of 117 regular salesmen. - Respondent Minister merely relied on the self-serving assertion of the respondent Union that it enjoyed the support of the majority of the salesmen, without subjecting such assertion to the test of competing claims. As pointed out by petitioner in its petition, what the respondent Minister achieved in rendering the assailed orders was to make a mockery of the procedure provided under the law for representation cases. Disposition Order REVERSED and SET ASIDE. 2. Employer Certification – Employer Voluntary Samahang Manggagawa sa Permex v. Secretary of Labor, 286 SCRA 692 (98) Facts: On January 15, 1991, a certification election was conducted among employees of Permex Producter and Exporter Corp, where maximum vote was "No Union" (existing union then was National Federation of Labor). Subsequesnt to the CE, some employees formed a labor organization known as Samahang Manggagawa sa Permex, registered it with DOLE then later affiliated with Philipine Integrated Industries Labor Union. On August 16, 1991, Samahan wrote the company requesting recognition as the sole and exclusive bargaining representatives of its employees. Permex recognized Samahan and entered into a CBA with it, which was ratified by majority its employees and certified by DOLE.On Feb 25, 1992, NFL filed a petition for CE which was diemissed by Med-Arbiter. Upon appeal, SOLE Usec LAguesma set aside Med-Arbiter and ordered CE with the following choices: 1) NFL, 2) Samahan, 3) No Union. Samahan moved for reconsideration but was denied, hence the current petiton. 79 | P a g e

Issue: 1) WON the order of holding a ceritifaction election is valid despite recognition by the company and it's entering into a CBA with Samahan. Held: Yes. CE is proper. Samaahan could not be the exclusive bargaining representative by virtue of employee recognition. Quoting Ilaw at Buklod ng Manggagawa v Ferrer-Calleja, "if a union asks the employer to voluntarily recognize it, it in efefct asks the employer to certify it as its bargaining representative of the employees - a certification which the employer has no authority to give for it is the employees' prerogative, not the employer's to determine whether they want a union to represent them. By EO 111, direct certification has been discontinued as the method for selecting the exclusive bargaining agent. Petitioner can neither invoke the contract-bar rule stating that petition for CE may only be entertained within 60 days prior to the expiration of an existing collective bargaining agreement. This case is exempted from contract bar rule notwithsatnding the existence of a CBA as Samahan entered into a CBA with Permex when its status as an exclusive bargaining agent was not yet established. 3. Effect One Union Only George and Peter Lines, Inc. v. Associated Labor Union, 134 SCRA 82 (85) NATURE Petition for certiorari to review the decision of the Bureau of Labor Relations. FACTS - George and Peter Lines, Inc. (petitioner) is involved in shipping, while Associated Labor Unions (ALU, respondent) is a legitimate labor organization. - July 16, 1878: a Petition for Direct Certification was filed by ALU praying that it be certified as the SOLE and EXCLUSIVE bargaining representative of all the rank and file employees of petitioner corporation, there being no labor union. - Petitioner opposed the petition stating that the Union does not represent the majority of the employees concerned, and that more than 80% of the licensed/ unlicensed crew of its vessels claim they are not members of any union. - August 25, 1978: Med-Arbiter issued an Order directly certifying ALU as the sole and exclusive bargaining agent. Petitioner moved for reconsideration alleging that 80% of the employees denied their membership. Corporation moved that a certification election should be called. - Bureau of Labor Relations Director, upon examination of the documents, opined that there existed a doubt regarding the majority of status of respondent ALU because of the withdrawal of the members, and directed a certification election. - Upon a motion for reconsideration by ALU, the BLR Director reconsidered its Resolution and directly certified ALU as sole bargaining agent. ISSUE 1. WON employees of the corporation are entitled to choose their sole and exclusive bargaining representative with petitioner thru a certification election; 2. WON petitioner is entitled to file petition for certification election. HELD 1. YES Ratio Employees have the constitutional right to choose the labor organization which it desires to join. The exercise of such right would be rendered nugatory and ineffectual if they would be denied the opportunity to choose in a certification election. Reasoning The holding of a certification election is a statutory policy that should not be circumvented. - The best forum to determine if there was indeed undue pressure exerted upon the employees to retract their membership is in the certification election itself (in secret ballot where they can freely express their choice). - The fact that there are no competing Unions should not affect the freedom of choice (they can always choose ALU or ‘No Union’). DISPOSITION The Regional Office concerned of MoLE is directed to cause the holding of a certification election Compare Policy on Certification Election, 256, 257, 258 and Voluntary Recognition – BV, IR, RI (bbb); BV, IR, R VII; BV; IR, RVIII, Sec. 23 4. Religion/Past Non-Participation Reyes v. Trajano, 209 SCRA 484 (92) Facts: · In the certification election for the exclusive representative of the bargaining unit for the Tri-Union Industries Corporation, two unions vie for the support of the workers. · The competing unions were Tri-Union Employees Union-Organized Labor Association in Line Industries and Agriculture (TUEU-OLALIA) and Trade Union of the Philippines and Allied Services (TUPAS). 348 workers were supposed to participate in the certification election but only 240 cast their vote. · Among the 240 employees, 141 were members of the Iglesia Ni Kristo who all voted for NO UNION. Their votes were challenged on the ground that they should not be allowed to vote as they are not members of any union and refused to participate in the previous certification elections. It is also argued that their religion does not allow membership in labor unions and thus they should not be allowed to vote in the certification elections.

80 | P a g e

· The INK employees filed a case before the Med-Arbiter to protest the exclusion of their votes. The Med-Arbiter dismissed the petition. On appeal to the BLR, Assistant Labor Secretary Cresenciano Trajano affirmed the Med-Arbiter’s decision as the INK employees are bereft of legal personality to protest their disenfranchisement. Issue: WON the votes of the 141 members of the INK can be validly excluded from the certification election. Held: · Guaranteed to all employees or workers is the right to self-organization and to form, join or assist labor organizations of their own choosing for purposes of collective bargaining. · The right not to join, affiliate with or assist any union and to disaffiliate or resign from a labor organization is subsumed in the right to join affiliate with or assist any union. The said right includes the right to refuse and refrain from exercising such right. Just as anyone cannot be denied the exercise of a right granted by law, neither can they be compelled to exercise such a conferred right. · The purpose of a certification election is to determine the wishers of majority of the employees in the appropriate bargaining unit as to whether to be or not to be represented by a labor organization and if yes, by what particular labor organization. If the results show that no union is desired by the workers, the minority cannot impose their will on the majority and they will have to wait for the next certification election to attempt a change. · The fact that the INK members, as employees voted for NO UNION, they were simply exercising that right of self-organization albeit in its negative aspect. The respondent’s argument that the petitioners are disqualified to vote because they are not constituted into a duly organized labor union but are members of INK which prohibits its followers, in religious grounds, from joining or forming any labor organization is specious. Neither law nor jurisprudence requires that only employees affiliated with any labor org can take part in a CE. · All bona fide employees possess such right. The fact that they did not participate in previous certification elections nor the fact that they are not members of any union does not deprive them of such right. 3. CERTIFICATION ELECTION – PROCESS A. The Union as Initiating Party – 212 (h); 242 (b) Art. 212. Definitions.(h) "Legitimate labor organization" means any labor organization duly registered with the Department of Labor and Employment, and includes any branch or local thereof. Art. 242. Rights of legitimate labor organizations. A legitimate labor organization shall have the right:To be certified as the exclusive representative of all the employees in an appropriate bargaining unit for purposes of collective bargaining; San Miguel Corporation, etc. v. San Miguel, etc. 467 SCRA 107 (05) The Union filed a petition for a cert. election and sought to represent the permanent rank and file EEs of the petitioner. SMC opposed mainly on the ground that the union is not listed or included in the roster of legit orgs. Another ground was contained in a Memorandum filed by SMC. It alleged that the bargaining unit that the Union sought to represent is no longer the same because of the dynamic nature of the petitioner's business, and a lot of chnages having occurred in the work environment. Moreover, four of the Union's officers are no longer connected with petitioner. Issue: WON the changes affect the standing of the Union Held: No. Assuming thatg these manifestations were true, they have no effect on the Court's ruling that a cert. Election should be immediately conducted with respondent union as one of the choices. Petitioner's manifestations adduce no reason why the cert. Election should not be conducted. If there were matters that have arisen since the filing of the petition that serves to delay or cancel the elections, these can be threshed out during the pre-election conferences. Neithe is the fact that some of its officers have resigned of any monent. The local chapter retains a separate legal personality from that of its officers or members that remain viable nothwithstanding any turnover in its officers or members. Lopez Sugar Corporation v. Secretary of Labor and Employment, 247 SCRA 1 (95) Facts -The Med-Arbiter, sustained by the Secretary of Labor and Employment, has ruled that Art. 257 is mandatory and give him no other choice than to conduct a certification election upon the receipt of the corresponding petition. "Art. 257. Petitions in unorganized establishments. - In any establishment where there is no certified bargaining agent, a certification election shall automatically be conducted by the Med-Arbiter upon the filing of a petition by a legitimate labor organization." -National Congress of Unions in the Sugar Industry of the Philippines-TUCP ("NACUSIP-TUCP") filed with the Department of Labor and Employment ("DOLE") a petition for direct certification or for certification election to determine the sole and exclusive collective bargaining representative of the supervisory employees of herein petitioner, Lopez Sugar Corporation ("LSC"). NACUSIP-TUCP averred that it was a legitimate national labor organization; that LSC was employing 55 supervisory employees, the majority of whom were members of the union; that no other labor organization was claiming membership over the supervisory employees; that there was no existing collective bargaining agreement covering said employees; and that there was no legal impediment either to a direct certification of NACUSIP-TUCP or to the holding of a certification election. 81 | P a g e

-LSC contended it. NACUSIP-TUCP submitted Charter Certificate No. 003-89, dated 20 July 1989, of the NACUSIP-TUCP Lopez Sugar Central Supervisory Chapter. -LSC appealed to the DOLE and asseverated that the order was a patent nullity and that the Med-Arbiter acted with grave abuse of discretion, Sec. of Labor denied it. Petition for certiorari was filed. Issue WON the certification election should push through Held No, because the labor organization is not legitimate. It was held in Progressive Development Corporation vs. Secretary, Department of Labor and Employment: "But while Article 257 cited by the Solicitor General directs the automatic conduct of a certification election in an unorganized establishment, it also requires that the petition for certification election must be filed by a legitimate labor organization. Article 212(h) defines a legitimate labor organization as 'any labor organization duly registered with the DOLE and includes any branch or local thereof.' Rule 1, Section 1(j), Book V of the Implementing Rules likewise defines a legitimate labor organization as 'any labor organization duly registered with the DOLE and includes any branch, local or affiliate thereof .' " Indeed, the law did not reduce the Med-Arbiter to an automaton which can instantly be set to impulse by the mere filing of a petition for certification election. He is still tasked to satisfy himself that all the conditions of the law are met, and among the legal requirements is that the petitioning union must be a legitimate labor organization in good standing. The petition for certification election, in the case at bench, was filed by the NACUSIP-TUCP, a national labor organization duly registered with the DOLE. The legitimate status of NACUSIP-TUCP might be conceded; being merely, however, an agent for the local organization (the NACUSIP-TUCP Lopez Sugar Central Supervisory Chapter), the federation's bona fide status alone would not suffice. The local chapter, as its principal, should also be a legitimate labor organization in good standing. Accordingly, in Progressive Development, we elucidated: "In the case of union affiliation with a federation, the documentary requirements are found in Rule II, Section 3(e), Book V of the Implementing Rules, which we again quote as follows: "'(c ) The local or chapter of a labor federation or national union shall have and maintain a constitution and by laws, set of officers and books of accounts. For reporting purposes, the procedure governing the reporting of independently registered unions, federations or national unions shall be observed.' "Since the 'procedure governing the reporting independently registered unions' refers to the certification and attestation requirements contained in Article 235, paragraph 2, it follows that the constitution and by-laws, set of officers and books of accounts submitted by the local and chapter must likewise comply with these requirements. The same rationale for requiring the submission of duly subscribed documents upon union registration exists in the case of union affiliation. Moreover, there is greater reason to exact compliance with the certification and attestation requirements because, as previously mentioned, several requirements applicable to independent union registration are no longer required in the case of the formation a local or chapter. The policy of the law in conferring greater bargaining power upon labor unions must be balanced with the policy of providing preventive measures against the commission of fraud. "A local or chapter therefore becomes a legitimate labor organization only upon submission of the following to the BLR: "1) A charter certificate, within 30 days from its issuance by the labor federation or national union, and "2) The constitution and by-laws, a statement on the set of officers, and the books of accounts all of which are certified under oath by the secretary or treasurer, as the case may be, of such local or chapter, and attested to by its president. "Absent compliance with these mandatory requirements, the local or chapter does not become legitimate labor organization." The only document extant on record to establish the legitimacy of the NACUSIP-TUCP Lopez Sugar Central Supervisory Chapter is a charter certificate and nothing else. Disposition WHEREFORE, the assailed Decision of the Secretary of Labor, dated 06 March 1990, affirming that of the Med-Arbiter, is ANNULLED and SET ASIDE. The petition for certification election is dismissed. Dunlop v. Secretary of Labor, 300 SCRA 120 (98) FACTS: Respondent union filed a Petition for Certification Election among the supervisory, office and technical employees of the petitioner company before DOLE Regional Office No. III, San Fernando, Pampanga. Petitioner company filed its Answer with Motion to Dismiss based on three (3) grounds, namely: (1) that the respondent union is comprised of supervisory and rank-and-file employees and cannot act as bargaining agent for the proposed unit; (2) that a single certification election cannot be conducted jointly among supervisory and rank-and-file employees; and (3) that the respondent union lacks legal standing since it failed to submit its books of accounts. In its Reply, the respondent union alleged that its members are supervisors and not rank-and-file employees, paid monthly by the petitioner company. Mediator Arbiter Ma. Carmen A. Espinosa granted the petition for certification election. Respondent Secretary of Labor and Employment affirmed the Arbiter's decision and denied the petitioner’s Motion for Reconsideration; hence this petition. ISSUE: whether or not the respondent union can file a petition for certification election to represent the supervisory employees of the petitioner company HELD, RATIO: Order and resolution of public respondent are annulled and set aside.

82 | P a g e

The respondent union has no legal right to file a certification election to represent a bargaining unit composed of supervisors for so long as it counts rank-and-file employees among its members. Supervisors can be an appropriate bargaining unit. This is in accord with our repeated ruling that "[a]n appropriate bargaining unit is a group of employees of a given employer, composed of all or less than the entire body of employees, which the collective interests of all the employees, consistent with equity to the employer, indicate to be best suited to serve reciprocal rights and duties of the parties under the collective bargaining provisions of law. A unit to be appropriate must effect a grouping of employees who have substantial, mutual interests in wages, hours, working conditions and other subjects of collective bargaining." The test of supervisory status as repeatedly ruled is whether an employee possesses authority to act in the interest of his employer, which authority should not be merely routinary or clerical in nature but requires the use of independent judgment. Corrollarily, what determines the nature of employment is not the employee's title, but his job description. The list of monthly paid employees submitted by the petitioner company contains the names of about twenty seven (27) supervisory employees, six (6) managerial employees, one (1) confidential employee and twenty six (26) office and technical employees holding various positions. The list reveals that the positions occupied by the twenty six (26) office and technical employees are in fact rank-and-file positions, i.e., A/C mechanic, draftsmen, storemen, etc. These positions cannot be considered as supervisory positions for they do not carry the authority to act in the interest of the employer or to recommend managerial actions. It is not decisive that these employees are monthly paid employees. Their mode of compensation is usually a matter of convenience and does not necessarily determine the nature and character of their job. Based Art. 245, a labor organization composed of both rank-and-file and supervisory employees is no labor organization at all. It cannot, for any guise or purpose, be a legitimate labor organization. Not being one, an organization which carries a mixture of rank-and-file and supervisory employees cannot possess any of the rights of a legitimate labor organization, including the right to file a petition for certification election for the purpose of collective bargaining. Samahang v. DOLE, 290 SCRA 680 (98) 1. Organized Establishment – 256 Art. 256. Representation issue in organized establishments. In organized establishments, when a verified petition questioning the majority status of the incumbent bargaining agent is filed before the Department of Labor and Employment within the sixty-day period before the expiration of the collective bargaining agreement, the Med-Arbiter shall automatically order an election by secret ballot when the verified petition is supported by the written consent of at least twenty-five percent (25%) of all the employees in the bargaining unit to ascertain the will of the employees in the appropriate bargaining unit. To have a valid election, at least a majority of all eligible voters in the unit must have cast their votes. The labor union receiving the majority of the valid votes cast shall be certified as the exclusive bargaining agent of all the workers in the unit. When an election which provides for three or more choices results in no choice receiving a majority of the valid votes cast, a run-off election shall be conducted between the labor unions receiving the two highest number of votes: Provided, that the total number of votes for all contending unions is at least fifty percent (50%) of the number of votes cast. At the expiration of the freedom period, the employer shall continue to recognize the majority status of the incumbent bargaining agent where no petition for certification election is filed. (As amended by Section 23, Republic Act No. 6715, March 21, 1989) a. Definition California Manufacturing Corporation v. Laguesma, 209 SCRA 606 (92) 1. May 24, 1990, a petition for certification election among the supervisors of California Manufacturing Corporation (CMC for brevity) was filed by the Federation of Free Workers (FFW) — California Manufacturing Corporation Supervisors Union Chapter (CALMASUCO), 2. CMC (the co.) filed answer saying that the petition does not have 25% support from barg. Unit, and that a big number of the supposed signatories to the petition are not actually supervisors as they have no subordinates to supervise, nor do they have the powers and functions which under the law would classify them as supervisors 3. FFW—CALMASUCO filed its reply maintaining that under the law, when there is no existing unit yet in a particular bargaining unit at the time a petition for certification election is filed, the 25% rule on the signatories does not apply; that the "organized establishment" contemplated by law does not refer to a "company" per se but rather refers to a "bargaining unit" which may be of different classifications in a single company; that CMC has at least two (2) different bargaining units, namely, the supervisory (unorganized) and the rank-and-file (organized); that the signatories to the petition have been performing supervisory functions; that since it is CMC which promoted them to the positions, of supervisors. it is already estopped from claiming that they are not supervisors; that the said supervisors were excluded from the coverage of the collective bargaining agreement of its rank-and-file employees; and that the contested signatories are indeed supervisors as shown in the "CMC Master List of Employees" 4. MA: hold CE; DOLE: hold CE I: whether CE should be held without showing of 25% support

83 | P a g e

SC: Article 257 of the Labor code is applicable to unorganized labor organizations and not to establishments where there exists a certified bargaining agent which had previously entered into a collective bargaining agreement with the management; Otherwise stated, the establishment concerned must have no certified bargaining agent the supervisors of CMC which constitute a bargaining unit separate and distinct from that of the rank-and-file, have no such agent. thus they correctly filed a petition for certification election thru union FFW-CALMASUCO, likewise indubitably a legitimate labor organization. CMC's insistence on the 25% subscription requirement, is clearly immaterial. The same has been expressly deleted by Section 24 of Republic Act No. 6715 and is presently prescribed only in organized establishments, that is, those with existing bargaining agents. Compliance with the said requirement need not even be established with absolute certainty. The Court has consistently ruled that "even conceding that the statutory requirement of 30% (now 25%) of the labor force asking for a certification election had not been strictly compiled with, the Director (now the Med-Arbiter) is still empowered to order that it be held precisely for the purpose of ascertaining which of the contending labor organizations shall be the exclusive collective bargaining agent requirement then is relevant only when it becomes mandatory to conduct a certification election. In all other instances, the discretion, according to the rulings of this Tribunal, ought to be ordinarily exercised in favor of a petition for certification CMC as employer has no standing to question a certification election The only exception is where the employer has to file the petition for certification election pursuant to Article 259 (now 258) of the Labor Code because it was requested to bargain collectively. Thereafter, the role of the employer in the certification process ceases. The employer becomes merely a bystander SC: hold CE. b. Freedom Period Atlantic Gulf and Pacific Co. Manila, Inc. v. Laguesma, 212 SCRA 281 (92) Atlantic Gulf executed a CBA with the United Rank and File Assoc (URFA) which was the bargaining unit of all the regular rank and file employees. The CBA was registered with the BLR on July 9 1990. On june 29 1990 Lakas – NFL filed a petition for certification election to be the bargaining agent of the regular non-project employees. The Med arbiter ordered the certification election. During this time the regular project employees sought to be represented by Lakas were issued regular employment appointments. AG&P alleged that all the employees sought to be represented by lakas are now regular employees and are included in the existing CBA. This being the case Lakas is barred from having certification election. Art 232 Prohibition on certification election – The bureau shall not entertain any petition for certification election or any other action which may disturb the administration of duly registered existing bargaining agreements affecting the parties except under Art 253, 253-A and 256 of this code”. If a CBA has been duly registered in accordance with Art 231 of the code, a petition for certification election or a motion for intervention can only be entertained within 60 days prior to the expiry of such an agreement. The existence of a duly registered CBA between AG & P and URFA bars any other labor organization from filing a petition for certification election except within the 60 day perior prior to the expiration of the CBA. c. When Applied Associated Labor Union v. Calleja, 179 SCRA 127 (89) The Associated Labor Union (ALU) had an existing CBA with the Philippine Associated Smelting and Refining Corporation (PASAR), which expired on April 1, 1987. Several days before its expiry, or on March 23, 1987, the National Federation of Labor Unions (NAFLU) filed a petition for certification election with the Bureau of Labor Relations. ALU moved to intervene and sought its dismissal, asserting that NAFLU failed to present the necessary signatures. The Med-Arbiter dismissed NAFLU's petition. The order was later set aside and a hearing was scheduled. PASAR was also enjoined from entering into a CBA with any union until the issue of representation has been resolved. NAFLU's petition was dismissed because of its failure to solicit the signatures of 20% of PASAR's employees. NAFLU appealed. Pending appeal, ALU and PASAR entered into a CBA, which was ratified by the members of the bargaining unit. ALU then moved for the dismissal of the appeal. BLU Director Ferrer-Calleja gave due course to NAFLU's appeal and ordered the holding of a certification election. Both parties moved for reconsideration. Calleja denied. ALU appealed, saying that the requirement of Art 257 should be observed, i.e. the signatures of at least 20% of the employees should be obtained first. Issues: 1. WON Art 257 is applicable 2. WON the renewed CBA can bar the holding of a certification election. Held: 1. No. The requirement imposed by 257 applies only to unorganized establishments. For organized establishments, Art 256 applies, i.e. the mere filing of a petition for certification election within the sixty-day freedom period is sufficient basis for the Director to order a certification election. PASAR was an organized establishment. NAFLU's petition was filed well within the freedom period. Hence 256 should apply. 2. No. PASAR and ALU were in bad faith when they entered into a CBA prematurely. A CBA prematurely entered into is not a bar to the holding of a certification election.

84 | P a g e

General Milling Corporation v. Court of Appeals, 422 SCRA 514 (04) General Milling Corporation (GMC) had a three-year CBA with its Union, effective from December 1, 1988 to November 30, 1991. A day before its expiration, the Union sent to it a proposed CBA, with a request that counter-proposals be sent to them within 10 days. As early as October 1991, however, GMC had been receiving letters from its employees stating that they had withdrawn from the Union. GMC relayed this to the Union officers through a letter, and said that there was no basis to negotiate with a union that no longer existed. The officers disclaimed this. A union member was dismissed, sparking protests from the Union which requested that the matter be submitted to the grievance procedure as outlined in the CBA. GMC advised them to refer to their earlier letter. The Union filed a complaint against GMC, alleging ULP for 1) refusal to bargain collectively, 2) interference with the right to self-organization, and 3) discrimination. The labor arbiter dismissed, with a recommendation that a certification election be held. The Union appealed to the NLRC, which set aside the LA's decision and ordered GMC to abide by the Union's proposed CBA for a period of two years. GMC moved for reconsideration. NLRC set aside its own decision, finding that GMC's doubts as to the Union's status were legitimate. The Union appealed to the CA, which ruled for it. After its Motion for Reconsideration was denied, GMC elevated the case to the SC. Issues: 1. WON GMC was guilty of ULP for refusal to bargain collectively and for interference with the workers' right to self-organization, 2. WON the draft CBA proposed by the Union should be imposed on GMC. Held: 1. Yes. Art 253-A mandates that the representation provision of a CBA should last for five years. The Union was still the certified collective bargaining agent of the workers because it sought renegotiation of the CBA within five years from the date of effectivity of their first CBA. The Union's proposal was also submitted within the prescribed three-year period of the CBA. Hence, the GMC had no reason to refuse to negotiate in good faith with the Union. The letters presented by GMC signifying the intention of the workers to resign from the Union were received during the pendency of the case before the LA. That showed that GMC exerted pressure on its employees, which amounted to interference. 2. Yes. Citing Kiok Loy and Divine Word University of Tacloban, the SC ruled that GMC had lost the right to bargain collectively when it acted in bad faith. Therefore, the Union's proposed CBA should be imposed on them. d. Form of Petition – Signature Verification and Verification of Pleadings National Mines and Allied Workers Union v. Secretary of Labor, 227 SCRA 821 (1993) · Petitioner and respondent FFW-SMQCC are local chapters of labor federations registered with DOLE. Petitioner is the exclusive bargaining agent of all the rank and file workers QCC, a domestic corporation engaged in the metal industry. · 38 days before the expiration of the Collective CBA between petitioner and QCC, respondent FFW-SMQCC through Reynito de Pedro filed with the DOLE a petition for certification election. The petition was accompanied by a list of signatures of company employees, who signified their consent to a certification election among the rank and file employees of QCC. · Petitioner moved to dismiss the petition of respondent FFW-SMQCC on the grounds that: (a) the required consent to the certification election of at least 25% of the rank and file employees had not been met; (b) the petition was not verified as required by law; and (c) Reynito de Pedro, who was also the president of petitioner, had no personality to file the petition on behalf of FFW-SMQCC. · FFW-SMQCC, filed a second petition for certification election, this time signed and verified by De Pedro. · Med-Arbiter granted the petition for certification election of FFW-SMQCC. Upon the conditions that: within twenty (20) days from receipt hereof with the usual pre-election conference of the parties will thresh out the mechanics and other details of the election. The payroll of the company three (3) months prior to the filing of the petition shall be used as the basis in determining the list of eligible voters.The choices are: a) (FFW)Samahang Mangagawa sa Quality Container Corporation Chapter; b) National Mines and Allied Workers' Union (NAMAWU); and c) No union. · Petitioner appealed this decision to the Secretary of Labor but was denied for lack of merit. Issue: WON the decision granting the petition for certification election is valid Held: Yes · First, although Reynito de Pedro was the duly elected president of petitioner, he had disaffiliated himself therefrom and joined respondent FFW-SMQCC before the petition for certification election was filed. · Second, verification of a pleading is a formal, not jurisdictional requisite (Buenaventura v. Uy), Even if verification is lacking and the pleading is formally defective, the courts may dispense with the requirement in the interest of justice and order of correction of the pleading accordingly. Generally, technical and rigid rules of procedure are not binding in labor cases; and this rule is specifically applied in certification election proceedings, which are nonlitigious but merely investigative and non-adversarial in character. Whatever formal defects existed in the first petition were cured and corrected in the second petition for certification election. · Third, attached to the original petition for certification election was a list of 141 supporting signatures out of the 300 employees belonging to the appropriate bargaining unit to be represented by respondent FFW-SMQCC. Respondent QCC sought to delete from the list some 36 signatures which are allegedly forged and falsified. Petitioner, likewise, submitted a joint affidavit of 13 employees, disclaiming the validity of the signatures therein. 85 | P a g e

Granting that 36 signatures were falsified and that 13 was disowned, this leaves 92 undisputed signatures which is definitely more than 75 — i.e., 25% of the total number of company employees required by law to support a petition for certification election. The disclaimer of 13 employees by their respective signatures covers only their own personal participation and cannot in any way be extended to include the rest of those who did not question the same. · Moreover, the fact that the list of signatures is undated does not necessarily mean that the signatures were obtained prior to the 60-day period before the expiration of the existing collective bargaining agreement. What is important is that the petition for certification election must be filed during the freedom period and that the 25% requirement of supporting signatures be met upon the filing thereof. These requirements have been compiled by respondent FFW-SMQCC in their first and second petitions. · If there are employees in the bargaining unit who refused to be represented by respondent FFW-SMQCC, with all the more reason should a certification election be held where the employees themselves can freely and voluntarily express by secret ballot their choice of bargaining representative. A certification election is the most effective and expeditious way to determine which labor organization can truly represent the working force in the appropriate bargaining unit of the company. Today’s Knitting v. Noriel, 75 SCRA 450 (77) The Philippine National Union Council (PNUC) filed with the Bureau of Labor Relations a petition for the holding of a certification election. It set forth therein that its ranks comprise a majority of membership at the Today's Knitting C company, Inc. A petition for intervention was filed by Today's Knitting Free Workers Union. It saw no need for a certification election, asserting that it had already been voluntarily recognized by the management as the bargaining representative. The Bureau of Labor Relations received resolutions by the rank-and-file employees of the Company, Inc. authorizing the filing of the petition for certification election with a total of two hundred (200) signatures. Company favored PNUC. Med-Arbiter ordered to grant the petition for certification election. Director Noriel ordered a certification election. Issue: WON the order to hold certification election may be invalidated. Held: No. Ratio: 1. There was evidence that more than a total of two-hundred signatures were obtained by respondent Union in seeking such a certification election in satisfaction of the 30% requirement under Art 257 of the Labor Code. Therefore, there is no basis to allow a rival labor organization to verify the authenticity of such signatures. If there is any doubt as to the required number having been met, the best way is to hold a certification election to ascertain which union really commands the allegiance of the rank-and-file employees. 2. To enable labor to determine which of the competing organizations should represent them for the purpose of a collective bargaining contracts strengthen the validity of the assailed order. 3. Attention of counsel must be called for him to be acquainted with applicable decisions to avoid the filing of this suit and thus hasten the effective and speedy implementation of a certification election. It is to live true to his duty as an officer of the court on whose trustworthiness reliance may be placed. 4. No much attention need be paid to the allegation in the petition that a memorandum circular of the Secretary of Labor was disregarded by respondent Director. The circular states that there is already a bona fide union negotiating a contract, hence, an application for a certification election should not be entertained. Firstly, a memorandum circular certainly cannot prevail as against the very language of the statute. Secondly, previous circulars of the secretary of labor before the passage of the Labor Code, were deemed superseded. e. Venue Cruz Valle, Inc. v. Laguesma, 238 SCRA 389 (1994) f. Substantial Support 1. Rationale and Computation Philippine Association of Free Labor Unions v. Ferrer-Calleja, 169 SCRA 490 (1989) St. James School v. St. James School, 476 SCRA 12 (2005) The Samahang Manggagawa sa St. James School of Quezon City (“Samahang Manggagawa”) filed a petition for certification election to determine the collective bargaining representative of the motor pool, construction and transportation employees of St. James School of Quezon City (“St. James”). 1. CE held, 149 eligible voters and 84 voters cast their votes. 2. St. James filed a certification election protest challenging the 84 votes; 179 rank and file employees did not vote in the CE; those who did vote wre construction workers of independent contractor: Architect Conrado Bacoy 3. MA: CE Void bec. Workers who voted not employees, and even if they were, not majority of R/F employees (84 out of 263) 4. on appeal to SOLE: reversed MA; Samahan wants to represent only the const’n, trans’n workers, not all the R/F, plus the list of 179 employees had only teaching and admin personnel; ORDER: open ballots and canvass 86 | P a g e

5. CA: no abuse of discretion I: is the labor union and the CE valid? SC: vis. whether labor union is composed of employees of independent contractor: St. James had previously filed for cancellation of union registration of Samahan based on lack of E-E rel. MA: cancelled reg’n, DOLE RD: cancelled reg’n, but BLR, CA and SC: reversed RD and MA and held that the 84 were employees in the const’n/trans’n department of St. James (although SC decision in that case was based on error of mode for appeal); decision is final, union is a legitimate labor org. Vis. CE: on issue of whether there was quorum: St James alleges that in all its 5 campuses, it has 570 R/F, while in the Tandang Sora campus alone, there are 179, so the 84 who voted are not the majority SC: computation of quorum based on what the Barg. Unit to be represented, i.e. the constn/transn pool, not all the R/F (see Sec. 2 Rule XII, Book V, IRR), this pool had 149 employees, so there was majority The fact that the names of the voters were not on St James list is negligible since what was submitted was a list of the teaching/admin personnel, which is not part of the barg. Unit. SC: CE valid, canvass votes 2. Compliance California Manufacturing Corporation v. Laguesma, 209 SCRA 609 (1992) 5. May 24, 1990, a petition for certification election among the supervisors of California Manufacturing Corporation (CMC for brevity) was filed by the Federation of Free Workers (FFW) — California Manufacturing Corporation Supervisors Union Chapter (CALMASUCO), 6. CMC (the co.) filed answer saying that the petition does not have 25% support from barg. Unit, and that a big number of the supposed signatories to the petition are not actually supervisors as they have no subordinates to supervise, nor do they have the powers and functions which under the law would classify them as supervisors 7. FFW—CALMASUCO filed its reply maintaining that under the law, when there is no existing unit yet in a particular bargaining unit at the time a petition for certification election is filed, the 25% rule on the signatories does not apply; that the "organized establishment" contemplated by law does not refer to a "company" per se but rather refers to a "bargaining unit" which may be of different classifications in a single company; that CMC has at least two (2) different bargaining units, namely, the supervisory (unorganized) and the rank-and-file (organized); that the signatories to the petition have been performing supervisory functions; that since it is CMC which promoted them to the positions, of supervisors. it is already estopped from claiming that they are not supervisors; that the said supervisors were excluded from the coverage of the collective bargaining agreement of its rank-and-file employees; and that the contested signatories are indeed supervisors as shown in the "CMC Master List of Employees" 8. MA: hold CE; DOLE: hold CE I: whether CE should be held without showing of 25% support SC: Article 257 of the Labor code is applicable to unorganized labor organizations and not to establishments where there exists a certified bargaining agent which had previously entered into a collective bargaining agreement with the management; Otherwise stated, the establishment concerned must have no certified bargaining agent the supervisors of CMC which constitute a bargaining unit separate and distinct from that of the rank-and-file, have no such agent. thus they correctly filed a petition for certification election thru union FFW-CALMASUCO, likewise indubitably a legitimate labor organization. CMC's insistence on the 25% subscription requirement, is clearly immaterial. The same has been expressly deleted by Section 24 of Republic Act No. 6715 and is presently prescribed only in organized establishments, that is, those with existing bargaining agents. Compliance with the said requirement need not even be established with absolute certainty. The Court has consistently ruled that "even conceding that the statutory requirement of 30% (now 25%) of the labor force asking for a certification election had not been strictly compiled with, the Director (now the Med-Arbiter) is still empowered to order that it be held precisely for the purpose of ascertaining which of the contending labor organizations shall be the exclusive collective bargaining agent requirement then is relevant only when it becomes mandatory to conduct a certification election. In all other instances, the discretion, according to the rulings of this Tribunal, ought to be ordinarily exercised in favor of a petition for certification CMC as employer has no standing to question a certification election The only exception is where the employer has to file the petition for certification election pursuant to Article 259 (now 258) of the Labor Code because it was requested to bargain collectively. Thereafter, the role of the employer in the certification process ceases. The employer becomes merely a bystander SC: hold CE. 3. Motion Intervention Port Workers Union of the Philippines v. Ferrer-Calleja, 207 SCRA 329 (1992) FACTS:

87 | P a g e

- The collective bargaining agreement of the International Container Terminal Services, Inc. (ICTSI) with private respondents Associate Port Checkers and Workers Union (APCWU), the incumbent union, was due to expire on April 14, 1990. Other unions were seeking to represent the laborers in the negotiation of the next CBA and were already plotting their moves. - on March 14, 1990, when the Sandigan ng Manggagawa sa Daungan (SAMADA) filed a petition for certification election. The consent signatures of at least 25% of the employees in the bargaining unit were submitted on March 26, 1990, or eleven days after the petition. - On April 2, 1990, herein petitioner Port Workers Union of the Philippines (PWUP) filed a petition for intervention. - another petition for certification election was filed by the Port Employees Association and Labor Union (PEALU), on April 6, 1990. The consent signatures were submitted on May 11, 1990, or thirty-five days after the filing of the petition. - On April 26, 1990, APCWU filed a motion to dismiss them on the ground that they did not comply with the requirement set forth in Section 6, Rule V, Book V of the Implementing Rules, quoted in part as follows: In a petition involving an organized establishment or enterprise where the majority status of the incumbent collective bargaining union is questioned through a verified petition by a legitimate labor organization, the Med-Arbiter shall immediately order the certification election by secret ballot if the petition is filed during the last sixty (60) days of the collective bargaining agreement and supported by the written consent of at least twenty-five percent (25%) of all the employees in the bargaining unit. Any petition filed before or after the sixty-day freedom period shall be dismissed outright. The twenty-five percent (25%) requirement shall be satisfied upon the filing of the petition, otherwise the petition shall be dismissed . (Emphasis supplied.) - the Med-Arbiter dismissed the consolidated petitions. PWUP appealed to the Secretary of Labor, arguing that Article 256 of the Labor Code did not require the written consent to be submitted simultaneously with the petition for certification election. DOLE Undersecretary Laguesma affirmed the order of the Med-Arbiter and dismissed PWUP's appeal. - ICTSI and APCWU resumed negotiations for a new collective bargaining agreement, which was concluded on September 28, 1990. This was ratified by a majority of the workers in the bargaining unit. - PWUP claims grave abuse of discretion on the part of the public respondent in the application of Article 256 of the Labor Code. The article provides in part as follows: Art. 256. Representation issue in organized establishments. ? In organized establishments, when a verified petition questioning the majority status of the incumbent bargaining agent is filed before the Department of Labor and Employment within the sixty-day period before the expiration of the collective bargaining agreement, the Med-Arbiter shall automatically order an election by secret ballot when the verified petition is supported by the written consent of at least twenty-five (25%) percent of all the employees in the bargaining unit to ascertain the will of the employees in the appropriate bargaining unit. . . . - The petitioner argues that under this article, the Med-Arbiter should automatically order election by secret ballot when the petition is supported by at least 25% of all employees in the bargaining unit. SAMADA and PEALU substantially complied with the law when they submitted the required consent signatures several days after filing the petition. The petitioner complains that the dismissal of the petitions for certification election, including its own petition for intervention, had the effect of indirectly certifying APCWU as the sole and exclusive bargaining representative of the ICTSI employees. - Private respondent ICTSI maintains that the dismissal was based on Article 256 of the Labor Code as implemented by Section 6, Rule V, Book V of the Implementing Rules, quoted above. Moreover, under Section 10, Rule V, Book V of the Implementing Rules, decisions of the Secretary in certification election cases shall be final and unappealable. - For its part, APCWU questions PWUP's personality in these proceedings in view of the lack of consent signatures in its petition, and argues as well that the petitioner has no authority to represent SAMADA or PEALU, which had not appealed. The private respondent also invokes Tupas and maintains that the ratification of the new CBA by the majority of the workers was an affirmation of their membership in the union that negotiated that agreement. ISSUE: WON there was indeed grave abuse of discretion amounting to lack or excess of jurisdiction on the part of public respondents when they dismissed the petitions for certification election because the consent signatures had not been submitted simultaneously with the petition HELD: YES - pursuant to the constitutional provision guaranteeing workers the right to self-organization and collective bargaining, "the constant and unwavering policy of the Court" has been "to require a certification election as the best means of ascertaining which labor organization should be the collective bargaining representative." - The certification election is the most democratic and expeditious method by which the laborers can freely determine the union that shall act as their representative in their dealings with the establishment where they are working. The holding of a certification election is a statutory policy that should not be circumvented. - the administrative rule requiring the simultaneous submission of the 25% consent signatures upon the filing of petition for certification election should not be strictly applied to frustrate the determination of the legitimate representative of the workers. Significantly, the requirement in the rule is not found in Article 256, the law it seeks to implement. This is all the more reason why the regulation should at best be given only a directory effect. - It is not denied that the petition to intervene filed by PWUP did not carry the 25% consent signatures, but that the requirement is in fact not applicable to a petition in intervention. - the certification election is not litigation but a mere investigation of a non-adversary character where the rules of procedure are not strictly applied. Technical rules and objections should not hamper the correct ascertainment of the labor union that has the support of confidence of the majority of the workers and is thus entitled to represent them in their dealings with management. 88 | P a g e

- Deviation from the contract-bar rule is justified only where the need for industrial stability is clearly shown to be imperative. Subject to this singular exception, contracts where the identity of the authorized representative of the workers is in doubt must be rejected in favor of a more certain indication of the will of the workers. The certification election is the best method of determining the will of the workers on the crucial question of who shall represent them in their negotiations with the management for a collective bargaining agreement that will best protect and promote their interests. It is essential that there be no collusion against this objective between an unscrupulous management and a union covertly supporting it while professing its loyalty to labor, or at least that the hopes of labor be not frustrated because of its representation by a union that does not enjoy its approval and support. It is therefore sound policy that any doubt regarding the real representation of the workers be resolved in favor of the holding of the certification election. This is preferable to the suppression of the voice of the workers through the prissy observance of technical rules that will exalt procedure over substantial justice. DISPOSITION Petition GRANTED. 4. Submission Oriental Tin Can Labor Union v. Secretary of Labor, 294 SCRA 640 (1998) FACTS: Oriental Tin Can Company, Inc. is engaged in the manufacture of tin can containers and metal sheets. On March 3, 1994, it entered into a CBA with the Petitioner Union (OTCLU) as the existing CBA was due to expire on April 15, 1994. Four days later, 248 of the company’s rank-and-file authorized the Federation of Free Workers (FFW) to file a petition for certification election. On March 10, 1994, however, this petition was repudiated via a written waiver by 115 of the signatories who, along with other employees totaling 897, ratified the CBA on the same date. On March 18, 1994, Private respondent Oriental Tin Can Workers Union – Federation of Free Workers (OTCWU-FFW) filed a petition for certification election. OTCLU filed a motion to dismiss on the ground that it was not endorsed by at least 25% of the employees of the bargaining unit. The OTCWU-FFW filed a reply, claiming that the retraction of support for the petition was “not verified under oath” and, therefore, had no legal effect. In the meantime, the DOLE issued a certificate of registration of the CBA pursuant to Article 231 of the Labor Code. As a result, the officers of the OTCWU-FFW walked out of their jobs, prompting the company to require them to explain in writing why no disciplinary action should be taken against them for walking out en masse. Two days later, the company directed said officers to report back to work within 48 hours, but none of them did. Med-Arbiter Paruñgo dismissed the petition for certification election. Noting that the petition was filed after the valid retractions were made, he concluded that by the withdrawal of support to the petition by 115 workers, the remaining 133 of the 1,020 employees were clearly less than the 25% subscription requirement. The OTCWU-FFW appealed to the Labor Secretary. However, during the pendency of the appeal, said union staged a strike that prevented the free ingress and egress of non-striking employees and vehicles. Upon complaint of the company, the NLRC issued a writ of preliminary injunction. Meanwhile, Undersecretary Laguesma, acting on the appeal issued a resolution IFO OTCWU-FFW, that the subject CBA was concluded during the 60-day freedom period and XXX hence no registered CBA in the respondent establishment could be invoked (to) pose as a bar to the holding of a certification election. In other words, when the said CBA was registered there was a pending representation case. Consequently, said CBA cannot bar the election being prayed for. Under the Rules and Regulations Implementing the Labor Code: Rule V, Section 4. Effects of early agreements. – The representation case shall not, however, be adversely affected by a collective bargaining agreement registered before or during the last sixty (60) days of a subsisting agreement or during the pendency of the representation case. “ On the issue of whether the 25% requirement for certification election had been met, Laguesma opined that in case of alleged retractions, the best forum for determining whether there were indeed retractions is the certification election. ISSUE: 1) WON the petition for certification election should be granted: YES HELD: It is uncontroverted that the petition for certification election in this case was filed on March 18, 1994, twenty-eight days before the expiration of the existing CBA on April 15, 1994, and well within the 60-day period provided for by the Code. The OTCLU, however, is concerned with the effect of the employees’ ratification of the new CBA. Would such ratification nullify the petition? NO. The filing of a petition for certification election during the 60-day freedom period gives rise to a representation case that must be resolved even though a new CBA has been entered into within that period. A petition for certification election is not necessary where the employees are one in their choice of a representative in the bargaining process. The intent of the legislative authority is to allow, if not encourage, the contending unions in a bargaining unit to hold a certification election during the freedom period. Hence, the Court held in the case of Warren Manufacturing Workers Union v. BLR that the agreement prematurely signed by the union and the company during the freedom period does not affect the petition for certification election filed by another union. As regards the 25% support requirement, we concur with public respondent’s finding that said requisite has been met in this case. The following pronouncement of the Court is relevant: “x x x. Even doubts as to the required 30% being met warrant (the) holding of the certification election. In fact, once the required percentage requirement has been reached, the employees’ withdrawal from union membership taking place after the filing of the petition for certification election will not affect the petition. On the contrary, the presumption arises that the withdrawal was not free but was procured through duress, coercion or for a valuable consideration.” The support requirement is a mere technicality which should be employed in determining the true will of the workers instead of frustrating the same. All doubts as to the number of employees actually supporting the holding of a certification election should, therefore, be resolved by going through such procedure. It is judicially settled

89 | P a g e

that a certification election is the most effective and expeditious means of determining which labor organization can truly represent the working force in the appropriate bargaining unit of the company. Port Workers Union of the Philippines v. DOLE, 207 SCRA 329 (1992) FACTS: - The collective bargaining agreement of the International Container Terminal Services, Inc. (ICTSI) with private respondents Associate Port Checkers and Workers Union (APCWU), the incumbent union, was due to expire on April 14, 1990. Other unions were seeking to represent the laborers in the negotiation of the next CBA and were already plotting their moves. - on March 14, 1990, when the Sandigan ng Manggagawa sa Daungan (SAMADA) filed a petition for certification election. The consent signatures of at least 25% of the employees in the bargaining unit were submitted on March 26, 1990, or eleven days after the petition. - On April 2, 1990, herein petitioner Port Workers Union of the Philippines (PWUP) filed a petition for intervention. - another petition for certification election was filed by the Port Employees Association and Labor Union (PEALU), on April 6, 1990. The consent signatures were submitted on May 11, 1990, or thirty-five days after the filing of the petition. - On April 26, 1990, APCWU filed a motion to dismiss them on the ground that they did not comply with the requirement set forth in Section 6, Rule V, Book V of the Implementing Rules, quoted in part as follows: In a petition involving an organized establishment or enterprise where the majority status of the incumbent collective bargaining union is questioned through a verified petition by a legitimate labor organization, the Med-Arbiter shall immediately order the certification election by secret ballot if the petition is filed during the last sixty (60) days of the collective bargaining agreement and supported by the written consent of at least twenty-five percent (25%) of all the employees in the bargaining unit. Any petition filed before or after the sixty-day freedom period shall be dismissed outright. The twenty-five percent (25%) requirement shall be satisfied upon the filing of the petition, otherwise the petition shall be dismissed . (Emphasis supplied.) - the Med-Arbiter dismissed the consolidated petitions. PWUP appealed to the Secretary of Labor, arguing that Article 256 of the Labor Code did not require the written consent to be submitted simultaneously with the petition for certification election. DOLE Undersecretary Laguesma affirmed the order of the Med-Arbiter and dismissed PWUP's appeal. - ICTSI and APCWU resumed negotiations for a new collective bargaining agreement, which was concluded on September 28, 1990. This was ratified by a majority of the workers in the bargaining unit. - PWUP claims grave abuse of discretion on the part of the public respondent in the application of Article 256 of the Labor Code. The article provides in part as follows: Art. 256. Representation issue in organized establishments. ? In organized establishments, when a verified petition questioning the majority status of the incumbent bargaining agent is filed before the Department of Labor and Employment within the sixty-day period before the expiration of the collective bargaining agreement, the Med-Arbiter shall automatically order an election by secret ballot when the verified petition is supported by the written consent of at least twenty-five (25%) percent of all the employees in the bargaining unit to ascertain the will of the employees in the appropriate bargaining unit. . . . - The petitioner argues that under this article, the Med-Arbiter should automatically order election by secret ballot when the petition is supported by at least 25% of all employees in the bargaining unit. SAMADA and PEALU substantially complied with the law when they submitted the required consent signatures several days after filing the petition. The petitioner complains that the dismissal of the petitions for certification election, including its own petition for intervention, had the effect of indirectly certifying APCWU as the sole and exclusive bargaining representative of the ICTSI employees. - Private respondent ICTSI maintains that the dismissal was based on Article 256 of the Labor Code as implemented by Section 6, Rule V, Book V of the Implementing Rules, quoted above. Moreover, under Section 10, Rule V, Book V of the Implementing Rules, decisions of the Secretary in certification election cases shall be final and unappealable. - For its part, APCWU questions PWUP's personality in these proceedings in view of the lack of consent signatures in its petition, and argues as well that the petitioner has no authority to represent SAMADA or PEALU, which had not appealed. The private respondent also invokes Tupas and maintains that the ratification of the new CBA by the majority of the workers was an affirmation of their membership in the union that negotiated that agreement. ISSUE: WON there was indeed grave abuse of discretion amounting to lack or excess of jurisdiction on the part of public respondents when they dismissed the petitions for certification election because the consent signatures had not been submitted simultaneously with the petition HELD: YES - pursuant to the constitutional provision guaranteeing workers the right to self-organization and collective bargaining, "the constant and unwavering policy of the Court" has been "to require a certification election as the best means of ascertaining which labor organization should be the collective bargaining representative." - The certification election is the most democratic and expeditious method by which the laborers can freely determine the union that shall act as their representative in their dealings with the establishment where they are working. The holding of a certification election is a statutory policy that should not be circumvented.

90 | P a g e

- the administrative rule requiring the simultaneous submission of the 25% consent signatures upon the filing of petition for certification election should not be strictly applied to frustrate the determination of the legitimate representative of the workers. Significantly, the requirement in the rule is not found in Article 256, the law it seeks to implement. This is all the more reason why the regulation should at best be given only a directory effect. - It is not denied that the petition to intervene filed by PWUP did not carry the 25% consent signatures, but that the requirement is in fact not applicable to a petition in intervention. - the certification election is not litigation but a mere investigation of a non-adversary character where the rules of procedure are not strictly applied. Technical rules and objections should not hamper the correct ascertainment of the labor union that has the support of confidence of the majority of the workers and is thus entitled to represent them in their dealings with management. - Deviation from the contract-bar rule is justified only where the need for industrial stability is clearly shown to be imperative. Subject to this singular exception, contracts where the identity of the authorized representative of the workers is in doubt must be rejected in favor of a more certain indication of the will of the workers. The certification election is the best method of determining the will of the workers on the crucial question of who shall represent them in their negotiations with the management for a collective bargaining agreement that will best protect and promote their interests. It is essential that there be no collusion against this objective between an unscrupulous management and a union covertly supporting it while professing its loyalty to labor, or at least that the hopes of labor be not frustrated because of its representation by a union that does not enjoy its approval and support. It is therefore sound policy that any doubt regarding the real representation of the workers be resolved in favor of the holding of the certification election. This is preferable to the suppression of the voice of the workers through the prissy observance of technical rules that will exalt procedure over substantial justice. DISPOSITION Petition GRANTED. 5. Effect Withdrawal Tagaytay Highlands International Gold Club, Inc. v. Tagaytay Highlands Employees Union, 395 SCRA 699 (2003) FACTS: On Oct 1997, the TAGAYTAY HIGHLANDS EMPLOYEES UNION-PTGWO (THEU- PTGWO), a legitimate labor organization said to represent majority of the rankand-file employees of Tagaytay Highlands Gold Club (THIGCI) filed a petition for certification election before the DOLE Med-Arb unit. THIGCI opposed the petition on the ground that the list of union members submitted was flawed as it includes supervisors, resigned employees, AWOL employees and non-employees of THIGCI. DOLE Med-Arbiter ordered holding of a certification election on the ground that the union is a legitimate labor federation, and that the issues on the members should be properly raised in exclusion-inclusion proceedings in the pre-election conference, THIGCI appealed to the DOLE Secretary which set aside the Med-Arbiter decision on the ground of clear absence of community of mutuality of interests” as THEU sought to represent separate bargaining units (supervisory emp and rank-and-file emp). Upon MR by THEU, DOLE Usec, by authority of the Secretary set aside the resolution dismissing the petition for certification election and held that rather than disregarding the legitimate status of THEU, the names of alleged supervisory employees should be simply removed form the roster of membership. THUGCI filed an MR but was denied. CA likewise denied. Issue: WON the inclusion of supervisory employees in the rank-and-file union affects the legitimacy of the union Held: No. While Article 245 expressly prohibits supervisory employees from joining a rank-and-file union, it does not provide what would be the effect if a rank-and-file union counts supervisory employees as members. After a certificate of registration is issued to a union, its legal personality cannot be subject to collateral attack. It may be questioned only in an independent petition for cancellation in accordance with Section 5 of Rule V, Book IV of the "Rules to Implement the Labor Code" (Implementing Rules) which section reads: Sec. 5. Effect of registration. The labor organization or workers' association shall be deemed registered and vested with legal personality on the date of issuance of its certificate of registration. Such legal personality cannot thereafter be subject to collateral attack, but may be questioned only in an independent petition for cancellation in accordance with these Rules. The inclusion in a union of disqualified employees is not among the grounds for cancellation, unless such inclusion is due to misrepresentation, false statement or fraud under the circumstances enumerated in Sections (a) and (c) of Article 239 of above-quoted Article 239 of the Labor Code. As for the lack of mutuality of interest, it does not lie, given the company’s failure to present substantial evidence that the assailed employees are actually occupying supervisory positions. While petitioner submitted a list of its employees with their corresponding job titles and ranks, there is nothing mentioned about the supervisor’s respective duties, powers, and prerogatives that would show that they can effectively recommend managerial actions with the required use of the independent judgment. 6. Discretion Rule Application Airtime Specialists, Inc. v. Director of BLR, 180 SCRA 749 (1990) Facts: Respondent Samahan ng mga Manggagawa sa Asia-FFW Chapter (SAMA-ASIA), Pinagbuklod ng Manggagawa sa Ataco-FFW Chapter (PMA for short) petitions for direct certification and/or certification election on behalf of the regular rank-and-file employees of the petitioners Airtime Specialists and Absolute Sound, Inc. and Country-Wealth Development, Ad Planner and Marketing Counsellors and Atlas Resources. Petitioners a motion to dismiss on the following grounds — disaffiliation of the rank and file employees, ineligibility of some signatories because they had less than one (1) year of service resulting in the non-compliance with the 30% requirement. Med Arbiter and BLR ordered the certification election. 91 | P a g e

Issue: WON employees with less than one year of service and even probationary employees are qualified participants in the certification election process. Held: Yes. In a certification election all rank-and-file employees in the appropriate bargaining unit are entitled to vote. This principle is clearly stated in Art. 255 of the Labor Code which states that the "labor organization designated or selected by the majority of the employees in an appropriate bargaining unit shall be the exclusive representative of the employees in such unit for the purpose of collective bargaining." Collective bargaining covers all aspects of the employment relation and the resultant CBA negotiated by the certified union binds all employees in the bargaining unit. Hence, all rank-and-file employees, probationary or permanent, have a substantial interest in the selection of the bargaining representative. The Code makes no distinction as to their employment status as basis for eligibility in supporting the petition for certification election. The law refers to "all" the employees in the bargaining unit. All they need to be eligible to support the petition is to belong to the "bargaining unit". Moreover, The BLR cannot exercise discretion if the petition is in compliance with the 30% requirement. However if the holding of certification election is supported by less than 30% (the Bureau of Labor Relations has discretion whether or not to order the holding of certification election depending on the circumstances of the case. Even assuming that the 20% requirement is not reached because of the alleged disaffiliation, this will not defeat the petition for certification election. It becomes more imperative to conduct one. The alleged disaffiliation from the petitioning unions (PMA-FFW and SAMA-ASIA-FFW) in favor of the ADLO-KMU raised a genuine representation issue which can best be tested in a certification election. Western Agusan Workers Union Local 101 of the United Lumber and General Workers of the Philippines v. Trajano, 196 SCRA 622 (1991) Facts: Petitioner, Western Agusan Workers Union (WAWU), became one of the local unions of United Lumber and General Workers of the Philippines (ULGWP), a national union. Private respondent Western Agusan Workers Union (KMU-WAWU) refers to the group of members of WAWU-ULGWP Local 101, who disaffiliated themselves from the mother union ULGWP while Philippine Transport and General Workers Organization (PTGWO) is a national labor organization. Nasipit Lumber Company has employed in its business a working force of more or less 2,100 workers in the level of rank and file, almost all of whom are members of the Labor Union of the Western Agusan Workers Union (WAWU). PTGWO instituted a petition for certification election alleging that 30% of the rank and file workers of the Nasipit Lumber Company and its sister companies supported its petition which was opposed by the compulsory intervenor WAWU-ULGWP Local 101 on the grounds that the petition does not meet the 30% consent requirement as the names and signatures appearing in the list submitted by PTGWO had been secured through fraud and that the purported signatures thereon were mere forgeries.Subsequently, the members of the Local KMU-WAWU adopted a resolution to disaffiliate from ULGWP invoking the law on disaffiliation which limits such actions only within the 60-day freedom period. WAWU contended that as an independently registered union, its disaffiliation from ULGWP did not extinguish its juridical personality; that it was the principal party to the CBA while ULGWP was only its agent from which it could disaffiliate at will; that while it was in favor of the certification election, the federation's name should not be attached to its own in the same ballot; that as the principal party to the CBA, it was the bargaining agent entitled to administer and enforce the agreement with the employer; that the issue of disaffiliation was not a prejudicial question to the settlement of representation proceedings; that, in fact, it could be treated in a separate proceeding. Further, WAWU enjoined NALCO from farther deducting union dues in favor of ULGWP. ULGWP argued that since it lacks the requisite support from the members of the bargaining unit, the petition should not be granted; that the disaffiliation of WAWU was void as only 429 out of total union membership of 2,149 voted in its favor and since no notice of disaffiliation was given to ULGWP by WAWU as required by the federation's constitution and by-laws; that WAWU had not acquired any legal personality to participate in the representation proceedings as it had not filed its intervention within the freedom period; that while it was true that majority of its officers had disaffiliated, ULGWP had reorganized its local and had a new set of officers elected, that the federation had issued a charter to WAWU-ULGWP Local 101 making it a local organization of the federation; that being the true and real compulsory intervenor, WAWU-ULGWP Local 101 should be placed on the ballot as a choice for the workers in the certification election. Moreover, ULGWP stressed to NALCO that WAWU's disaffiliation was void and, hence, it was still entitled to receive the dues checked-off from the members of the bargaining unit. Med-Arbiter granted the certification election sought by PTGWO; declaring the disaffiliation of WAWU from ULGWP meritorious, legal and valid, BLR affirmed. Issue: WON certification election is valid. Held: Yes. The sentiments/loyalties of the rank-and-file employees is best determined through certification election. The fact that members of the mother organization or unit have disaffiliated is an added reason for certification election. Once the fact of disaffiliation has been demonstrated beyond doubt, a certification election is the most expeditious way of determining which labor organization is to be the exclusive bargaining representative. *Case was dismissed for being moot and academic. WAWU members who were not present during the election certified that they were in favor of disaffiliation of WAWU from ULGWP and WAWU entered into a new Collective Bargaining Agreement with NALCO on December 4, 1985 effective for three (3) years from July 1, 1985 to June, 1988. 2. Unorganized Establishment – 212 (h); 257; 242 (b) Art. 212. Definitions.(h) "Legitimate labor organization" means any labor organization duly registered with the Department of Labor and Employment, and includes any branch or local thereof. Art. 257. Petitions in unorganized establishments. In any establishment where there is no certified bargaining agent, a certification election shall automatically be conducted by the Med-Arbiter upon the filing of a petition by a legitimate labor organization. (As amended by Section 24, Republic Act No. 6715, March 21, 1989) Art. 242. Rights of legitimate labor organizations. A legitimate labor organization shall have the right:To be certified as the exclusive representative of all the employees in an appropriate bargaining unit for purposes of collective bargaining; 92 | P a g e

a. Definition Philippine Telegraph and Telephone Corporation v. Laguesma, 223 SCRA 452 (1993) California Manufacturing Corporation v. Laguesma, 209 SCRA 609 (1992) 1. May 24, 1990, a petition for certification election among the supervisors of California Manufacturing Corporation (CMC for brevity) was filed by the Federation of Free Workers (FFW) — California Manufacturing Corporation Supervisors Union Chapter (CALMASUCO), 2. CMC (the co.) filed answer saying that the petition does not have 25% support from barg. Unit, and that a big number of the supposed signatories to the petition are not actually supervisors as they have no subordinates to supervise, nor do they have the powers and functions which under the law would classify them as supervisors 3. FFW—CALMASUCO filed its reply maintaining that under the law, when there is no existing unit yet in a particular bargaining unit at the time a petition for certification election is filed, the 25% rule on the signatories does not apply; that the "organized establishment" contemplated by law does not refer to a "company" per se but rather refers to a "bargaining unit" which may be of different classifications in a single company; that CMC has at least two (2) different bargaining units, namely, the supervisory (unorganized) and the rank-and-file (organized); that the signatories to the petition have been performing supervisory functions; that since it is CMC which promoted them to the positions, of supervisors. it is already estopped from claiming that they are not supervisors; that the said supervisors were excluded from the coverage of the collective bargaining agreement of its rank-and-file employees; and that the contested signatories are indeed supervisors as shown in the "CMC Master List of Employees" 4. MA: hold CE; DOLE: hold CE I: whether CE should be held without showing of 25% support SC: Article 257 of the Labor code is applicable to unorganized labor organizations and not to establishments where there exists a certified bargaining agent which had previously entered into a collective bargaining agreement with the management; Otherwise stated, the establishment concerned must have no certified bargaining agent the supervisors of CMC which constitute a bargaining unit separate and distinct from that of the rank-and-file, have no such agent. thus they correctly filed a petition for certification election thru union FFW-CALMASUCO, likewise indubitably a legitimate labor organization. CMC's insistence on the 25% subscription requirement, is clearly immaterial. The same has been expressly deleted by Section 24 of Republic Act No. 6715 and is presently prescribed only in organized establishments, that is, those with existing bargaining agents. Compliance with the said requirement need not even be established with absolute certainty. The Court has consistently ruled that "even conceding that the statutory requirement of 30% (now 25%) of the labor force asking for a certification election had not been strictly compiled with, the Director (now the Med-Arbiter) is still empowered to order that it be held precisely for the purpose of ascertaining which of the contending labor organizations shall be the exclusive collective bargaining agent requirement then is relevant only when it becomes mandatory to conduct a certification election. In all other instances, the discretion, according to the rulings of this Tribunal, ought to be ordinarily exercised in favor of a petition for certification CMC as employer has no standing to question a certification election The only exception is where the employer has to file the petition for certification election pursuant to Article 259 (now 258) of the Labor Code because it was requested to bargain collectively. Thereafter, the role of the employer in the certification process ceases. The employer becomes merely a bystander SC: hold CE. Celine Marketing Corporation v. Laguesma, 205 SCRA 849 (1992) b. Mandatory Election Sugbuanon Rural Bank, Inc. v. Laguesma, 324 SCRA 425 (2000) Facts: Petitioner Sugbuanon Rural Bank, Inc., (Sugbuanon) is a duly-registered banking institution with principal office in Cebu City and a branch in Mandaue City. Private respondent SRBI-Association of Professional, Supervisory, Office, and Technical Employees Union (Union) is a legitimate labor organization affiliated with the Trade Unions Congress of the Philippines (TUCP) and was granted by the DOLE Regional Office in Cebu City a Certificate of Registration. The union filed a petition for certification election of the supervisory employees of Sugbuanon. Sugbuanon sought to have the Union's petition dismissed, but was denied by the Med-Arbiter and DOLE. Petitioner’s prayer: To cancel respondent union's registration on the ground that the APSOTEU-TUCP members were actually managerial employees who were prohibited by law from joining or organizing unions. DOLE: Petition denied for lack of merit on the ground that APSOTEU- TUCP was a legitimate labor organization, and as such, it had the legal right to represent its members for collective bargaining purposes until and unless a final order is issued canceling APSOTEU- TUCP's registration certificate Issue: 1. WON members of the respondent union are managerial employees and/or highly-placed confidential employees

93 | P a g e

2. WON the Med-Arbiter may validly order the holding of a certification election upon the filing of a petition for certification election by a registered union, despite the petitioner’s appeal pending before the DOLE Secretary against the issuance of the union’s registration Ruling: 1. NO. The employees in question only had recommendatory powers subject to evaluation, review, and final decision by the bank's management. The job description forms submitted by petitioner clearly show that the union members in question may not transfer, suspend, lay-off, recall, discharge, assign, or discipline employees. Moreover, the forms also do not show that the Cashiers, Accountants, and Acting Chiefs of the loans Department formulate and execute management policies which are normally expected of management officers. Neither are they confidential employees whose duties are specifically connected to labor relations. 2. YES. One of the rights of a legitimate labor organization under Article 242(b) of the Labor Code is the right to be certified as the exclusive representative of all employees in an appropriate bargaining unit for purposes of collective bargaining. Having complied with the requirements of Art. 234, respondent union is a legitimate labor union. Article 257 of the Labor Code mandates that a certification election shall automatically be conducted by the Med-Arbiter upon the filing of a petition by a legitimate labor organization. Nothing is said therein that prohibits such automatic conduct of the certification election if the management appeals on the issue of the validity of the union's registration. B. The employer as initiating party – 212 (e); 258; 258-A Art. 212. Definitions.(e) "Employer" includes any person acting in the interest of an employer, directly or indirectly. The term shall not include any labor organization or any of its officers or agents except when acting as employer. Art. 258. When an employer may file petition. When requested to bargain collectively, an employer may petition the Bureau for an election. If there is no existing certified collective bargaining agreement in the unit, the Bureau shall, after hearing, order a certification election. All certification cases shall be decided within twenty (20) working days. The Bureau shall conduct a certification election within twenty (20) days in accordance with the rules and regulations prescribed by the Secretary of Labor. Role Employer Republic v. Kawashima Textile, etc. 559 SCRA 386 (2008) Facts: Petitioner Kawashima Free Workers Union (KFWU) was a certified chapter of the National Federation Phil. Transport & General Workers Organization and had been listed in the registry of legitimate labor organizations. It filed with the DOLE Regional Office a Petition for Certification Election to be conducted in the bargaining unit composed of rank-and-file employees of respondent Kawashima Textile Mfg. Phils., Inc. (Kawashima). Petitioner’s prayer: To be allowed to conduct a certification election Respondent’s prayer: To dismiss the petition for certification election on the ground that KFWU did not acquire any legal personality because its membership consisted of rank-and- file and supervisory employees, and of its failure to submit its books of account; consequently the registration of petitioner Union ought to be canceled Med-Arbiter: Petition for certification election dismissed on the ground that KFWU was not a legitimate labor organization because at least two of its members were supervisory employees citing Toyota Motor Philippines Corporation v. Toyota Motor Philippines Corporation Labor Union and Dunlop Slazenger (Phils.), Inc. vs. Honorable Secretary of Labor and Employment DOLE: Petition for certification election granted on the ground that while Article 245 declares supervisory employees ineligible for membership in a labor organization for rank-and-file employees, the provision did not state the effect of such prohibited membership on the legitimacy of the labor organization and its right to file for certification election. Section 11, Paragraph II, Rule XI of Department Order No. 9 "provides for the dismissal of a petition for certification election based on lack of legal personality of a labor organization only on the following grounds: (1) [KFWU] is not listed by the Regional Office or the Bureau of Labor Relations in its registry of legitimate labor organizations; or (2) [KFWU's] legal personality has been revoked or canceled with finality." Neither was present in this case; on the contrary, KFWU's legal personality was well-established. As to the failure of KFWU to file its books of account, the DOLE held that such omission was not a ground for revocation of union registration or dismissal of petition for certification election, for under Section 1, Rule VI of Department Order No. 9, a local or chapter like KFWU was no longer required to file its books of account. CA: Petition for certification election denied on the ground that KFWU cannot qualify as a legitimate labor organization , which cannot be corrected in the inclusionexclusion proceedings during the pre-election conference. Issues: 1. WON a mixed membership of rank-and-file and supervisory employees in a union is a ground for the dismissal of a petition for certification election; and 2. WON the legitimacy of a duly registered labor organization can be collaterally attacked in a petition for a certification election through a motion to dismiss filed by an employer Ruling: Under sec. 9 of R.A. No. 9481, article 245-A is inserted into the Labor Code to read as follows: ‘Art. 245-A. Effect of Inclusion as Members of Employees Outside the Bargaining Unit. - The inclusion as union members of employees outside the bargaining unit shall not be a ground for the cancellation of the registration of the union. Said employees are automatically deemed removed from the list of membership of said union .’ Furthermore, under Section 12 of R.A. No. 9481, employers are merely a bystander with no personality to interfere with or thwart a petition for certification election filed by a legitimate labor organization. However, R.A. No. 9481 took effect only on June 14, 2007. As the petition for certification election was filed by KFWU on January 24, 2000, R.A. No. 9481 cannot apply to it. Instead, the law and rules in force at the time of the filing by KFWU of the petition for certification election on January 24, 2000 are R.A. No. 6715, amending Book 94 | P a g e

V of Presidential Decree (P.D.) No. 442 (Labor Code), as amended, and the Rules and Regulations Implementing R.A. No. 6715, as amended by Department Order No. 9, series of 1997. It is within the parameters of R.A. No. 6715 and the Implementing Rules that the Court will now resolve the two issues raised by petitioner. 1. NO. Under sec. 3 of R.A. No. 875, as interpreted in Lopez v. Chronicle Publication Employees Association, the invalidity of membership of a supervisor in a union of rank-and-file employees does not make the union illegal, where the requirements of the law for the organization thereof are, nevertheless, satisfied and met. Then the Labor Code was enacted in 1974 without reproducing Sec. 3 of R.A. No. 875. The provision in the Labor Code closest to Sec. 3 is Article 290, which is deafeningly silent on the prohibition against supervisory employees mingling with rank-and-file employees in one labor organization. While amending certain provisions of Book V of the Labor Code, E.O. No. 111 and its implementing rules continued to recognize the right of supervisory employees, who do not fall under the category of managerial employees, to join a rank- and-file labor organization. Effective 1989, R.A. No. 6715 restored the prohibition against the questioned mingling in one labor organization. Unfortunately, just like R.A. No. 875, R.A. No. 6715 omitted specifying the exact effect any violation of the prohibition would bring about on the legitimacy of a labor organization . It was the Rules and Regulations Implementing R.A. No. 6715 (1989 Amended Omnibus Rules) which supplied the deficiency by introducing the following amendment to Rule II (Registration of Unions). By that provision, any questioned mingling will prevent an otherwise legitimate and duly registered labor organization from exercising its right to file a petition for certification election. Thus, in Toyota and Dunlop, the Court, citing Article 245 of the Labor Code, as amended by R.A. No. 6715, held that a labor organization composed of both rank-andfile and supervisory employees is no labor organization at all. Not being one, an organization which carries a mixture of rank-and-file and supervisory employees cannot possess any of the rights of a legitimate labor organization, including the right to file a petition for certification election for the purpose of collective bargaining. But then, on June 21, 1997, the 1989 Amended Omnibus Rules was further amended by Department Order No. 9, series of 1997 (1997 Amended Omnibus Rules). Specifically, the requirement under Sec. 2(c) of the 1989 Amended Omnibus Rules - that the petition for certification election indicate that the bargaining unit of rankand-file employees has not been mingled with supervisory employees - was removed. Then came Tagaytay Highlands Int'l. Golf Club, Inc. v. Tagaytay Highlands Employees Union-PGTWO in which the core issue was whether mingling affects the legitimacy of a labor organization and its right to file a petition for certification election. Given the altered legal milieu, the Court abandoned the view in Toyota and Dunlop and reverted to its pronouncement in Lopez that while there is a prohibition against the mingling of supervisory and rank-and-file employees in one labor organization, the Labor Code does not provide for the effects thereof. Thus, the Court held that after a labor organization has been registered, it may exercise all the rights and privileges of a legitimate labor organization. In San Miguel Corp. (Mandaue Packaging Products Plants) v. Mandaue Packing Products Plants-San Miguel Packaging Products-San Miguel Corp. Monthlies Rank-and-File Union-FFW, the Court explained that since the 1997 Amended Omnibus Rules does not require a local or chapter to provide a list of its members, it would be improper for the DOLE to deny recognition to said local or chapter on account of any question pertaining to its individual members. More to the point is Air Philippines Corporation v. Bureau of Labor Relations, which involved a petition for cancellation of union registration filed by the employer in 1999 against a rank-and-file labor organization on the ground of mixed membership; the Court therein reiterated its ruling in Tagaytay Highlands that the inclusion in a union of disqualified employees is not among the grounds for cancellation, unless such inclusion is due to misrepresentation, false statement or fraud under the circumstances enumerated in Sections (a) and (c) of Article 239 of the Labor Code. All said, while the latest issuance is R.A. No. 9481, the 1997 Amended Omnibus Rules, as interpreted by the Court in Tagaytay Highlands, San Miguel and Air Philippines, had already set the tone for it. Toyota and Dunlop no longer hold sway in the present altered state of the law and the rules. NO. Except when it is requested to bargain collectively, an employer is a mere bystander to any petition for certification election; such proceeding is non-adversarial and merely investigative, for the purpose thereof is to determine which organization will represent the employees in their collective bargaining with the employer. The choice of their representative is the exclusive concern of the employees; the employer cannot have any partisan interest therein; it cannot interfere with, much less oppose, the process by filing a motion to dismiss or an appeal from it; not even a mere allegation that some employees participating in a petition for certification election are actually managerial employees will lend an employer legal personality to block the certification election. The employer's only right in the proceeding is to be notified or informed thereof. Notre Dame v. Laguesma, 433 SCRA 224 (2004) FACTS: Oct 14, 1991 – Notre Dame of Greater Manila Teachers & Employees Union filed with the DOLE a petition for direct certification as the sole and exclusive bargaining agent or certification election among the rank and file employees of petitioner NDGM. November 18, 1991 – Med-Arbiter granted certification election instead of direct certification and ordered conduct of pre-election conference, taking into account the following choices: 1. Notre Dame of Greater Manila Teachers and Employees Union (NDGMTEU); and 2. No Union. January8, 1992 - a pre-election conference was conducted wherein the parties agreed, among others, that the certification election shall be conducted on January 18, 1992 from 10:00 o’clock in the morning to 2:00 o’clock in the afternoon and that the eligible voters shall be ‘those employees appearing in the list submitted by management as agreed upon by the parties by affixing their signatures on said list.’ 95 | P a g e

January 13, 1992, petitioner NDGM registered a motion to include probationary and substitute employees in the list of qualified voters. On the same day, respondent Med-Arbiter Falconitin denied said motion by handwritten notation on the motion itself – ‘1/13/92 – The Rep. officer allow[s] only regular employees to vote’. January 17, 1992 – NDGM filed appeal January 18, 1992 – certification election wherein NGMTEU won with 56 votes over the “No votes” of 23; petitioner filed written notice of protest March 16, 1992 - certified private respondent NDGMTEU as the sole and exclusive bargaining agent of all the rank-and-file employees of petitioner and accordingly dismissed petitioner’s protest. March 30, 1992 – petitioner lodged an appeal July 23, 1992 – Usec. Laguesma dismissed appeal for lack of merit. ISSUE: WON petitioner Notre Dame of Greater Manila can question list of voters. HELD: No. RATIO: CA: The CA ruled that petitioner had no standing to question the qualification of the workers who should be included in the list of voters because, in the process of choosing their collective bargaining representative, the employer was definitely an intruder. SC: Employer has no standing to question the election, which is the sole concern of the workers. In Joya v. PCGG,14 this Court explained that "‘[l]egal standing’ means a personal and substantial interest in the case such that the party has sustained or will sustain direct injury as a result of the x x x act that is being challenged. The term ‘interest’ is material interest, an interest in issue and to be affected by the decree, as distinguished from mere interest in the question involved, or a mere incidental interest. Moreover, the interest of the party plaintiff must be personal and not one based on a desire to vindicate the constitutional right of some third and unrelated party."15 Clearly, petitioner did not and will not sustain direct injury as a result of the non-inclusion of some of its employees in the certification election. Hence, it does not have any material interest in this case. Only the employees themselves, being the real parties-in-interest,16 may question their removal from the voters’ list. This Court would be the last agency to support an attempt to interfere with a purely internal affair of labor.24 The provisions of the Labor Code relating to the conduct of certification elections were enacted precisely for the protection of the right of the employees to determine their own bargaining representative. Employers are strangers to these proceedings. They are forbidden from influencing or hampering the employees’ rights under the law. They should not in any way affect, much less stay, the holding of a certification election by the mere convenience of filing an appeal with the labor secretary. To allow them to do so would do violence to the letter and spirit of welfare legislations intended to protect labor and to promote social justice. SMC Quarry Workers Union v. Titan Megabags Industrial Corporation 428 SCRA 524 (2004) 4. NATURE OF PROCEEDINGS A. Nature Young Men Labor Union Stevedores v. CIR, 13 SCRA 285 (1965) Certification proceedings are investigatory in nature. Object of the proceedings is not the decision of any alleged commission of wrong nor asserted deprivation of right, but is merely the determination of proper bargaining units and the ascertainment of the will and choice and choice of the employees in respect of the selection of the bargaining representative. The determination of the proceeding does not entail the entry of remedial orders or redress of rights, but culmination solely in an official designation of bargaining units and an affirmation of the employees expressed choice of bargaining agent. B. Certification Election and Run-off – Process and Procedure – 256, 257, 258 Art. 256. Representation issue in organized establishments. In organized establishments, when a verified petition questioning the majority status of the incumbent bargaining agent is filed before the Department of Labor and Employment within the sixty-day period before the expiration of the collective bargaining agreement, the Med-Arbiter shall automatically order an election by secret ballot when the verified petition is supported by the written consent of at least twenty-five percent (25%) of all the employees in the bargaining unit to ascertain the will of the employees in the appropriate bargaining unit. To have a valid election, at least a majority of all eligible voters in the unit must have cast their votes. The labor union receiving the majority of the valid votes cast shall be certified as the exclusive bargaining agent of all the workers in the unit. When an election which provides for three or more choices results in no choice receiving a majority of the valid votes cast, a run-off election shall be conducted between the labor unions receiving the two highest number of votes: Provided, that the total number of votes for all contending unions is at least fifty percent (50%) of the number of votes cast. At the expiration of the freedom period, the employer shall continue to recognize the majority status of the incumbent bargaining agent where no petition for certification election is filed. (As amended by Section 23, Republic Act No. 6715, March 21, 1989) Art. 257. Petitions in unorganized establishments. In any establishment where there is no certified bargaining agent, a certification election shall automatically be conducted by the Med-Arbiter upon the filing of a petition by a legitimate labor organization. (As amended by Section 24, Republic Act No. 6715, March 21, 1989) Art. 258. When an employer may file petition. When requested to bargain collectively, an employer may petition the Bureau for an election. If there is no existing certified collective bargaining agreement in the unit, the Bureau shall, after hearing, order a certification election. 96 | P a g e

All certification cases shall be decided within twenty (20) working days. The Bureau shall conduct a certification election within twenty (20) days in accordance with the rules and regulations prescribed by the Secretary of Labor. a) Effect of Private Agreement Ilaw at Buklod ng Manggagawa v. Director of BLR, 91 SCRA 482 (1979) HELD: Referral of the appeal to the Trade Union Congress of the Philippines (TUCP), a federation of labor unions is glaringly illegal and void. The Labor Code never intended that the Director of Labor Relations should abdicate, delegate and relinquish his arbitrational prerogatives in favor of a private person or entity or to a federation of trade unions. Article 226, 259, 260 are mandatory and should be strictly adhered to. They are part and parcel of the adequate administrative machinery established by the Labor Code for the expeditious settlement of labor disputes. The Director's act of referring the appeal is patent nullification of the policy of the Labor Code to avoid delay in the adjudication of labor controversies. Labor Code never intended that the original record of a labor case, an official public record, should be removed from the legitimate custodian and entrusted to a private person. Delivery of an official public record to a private person is fraught with mischievous consequences. Director placed himself in ridiculous situation of having to beg the TUCP for the return of the record. Director could have reconstituted the record and the Director could have decided the appeal on the basis of the reconstituted record. PLUM Foundation of Industrial and Agrarian Workers v. Noriel, 119 SCRA 299 (1982) Plum Federation of Industrial and Agrarian Workers filed a petition to bee certified as the sole and exclusive bargaining agent of the rank-and-file workers of Manila Jockey Club, Inc. However, the Manila Jockey Club Race Day Operation Employees Labor Union-PTGWO (Manila Jockey Union for brevity) filed a motion to intervene and opposition to said petition and alleged that it is the recognized collective bargaining representative of all the employees of the company and that it is in the process of negotiating a modification of the collective bargaining agreement. Another supplemental motion to dismiss was filed by intervenor PTGWO, this time invoking (1) the "No Union Raiding Clause" of the "Code of Ethics" adopted by the members of the Trade Union Congress of the Philippines (T.U.C.P.) wherein both petitioner and intervenor are members, and (2) claiming that the petition failed to satisfy the 30% requirement of the law. The entire record of the case was forwarded to the Office of the President of the TUCP. for the purpose of submitting the matter to the Congress for decision. TUCP wrote a letter to the Secretary of Labor, who transmitted the same to the Burea of Labor Relations Office, which letter stated that in a National Executive Board meeting of TUCP, it was duly approved that The Manila Jockey Union be declared as the sole and exclusive bargaining agent, thus dismissing the petition of PLUM. Thus MedArbiter dismissed the case pursuant to the letter. Petitioner PLUM filed an appeal to the Bureau of Labor Relations on the ground that TUCP has no authority in law to grant or deny election under the Labor Code, which mandated the secret ballot to elect the true union representative. The Bureau Director dismissed the appeal, ruling that while it may be true that the facts of the case may warrant the holding of a certification election in the bargaining unit concerned, it shall sustain the decision arrived at by the National Executive Board of TUCP since both contenders are members of TUCP, and that the internal rules including their Code of Ethics must be respected and enforced also by the Bureau. PLUM asserts by way of motion for reconsideration that nowhere in the Labor Code or in the Constitution has TUCP been granted any authority to supersede or impair the holding of a certification election or deny the majority employees of their right to elect their own union, and that it is not within the Code of Ethics to suppress the employees' freedom to choose their own union. ISSUE: W/N the “No Union Raiding Clause” in TUCP’s Code of Ethics bars the petition for certification election? HELD/RATIO: NO. Certification election is the fairest and most effective way of determining which labor organization can truly represent the working force. It is a fundamental postulate that the win of the majority if given expression in an honest election with freedom on the part of the voters to make their choice, is controlling. 4 Protection to labor and freedom of peaceful assembly and association are guaranteed by the Constitution. Moreover, a letter from the president of respondent union reveals the present state of affairs of the employees wherein they are deprived of the benefits of a collective bargaining agreement, for management refused to bargain with the union. If this situation continues, the employees would stand to lose a long-line of cases that the workers' welfare can be promoted through the bargaining process As to the issue of whether or not the 30% minimum subscription requirement was met, it was held that the Director is still empowered to call for a certification election provided there was no abuse of discretion. However, in the case at bar, instead of ordering an election, respondent Director dismissed the appeal of PLUM based on the decision of the TUCP, which the Court considers an impairment of the freedom of the workers to voice out their choice of the union to represent them. If there is any doubt as to the required number having met, there would be no better way than the holding of a certification election to ascertain which union really commands the allegiance of the rank-and-file employees. 5. ELECTION – MECHANICS OF PROCESS A. Posting Notice B. Waiver Jiescor Independent Union v. Torres, 221 SCRA 699 (1993)

97 | P a g e

Petitioner JISSCOR Independent Union (JIU) filed a petition for certification election among the rank-and-file employees of the Jacinto Iron and Steel Sheets Corporation (JISSCOR) In a pre-election conference held in the DOLE, an agreement was entered into by JIU, JISSCOR and SMJ-ALU, providing that the election would be conducted on September 6, 1990 from 8:00 A.M. to 3:00 P.M., and that "the mandatory five (5) days posting is hereby waived by agreement of the parties". SMJ-ALU won the certification election with 50 votes, JIU placed second at 46 votes. JIU registered a protest in the minutes of the election stating that: "we file protest on the following grounds using visor, emblem". It also filed a formal protest before the Department of Labor, National Capital Region, on the following grounds that (1) No compliance of (sic) mandatory posting of notice of certification election and necessary list of qualified voters in accordance with the Implementing Rules and Regulations, resulting in disorderly elections; (2) The lack of the required posting had misled / misinformed the voters/workers of the manner of voting, thus it resulted to some spoiled votes; (3) Forcing the workers to vote for SMJ-ALU by posting of its campaign streamer and wearing of sunvisors and pins with VOTE! SMJ-ALU before and during voting inside the polling place. Med-Arbiter Tomas F. Falconitin issued an Order declaring the September 6, 1990 certification election null and void. Respondent SMJ-ALU appealed to the DOLE Secretary and prayed that it be declared the sole and exclusive bargaining agent, which was granted. ISSUE: W/N JISSCOR may impugn the validity of the certification elections on the grounds of (1) non compliance with mandatory posting of notice, and (2) usage of ALU of campaign materials? HELD/RATIO: 1. NO. There is no merit in the petitioner's contention that the non-posting of the notice of the certification election as prescribed by Section 1, Rule VI, Book V of the Onmibus Rules Implementing the labor Code misled and confused the workers regarding the mechanics of the election. The petitioner is estopped from raising that issue for it signed an agreement with the private respondent to waive the mandatory five (5) days posting of election notices. 2. NO. Section 3, Rule VI, Book V of the Omnibus Rules implementing the Labor Code provides that the grounds of a protest may be filed on the spot or in writing with the representation officer and shall be contained in the minutes of the proceedings. Protests not so raised are deemed waived. The minutes of the certification election show, however, that JIU only protested against the use of emblem, visor, pin. Hence, other "protests [such as the posting in the chapel entrance of a huge streamer with the words: "Vote! Samahang Manggagawa ng JISSCOR-ALU"]not so raised are deemed waived" Moreover, the results of the certification election belie the petitioner's allegation that the workers were misinformed about the election for the records show that out of 104 eligible voters, 99 were able to cast their votes and only 3 were spoiled ballots. C. Voting List and Voters Acoje Workers Union v. National Mines & Allied Workers Union, 7 SCRA 730 (63) FACTS: June 9, 1961 - The Department of Labor conducted a “consent election” among the workers of Acoje Mining Company in which 5 labor unions participated, including Acoje Workers’ Union (AWU) and National Mines and Allied Workers’ Union (NAMAWU). NAMAWU won in the elections with 560 votes. AWU followed with 278 votes. AWU was defeated by NAMAWU by a margin of 282 votes. June 12, 1961 – AWU filed a motion to invalidate election upon several grounds. July 21, 1961 – lower court issued order appealed from, dismissing motion and certifying NAMAWU as the sole and exclusive bargaining agent of company workers. AWU appealed by certiorari from the order of the Court of Industrial Relations certifying the NAMAWU as the sole and exclusive bargaining agent of all workers in for purposes of collective bargaining said company, alleging among others, that the election should be invalidated for the same was “the result of acts of terrorism, force, threat and intimidation employed by" agents of respondent Union. They also alleged that, “. . . d) many workers were unable to vote for justified causes; and e) many workers, subject of unfair labor practice case actually pending in Court, were not allowed to vote, contrary to law.” ISSUE: WON AWU can question the validity of the “consent election” on the last two grounds mentioned. HELD: NO. RATIO: The last two (2) grounds are clearly untenable. It appears that on April 24, 1961, the labor unions concerned agreed, not only to the holding of the aforementioned election, but, also, to the use of the Company payroll of March 31, 1961, as the basis for determining who are qualified to vote subject to the approval of the lower court. On May 8, 1961, the Company presented its aforementioned payroll to said court and stated that the labor unions had been furnished copy thereof, at least three (3) days prior thereto. Said labor unions were given an opportunity to make their comments and observations on the list of workers contained in the payroll and to ask or suggest the inclusion or exclusion of names therein or therefrom. Petitioner's representative then stated that it would abide by whatever ruling the court may make on the matter of inclusion and exclusion of voters. Indeed, on May 19, 1961, the court issued the corresponding order for the holding of the election and in made its ruling on the question as to who were qualified to vote, and petitioner did not move for a reconsideration of said ruling, although two (2) other Labor Unions and that Company did so, and their motions for reconsideration were denied by the Court en banc. Hence, petitioner may no longer contest the accuracy of the aforementioned voters list. Pursuant thereto that Company had 1,019 workers, excluding department heads foremen, but including 48 security guards. Excluding the latter, there were, therefore, only 971 qualified voters. Of these, 904 had voted, so that only 67 qualified voters had to cast their votes. It is obvious, that this number plus the 19 ballots challenged in the election are insufficient to offset the plurality of 282 votes obtained by respondent Union. Notre Dame v. Laguesma, 433 SCRA 225 (04) 98 | P a g e

FACTS: Oct 14, 1991 – Notre Dame of Greater Manila Teachers & Employees Union filed with the DOLE a petition for direct certification as the sole and exclusive bargaining agent or certification election among the rank and file employees of petitioner NDGM. November 18, 1991 – Med-Arbiter granted certification election instead of direct certification and ordered conduct of pre-election conference, taking into account the following choices: 1. Notre Dame of Greater Manila Teachers and Employees Union (NDGMTEU); and 2. No Union. January8, 1992 - a pre-election conference was conducted wherein the parties agreed, among others, that the certification election shall be conducted on January 18, 1992 from 10:00 o’clock in the morning to 2:00 o’clock in the afternoon and that the eligible voters shall be ‘those employees appearing in the list submitted by management as agreed upon by the parties by affixing their signatures on said list.’ January 13, 1992, petitioner NDGM registered a motion to include probationary and substitute employees in the list of qualified voters. On the same day, respondent Med-Arbiter Falconitin denied said motion by handwritten notation on the motion itself – ‘1/13/92 – The Rep. officer allow[s] only regular employees to vote’. January 17, 1992 – NDGM filed appeal January 18, 1992 – certification election wherein NGMTEU won with 56 votes over the “No votes” of 23; petitioner filed written notice of protest March 16, 1992 - certified private respondent NDGMTEU as the sole and exclusive bargaining agent of all the rank-and-file employees of petitioner and accordingly dismissed petitioner’s protest. March 30, 1992 – petitioner lodged an appeal July 23, 1992 – Usec. Laguesma dismissed appeal for lack of merit. ISSUE: WON petitioner Notre Dame of Greater Manila can question list of voters. HELD: No. RATIO: CA: The CA ruled that petitioner had no standing to question the qualification of the workers who should be included in the list of voters because, in the process of choosing their collective bargaining representative, the employer was definitely an intruder. SC: Employer has no standing to question the election, which is the sole concern of the workers. In Joya v. PCGG,14 this Court explained that "‘[l]egal standing’ means a personal and substantial interest in the case such that the party has sustained or will sustain direct injury as a result of the x x x act that is being challenged. The term ‘interest’ is material interest, an interest in issue and to be affected by the decree, as distinguished from mere interest in the question involved, or a mere incidental interest. Moreover, the interest of the party plaintiff must be personal and not one based on a desire to vindicate the constitutional right of some third and unrelated party."15 Clearly, petitioner did not and will not sustain direct injury as a result of the non-inclusion of some of its employees in the certification election. Hence, it does not have any material interest in this case. Only the employees themselves, being the real parties-in-interest,16 may question their removal from the voters’ list. This Court would be the last agency to support an attempt to interfere with a purely internal affair of labor.24 The provisions of the Labor Code relating to the conduct of certification elections were enacted precisely for the protection of the right of the employees to determine their own bargaining representative. Employers are strangers to these proceedings. They are forbidden from influencing or hampering the employees’ rights under the law. They should not in any way affect, much less stay, the holding of a certification election by the mere convenience of filing an appeal with the labor secretary. To allow them to do so would do violence to the letter and spirit of welfare legislations intended to protect labor and to promote social justice. St. James School v. Samahang, 476 SCRA 12 (05) The Samahang Manggagawa sa St. James School of Quezon City (“Samahang Manggagawa”) filed a petition for certification election to determine the collective bargaining representative of the motor pool, construction and transportation employees of St. James School of Quezon City (“St. James”). 1. CE held, 149 eligible voters and 84 voters cast their votes. 2. St. James filed a certification election protest challenging the 84 votes; 179 rank and file employees did not vote in the CE; those who did vote wre construction workers of independent contractor: Architect Conrado Bacoy 3. MA: CE Void bec. Workers who voted not employees, and even if they were, not majority of R/F employees (84 out of 263) 4. on appeal to SOLE: reversed MA; Samahan wants to represent only the const’n, trans’n workers, not all the R/F, plus the list of 179 employees had only teaching and admin personnel; ORDER: open ballots and canvass 5. CA: no abuse of discretion I: is the labor union and the CE valid? SC: vis. whether labor union is composed of employees of independent contractor: St. James had previously filed for cancellation of union registration of Samahan based on lack of E-E rel. MA: cancelled reg’n, DOLE RD: cancelled reg’n, but BLR, CA and SC: reversed RD and MA and held that the 84 were employees in the const’n/trans’n department of St. James (although SC decision in that case was based on error of mode for appeal); decision is final, union is a legitimate labor org. 99 | P a g e

Vis. CE: on issue of whether there was quorum: St James alleges that in all its 5 campuses, it has 570 R/F, while in the Tandang Sora campus alone, there are 179, so the 84 who voted are not the majority SC: computation of quorum based on what the Barg. Unit to be represented, i.e. the constn/transn pool, not all the R/F (see Sec. 2 Rule XII, Book V, IRR), this pool had 149 employees, so there was majority The fact that the names of the voters were not on St James list is negligible since what was submitted was a list of the teaching/admin personnel, which is not part of the barg. Unit. SC: CE valid, canvass votes D. Voters All Employees – 212 (f); 256 Art. 212. Definitions.(f) "Employee" includes any person in the employ of an employer. The term shall not be limited to the employees of a particular employer, unless the Code so explicitly states. It shall include any individual whose work has ceased as a result of or in connection with any current labor dispute or because of any unfair labor practice if he has not obtained any other substantially equivalent and regular employment. Art. 256. Representation issue in organized establishments. In organized establishments, when a verified petition questioning the majority status of the incumbent bargaining agent is filed before the Department of Labor and Employment within the sixty-day period before the expiration of the collective bargaining agreement, the Med-Arbiter shall automatically order an election by secret ballot when the verified petition is supported by the written consent of at least twenty-five percent (25%) of all the employees in the bargaining unit to ascertain the will of the employees in the appropriate bargaining unit. To have a valid election, at least a majority of all eligible voters in the unit must have cast their votes. The labor union receiving the majority of the valid votes cast shall be certified as the exclusive bargaining agent of all the workers in the unit. When an election which provides for three or more choices results in no choice receiving a majority of the valid votes cast, a run-off election shall be conducted between the labor unions receiving the two highest number of votes: Provided, that the total number of votes for all contending unions is at least fifty percent (50%) of the number of votes cast. At the expiration of the freedom period, the employer shall continue to recognize the majority status of the incumbent bargaining agent where no petition for certification election is filed. (As amended by Section 23, Republic Act No. 6715, March 21, 1989) Yokohama Tire Philippines, Inc. v. Yokohama Employees Union, 539 SCRA 556 (07) Airtime Specialists Inc. v. Director of BLR, 180 SCRA 749 (89) Facts: Respondent Samahan ng mga Manggagawa sa Asia-FFW Chapter (SAMA-ASIA), Pinagbuklod ng Manggagawa sa Ataco-FFW Chapter (PMA for short) petitions for direct certification and/or certification election on behalf of the regular rank-and-file employees of the petitioners Airtime Specialists and Absolute Sound, Inc. and Country-Wealth Development, Ad Planner and Marketing Counsellors and Atlas Resources. Petitioners a motion to dismiss on the following grounds — disaffiliation of the rank and file employees, ineligibility of some signatories because they had less than one (1) year of service resulting in the non-compliance with the 30% requirement. Med Arbiter and BLR ordered the certification election. Issue: WON employees with less than one year of service and even probationary employees are qualified participants in the certification election process. Held: Yes. In a certification election all rank-and-file employees in the appropriate bargaining unit are entitled to vote. This principle is clearly stated in Art. 255 of the Labor Code which states that the "labor organization designated or selected by the majority of the employees in an appropriate bargaining unit shall be the exclusive representative of the employees in such unit for the purpose of collective bargaining." Collective bargaining covers all aspects of the employment relation and the resultant CBA negotiated by the certified union binds all employees in the bargaining unit. Hence, all rank-and-file employees, probationary or permanent, have a substantial interest in the selection of the bargaining representative. The Code makes no distinction as to their employment status as basis for eligibility in supporting the petition for certification election. The law refers to "all" the employees in the bargaining unit. All they need to be eligible to support the petition is to belong to the "bargaining unit". Moreover, The BLR cannot exercise discretion if the petition is in compliance with the 30% requirement. However if the holding of certification election is supported by less than 30% (the Bureau of Labor Relations has discretion whether or not to order the holding of certification election depending on the circumstances of the case. Even assuming that the 20% requirement is not reached because of the alleged disaffiliation, this will not defeat the petition for certification election. It becomes more imperative to conduct one. The alleged disaffiliation from the petitioning unions (PMA-FFW and SAMA-ASIA-FFW) in favor of the ADLO-KMU raised a genuine representation issue which can best be tested in a certification election. Barrera v. CIR, 107 SCRA 596 (81) If it were a labor organization objecting to the participation in a certification election of a company-dominated union, as a result of which a complaint for an unfair labor practice case against the employer was filed, the status of the latter union must be first cleared in such a proceeding before such voting could take place. The reason that justifies the postponement of a certification election pending an inquiry, as to the bona fides of a labor union does not apply in this case. If management is allowed to have its way, the result might be to dilute strength of an organization bent on a more zealous defense of labor's prerogatives. This is not to say that management is to be precluded from filing an unfair labor practice case. It is merely to stress that such a suit should not be allowed to lend itself as a means, whether intended or not, to prevent a truly free expression of the will of the labor group as to the organization that will represent it. 100 | P a g e

It is not only the loss of time involved but also the fear engendered in the mind of an ordinary employee that management has many weapons in its arsenal to bring the full force of its undeniable power against those of its employees dissatisfied with things as they are. This is one instance that calls for the application of the maxim, lex dilationes semper exhorret. Even on the assumption that the vigorous condemnation of the strike and the picketing were attended by violence, it does not automatically follow that thereby the strikers are no longer entitled to participate in the certification election for having automatically lost their jobs. Confederation of Citizens Labor Unions v. Noriel, 98 SCRA 474 (80) Eastland Manufacturing Co., Inc. v. Noriel, 111 SCRA 674 (82) R. Transport Corporation v. Laguesma, 227 SCRA 827 (93) Company argues that the employment status of the members of CLOP who joined the strike must first be resolved before a certification election can be conducted. This is untenable. In Philippine Fruits v. Torres, employees who have been improperly laid off but who have a present, unabandoned right to or expectation of reemployment, are eligible to vote in certification elections. Employees who participated in the strike, legally remain as such, until either the motion to declare their employment status legally terminated or their complaint for illegal dismissal is resolved by the NLRC. E. Effect Non-Participation Previous Election Reyes v. Trajano, 209 SCRA 484 (92) Facts: • In the certification election for the exclusive representative of the bargaining unit for the Tri-Union Industries Corporation, two unions vie for the support of the workers. • The competing unions were Tri-Union Employees Union-Organized Labor Association in Line Industries and Agriculture (TUEU-OLALIA) and Trade Union of the Philippines and Allied Services (TUPAS). 348 workers were supposed to participate in the certification election but only 240 cast their vote. • Among the 240 employees, 141 were members of the Iglesia Ni Kristo who all voted for NO UNION. Their votes were challenged on the ground that they should not be allowed to vote as they are not members of any union and refused to participate in the previous certification elections. It is also argued that their religion does not allow membership in labor unions and thus they should not be allowed to vote in the certification elections. • The INK employees filed a case before the Med-Arbiter to protest the exclusion of their votes. The Med-Arbiter dismissed the petition. On appeal to the BLR, Assistant Labor Secretary Cresenciano Trajano affirmed the Med-Arbiter’s decision as the INK employees are bereft of legal personality to protest their disenfranchisement. Issue: WON the votes of the 141 members of the INK can be validly excluded from the certification election. Held: • Guaranteed to all employees or workers is the right to self-organization and to form, join or assist labor organizations of their own choosing for purposes of collective bargaining. • The right not to join, affiliate with or assist any union and to disaffiliate or resign from a labor organization is subsumed in the right to join affiliate with or assist any union. The said right includes the right to refuse and refrain from exercising such right. Just as anyone cannot be denied the exercise of a right granted by law, neither can they be compelled to exercise such a conferred right. • The purpose of a certification election is to determine the wishers of majority of the employees in the appropriate bargaining unit as to whether to be or not to be represented by a labor organization and if yes, by what particular labor organization. If the results show that no union is desired by the workers, the minority cannot impose their will on the majority and they will have to wait for the next certification election to attempt a change. • The fact that the INK members, as employees voted for NO UNION, they were simply exercising that right of self-organization albeit in its negative aspect. • All bona fide employees possess such right. The fact that they did not participate in previous certification elections nor the fact that they are not members of any union does not deprive them of such right. Voting Day Asian Design and Manufacturing Corporation v. Calleja, 174 SCRA 477 (89) Facts:  Petitioner Asian Design and Manufacturing Corporation (ADMACOR) is a corporation that operates a rattan furniture factory in Cebu.  A certification election was scheduled to be held on 21 May 1986; on a regular business day, pursuant to Section 2, Rule VI, Book V of the Omnibus Rules Implementing the Labor Code, to wit: Section 2. The election shall be set during the regular business day of the company unless otherwise agreed upon by the parties. (now in Book V, Rule XII, Section 3 of the Implementing Rules) 101 | P a g e

On 19 May 1986, several factory workers of ADMACOR held a strike. No previous notice of strike was filed. Despite the strike, the scheduled certification election was conducted on 21 May 1986. Of the 423 workers who voted, 413 voted for Southern Philippines Federation of Labor (SPFL) as their exclusive bargaining agent.  On the same day of the election, ADMACOR filed a complaint for illegal strike and for illegal picketing. Furthermore, on 23 May 1986, it filed a petition to declare the certification election conducted on 21 May 1986 as null and void on the ground that there being a strike by some workers in the premises of the factory on the day of the certification election, such day cannot be considered a regular business day. Issue: WON the certification election was valid despite the strike? Held: Yes, the certification election was valid. The strike and/or picketing of some employees at the company's premises, even though eventually declared illegal, which coincided with the actual conduct of certification election, might have affected the actual performance of works by some employees, but did not necessarily make said date an irregular business day of the company to go against the aforesaid Rule. It cannot be denied that an actual election was conducted on said date where, of the 423 workers who voted, 413 voted for SPFL as its exclusive bargaining agent. Such concern over the validity of certification election must come from the employees themselves. Cited case: Trade Union Congress of the Philippines and Allied Services vs. Trajano, G.R. No. 61153, January 17, 1983, 120 SCRA 64 A certification election is the sole concern of the workers. On a matter that should be the exclusive concern of labor, the choice of a collective bargaining representative, the employer is definitely an intruder.   F. Validity Samahan ng Manggagawa sa Pacific Plastic v. Laguesma, 267 SCRA 303 (97) Facts:  Petitioner Samahan ng Manggagawa sa Pacific Plastic (SAMAHAN) and respondent Malayang Nagkakaisang Manggagawa ng Pacific Plastic (MNMPP) are labor unions of rank and file employees at the Pacific Plastic Corporation (PPC) in Valenzuela, Metro Manila.  MNMPP filed a petition for certification election on 24 August 1990, alleging that there were more or less 130 rank and file employees at PPC whom it was seeking to represent.  Before the certification election, SAMAHAN: o sought to cancel MNMPP’s union registration; o argued that there existed a collective bargaining agreement between PPC and SAMAHAN which was a bar to the certification election; and o claimed that, according to the Implementing Rules, the basis for the list of eligible voters should have been the payroll three (3) months preceding the filing of the petition for certification election instead of the list furnished by the SSS.  The Secretary of Labor ordered that a certification election be held on 06 October 1992, wherein MNMPP won by garnering 56 out of 98 eligible votes. Issue: WON the certification election was valid despite the contentions of SAMAHAN? Held: Yes, the certification election held on 06 October 1992 was valid. Art. 256 of the labor Code provides that in order to have a valid election, at least a majority of all eligible voters in the unit must have cast their votes. The unjustified refusal of PPC to submit the payroll in its custody, despite demands made by DOLE to produce it, compelled resort to the SSS list as the next best source of information to determine the list of eligible voters. Based on this list, election results show that more than a majority, i.e. 62 out of a total of 98 eligible voters cast their votes. Hence, the legal requirement for a valid election was met. (The 130 figure was only an allegation by MNMPP and thus irrelevant.) In the case of ALU-TUCP v. Trajano, the representation case will not be adversely affected by a CBA registered before or during the freedom period or during the pendency of the representation case. In ALU v. Calleja, a CBA, which was prematurely renewed, is not a bar to the holding of a certification election. Hence, the CBA entered into between SAMAHAN and PPC during the pendency of the representation case and after the filing of the petition for certification election on 24 August 1990 by MNMPP, cannot possibly prejudice the certification election nor render it moot. A certification election can also be conducted despite pendency of a petition to cancel the union registration certificate. For the fact is that at the time the respondent union filed its petition for certification, it still had the legal personality to perform such act absent an order directing its cancellation. G. Protest Period Timbungco v. Castro, 183 SCRA 140 (90) DHL Philippines Corporation United Rank and File Association-Federation of Free Workers v. Buklod ng Manggagawa ng DHL Philippines Corporation, 434 SCRA 670 (04) 102 | P a g e

H. Appeal – 259 Art. 259. Appeal from certification election orders. Any party to an election may appeal the order or results of the election as determined by the Med-Arbiter directly to the Secretary of Labor and Employment on the ground that the rules and regulations or parts thereof established by the Secretary of Labor and Employment for the conduct of the election have been violated. Such appeal shall be decided within fifteen (15) calendar days. (As amended by Section 25, Republic Act No. 6715, March 21, 1989) Philippine Fruits and Vegetable Industries v. Torres, 211 SCRA 95 (92) FACTS: • Med-Arbiter Basa issued an Order granting the petition for Certification election filed by TUPAS, to be held among the regular and seasonal workers of the Philippine Fruits and Vegetables, Inc. • After a series of pre-election conferences, the issue on the voting qualifications of the 194 workers enumerated in the list of qualified voters wer left unresolved. • Thereafter, the Med-Arbiterissued an Order dated December 9, 1988 allowing 184 of the 194 questioned workers to vote, subject to challenge, in the certification election to be held on December 16, 1989. Copies of said Order were furnished the parties and on December 12, 1988 the notice of certification election was duly posted. • In the scheduled certification election, petitioner objected to the proceeding alleging that the posting of the list of eligible voters authorized to participate in the certification election was short of the five (5) days provided by law. • By agreement of petitioner and TUPAS, workers whose names were inadvertently omitted in the list of qualified voters were allowed to vote, subject to challenge • In the election results excluding the challenged votes, yes votes failed to obtain the majority of the votes cast in said certification election. Hence, the challenged votes were opened to determine the true will of the employees. The challenged votes were opened on February 2, 1989 and as canvassed, the results showed 165 votes for yes, 0 for no, while 3 votes were spoiled. • Petitioner, February 23, 1989, formally filed a Protest claiming that the required five day posting of notice was not allegedly complied with. ISSUE: WON the protest filed by petitioner company would prosper. HELD: NO • Requirements in order that a protest filed would prosper: (1) The protest must be filed with the representation officer and made of record in the minutes of the proceedings before the close of election proceedings, and (2) The protest must be formalized before the Med-Arbiter within five (5) days after the close of the election proceedings. Phil. Fruits after filing a manifestation of protest on election day, only formalized more than two months after the close of election proceedings. • The phrase "close of election proceedings" as used in Sections 3 and 4 of Rule VI, Book V of the Implementing Rules of the Labor Code refers to that period from the closing of the polls to the counting and tabulation of the votes as it could not have been the intention of the Implementing Rules to include in the term "close of the election proceedings" the period for the final determination of the challenged votes and the canvass thereof, as in the case at bar which may take a very long period. • The petitioner, after filing a manifestation of protest on December 16, 1988, election day, only formalized the same on February 20, 1989, or more than two months after the close of election proceedings. Samahan ng Manggagawa sa Filsystems v. Secretary of Labor, 290 SCRA 680 (98) FACTS: • Petitioner Unon, a registered labor union, filed a Petition for Certification Election among the rank-and-file employees of private respondent FILSYSTEMS, Inc. before the DOLE. • Private respondent opposed the petition, questioning the status of petitioner as a legitimate labor organization on the ground of lack of proof that its contract of affiliation with the NAFLU-KMU has been submitted to the BLR within 30 days from its execution. • The Med-Arbiter dismissed the petition for certification election, ruling that petitioner, as an affiliate of NAFLU-KMU, has no legal personality on account of its failure to comply with paragraphs (a), (b) and (e) of Section 3, Rule II of the Implementing Rules of Book V of the Labor Code. • Petitioner appealed contending that it has the right to file a petition for certification election regardless of its failure to prove its affiliation with NAFLU-KMU. 103 | P a g e



Petitioner opposed the Motion to Dismiss, contending that its appeal is not moot as the certification election held was void and that the CBA executed between the FWU and the private respondent could not affect its pending representation case. • Respondent Secretary dismissed the appeal on the ground that it has been rendered moot by the certification of FWU as the sole and exclusive bargaining agent of the rank-and-file workers of respondent company. ISSUE: WON the appeal filed by the petitioner was rendered moot and academic by the subsequent certification election ordered by the Med-Arbiter, won by the FWU and which culminated in a CBA with private respondent. HELD: No, the order of the Med-Arbiter dismissing petitioner's petition for certification election was seasonably appealed. • The appeal stopped the holding of any certification election. There was thus an unresolved representation case at the time the CBA was entered between FWU and private respondent. Such CBA cannot and will not prejudice petitioner's pending representation case or render the same moot. • Accordingly, there was an unresolved representation case at the time the CBA was entered between FWU and private respondent. Following Section 4, Rule V of the Implementing Rules of Book V of the Labor Code, such CBA cannot and will not prejudice petitioner's pending representation case or render the same moot. Notre Dame Megabags Industrial Corporation v. Laguesma, 433 SCRA 224 (04) FACTS: Oct 14, 1991 – Notre Dame of Greater Manila Teachers & Employees Union filed with the DOLE a petition for direct certification as the sole and exclusive bargaining agent or certification election among the rank and file employees of petitioner NDGM. November 18, 1991 – Med-Arbiter granted certification election instead of direct certification and ordered conduct of pre-election conference, taking into account the following choices: 1. Notre Dame of Greater Manila Teachers and Employees Union (NDGMTEU); and 2. No Union. January8, 1992 - a pre-election conference was conducted wherein the parties agreed, among others, that the certification election shall be conducted on January 18, 1992 from 10:00 o’clock in the morning to 2:00 o’clock in the afternoon and that the eligible voters shall be ‘those employees appearing in the list submitted by management as agreed upon by the parties by affixing their signatures on said list.’ January 13, 1992, petitioner NDGM registered a motion to include probationary and substitute employees in the list of qualified voters. On the same day, respondent Med-Arbiter Falconitin denied said motion by handwritten notation on the motion itself – ‘1/13/92 – The Rep. officer allow[s] only regular employees to vote’. January 17, 1992 – NDGM filed appeal January 18, 1992 – certification election wherein NGMTEU won with 56 votes over the “No votes” of 23; petitioner filed written notice of protest March 16, 1992 - certified private respondent NDGMTEU as the sole and exclusive bargaining agent of all the rank-and-file employees of petitioner and accordingly dismissed petitioner’s protest. March 30, 1992 – petitioner lodged an appeal July 23, 1992 – Usec. Laguesma dismissed appeal for lack of merit. ISSUE: WON petitioner Notre Dame of Greater Manila can question list of voters. HELD: No. RATIO: CA: The CA ruled that petitioner had no standing to question the qualification of the workers who should be included in the list of voters because, in the process of choosing their collective bargaining representative, the employer was definitely an intruder. SC: Employer has no standing to question the election, which is the sole concern of the workers. In Joya v. PCGG,14 this Court explained that "‘[l]egal standing’ means a personal and substantial interest in the case such that the party has sustained or will sustain direct injury as a result of the x x x act that is being challenged. The term ‘interest’ is material interest, an interest in issue and to be affected by the decree, as distinguished from mere interest in the question involved, or a mere incidental interest. Moreover, the interest of the party plaintiff must be personal and not one based on a desire to vindicate the constitutional right of some third and unrelated party."15 Clearly, petitioner did not and will not sustain direct injury as a result of the non-inclusion of some of its employees in the certification election. Hence, it does not have any material interest in this case. Only the employees themselves, being the real parties-in-interest,16 may question their removal from the voters’ list. This Court would be the last agency to support an attempt to interfere with a purely internal affair of labor.24 The provisions of the Labor Code relating to the conduct of certification elections were enacted precisely for the protection of the right of the employees to determine their own bargaining representative. Employers are strangers to these proceedings. They are forbidden from influencing or hampering the employees’ rights under the law. They should not in any way affect, much less stay, the 104 | P a g e

• • • •

Meanwhile, another union, the Filsystem Workers Union (FWU), filed a Petition for Certification Election in the same bargaining unit. The election was won by FWU. Thereafter, FWU and the private respondent negotiated a CBA. Private respondent filed a Motion to Dismiss Appeal of petitioner as it has become moot and academic.

holding of a certification election by the mere convenience of filing an appeal with the labor secretary. To allow them to do so would do violence to the letter and spirit of welfare legislations intended to protect labor and to promote social justice. SMC Quarry Workers Union v. Titan Megabags Industrial Corporation, 428 SCRA 524 (04) SMC Quarry Union filed a petition for certification, alleging that it seeks to represent regular rank-and-file workers. Titan Megabags opposed petition contending that members of union are not its employees but that of Stitchers Multipurpose - an independent contractor. Med-Arbiter held tha Titan is employer of said members and directed certification election to be conducted. Titan filed MR (denied for being late by 7 days) then petition for certiorari with CA. CA disallowed certification election. SMC Quarry Union - petition for review. Issue: Whether or not CA is correct in setting aside final and executory resolutions of Secretary of DOLE. Held: No. 1. Art. 259 LC - any party to a certification election may appeal the order of Med-Arbiter directly to the Sec. of Labor. Decision of Secretary of Labor shall be final and executory - unless restrained by appropriate court. Remedy of aggrieved party in decision of Secretary of DOLE - file MR then certiorari (Rule 65). 2. Titan's failure to file MR (late by 7 days) is fatal to its cause and rendered final and executory the resolution of Secreatry of DOLE. 3. In any case, Titan as employer is a bystander to certification elections and has no right or material interest to assail the same. PART 5 COLLECTIVE BARGAINING PROCESS, PROCEDURES & ISSUES A. GENERAL CONCEPTS 1. Policy Declaration – 211 (a); Art. XII, Sec. 3, Constitution Art. 211. Declaration of Policy. A. It is the policy of the State: a. To promote and emphasize the primacy of free collective bargaining and negotiations, including voluntary arbitration, mediation and conciliation, as modes of settling labor or industrial disputes; Kiok Loy v. NLRC, 141 SCRA 179 (86) NATURE: Petition for certiorari to annul the decision of the National Labor Relations Commission FACTS: In a certification election held on October 3, 1978, the Pambansang Kilusang Paggawa (Union for short) was subsequently certified in a resolution dated November 29, 1978 by the Bureau of Labor Relations as the sole and exclusive bargaining agent of the rank-and-file employees of Sweden Ice Cream Plant (Company for short). The Company's motion for reconsideration of the said resolution was denied on January 25, 1978. December 7, 1978, the Union furnished the Company with two copies of its proposed collective bargaining agreement. It also requested the Company for its counter proposals. Both requests were ignored and remained unacted upon by the Company. The Union, on February 14, 1979, filed a "Notice of Strike", with the Bureau of Labor Relations (BLR) on ground of unresolved economic issues in collective bargaining. Conciliation proceedings then followed during the thirty-day statutory cooling-off period. The Bureau of Labor Relations to certify the case to the National Labor Relations Commission for compulsory arbitration. The labor arbiter set the initial hearing for April 29, 1979. For failure however, of the parties to submit their respective position papers as required, the said hearing was cancelled and reset to another date. The Union submitted its position paper. On July 20, 1979, the National Labor Relations Commission rendered its decision declaring the respondent guilty of unjustified refusal to bargain Petitioner contends that the National Labor Relations Commission acted without or in excess of its jurisdiction or with grave abuse of discretion amounting to lack of jurisdiction in rendering the challenged decision. Petitioner further contends that the National Labor Relations Commission's finding of unfair labor practice for refusal to bargain is not supported by law ISSUE/S: WON the respondent is guilty of unjustified refusal to bargain HELD: YES unfair labor practice is committed when it is shown that the respondent employer, after having been served with a written bargaining proposal by the petitioning Union, did not even bother to submit an answer or reply to the said proposal Ratio Unfair labor practice is committed when it is shown that the respondent employer, after having been served with a written bargaining proposal by the petitioning Union, did not even bother to submit an answer or reply to the said proposal 105 | P a g e

Reasoning: Collective bargaining which is defined as negotiations towards a collective agreement, is one of the democratic frameworks under the New Labor Code, designed to stabilize the relation between labor and management and to create a climate of sound and stable industrial peace. It is a mutual responsibility of the employer and the Union and is characterized as a legal obligation. So much so that Article 249, par. (g) of the Labor Code makes it an unfair labor practice for an employer to refuse "to meet and convene promptly and expeditiously in good faith for the purpose of negotiating an agreement with respect to wages, hours of work, and all other terms and conditions of employment including proposals for adjusting any grievance or question arising under such an agreement and executing a contract incorporating such agreement, if requested by either party. While it is a mutual obligation of the parties to bargain, the employer, however, is not under any legal duty to initiate contract negotiation. The mechanics of collective bargaining is set in motion only when the following jurisdictional preconditions are present, namely, (1) possession of the status of majority representation of the employees' representative in accordance with any of the means of selection or designation provided for by the Labor Code; (2) proof of majority representation; and (3) a demand to bargain under Article 251, par. (a) of the New Labor Code . - From the over-all conduct of petitioner company in relation to the task of negotiation, there can be no doubt that the Union has a valid cause to complain against its (Company's) attitude, the totality of which is indicative of the latter's disregard of, and failure to live up to, what is enjoined by the Labor Code to bargain in good faith. DISPOSITION: Petition dismissed Scout Albano Memorial College, 85 SCRA 494 B. Nature, Purpose and Rules of Interpretation Suarez v. National Steel Corporation, 562 SCRA 495 (2008) Facts: Respondent National Steel Corporation was engaged in the business of manufacturing steel products needed for pipe making, shipbuilding, can-making and production of appliances. When respondent suffered substantial financial losses, it retrenched herein petitioners who received a separation package in accordance with the retrenchment program. After having been paid their separation benefits, petitioners executed and signed a release and quitclaim. Subsequently, respondent and the National Steel Labor Union-Federation of Free Workers (NASLU-FFW), the certified collective bargaining agent of respondent’s rank-and-file employees, signed a new CBA. Pursuant thereto, petitioners were given their salary differentials, for which they executed and signed another release and quitclaim. Nothing was heard from petitioners, until about 2½ years after their separation from the company, they wrote respondent demanding payment of retirement benefits under the CBA. Petitioners’ prayer: To be paid retirement benefits in addition to the separation pay they received from respondent pursuant to Article XIV of the existing CBA providing for retirement benefits Respondent’s prayer: To have the complaint dismissed on the ground that its retirement plan expressly prohibits the payment of retirement benefits to employees terminated for cause Labor Arbiter: Complaint dismissed for lack of merit. NLRC: Appeal granted, ruling of the Labor Arbiter reversed. CA: Petitioners were no longer entitled to retirement benefits after having received the separation pay, and were precluded from claiming such benefits because of their quitclaims. Issues: WON petitioners as retrenched employees who already received their separation pay can still recover retirement benefits. Ruling: NO. While the CBA, on its face, does not contain an express prohibition of payment of retirement benefits to retrenched employees, the parties may still prove it by means of contemporaneous and subsequent acts of the parties to the agreement, such as the execution of the affidavits by the NASLU-FFW officers and respondent’s managers. According to the officers of NASLU-FFW, the duly certified bargaining agent of respondent’s rank-and-file employees, the intent of the parties in drafting the CBA was to make payment of the separation package for retrenched employees exclusive of retirement benefits. Ratio: If the terms of the CBA are clear and have no doubt upon the intention of the contracting parties, the literal meaning of its stipulation shall prevail. However, if the CBA imports ambiguity, then the parties’ intention as shown by their conduct, words, actions and deeds – prior to, during, and after executing the agreement, must be ascertained. Lepanto Consolidated Mining Corporation v. Lepanto Local Stuff Union, 562 SCRA 495 (2008) Facts: Lepanto Consolidated Mining Company (petitioner) is a domestic mining corporation. Lepanto Local Staff Union (respondent) is the duly certified bargaining agent of petitioner's employees occupying staff positions. Petitioner and respondent entered into their 4 th CBA, which provides: ‘ARTICLE VIII - NIGHT SHIFT DIFFERENTIAL Section 3. Night Differential pay. - The Company shall continue to pay night shift differential for work during the first (11:00 p.m. to 7:00 a.m.) and third (3:00 p.m. to 11:00 p.m.) shifts to all covered employees within the bargaining unit. However, for overtime work, which extends beyond the regular day shift (7:00 a.m. to 3:00 p.m.), there will be no night differential pay added before the overtime pay is calculated.’ Petitioner’s prayer: To have the complaint dismissed due to a wrong interpretation of the CBA Respondent’s prayer: To compel petitioner to pay employees in the second shift night shift differential as provided in the CBA

106 | P a g e

Voluntary Arbiter: Petition granted; paragraph 3, Section 3, Article VIII of the 4 th CBA, only meant that an employee who extends work beyond the second shift shall receive overtime pay which shall be computed before the night shift differential pay. In other words, it excludes the night shift differential in the computation of overtime pay. CA: Voluntary Arbitrator's decision affirmed; paragraph 3, Section 3, Article VIII was clear and unequivocal in granting night shift differential pay to employees of the second shift for work rendered beyond their regular day shift. However, the night shift differential was excluded in the computation of the overtime pay. Issues: WON, under the 4th CBA, employees in the second shift are entitled to night shift differential for work performed beyond the regular day shift Ruling: YES. The provision in question was contained in the 1 st, 2nd, and 3rd CBAs between petitioner and respondent. During the effectivity of the first three CBAs, petitioner paid night shift differentials to other workers who were members of respondent for work performed beyond 3:00 p.m. Petitioner also paid night shift differential for work beyond 3:00 p.m. during the effectivity of the 4 th CBA. In fact, even after the promulgation of the Voluntary Arbitrator's decision and while the case was pending appeal, petitioner still paid night shift differential for work performed beyond 3:00 p.m. All these affirm the intention of the parties to the CBA to grant night shift differential for work performed beyond 3:00 p.m. Ratio: To ascertain the intention of the contracting parties, their contemporaneous and subsequent acts as well as their negotiating and contractual history and evidence of past practices shall be considered. United Employees Union of Gelmart Industries, Philippines v. Noriel, 67 SCRA 267 (1975) Petitioner (United Employees Union of Gelmart Industries -UEUGIP) seeks the nullification of the certification election which declared GATCORD (National Union of Garments, Textile, Cordage and Allied Workers of the Phils.) the exclusive bargaining agent of Gelmart Industries. Petitioner alleged that in the pre-election conference, it was agreed that petitioner will be listed as UEUGIP; that in the notice of certification election, it was willfully deleted and replaced by Philippine Social Security Labor Union-PSSLU; that PSSLU, although an existing labor federation, has no right of participation in the certification election; that PSSLU appeared in the sample ballot not UEUGIP. Petitioner alleged that, as a result, there was confusion in the minds of the independent workers and demoralization in the ranks of those inclined to favor petitioner. Petitioner filed a protest, but not on the ground as above alleged, but on the alleged electioneering of nuns and a priest as inspectors or observers in behalf of private respondent. Respondents denied the imputation of irregularity. Respondents made clear that petitioners only garnered 4.5% of the total votes cast (placing only fifth) while respondent GATCORD garnered 63% (it characterized such votes as “unassailable majority”); that the Certification Election was conducted pursuant to an order of the BLR and was supervised by the BLR; that UEUGIP did not lodge a protest on the alleged misprinting on the sample ballot before, during or shortly after the election; that when GATCORD petitioned for certification election, the petitioner intervened represented by Mr. Escreza (the union president) and Antonio Diaz who intervened not for UEUGIP but for UEUGIP-Workers Faction; that since UEUGIP-Workers Faction is only a faction of UEUGIP, the BLR included only UEUGIP as one of the contending unions. Subsequently, Philippine Transport and General Workers Organization (PTGWO) intervened, claiming that UEUGIP affiliated with PTWGO and moved for the correction of the name. However, during the pre-election conference, Mr. Diaz appeared no longer representing UEUGIP but entered a new union (PSSLU). It was alleged by respondent that, it was at this time, when the parties have not yet agreed on how their names should appear in the ballot, that the sample ballot was printed with the name PSSLU. (Note that after election notice was posted with the said sample ballot, the parties came to agree that UEUGIP shall appear on the ballot without PSSLU.) Issue: Whether the certification election is null and void on the basis of the alleged lack of procedural due process. Ruling: The Court ruled in favor of respondents. Reasoning of the court: • The institution of collective bargaining is, to recall Cox, a prime manifestation of industrial democracy at work. The two parties to the relationship, labor and management, make their own rules by coming to terms. That is to govern themselves in matters that really count. As labor, however, is composed of a number of individuals, it is indispensable that they be represented by a labor organization of their choice. Thus may be discerned how crucial is a certification election. So our decisions from the earliest case of PLDT Employees Union v. PLDT Co. Free Telephone Workers Union 14 to the latest, Philippine Communications, Electronics & Electricity Workers' Federation (PCWF) v. Court of Industrial Relations, have made clear. • Thus is one of the earliest cases, The Standard Cigarette Workers' Union v. Court of Industrial Relations , it was made clear in the opinion of Justice J. B. L. Reyes that "a complaint for unfair labor practice may be considered a prejudicial question in a proceeding for certification election when it is charged therein that one or more labor unions participating in the election are being aided, or are controlled, by the company or employer. The reason is that the certification election may lead to the selection of an employer-dominated or company union as the employees' bargaining representative, and when the court finds that said union is employer-dominated in the unfair labor practice case, the union selected would be decertified and the whole election proceedings would be rendered useless and nugatory." • xxx in the language of former Chief Justice Concepcion, in the leading case of LVN Pictures v. Philippine Musicians Guild "to insure the fair and free choice of bargaining representatives by employees." There must be such an opportunity to determine which labor organization shall act on their behalf. It is precisely

107 | P a g e

because respect must be accorded to the will of labor thus ascertained that a general allegation of duress is not sufficient to invalidate a certification election; it must be shown by competent and credible proof. That is to give substance to the principle of majority rule, one of the basic concepts of a democratic polity. • The matter is summarized thus in one of the latest decisions of this Court, Federation of the United Workers Organization v. Court of Industrial Relations : "The slightest doubt cannot therefore be entertained that what possesses significance in a petition for certification is that through such a device the employees are given the opportunity to make known who shall have the right to represent them. What is equally important is that not only some but all of them should have the right to do so." It would be apparent that the grievance spoken of is more fancied than real, the assertion of confusion and demoralization based on conjecture rather than reality. The mode and manner in which Antonio Diaz demonstrated how militant and articulate he could be in presenting his side of the controversy could hardly argue for the accuracy of his claim that his men did lose heart by what appeared at the most to be an honest mistake, if it could be characterized as one. Certainly then, the accusation that there was abuse of discretion, much less a grave one, falls to the ground. P.I. Manufacturing, Inc. v. P.I. Manufacturing Supervisors and Foreman Association, 543 SCRA 613 (2008) On 10 December 1987, the President signed into law RA No. 6640 providing an increase in the statutory minimum wage and salary rates of employees and workers in the private sector. On 18 December 1987, the petitioner (PI Manufacturing) and respondent (PI Manufacturing Supervisors and Foremen Association – PIMASUFA) entered into a Collective Bargaining Agreement whereby supervisors were granted an increase of P625.00 per month and the foremen an increase of P475.00 per month. The increases were made retroactive to May 12, 1987, prior to the passage of RA No. 6640, and every year thereafter. The respondent union and the federation (National Labor Union-NLU) filed a complaint with the NLRC charging the petitioner with violation of R.A. No. 6640 and attached to their complaint an illustration of the wage distortion resulting from the implementation of R.A. 6640. The Labor Arbiter ruled in favor of respondents and ordered the petitioners to grant PIMASUFA wage increases equivalent to 13.5% of their basic pay. NLRC affirmed. Court of Appeals affirmed the NLRC with modification by raising wage increase from 13.5% to 18.5%. Issue: Whether the implementation of R.A. No. 6640 resulted in a wage distortion and whether such wage distortion was cured by the 1987 CBA Ruling: The CA correctly ruled that a wage distortion occurred due to the implantation of R.A. No. 6640. The implementation of R.A. No. 6640 resulted in the increase of P10.00 in the wage rates the petitioner’s lowest paid supervisor and foremen, namely, Alcantara (supervisor), and Morales and Salvo (foremen). The increased wage rates of foremen Morales and Salvo exceeded that of supervisor Buencuchillo. Also, the increased wage rate of supervisor Alcantara exceeded those of supervisors Buencuchillo and Del Prado. Consequently, the P9.79 gap or difference between the wage rate of supervisor Del Prado and that of supervisor Alcantara was eliminated. Instead, the latter gained a P.21 lead over Del Prado. However, we are convinced that the wage distortions were remedied when respondent PIMASUFA entered into the 1987 CBA with petitioner after the effectivity of R.A. No. 6640. The 1987 CBA increased the monthly salaries of the supervisors by P625.00 and the foremen, by P475.00, effective May 12, 1987. These increases reestablished and broadened the gap, not only between the supervisors and the foremen, but also between them and the rank-and-file employees. Significantly, the 1987 CBA wage increases almost doubled that of the P10.00 increase under R.A. No. 6640. The P625.00/month means P24.03 increase per day for the supervisors, while the P475.00/month means P18.26 increase per day for the foremen. These increases were to be observed every year, starting May 12, 1987 until July 26, 1989. Clearly, the gap between the wage rates of the supervisors and those of the foremen was inevitably re-established. It continued to broaden through the years. Such gap as re-established by virtue of the CBA is more than a substantial compliance with R.A. No. 6640. Requiring petitioner to pay all the members of PIMASUFA a wage increase of 18.5%, over and above the negotiated wage increases (as ruled by the CA) provided under the 1987 CBA, is highly unfair and oppressive to the former. It was not the intention of R.A. No. 6640 to grant an across-the-board increase in pay to all the employees of petitioner. Section 2 of R.A. No. 6640 mandates only the following increases in the private sector: (1) P10.00 per day for the employees in the private sector, whether agricultural or non-agricultural, who are receiving the statutory minimum wage rates; (2) P11.00 per day for non-agricultural workers and employees outside Metro Manila; and (3) P10.00 per day for those already receiving the minimum wage up to P100.00. To be sure, only those receiving wages P100.00 and below are entitled to the P10.00 wage increase. The apparent intention of the law is only to upgrade the salaries or wages of the employees specified therein. Almost all of the members of PIMASUFA have been receiving wage rates above P100.00 and, therefore, not entitled to the P10.00 increase. Only three (3) of them are receiving wage rates below P100.00, thus, entitled to such increase. Now, to direct petitioner to grant an across-the-board increase to all of them, regardless of the amount of wages they are already receiving, would be harsh and unfair to the former. Petitioner’s motion for reconsideration GRANTED. Almario v. Philippine Airlines, 532 SCRA 614 (2007) Facts: Almario was initially hired by PAL as a Boeing 747 Systems Engineer. He was eventually promoted to the position of Airbus 300 First Officer. In order to comply with the requirements of this higher position, Almario had to undergo more than five months of training which included ground schooling in Manila and flight simulation 108 | P a g e

in Australia. PAL paid for Almario’s training. After the training, Alamario served PAL in his new rank as A-300 First Officer. However, after eight months of service, he tendered his resignation for “personal reasons.” In response to his resignation letter, PAL sent Almario a letter reminding him that the company invested heavily on his personal training on the basis that he would continue to serve the company for at least three years. The letter also informed Almario that if he would choose to resign, he would have to reimburse the company an amount representing the cost of his training. Almario pushed through with his resignation. PAL filed a Complaint against Almario for reimbursement of training costs at the Makati RTC. The company contended that it had an innominate contract do ut facias (I give that you may do) with Almario—that the company spent for Almario’s training with the understanding that he would render service to PAL until the costs of said training were recovered in at least three years. Almario denied the existence of any agreement with PAL that he would have to render service to it for three years after his training failing which he would reimburse the training costs. He pointed out that the 1991-1994 CBA between PAL and ALPAP, of which he was a member, carried no such agreement. The RTC ruled in favor of Almario. The RTC found no provision in the CBA between PAL and ALPAP stipulating that a pilot who underwent training must serve PAL for at least three years failing which s/he should reimburse the training expenses. The CA reversed the RTC and ruled in favor of PAL. The CA found Almario liable under the CBA between PAL and ALPAP and under Art 22 of the Civil Code. Issue: Whether or not Almario should be ordered to reimburse the costs for his training as a consequence of resigning before the three-year period. Held: Almario is liable; he must reimburse. 1. Almario is liable under the CBA (cf. Sec 1, Art XXIII). This provision of the CBA does not explicitly state the requirement that a pilot who has underwent training must render service to the company for at least three years, lest s/he reimburse the cost of the training. However, the Court held that this provision—and CBA provisions, in general—should be construed liberally rather than narrowly and technically, and the courts must place a practical and realistic construction upon it, giving due consideration to the context in which it is negotiated and purpose which it is intended to serve. Sec 1, Art XXXIII was incorporated into the CBA taking into consideration the reality of prohibitive training costs and the need for PAL to have a return on the investment of spending for a pilot’s training. The Court also gave credence to the company’s testimony that it has been the policy or practice of PAL to underwrite the training costs of its pilots, with the expectation of benefiting therefrom in order to recover the costs of training. 2. Almario is liable under Art 22 of the Civil Code. One may not enrich himself at the expense of another. PAL invested for the training of Almario to enable him to acquire a higher level of proficiency. PAL was justified in expecting to recover the training costs by availing itself of Almario’s services for at least three years. This expectation of PAL was not fully realized because of Almario’s untimely resignation. He cannot refuse to reimburse; otherwise, he will be violating the principle of unjust enrichment C. Waiver Rivera v. Espiritu, 374 SCRA 351 (2002) Nature Special civil action for certiorari and prohibition Facts As a result of a three week strike staged by PAL pilots affiliated with the Airline Pilots Association of the Philippines (ALPAP) PAL which was already financially beleaguered suffered serious losses, PAL’s financial situation went from bad to worse. Faced with bankruptcy, PAL adopted a rehabilitation plan and downsized its labor force by more than one-third. In protest to such action PALEA went on strike which when PAL and PALEA agreed to a more systematic reduction in PAL’s work force and the payment of separation benefits to all retrenched employees. President Estrada thru AO 16 created an Inter-Agency Task Force to address the problems of PAL. PAL management submitted to the Task Force an offer by Lucio Tan, Chairman a plan to transfer shares of stock to its employees which has a provision regarding the suspension of the Collective Bargaining Agreements (CBAs) for 10 years. PALEA Members rejected the offer.Subsequently, PAL informed the Task Force that it was shutting down its operations because given its labor problems, rehabilitation was no longer feasible, and hence, the airline had no alternative but to close shop. PALEA sought the intervention of the Office of the President in immediately convening the parties, the PAL management, PALEA, ALPAP, and FASAP, including the SEC under the direction of the Inter-Agency Task Force, to prevent the imminent closure of PAL.After several negotiations a the questioned PAL- PALEA Agreement which provided for among others the suspension of the PAL-PALEA CBA for a period of ten (10) years, provided the certain safeguards are in place. Issue WON the PAL-PALEA agreement stipulating the suspension of the PAL-PALEA CBA unconstitutional and contrary to public policy Held No. A CBA is “a contract executed upon request of either the employer or the exclusive bargaining representative incorporating the agreement reached after negotiations with respect to wages, hours of work and all other terms and conditions of employment, including proposals for adjusting any grievances or questions arising under such agreement.” The primary purpose of a CBA is the stabilization of labor-management relations in order to create a climate of a sound and stable industrial peace. In construing a CBA, the courts must be practical and realistic and give due consideration to the context in which it is negotiated and the purpose which it is intended to serve. The assailed PAL-PALEA agreement was the result of voluntary collective bargaining negotiations undertaken in the light of the severe financial situation faced by the employer, with the peculiar and unique intention of not merely promoting industrial peace at PAL, but preventing the latter’s closure. We find no conflict between said agreement and Article 253-A of the Labor Code. Article 253-A has a two-fold purpose. One is to promote industrial stability and predictability. Inasmuch as the agreement sought to promote industrial peace at PAL during its rehabilitation, said agreement satisfies the first purpose of Article 253109 | P a g e

A. The other is to assign specific timetables wherein negotiations become a matter of right and requirement. Nothing in Article 253-A, prohibits the parties from waiving or suspending the mandatory timetables and agreeing on the remedies to enforce the same. In the instant case, it was PALEA, as the exclusive bargaining agent of PAL’s ground employees, that voluntarily entered into the CBA with PAL. It was also PALEA that voluntarily opted for the 10-year suspension of the CBA. Either case was the union’s exercise of its right to collective bargaining. The right to free collective bargaining, after all, includes the right to suspend it. The acts of public respondents in sanctioning the 10-year suspension of the PAL-PALEA CBA did not contravene the “protection to labor” policy of the Constitution. The agreement afforded full protection to labor; promoted the shared responsibility between workers and employers; and the exercised voluntary modes in settling disputes, including conciliation to foster industrial peace." Disposition petition is DISMISSED. B. BARGAINING PROCEDURE 1. Private Procedure – 251 Art. 251. Duty to bargain collectively in the absence of collective bargaining agreements. In the absence of an agreement or other voluntary arrangement providing for a more expeditious manner of collective bargaining, it shall be the duty of employer and the representatives of the employees to bargain collectively in accordance with the provisions of this Code. 2. Code Procedure – 250; 212 (c) Art. 250. Procedure in collective bargaining. The following procedures shall be observed in collective bargaining: a. When a party desires to negotiate an agreement, it shall serve a written notice upon the other party with a statement of its proposals. The other party shall make a reply thereto not later than ten (10) calendar days from receipt of such notice; b. Should differences arise on the basis of such notice and reply, either party may request for a conference which shall begin not later than ten (10) calendar days from the date of request. If the dispute is not settled, the Board shall intervene upon request of either or both parties or at its own initiative and immediately call the parties to conciliation meetings. The Board shall have the power to issue subpoenas requiring the attendance of the parties to such meetings. It shall be the duty of the parties to participate fully and promptly in the conciliation meetings the Board may call; d. During the conciliation proceedings in the Board, the parties are prohibited from doing any act which may disrupt or impede the early settlement of the disputes; and e. The Board shall exert all efforts to settle disputes amicably and encourage the parties to submit their case to a voluntary arbitrator. (As amended by Section 20, Republic Act No. 6715, March 21, 1989) Art. 212. Definitions. c. "Board" means the National Conciliation and Mediation Board established under Executive Order No. 126. 3. Nature of Procedure – 253 Art. 253. Duty to bargain collectively when there exists a collective bargaining agreement. When there is a collective bargaining agreement, the duty to bargain collectively shall also mean that neither party shall terminate nor modify such agreement during its lifetime. However, either party can serve a written notice to terminate or modify the agreement at least sixty (60) days prior to its expiration date. It shall be the duty of both parties to keep the status quo and to continue in full force and effect the terms and conditions of the existing agreement during the 60-day period and/or until a new agreement is reached by the parties. Associated Labor Unions v. Calleja, 173 SCRA 178 (1989) Federation SPFL and Union NAMGAW undertook a strike after it failed to get the management of GAW Trading to sit for a conference re union demands and to reconsider its standing recognition of ALU as the sole and exclusive bargaining representatives of its employees. Federation SPFL sought direct recognition as bargaining agent and federation GALLU-PSSLU filed a petition for certification election, both with the Med-Arbiter. In the meantime, ALU and GAW Trading executed a CBA. Med-Arbiter ruled for the holding of a certification election, which was reversed by the BLR Director Trajano on the ground that the CBA has been effective and valid and the contract-bar rule is applicable in this case. On motion for reconsideration, BLR Director Ferrer-Calleja reversed the decision of the former director because the contract-bar rule relied upon by Trajano does not apply - the CBA was defective because it was not duly submitted in accordance with the rules requiring (1) posting in at least 2 conspicuous places in the establishment at least 5 days before its ratification and (2) ratification by the majority of the employees in the bargaining unit. Issue: Whether or not CBA was defective. Held: Yes. Mechanics of collective bargaining are set in motion only when the following jurisdictional preconditions are present: 1. Possesion of the status of majority representation by the employees' rep in accordance with an of the means of selection and/or designation as per LC 2. Proof of majority representation 110 | P a g e c.

3. Demand to bargain under Art. 251(a), LC Supreme Court found that the status as exclusive bargaining representative of ALU is dubious. The only express recognition of ALU as employees' bargaining rep was the CBA. Also, there was precipitate haste on the part of GAW Trading in recognizing said union even amidst other unions existing in the unit. There was also failure to post the CBA in at least 2 conspicuous places in the establishment at least 5 days before its ratification. The assembly of the members of ALU wherein the agreement was allegedly explained does not cure the defect, as even non-members must be apprised of the terms. Also, 181 out of 281 workers who "ratified" the CBA have repudiated the same. Caltex v. Brillantes, 279 SCRA 218 (1997) Deadlock in negotioations before the NCMB, leading to a strike. Problem referred to the SOLE. Union now questions before the Supreme Court the resolution of the SOLE as regards 5 issues of contention - wage increase, union security clause, retirement benefits or application of the new retirement plan, signing bonus, and grievance and arbitration machineries. Issue: Whether or not SOLE acted with grave abuse of discretion in resolving labor dispute. Held: No. Claim of union of grave abuse of discretion is hinged upon the fact that SOLE adopted largely proposals of Caltex. It should be understood that bargaining is not equivalent to an adversarial litigation where rights and obligations are deliniated and remedies applied. It is simply a process of finding a reasonable solution to a conflict and harmonizing opposite positions into a fair and reasonable compromise. When parties agree to submit unresolved issues to the SOLE for his resolution, they should not expect their positions to be adopted in toto. It is understood that they defer to his wisdom and objectivity in insuring industrial peace. And unless they clearly demonstrate bias, arbitrariness, capriciousness, or personal hostility, the Court cannot interfere or substitute offcier's judgment. CBA - question is rarely one of legal right or wrong, but one of wisdom, cogency and compromise as to what is possible, fair, and reasonable under the circumstances. American President Lines v. Clave, 114 SCRA 826 (1982) NATURE Petition for certiorari FACTS - On January 4, 1960, the petitioner entered into a contract with the Marine Security Agency for the latter to guard and protect the petitioner's vessels while they were moored at the port of Manila. It was stipulated in the contract that its term was for one year commencing from the date of its execution and it may be terminated by either party 30 days' notice to the other. The relationship between the petitioner and Marine Security Agency is such that it was the latter who hired and assigned the guards who kept watching over the petitioner's vessels. The guards were not known to petitioner who dealt only with the agency on matters pertaining to the service of the guards. A lump sum would be paid by petitioner to the agency who in turn determined and paid the compensation of the individual watchmen. - Upon prior notice given by the petitioner to the Marine Security Agency, the contract was terminated on January 4, 1961, after it had run its term. After the termination of its contract with Marine Security Agency, the petitioner executed a new contract with the Philippine Scout Veterans Security and Investigation Agency also for the purpose of having its vessels protected while they called at the port of Manila, and this contract was also for a fixed period of one year. - private respondents protested against the termination - On February 6, 1961, the respondent Union passed a resolution abolishing itself with the following reasons: 1. Termination of Contract of the Marine Security Agency with the American President Lines. 2. Inability of the Marine Security Agency to provide employment 3. Inability of the members and the Union to provide maintenance in the coming months. - On December 10, 1962, the respondent union passed another resolution reviving itself. - On March 21, 1963, the Maritime Security Union, through private respondents filed a complaint against the petitioner for unfair labor practice under RA 875. Their complaint, wherein they charged that the petitioner had refused to negotiate an agreement with them and had discriminated against them with regard to their tenure of employment by dismissing them for no other reason than their membership with the union and union activities, was lodged with the defunct Court of Industrial Relations. However, before that court could resolve the case, the Labor Code was enacted and the case was transferred to the NLRC under Arbiter Lomabao. - Arbiter Lomabao found the petitioner to be an employer of the private respondents and guilty of ULP against them. - The NLRC affirmed with the qualification that only those complainants who are 60 years old or younger and capacitated to discharge their former duties should be reinstated without loss of seniority rights and other privileges, and with three years of backwages; and those who could not be so reinstated should be given separation pay in addition to their backwages for three years. The Minister of Labor affirmed, and the Office of the President affirmed as well. ISSUES 1. WON there existed an employer-employee relationship between the petitioner and the individual watchmen of the Marine Security Agency who are alleged to be members of the respondent union. 2. WON the petitioner refused to negotiate a CBA with the said individual watchmen and discriminated against them in respect to their tenure of employment by terminating their contract on January 1, 1961, because of their union activities (an unfair labor practice). HELD 1. NO. 111 | P a g e

Ratio The following elements are generally considered to determine whether an employer-employee relationship exists: (1) the selection and engagement of the employee; (2) the payment of wages; (3) the power of dismissal; and (4) the power to control the employee's conduct-although the latter is the most important element. Reasoning In the present case, it is the agency that recruits, hires, and assigns the work of its watchmen. Hence, a watchman can not perform any security service for the petitioner's vessels unless the agency first accepts him as its watchman. With respect to his wages, the amount to be paid to a security guard is beyond the power of the petitioner to determine. Certainly, the lump sum amount paid by the petitioner to the agency in consideration of the latter's service is much more than the wages of any one watchman. In point of fact, it is the agency that quantifies and pays the wages to which a watchman is entitled. Neither does the petitioner have any power to dismiss the security guards. Since the petitioner has to deal with the agency, and not the individual watchmen, on matters pertaining to the contracted task, it stands to reason that the petitioner does not exercise any power over the watchmen's conduct. Always, the agency stands between the petitioner and the watchmen; and it is the agency that is answerable to the petitioner for the conduct of its guards. 2. NO. Ratio In view of Our finding that there is no employer-employee relationship between the petitioner and the members of the respondent agency, it should necessarily follow that the petitioner cannot be guilty of unfair labor practice as charged by the private respondents. Under RA 975 Sec.13, an unfair labor practice may be committed only within the context of an employer-employee relationship. - We find it difficult to believe that the members of the respondent agency made "repeated requests" upon the petitioner through its Captain Morris to negotiate a CBA with the respondent union. Apart from their oral declaration, the private respondents have not presented any written proof that such requests were made. Under RA 875, Sec.14(a), the desire to negotiate an agreement should be expressed through a written notice. At the time the members of the agency were allegedly presenting "repeated requests" for negotiation, they were represented by counsel. If such requests were in fact made, counsel would not have failed to advise his clients to tender their requests in the manner required by law. With regard to the termination of the contract between the petitioner and the respondent agency, We find no evidence, that it bears any relationship to the alleged union activities of the individual members of the agency. The hard fact is that the contract had a lifetime of one year. Hence, after that period, and without it being renewed, it lived out its term. While the expiration of the contract might have rendered the members of the respondent agency jobless, it can hardly be attributed to any adverse act by the petitioner. Disposition WHEREFORE the complaint for unfair labor practice against petitioner is hereby dismissed. SEPARATE OPINIONS AQUINO, Concurring: I concur in the result. The watchmen were employees of the American President Lines while guarding the ships. Since the watchmen were hired only for a period of one year, they ceased, after that period, to be employees of the APL. APL was not obligated to renew the contract of employment. Hence, the non-renewal of their employment and the act of the American President Lines in hiring the watchmen of another security agency cannot be regarded as an unfair labor practice. Moreover, the watchmen in filing their complaint for unfair labor practice and reinstatement only two years and two months after the expiration of their employments contract were guilty of laches. ABAD SANTOS, Dissenting: - The pivotal question in this case is one of fact, i.e. whether or not there existed an employer-employee relationship between the APL and the individual complainants. The Executive Department of the government starting from the Labor Arbiter, to the NLRC, the Minister of Labor and finally the Office of the President found as a fact that there was an employer-employee relationship. This finding of fact is supported by substantial evidence: "evidence on record undisputably shows that private respondents became employees of the APL when they were hired much earlier even before 1961 after they had been recruited by the Marine Security Agency for the said shipping company who then hired them to perform guarding duties over its vessels on dock in the Manila ports. This arrangement became the practice starting the early part of 1951 to evade the preferential hiring of union men and the maintenance of the rates of pay then obtaining. This arrangement gave birth to the Marine Security Agency which was contracted for the sole purpose of recruiting and supplying watchmen on ships and vessels of the American President Lines. It was also observed that the Marine Security Agency which had recruited herein private respondents for the said shipping company was not an 'independent contractor' but a 'mere agent which served as extension of the office' of the said shipping company 'in the recruitment of the watchmen, the computation of the watchmen's wages; and the placement of supervisors of the watchmen.' These reveal that a certain degree of control exercised by the shipping company over these watchmen. The services of these watchmen were availed of and their compensation paid in lump sum by the shipping company through the watchmen's agency, even if such were done through the said watchmen agency without the direct intervention of the said shipping company. While working as regular employees of APL, private respondents herein formed and organized on August 3, 1958 the Maritime Security Union. The foregoing factors or indicia demonstrate that employer-employee relationship existed between APL and herein private respondents, - On the question as to whether or not the APL is guilty of unfair labor practice, it suffices to quote from the decision of the Office of the President, thus: "This Office also found that there was indeed an unfair labor practice committed by the respondent-appellant. The evidence indubitably show that the repeated requests of members of the complainant union to negotiate in behalf of the union with Capt. Edward Morris were unheeded. As such, refusal to negotiate and eventually separating individual complaints are, to our mind, acts constituting unfair labor practice." National Union of Restaurant Workers v. CIR, 10 SCRA 843 (1964) 112 | P a g e

June 9, 1960 – ULP complaint filed against Tres Hermanas Restaurant, particularly Mrs. Felisa Herrera. Grounds: Refusal to bargain collectively with petitioner union Tres Hermanas made a counter-proposal saying that they will bargain with the union if: The union becomes a company union One Martin Briones, an employee, is separated from the service because he was found to be the organizer and adviser of the complaining union July 28, 1961 – Judge Tagbine (CIR) dismissed complaint as he found that the charges were not proven. CIR en banc affirmed Judge Tagbine’s decision. CIR Decision: Tres Hermanas did not refuse to bargain collectively with the union as in fact they met its members with the only particularity that they were not able to accept all the demands of the union. The decision relied on Exhibit 1: A letter sent by the Union to Tres Hermanas containing: 1. its demands, with certain marks opposite each demand; 2. a check for those demands to which Mrs. Felisa Herrera was agreeable 3. a cross signifying the disapproval of Mrs. Herrera 4. a circle regarding those demands which were left open for discussion on some future occasion that the parties may deem convenient. 5. Such markings were made during the discussion of the demands in the meeting called by respondents on May 3, 1960 at their restaurant in Quezon City Tres Hermanas did not interfere, coerce or restrain their employees in the exercise of their right to join the complaining union the dismissal of Martin Briones was due to the concern of Mrs. Herrera for her life on account of the hatred that Briones had entertained against her, she being always with him in the car he used to drive during their business routine ISSUE: WON Tres Hermanas refused to bargain, therefore guilty of ULP. HELD: Tres Hermanas did not refuse to bargain. RATIO: The inference that respondents did not refuse to bargain collectively with the complaining union because they accepted some of the demands while they refused the others even leaving open other demands for future discussion is correct, especially so when those demands were discussed at a meeting called by respondents themselves precisely in view of the letter sent by the union on April 29, 1960. It is true that under Section 14 of Republic Act 875 whenever a party serves a written notice upon the employer making some demands the latter shall reply thereto not later than 10 days from receipt thereof, but this rendition is merely procedural and as such its non-compliance cannot be deemed to be an act of unfair labor practice. The fact is that respondents did not ignore the letter sent by the union so much so that they called a meeting to discuss its demands, as already stated elsewhere. Collegio de San Juan de Letran v. Association of Employees, 340 SCRA 587 (2000) Facts: Salvador Abtria, then President of respondent union, Association of Employees and Faculty of Letran, initiated the renegotiation of its Collective Bargaining Agreement with petitioner Colegio de San Juan de Letran for the last two (2) years of the CBA's five (5) year lifetime from 1989-1994. In the same year, the union elected a new set of officers wherein private respondent Eleanor Ambas emerged as the newly elected President. On January 18, 1996, the parties agreed to disregard the unsigned CBA and to start negotiation on a new five-year CBA starting 1994-1999. On February 7, 1996, the union submitted its proposals to petitioner, which notified the union six days later, or on February 13, 1996, that the same had been submitted to its Board of Trustees. In the meantime, Ambas was informed through a letter dated February 15, 1996 from her superior that her work schedule was being changed from Monday to Friday to Tuesday to Saturday. Ambas protested and requested management to submit the issue to a grievance machinery under the old CBA. On March 29, 1996, the union received petitioner's letter dismissing Ambas for alleged insubordination. Hence, the union amended its notice of strike to include Ambas' dismissal. On April 20, 1996, both parties again discussed the ground rules for the CBA renegotiation. However, petitioner stopped the negotiations after it purportedly received information that a new group of employees had filed a petition for certification election. On June 18, 1996, the union finally struck. On July 2, 1996, public respondent Secretary of Labor and Employment assumed jurisdiction and ordered all striking employees, including the union president, to return to work, and for petitioner to accept them back under the same terms and conditions before the actual strike. Petitioner readmitted the striking members except Ambas. Public respondent issued an order declaring petitioner guilty of unfair labor practice on two counts and directing the reinstatement of private respondent Ambas with backwages. Petitioner filed a motion for reconsideration which was denied. Petitioner sought a review of the order of the Secretary of Labor and Employment before the Court of Appeals. The appellate court dismissed the petition and affirmed the findings of the Secretary of Labor and Employment. Hence, the present petition. Issue: (1) WON petitioner is guilty of unfair labor practice by refusing to bargain with the union when it unilaterally suspended the ongoing negotiations for a new Collective Bargaining Agreement (CBA) upon mere information that a petition for certification has been filed by another legitimate labor organization (2) WON the termination of the union president amounts to an interference of the employees' right to self-organization. Held: The Supreme Court denied the petition. The petitioner's actuation showed a lack of sincere desire to negotiate rendering it guilty of unfair labor practice. Petitioner's utter lack of interest in bargaining with the union is obvious in its failure to make a timely reply to the proposals presented by the latter. More than a month after the proposals were submitted by the union, petitioner still had not made any counter-proposals. Said inaction on the part of petitioner prompted the union to file 113 | P a g e

its second notice of strike on March 13, 1996. Petitioner could only offer a feeble explanation that the Board of Trustees had not yet convened to discuss the matter as its excuse for failing to file its reply. Petitioner clearly violated Article 250 of the Labor Code governing the procedure in collective bargaining. Concerning the issue on the validity of the termination of the union president, the Court ruled that the dismissal was effected in violation of the employees' right to self-organization. The factual backdrop of the termination of Ms. Ambas led to no other conclusion that she was dismissed in order to strip the union of a leader who would fight for the right of her co-workers at the bargaining table. The record showed that it was when Ms. Ambas was the union president and during the period of tense and difficult negotiations when her work schedule was altered from Mondays to Fridays to Tuesdays to Saturdays. When she did not budge, although her schedule was changed, she was outrightly dismissed for alleged insubordination. EIAHcC The mere filing of a petition for certification election does not ipso facto justify the suspension of negotiation by the employer. The petition must first comply with the provisions of the Labor Code and its Implementing Rules. No petition for certification election for any representation issue may be filed after the lapse of the sixty-day freedom period. The old CBA is extended until a new one is signed. The rule is that despite the lapse of the formal effectivity of the CBA the law still considers the same as continuing in force and effect until a new CBA shall have been validly executed. Hence, the contract bar rule still applies. The purpose is to ensure stability in the relationship of the workers and the company by preventing frequent modifications of any CBA earlier entered into by them in good faith and for the stipulated original period. Kiok Loy v. NLRC, 141 SCRA 179 (1988) supra D. Conciliation Procedure – 212 (c); 233 Art. 212. Definitions. c. "Board" means the National Conciliation and Mediation Board established under Executive Order No. 126. Art. 233. Privileged communication. Information and statements made at conciliation proceedings shall be treated as privileged communication and shall not be used as evidence in the Commission. Conciliators and similar officials shall not testify in any court or body regarding any matters taken up at conciliation proceedings conducted by them. Nissan Motors v. Secretary of DOLE, 491 SCRA 605 (2006) [G.R. Nos. 158190-91. October 31, 2006.] NISSAN MOTORS PHILIPPINES, INC., petitioner, vs. SECRETARY OF LABOR AND EMPLOYMENT and BAGONG NAGKAKAISANG LAKAS SA NISSAN MOTORS PHILIPPINES, INC. (BANAL-NMPI-OLALIA-KMU), respondents. [G.R. Nos. 158276 and 158283. October 31, 2006.] BAGONG NAGKAKAISANG LAKAS SA NISSAN MOTORS PHILIPPINES, INC. (BANAL-NMPI-OLALIA-KMU), petitioner, vs. COURT OF APPEALS (SPECIAL DIVISION OF FIVE), SECRETARY OF LABOR and EMPLOYMENT and NISSAN MOTORS PHILIPPINES, INC., respondents. Facts: There was collective bargaining deadlock between the petitioner corporation and respondent union. Thereafter 4 notices of strike were filed by the respondent. (1) on account of the alleged suspension of about 140 employees following a disruption of company operations; and (2) the dismissal from the service of a number of company employees. DOLE enjoined any strike or lockout and gave a cease and desist order. Later, DOLE issued an order affirming the dismissal of the employees and union officers. CA Affirmed. Issue: WON the strike was illegal. Yes. The Union engaged in work slowdown which under the circumstances in which constitute illegal strike. The company is therefore right in dismissing the subject Union officers in accordance with Article 264 (a) of the Labor Code, for participating in illegal strike in defiance of the assumption of jurisdiction order by the Labor Secretary. However, dismissal is a harsh penalty there being no evidence that they engaged or participated in the commission of illegal activities during the said strike. One month suspension is a reasonable disciplinary measure which should be deemed served during the time they out of their jobs. C. DUTY TO BARGAIN – 250-253; 242 (c) Art. 250. Procedure in collective bargaining. The following procedures shall be observed in collective bargaining: a. When a party desires to negotiate an agreement, it shall serve a written notice upon the other party with a statement of its proposals. The other party shall make a reply thereto not later than ten (10) calendar days from receipt of such notice; b. Should differences arise on the basis of such notice and reply, either party may request for a conference which shall begin not later than ten (10) calendar days from the date of request. c. If the dispute is not settled, the Board shall intervene upon request of either or both parties or at its own initiative and immediately call the parties to conciliation meetings. The Board shall have the power to issue subpoenas requiring the attendance of the parties to such meetings. It shall be the duty of the parties to participate fully and promptly in the conciliation meetings the Board may call; d. During the conciliation proceedings in the Board, the parties are prohibited from doing any act which may disrupt or impede the early settlement of the disputes; and 114 | P a g e

The Board shall exert all efforts to settle disputes amicably and encourage the parties to submit their case to a voluntary arbitrator. (As amended by Section 20, Republic Act No. 6715, March 21, 1989) Art. 251. Duty to bargain collectively in the absence of collective bargaining agreements. In the absence of an agreement or other voluntary arrangement providing for a more expeditious manner of collective bargaining, it shall be the duty of employer and the representatives of the employees to bargain collectively in accordance with the provisions of this Code. Art. 252. Meaning of duty to bargain collectively. The duty to bargain collectively means the performance of a mutual obligation to meet and convene promptly and expeditiously in good faith for the purpose of negotiating an agreement with respect to wages, hours of work and all other terms and conditions of employment including proposals for adjusting any grievances or questions arising under such agreement and executing a contract incorporating such agreements if requested by either party but such duty does not compel any party to agree to a proposal or to make any concession. Art. 253. Duty to bargain collectively when there exists a collective bargaining agreement. When there is a collective bargaining agreement, the duty to bargain collectively shall also mean that neither party shall terminate nor modify such agreement during its lifetime. However, either party can serve a written notice to terminate or modify the agreement at least sixty (60) days prior to its expiration date. It shall be the duty of both parties to keep the status quo and to continue in full force and effect the terms and conditions of the existing agreement during the 60-day period and/or until a new agreement is reached by the parties. 1. Meaning of Duty Union of Filipro Employees, etc. v. Nestle Philippines, Inc., 547 SCRA 323 (2008) FACTS - Due to the impending expiration of the existing collective bargaining agreement (CBA) between Nestlé and UFE-DFA-KMU, the Presidents of the Alabang and Cabuyao Divisions of UFE-DFA-KMU, informed Nestlé of their intent to “open our new Collective Bargaining Negotiation for the year 2001-2004 x x x as early as June 2001.” - Nestlé acknowledged receipt of the aforementioned letter. It also informed UFE-DFA-KMU that it was preparing its own counter-proposal and proposed ground rules that shall govern the conduct of the collective bargaining negotiations. - Nestlé underscored its position that “unilateral grants, one-time company grants, company-initiated policies and programs, which include, but are not limited to the Retirement Plan, Incidental Straight Duty Pay and Calling Pay Premium, are by their very nature not proper subjects of CBA negotiations and therefore shall be excluded therefrom.” - In addition, it clarified that with the closure of the Alabang Plant, the CBA negotiations will only be applicable to the covered employees of the Cabuyao Plant; hence, the Cabuyao Division of UFE-DFA-KMU became the sole bargaining unit involved in the subject CBA negotiations. - Thereafter, dialogue between the company and the union ensued. - Nestlé, claiming to have reached an impasse in said dialogue, requested the National Conciliation and Mediation Board to conduct preventive mediation proceedings between it and UFE-DFA-KMU. - Conciliation proceedings nevertheless proved ineffective. Complaining, in essence, of bargaining deadlock – pertaining to economic issues, i.e., “retirement (plan), panel composition, costs and attendance, and CBA,” UFE-DFA-KMU filed a Notice of Strike . - One week later, another Notice of Strike was filed by the UFE-DFA-KMU, this time predicated on Nestlé’s alleged unfair labor practices i.e., bargaining in bad faith in that it was setting pre-conditions in the ground rules by refusing to include the issue of the Retirement Plan in the CBA negotiations. - In view of the looming strike, Nestlé filed with the DOLE a Petition for Assumption of Jurisdiction - Sto. Tomas issued an Order assuming jurisdiction over the subject labor dispute between the parties stating that any strike or lockout is enjoined, that the parties are further directed to meet and convene for the discussion of the union proposals and company counter-proposals before the National Conciliation and Mediation Board and that if no settlement of all the issues is reached, the Office shall define the outstanding issues and order the filing of position papers for a ruling on the merits. - UFE-DFA-KMU sought reconsideration of the Assumption of Jurisdiction Order - In an Order, Sec. Sto. Tomas denied the aforequoted motion for reconsideration - The employee members of UFE-DFA-KMU at the Nestlé Cabuyao Plant went on strike. - Notwithstanding a Return-To-Work Order, the members of UFE-DFA-KMU continued with their strike and refused to go back to work as instructed. Thus, Sec. Sto. Tomas sought the assistance of the Philippine National Police (PNP) for the enforcement of said order. - At the hearing called, Nestlé and UFE-DFA-KMU filed their respective position papers. - Tomas denied the motion for reconsideration of UFE-DFA-KMU. - Frustrated with the foregoing turn of events, UFE-DFA-KMU filed a petition for certiorari with application for the issuance of a temporary restraining order or a writ of preliminary injunction before the Court of Appeals. - Meanwhile, in an attempt to finally resolve the crippling labor dispute between the parties, then Acting Secretary of the DOLE, Hon. Arturo D. Brion, came out with an Order - UFE-DFA-KMU moved to reconsider the aforequoted position of the DOLE. - Secretary of the DOLE, Hon. Sto. Tomas, issued the last of the assailed Orders. This order resolved to deny the preceding motion for reconsideration of UFE-DFA-KMU. - Undaunted still, UFE-DFA-KMU, for the second time, went to the Court of Appeals 115 | P a g e

e.

- The Court of Appeals, acting on the twin petitions for certiorari, determined the issues in favor of UFE-DFA-KMU - Dissatisfied, both parties separately moved for the reconsideration of the abovequoted decision - The Court of Appeals stood pat in its earlier pronouncements and denied the motions for reconsideration ISSUES 1. WON the Retirement Plan was a proper subject to be included in the CBA negotiations between the parties hence, negotiable. 2. WON the assumption powers of the Secretary of Labor and Employment should have been limited merely to the grounds alleged in the second Notice of Strike 3. WON Nestlé was guilty of unfair labor practice HELD 1. YES - In Nestlé Philippines, Inc. v. NLRC, ironically involving the same parties herein, the Court has had the occasion to affirm that a retirement plan is consensual in nature. The Court, through Madame Justice Griño-Aquino, declared that: The fact that the retirement plan is non-contributory, i.e., that the employees contribute nothing to the operation of the plan, does not make it a non-issue in the CBA negotiations. As a matter of fact, almost all of the benefits that the petitioner has granted to its employees under the CBA – salary increases, rice allowances, midyear bonuses, 13th and 14th month pay, seniority pay, medical and hospitalization plans, health and dental services, vacation, sick & other leaves with pay – are noncontributory benefits. Since the retirement plan has been an integral part of the CBA since 1972, the Union’s demand to increase the benefits due the employees under said plan, is a valid CBA issue. x x x [E]mployees do have a vested and demandable right over existing benefits voluntarily granted to them by their employer. The latter may not unilaterally withdraw, eliminate or diminish such benefits - In the case at bar, it cannot be denied that the CBA that was about to expire at that time contained provisions respecting the Retirement Plan. As the latter benefit was already subject of the existing CBA, the members of UFE-DFA-KMU were only exercising their prerogative to bargain or renegotiate for the improvement of the terms of the Retirement Plan just like they would for all the other economic, as well as non-economic benefits previously enjoyed by them. - The purpose of collective bargaining is the acquisition or attainment of the best possible covenants or terms relating to economic and non-economic benefits granted by employers and due the employees. The Labor Code has actually imposed as a mutual obligation of both parties, this duty to bargain collectively. - The duty to bargain collectively is categorically prescribed by Article 252 of the said code. It states: ART. 252. MEANING OF DUTY TO BARGAIN COLLECTIVELY. – The duty to bargain collectively means the performance of a mutual obligation to meet and confer promptly and expeditiously and in good faith for the purpose of negotiating an agreement with respect to wages, hours of work, and all other terms and conditions of employment including proposals for adjusting any grievances or questions arising under such agreement and executing a contract incorporating such agreement if requested by either party, but such duty does not compel any party to agree to a proposal or to make any concession. - Further, Article 253, also of the Labor Code, defines the parameter of said obligation when there already exists a CBA, viz: ART. 253. DUTY TO BARGAIN COLLECTIVELY WHEN THERE EXISTS A COLLECTIVE BARGAINING AGREEMENT. – The duty to bargain collectively shall also mean that either party shall not terminate nor modify such agreement during its lifetime. However, either party can serve a written notice to terminate or modify the agreement at least sixty (60) days prior to its expiration date. It shall be the duty of both parties to keep the status quo and to continue in full force and effect the terms and conditions of the existing agreement during the sixty day period and/or until a new agreement is reached by the parties. - In demanding that the terms of the Retirement Plan be opened for renegotiation, the members of UFE-DFA-KMU are acting well within their rights as we have, indeed, declared that the Retirement Plan is consensual in character; and so, negotiable. 2. NO. - Declaring the Secretary of the DOLE to have acted with grave abuse of discretion for ruling on substantial matters or issues and not restricting itself merely on the ground rules, the appellate court and UFE-DFA-KMU would have the Court treat the subject labor dispute in a piecemeal fashion. - The power granted to the Secretary of the DOLE by Paragraph (g) of Article 263 of the Labor Code, authorizes her to assume jurisdiction over a labor dispute, causing or likely to cause a strike or lockout in an industry indispensable to the national interest, and correlatively, to decide the same. - In the case at bar, the Secretary of the DOLE simply relied on the Notices of Strike that were filed by UFE-DFA-KMU. Thus, based on the Notices of Strike filed by UFEDFA-KMU, the Secretary of the DOLE rightly decided on matters of substance. - The issue of whether or not the Secretary of the DOLE could decide issues incidental to the subject labor dispute had already been answered in the affirmative. The Secretary’s assumption of jurisdiction power necessarily includes matters incidental to the labor dispute, that is, issues that are necessarily involved in the dispute itself, not just to those ascribed in the Notice of Strike; or, otherwise submitted to him for resolution. - In any event, the query as to whether or not the Retirement Plan is to be included in the CBA negotiations between the parties ineluctably dictates upon the Secretary of the DOLE to go into the substantive matter of the CBA negotiations. 3. NO. - Basic is the principle that good faith is presumed and he who alleges bad faith has the duty to prove the same. By imputing bad faith unto the actuations of Nestlé, it was UFE-DFA-KMU, therefore, who had the burden of proof to present substantial evidence to support the allegation of unfair labor practice. - A perusal of the allegations and arguments raised by UFE-DFA-KMU in the Memorandum will readily disclose that it failed to discharge said onus probandi as there is still a need for the presentation of evidence other than its bare contention of unfair labor practice in order to make certain the propriety or impropriety of the unfair labor practice charge hurled against Nestlé. 116 | P a g e

- There is no per se test of good faith in bargaining. Good faith or bad faith is an inference to be drawn from the facts, to be precise, the crucial question of whether or not a party has met his statutory duty to bargain in good faith typically turns on the facts of the individual case. Necessarily, a determination of the validity of the Nestlé’s proposition involves an appraisal of the exercise of its management prerogative. - Employers are accorded rights and privileges to assure their self-determination and independence and reasonable return of capital. This mass of privileges comprises the so-called management prerogatives. In this connection, the rule is that good faith is always presumed. As long as the company’s exercise of the same is in good faith to advance its interest and not for purpose of defeating or circumventing the rights of employees under the law or a valid agreement, such exercise will be upheld. Disposition Petition seeking that Nestlé be declared to have committed unfair labor practice was DENIED. The Petition was PARTLY GRANTED, the ruling of the Court of Appeals was REVERSED in so far as it ruled that the Secretary of the DOLE gravely abused her discretion in failing to confine her assumption of jurisdiction power over the ground rules of the CBA negotiations; but the ruling of the Court of Appeals on the inclusion of the Retirement Plan as a valid issue in the collective bargaining negotiations between UFE-DFA-KMU and Nestlé is AFFIRMED. Kiok Loy v. NLRC, 141 SCRA 179 (1986) supra Standard Chartered Bank Employees Union v. Confessor, 432 SCRA 304 (2004) FACTS - Standard Chartered Bank is a foreign banking corporation doing business in the Philippines. The exclusive bargaining agent of the rank and file employees of the Bank is the Standard Chartered Bank Employees Union - The Union sought to renegotiate the terms of the CBA and initiated the negotiations. - Through its President, Eddie L. Divinagracia, it sent a letter containing its proposals covering political and economic provisions. - The Bank, took note of the Union’s proposals. The Bank attached its counter-proposal to the non-economic provisions proposed by the Union. - Before the commencement of the negotiation, the Union, through Divinagracia, suggested to the Bank’s Human Resource Manager and head of the negotiating panel, Cielito Diokno, that the bank lawyers should be excluded from the negotiating team. The Bank acceded. - Meanwhile, Diokno suggested to Divinagracia that Jose P. Umali, Jr., the President of the National Union of Bank Employees (NUBE), the federation to which the Union was affiliated, be excluded from the Union’s negotiating panel. However, Umali was retained as a member thereof. - The parties met and set the ground rules for the negotiation. Diokno suggested that the negotiation be kept a “family affair.” - Even during the final reading of the, there were still non-economic provisions on which the Union and the Bank could not agree. Both parties agreed to place the notation “DEFERRED/DEADLOCKED.” - The negotiation for economic provisions commenced. Except for the provisions on signing bonus and uniforms, the Union and the Bank failed to agree on the remaining economic provisions of the CBA. The Union declared a deadlock and filed a Notice of Strike before the National Conciliation and Mediation Board - The Bank filed a complaint for Unfair Labor Practice (ULP) and Damages before the NLRC in Manila alleging that the Union violated its duty to bargain, as it did not bargain in good faith. It contended that the Union demanded “sky high economic demands,” indicative of blue-sky bargaining. - Then Secretary of Labor and Employment (SOLE) Nieves R. Confesor, assumed jurisdiction over the labor dispute and issued an Order dismissing the Bank and the Union’s charges for unfair labor practice - The Union filed a motion for reconsideration with clarification, while the Bank filed a motion for reconsideration. The SOLE issued a Resolution denying the motions. The Union filed a second motion for reconsideration, which was, likewise, denied - The Union filed this petition - The Union alleges that the Bank violated its duty to bargain; hence, committed ULP under Article 248(g) when it engaged in surface bargaining. It alleged that the Bank just went through the motions of bargaining without any intent of reaching an agreement, as evident in the Bank’s counter-proposals. ISSUE WON the SOLE committed grave abuse of discretion amounting to lack of jurisdiction in dismissing the union’s charge of unfair labor practice. HELD NO. - Surface bargaining: “going through the motions of negotiating” without any legal intent to reach an agreement. - The resolution of surface bargaining allegations never presents an easy issue. The determination of whether a party has engaged in unlawful surface bargaining is usually a difficult one because it involves, at bottom, a question of the intent of the party in question, and usually such intent can only be inferred from the totality of the challenged party’s conduct both at and away from the bargaining table. It involves the question of whether an employer’s conduct demonstrates an unwillingness to bargain in good faith or is merely hard bargaining. - The minutes of meetings do not show that the Bank had any intention of violating its duty to bargain with the Union. Records show that after the Union sent its proposal to the Bank, the latter replied with a list of its counter-proposals. Thereafter, meetings were set for the settlement of their differences. The minutes of the meetings show that both the Bank and the Union exchanged economic and non-economic proposals and counter-proposals. - The Union has not been able to show that the Bank had done acts, both at and away from the bargaining table, which tend to show that it did not want to reach an agreement with the Union or to settle the differences between it and the Union. Admittedly, the parties were not able to agree and reached a deadlock. However, it is herein emphasized that the duty to bargain “does not compel either party to agree to a proposal or require the making of a concession.” Hence, the parties’ failure to agree did not amount to ULP under Article 248(g) for violation of the duty to bargain. 117 | P a g e

- The inference that respondents did not refuse to bargain collectively with the complaining union because they accepted some of the demands while they refused the others even leaving open other demands for future discussion is correct, especially so when those demands were discussed at a meeting called by respondents themselves precisely in view of the letter sent by the union - The Court also does not agree that the Union is guilty of ULP for engaging in blue-sky bargaining or making exaggerated or unreasonable proposals. - The Bank failed to show that the economic demands made by the Union were exaggerated or unreasonable. The minutes of the meeting show that the Union based its economic proposals on data of rank and file employees and the prevailing economic benefits received by bank employees from other foreign banks doing business in the Philippines and other branches of the Bank in the Asian region. Disposition Resolutions of the SOLE are AFFIRMED. Union of Filipro, etc. v. Nestle Philippines, 499 SCRA 521 (2006) FACTS - Due to the impending expiration of the existing collective bargaining agreement (CBA) between Nestlé and UFE-DFA-KMU, the Presidents of the Alabang and Cabuyao Divisions of UFE-DFA-KMU, informed Nestlé of their intent to “open our new Collective Bargaining Negotiation for the year 2001-2004 x x x as early as June 2001.” - Nestlé acknowledged receipt of the aforementioned letter. It also informed UFE-DFA-KMU that it was preparing its own counter-proposal and proposed ground rules that shall govern the conduct of the collective bargaining negotiations. - Nestlé underscored its position that “unilateral grants, one-time company grants, company-initiated policies and programs, which include, but are not limited to the Retirement Plan, Incidental Straight Duty Pay and Calling Pay Premium, are by their very nature not proper subjects of CBA negotiations and therefore shall be excluded therefrom.” - In addition, it clarified that with the closure of the Alabang Plant, the CBA negotiations will only be applicable to the covered employees of the Cabuyao Plant; hence, the Cabuyao Division of UFE-DFA-KMU became the sole bargaining unit involved in the subject CBA negotiations. - Thereafter, dialogue between the company and the union ensued. - Nestlé, claiming to have reached an impasse in said dialogue, requested the National Conciliation and Mediation Board to conduct preventive mediation proceedings between it and UFE-DFA-KMU. - Conciliation proceedings nevertheless proved ineffective. Complaining, in essence, of bargaining deadlock – pertaining to economic issues, i.e., “retirement (plan), panel composition, costs and attendance, and CBA,” UFE-DFA-KMU filed a Notice of Strike . - One week later, another Notice of Strike was filed by the UFE-DFA-KMU, this time predicated on Nestlé’s alleged unfair labor practices i.e., bargaining in bad faith in that it was setting pre-conditions in the ground rules by refusing to include the issue of the Retirement Plan in the CBA negotiations. - In view of the looming strike, Nestlé filed with the DOLE a Petition for Assumption of Jurisdiction - Sto. Tomas issued an Order assuming jurisdiction over the subject labor dispute between the parties stating that any strike or lockout is enjoined, that the parties are further directed to meet and convene for the discussion of the union proposals and company counter-proposals before the National Conciliation and Mediation Board and that if no settlement of all the issues is reached, the Office shall define the outstanding issues and order the filing of position papers for a ruling on the merits. - UFE-DFA-KMU sought reconsideration of the Assumption of Jurisdiction Order - In an Order, Sec. Sto. Tomas denied the aforequoted motion for reconsideration - The employee members of UFE-DFA-KMU at the Nestlé Cabuyao Plant went on strike. - Notwithstanding a Return-To-Work Order, the members of UFE-DFA-KMU continued with their strike and refused to go back to work as instructed. Thus, Sec. Sto. Tomas sought the assistance of the Philippine National Police (PNP) for the enforcement of said order. - At the hearing called, Nestlé and UFE-DFA-KMU filed their respective position papers. - Tomas denied the motion for reconsideration of UFE-DFA-KMU. - Frustrated with the foregoing turn of events, UFE-DFA-KMU filed a petition for certiorari with application for the issuance of a temporary restraining order or a writ of preliminary injunction before the Court of Appeals. - Meanwhile, in an attempt to finally resolve the crippling labor dispute between the parties, then Acting Secretary of the DOLE, Hon. Arturo D. Brion, came out with an Order - UFE-DFA-KMU moved to reconsider the aforequoted position of the DOLE. - Secretary of the DOLE, Hon. Sto. Tomas, issued the last of the assailed Orders. This order resolved to deny the preceding motion for reconsideration of UFE-DFA-KMU. - Undaunted still, UFE-DFA-KMU, for the second time, went to the Court of Appeals - The Court of Appeals, acting on the twin petitions for certiorari, determined the issues in favor of UFE-DFA-KMU - Dissatisfied, both parties separately moved for the reconsideration of the abovequoted decision - The Court of Appeals stood pat in its earlier pronouncements and denied the motions for reconsideration ISSUES 1. WON the Retirement Plan was a proper subject to be included in the CBA negotiations between the parties hence, negotiable. 2. WON the assumption powers of the Secretary of Labor and Employment should have been limited merely to the grounds alleged in the second Notice of Strike 118 | P a g e

3. WON Nestlé was guilty of unfair labor practice HELD 1. YES - In Nestlé Philippines, Inc. v. NLRC, ironically involving the same parties herein, the Court has had the occasion to affirm that a retirement plan is consensual in nature. The Court, through Madame Justice Griño-Aquino, declared that: The fact that the retirement plan is non-contributory, i.e., that the employees contribute nothing to the operation of the plan, does not make it a non-issue in the CBA negotiations. As a matter of fact, almost all of the benefits that the petitioner has granted to its employees under the CBA – salary increases, rice allowances, midyear bonuses, 13th and 14th month pay, seniority pay, medical and hospitalization plans, health and dental services, vacation, sick & other leaves with pay – are noncontributory benefits. Since the retirement plan has been an integral part of the CBA since 1972, the Union’s demand to increase the benefits due the employees under said plan, is a valid CBA issue. x x x [E]mployees do have a vested and demandable right over existing benefits voluntarily granted to them by their employer. The latter may not unilaterally withdraw, eliminate or diminish such benefits - In the case at bar, it cannot be denied that the CBA that was about to expire at that time contained provisions respecting the Retirement Plan. As the latter benefit was already subject of the existing CBA, the members of UFE-DFA-KMU were only exercising their prerogative to bargain or renegotiate for the improvement of the terms of the Retirement Plan just like they would for all the other economic, as well as non-economic benefits previously enjoyed by them. - The purpose of collective bargaining is the acquisition or attainment of the best possible covenants or terms relating to economic and non-economic benefits granted by employers and due the employees. The Labor Code has actually imposed as a mutual obligation of both parties, this duty to bargain collectively. - The duty to bargain collectively is categorically prescribed by Article 252 of the said code. It states: ART. 252. MEANING OF DUTY TO BARGAIN COLLECTIVELY. – The duty to bargain collectively means the performance of a mutual obligation to meet and confer promptly and expeditiously and in good faith for the purpose of negotiating an agreement with respect to wages, hours of work, and all other terms and conditions of employment including proposals for adjusting any grievances or questions arising under such agreement and executing a contract incorporating such agreement if requested by either party, but such duty does not compel any party to agree to a proposal or to make any concession. - Further, Article 253, also of the Labor Code, defines the parameter of said obligation when there already exists a CBA, viz: ART. 253. DUTY TO BARGAIN COLLECTIVELY WHEN THERE EXISTS A COLLECTIVE BARGAINING AGREEMENT. – The duty to bargain collectively shall also mean that either party shall not terminate nor modify such agreement during its lifetime. However, either party can serve a written notice to terminate or modify the agreement at least sixty (60) days prior to its expiration date. It shall be the duty of both parties to keep the status quo and to continue in full force and effect the terms and conditions of the existing agreement during the sixty day period and/or until a new agreement is reached by the parties. - In demanding that the terms of the Retirement Plan be opened for renegotiation, the members of UFE-DFA-KMU are acting well within their rights as we have, indeed, declared that the Retirement Plan is consensual in character; and so, negotiable. 2. NO. - Declaring the Secretary of the DOLE to have acted with grave abuse of discretion for ruling on substantial matters or issues and not restricting itself merely on the ground rules, the appellate court and UFE-DFA-KMU would have the Court treat the subject labor dispute in a piecemeal fashion. - The power granted to the Secretary of the DOLE by Paragraph (g) of Article 263 of the Labor Code, authorizes her to assume jurisdiction over a labor dispute, causing or likely to cause a strike or lockout in an industry indispensable to the national interest, and correlatively, to decide the same. - In the case at bar, the Secretary of the DOLE simply relied on the Notices of Strike that were filed by UFE-DFA-KMU. Thus, based on the Notices of Strike filed by UFEDFA-KMU, the Secretary of the DOLE rightly decided on matters of substance. - The issue of whether or not the Secretary of the DOLE could decide issues incidental to the subject labor dispute had already been answered in the affirmative. The Secretary’s assumption of jurisdiction power necessarily includes matters incidental to the labor dispute, that is, issues that are necessarily involved in the dispute itself, not just to those ascribed in the Notice of Strike; or, otherwise submitted to him for resolution. - In any event, the query as to whether or not the Retirement Plan is to be included in the CBA negotiations between the parties ineluctably dictates upon the Secretary of the DOLE to go into the substantive matter of the CBA negotiations. 3. NO. - Basic is the principle that good faith is presumed and he who alleges bad faith has the duty to prove the same. By imputing bad faith unto the actuations of Nestlé, it was UFE-DFA-KMU, therefore, who had the burden of proof to present substantial evidence to support the allegation of unfair labor practice. - A perusal of the allegations and arguments raised by UFE-DFA-KMU in the Memorandum will readily disclose that it failed to discharge said onus probandi as there is still a need for the presentation of evidence other than its bare contention of unfair labor practice in order to make certain the propriety or impropriety of the unfair labor practice charge hurled against Nestlé. - There is no per se test of good faith in bargaining. Good faith or bad faith is an inference to be drawn from the facts, to be precise, the crucial question of whether or not a party has met his statutory duty to bargain in good faith typically turns on the facts of the individual case. Necessarily, a determination of the validity of the Nestlé’s proposition involves an appraisal of the exercise of its management prerogative.

119 | P a g e

- Employers are accorded rights and privileges to assure their self-determination and independence and reasonable return of capital. This mass of privileges comprises the so-called management prerogatives. In this connection, the rule is that good faith is always presumed. As long as the company’s exercise of the same is in good faith to advance its interest and not for purpose of defeating or circumventing the rights of employees under the law or a valid agreement, such exercise will be upheld. Disposition Petition seeking that Nestlé be declared to have committed unfair labor practice was DENIED. The Petition was PARTLY GRANTED, the ruling of the Court of Appeals was REVERSED in so far as it ruled that the Secretary of the DOLE gravely abused her discretion in failing to confine her assumption of jurisdiction power over the ground rules of the CBA negotiations; but the ruling of the Court of Appeals on the inclusion of the Retirement Plan as a valid issue in the collective bargaining negotiations between UFE-DFA-KMU and Nestlé is AFFIRMED. General Milling Corporation v. CA, 422 SCRA 514 (2004) Facts: Corporation and union entered in CBA. Corp received collective and individual letters from workers stating they had withdrawn from their union membership, on grounds of religious affiliation and personal differences. Relying on this, GMC did not send any counter-proposal in the CBA. Union disclaimed disaffiliation. Union filed complaint alleging unfair labor practice against GMC for: (1) refusal to bargain collectively; (2) interference with the right to self-organization; and (3) discrimination. Issue: WON the corporation refused to bargain collectively Held: Yes Ratio: The union lived up to this obligation when it presented proposals for a new CBA to within three (3) years from the effectivity of the original CBA. But GMC failed in its duty under Article 252 to make a counter-proposal. Union devised a flimsy excuse by questioning the existence of the union and the status of its membership to prevent any negotiation. Corporation’s failure to make a timely reply to the proposals presented by the union is indicative of its utter lack of interest in bargaining with the union. Its excuse that it felt the union no longer represented the workers, was mainly dilatory as it turned out to be utterly baseless. This is an indication of bad faith. Where the employer did not even bother to submit an answer to the bargaining proposals of the union, there is a clear evasion of the duty to bargain collectively. Since it was GMC which violated the duty to bargain collectively (based on Kiok Loy and Divine Word University of Tacloban) it had lost its statutory right to negotiate or renegotiate the terms and conditions of the draft CBA proposed by the union. Collegio de San Juan de Letran v. Association of Employees, 340 SCRA 587 (2000) supra Republic Savings Bank v. CIR, 21 SCRA 226 (1967) I: whether the dismissal of the eight (8) respondent employees by the petitioner Republic Bank (hereinafter referred to as the Bank) constituted an unfair labor practice within the meaning and intendment of the Industrial Peace Act (Republic Act 875). 1. respondents Rosendo T. Resuello, Benjamin Jara, Florencio Allasas, Domingo B. Jola, Diosdado S. Mendiola, Teodoro de la Cruz, Narciso Macaraeg and Mauro A. Rovillos were employees of Republic Savings who were dismissed “for having written and published "a patently libelous letter . . . tending to cause the dishonor, discredit or contempt not only of officers and employees of this bank, but also of your employer, the bank itself” 2. the letter accused the bank president of: nepotism, favoritism, illicit relations with bank employees who later promoted, and of choosing them for training abroad so they could be with him (although only one such employee was mentioned in the letter). Copies of this letter were admittedly given to the chairman of the board of directors of the Bank, and the Governor of the Central Bank. 3. The employees filed a complaint for Illegal dismissal/ULP in the CIR: alleging that the Bank's conduct violated section 4(a) (5) of the Industrial Peace Act which makes it an unfair labor practice for an employer "to dismiss, discharge or otherwise prejudice or discriminate against an employee for having filed charges or for having given or being about to give testimony under this Act." 4. In 1960, however, this Court overruled the decision of the CIR in the Royal Interocean case and held that "the charge, the filing of which is the cause of the dismissal of the employee, must be related to his right to self-organization in order to give rise to unfair labor practice on the part of the employer," because "under subsection 5 of section 4(a), the employee's (1) having filed charges or (2) having given testimony or (3) being about to give testimony, are modified by 'under this Act' appearing after the last item." 2 The Bank therefore renewed its motion to dismiss, but the court held the motion in abeyance and proceeded with the hearing 5. CIR: there is ULP; Bank appealed relying on interocean SC: Assuming that the latter acted in their individual capacities when they wrote the letter-charge they were nonetheless protected for they were engaged in concerted activity, in the exercise of their right of self-organization that includes concerted activity for mutual aid and protection, 5 interference with which constitutes an unfair labor practice under section 4(a)(1). This is the view of some members of this Court. For, as has been aptly stated, the joining in protests or demands, even by a small group of employees, if in furtherance of their interests as such, is a concerted activity protected by the Industrial Peace Act. It is not necessary that union activity be involved or that collective bargaining be contemplated. 6

120 | P a g e

Indeed, when the respondents complained against nepotism, favoritism and other management practices, they were acting within an area marked out by the Act as a proper sphere of collective bargaining. Even the reference to immorality was not irrelevant as it was made to support the respondents' other charge that the bank president had failed to provide wholesome working conditions, let alone a good moral example, for the employees by practicing discrimination and favoritism in the appointment and promotion of certain employees on the basis of illicit relations or blood relationship with them. Some other members of this Court believe, without necessarily expressing approval of the way the respondents expressed their grievances, that what the Bank should have done was to refer the letter-charge to the grievance committee. This was its duty, failing which it committed an unfair labor practice under section 4(a) (6). For collective bargaining does not end with the execution of an agreement. It is a continuous process. The duty to bargain imposes on the parties during the term of their agreement the mutual obligation "to meet and confer promptly and expeditiously and in good faith . . . for the purpose of adjusting any grievances or question arising under such agreement"8 and a violation of this obligation is, by section 4 (a) (6) and (b) (3) an unfair labor practice Instead of stifling criticism, the Bank should have allowed the respondents to air their grievances. Good faith bargaining required of the Bank an open mind and a sincere desire to negotiate over grievances.11 The grievance committee, created in the collective bargaining agreements, would have been an appropriate forum for such negotiation. Indeed, the grievance procedure is a part of the continuous process of collective bargaining. 12 It is intended to promote, as it were, a friendly dialogue between labor and management as a means of maintaining industrial peace. In Royal Interocean, the employee's letter to the home office, for writing which she was dismissed, complained of the local manager's "inconsiderate and untactful attitude"20 — a grievance which, the court found, "had nothing to do with or did not arise from her union activities." Nor did the court find evidence of discriminatory discharge in Lakas ng Pagkakaisa as the letter, which the employee wrote to the mother company in violation of the local company's rule, denounced "wastage of company funds." In contrast, the express finding of the court in this case was that the dismissal of the respondents was made on account of the letter they had written, in which they demanded the resignation of the bank president for a number of reasons touching labor-management relations SEPARATE OPINION: FERNANDO No one can doubt that we are in the process of evolving an indigenous labor jurisprudence. Notwithstanding the clearly American background of the Industrial Peace Act, based as it is mainly on the Wagner Act,1 labor relations in the Philippines with their peculiar problems and the ingenuity of Filipino lawyers have resulted in a growing body of decisions notable for their suitability to local condition and their distinctly local flavor. This is as it should be. At the very least, it may indicate that while the problem posed could have arisen there, this particular response of labor was quite unique. On the assumption which I have here hypothetically made that there was indeed a valid cause for grievance, a more diplomatic approach could have been attempted. Or at the very least the procedure indicated for the adjustment of a grievance could have been followed. That was not done. What respondents did was to issue an ultimatum. Collective bargaining whether in its formative stage preparatory to a labor contract or in the adjustment of a labor problem in accordance with the procedure set forth in an existing agreement presupposes the give-and-take of discussion. No party adopts, at least in its initial stages, a hard-line position, from which there can be no retreat. That was not the situation here. Respondents as labor leaders appeared adamantine in their attitude to terminate the services of the then president of the Republic Savings Bank. Nor did they mince words in describing his alleged misdeeds. They were quite certain that he had offended most grievously. They wanted him out. There was no room for discussion. That for me is not bargaining as traditionally and commonly understood. It is for that reason that I find it difficult to agree fully with the view that their dismissal could be construed as a refusal to bargain collectively. .. Nonetheless, concurrence with the decision arrived at by the Court is called for in view of their mass dismissal In the Philippines as in the United States then, the first subsection on "interference, restraint or coercion" covering as it does such a broad range of undesirable practices on the part of employers could easily be seized upon, where a borderline case, inimical to the right of self-organization or to collective bargaining, presents itself as justifying a finding of an unfair labor practice. San Miguel Corporation v. NLRC, 304 SCRA 1 (1999) 1. San Miguel Cooperation, alleging the need to streamline its operations due to financial loses, shut down some of its plants and declared 55 positions as redundant listed as follows: seventeen (17) employees in the Business Logistics Division ("BLD"), seventeen (17) in the Ayala Operations Center (AOC), and eighteen (18) in the Magnolia-Manila Buying Station ("Magnolia-MBS") 2. respondent union filed several grievance cases for the said retrenched employees, praying for the redeployment of the said employees to the other divisions of the company. 3. The grievance proceedings were conducted pursuant to Sections 5 and 8, Article VIII of the parties' 1990 Collective Bargaining Agreement 4. During the grievance proceedings, however, most of the employees were redeployed, while others accepted early retirement. As a result only 17 employees remained when the parties proceeded to the third level (Step 3) of the grievance procedure. In a meeting on October 26, 1990, petitioner informed private respondent union that if by October 30, 1990, the remaining 17 employees could not yet be redeployed, their services would be terminated on November 2, 1990. The said meeting adjourned when Mr. Daniel S. L. Borbon II, a representative of the union, declared that there was nothing more to discuss in view of the deadlock.

121 | P a g e

union gave notice of strike based on: a) bargaining deadlock; b) union busting; c) gross violation of the Collective Bargaining Agreement (CBA), such as noncompliance with the grievance procedure; d) failure to provide private respondent with a list of vacant positions pursuant to the parties side agreement that was appended to the 1990 CBA; and e) defiance of voluntary arbitration award 6. SMC filed complaint with NLRC based on: : (1) the dismissal the notice of strike; (2) an order compelling the respondent union to submit to grievance and arbitration the issue listed in the notice of strike; (3) the recovery of the expenses of litigation. 7. NLRC dismissed complaint SC: In the case under consideration, the grounds relied upon by the private respondent union are non-strikeable. The issues which may lend substance to the notice of strike filed by the private respondent union are: collective bargaining deadlock and petitioner's alleged violation of the collective bargaining agreement. These grounds, however, appear more illusory than real. Collective Bargaining Deadlock is defined as "the situation between the labor and the management of the company where there is failure in the collective bargaining negotiations resulting in a stalemate" 11 This situation, is non-existent in the present case since there is a Board assigned on the third level (Step 3) of the grievance machinery to resolve the conflicting views of the parties. Instead of asking the Conciliation Board composed of five representatives each from the company and the union, to decide the conflict, petitioner declared a deadlock, and thereafter, filed a notice of strike. For failing to exhaust all the steps in the grievance machinery and arbitration proceedings provided in the Collective Bargaining Agreement, the notice of strike should have been dismissed by the NLRC and private respondent union ordered to proceed with the grievance and arbitration proceedings. As regards the alleged violation of the CBA, we hold that such a violation is chargeable against the private respondent union. In abandoning the grievance proceedings and stubbornly refusing to avail of the remedies under the CBA. private respondent violated the mandatory provisions of the collective bargaining agreement. Abolition of departments or positions in the company is one of the recognized management prerogatives. WHEREFORE, Petitioner San Miguel Corporation and private respondent San Miguel Corporation Employees Union — PTGWO are hereby directed to complete the third level (Step 3) of the Grievance Procedure and proceed with the Arbitration proceedings if necessary. 2. Deadlock 3. Minutes of Negotiation Samahang Manggagawa sa Top Form Manufacturing United Workers of the Philippines v. NLRC, 295 SCRA 171 (1998) FACTS: The charge arose from the employer's refusal to grant across-the-board increases to its employees in implementing Wage Orders Nos. 01 and 02 of the Regional Tripartite Wages and Productivity Board of the National Capital Region (RTWPB-NCR). Such refusal was aggravated by the fact that prior to the issuance of said wage orders, the employer allegedly promised at the collective bargaining conferences to implement any government-mandated wage increases on an across-theboard basis. Petitioner Samahang Manggagawa sa Top Form Manufacturing United Workers of the Philippines (SMTFM) was the certified collective bargaining representative of all regular rank and file employees of private respondent Top Form Manufacturing Philippines, Inc. At the collective bargaining negotiation, the parties agreed to discuss unresolved economic issues. According to the minutes of the meeting the Union proposed that any future wage increase given by the government should be implemented by the company across-the-board or non-conditional. Management requested the union to retain this provision since their sincerity was already proven when the P25.00 wage increase was granted across-the-board. The union acknowledges management's sincerity but they are worried that in case there is a new set of management, they can just show their CBA. The union decided to defer this provision. In their joint affidavit dated January 30, 1992, union members Salve L. Barnes, Eulisa Mendoza, Lourdes Barbero and Concesa Ibañez affirmed that at the subsequent collective bargaining negotiations, the union insisted on the incorporation in the collective bargaining agreement (CBA) of the union proposal on "automatic across-the-board wage increase." On October 15, 1990, the RTWPB-NCR issued Wage Order No. 01 granting an increase of P17.00 per day in the salary of workers. This was followed by Wage Order No. 02 dated December 20, 1990 providing for a P12.00 daily increase in salary. As expected, the union requested the implementation of said wage orders. However, they demanded that the increase be on an across-the-board basis. Private respondent refused to accede to that demand. Instead, it implemented a scheme of increases purportedly to avoid wage distortion. On October 24, 1991, the union, through its legal counsel, wrote private respondent a letter demanding that it should "fulfill its pledge of sincerity to the union by granting an across-the-board wage increase to all employees under the wage orders." Several conferences between the parties notwithstanding, private respondent adamantly maintained its position on the salary increases it had granted that were purportedly designed to avoid wage distortion. Consequently, the union filed a complaint with the NCR NLRC alleging that private respondent's act of "reneging on its undertaking/promise clearly constitutes act of unfair labor practice through bargaining in bad faith." It charged private respondent with acts of unfair labor practices or violation of Article 247 of the Labor Code, as amended, specifically "bargaining in bad faith," and prayed that it be awarded actual, moral and exemplary damages. 122 | P a g e

5.

Private respondent, on the other hand, contended that in implementing Wage Orders Nos. 01 and 02, it had avoided "the existence of a wage distortion" that would arise from such implementation. It emphasized that only "after a reasonable length of time from the implementation" of the wage orders "that the union surprisingly raised the question that the company should have implemented said wage orders on an across-the-board basis." It asserted that there was no agreement to the effect that future wage increases mandated by the government should be implemented on an across-the-board basis. Otherwise, that agreement would have been incorporated and expressly stipulated in the CBA. On March 11, 1992, Labor Arbiter Jose G. de Vera rendered a decision dismissing the complaint for lack of merit. Not satisfied, petitioner appealed to the NLRC that, in turn, promulgated the assailed Resolution of April 29, 1993 dismissing the appeal for lack of merit. Still dissatisfied, petitioner sought reconsideration which, however, was denied by the NLRC in the Resolution dated January 17, 1994. ISSUE: WON private respondent committed an unfair labor practice HELD: NO Reasoning: - If there was indeed a promise or undertaking on the part of private respondent to obligate itself to grant an automatic across-the-board wage increase, petitioner union should have requested or demanded that such "promise or undertaking" be incorporated in the CBA. After all, petitioner union has the means under the law to compel private respondent to incorporate this specific economic proposal in the CBA. It could have invoked Article 252 of the Labor Code defining "duty to bargain," thus, the duty includes "executing a contract incorporating such agreements if requested by either party." - The CBA is the law between the contracting parties, the collective bargaining representative and the employer-company. Compliance with a CBA is mandated by the expressed policy to give protection to labor. In the same vein, CBA provisions should be "construed liberally rather than narrowly and technically, and the courts must place a practical and realistic construction upon it, giving due consideration to the context in which it is negotiated and purpose which it is intended to serve." This is founded on the dictum that a CBA is not an ordinary contract but one impressed with public interest. It goes without saying, however, that only provisions embodied in the CBA should be so interpreted and complied with. Where a proposal raised by a contracting party does not find print in the CBA, it is not a part thereof and the proponent has no claim whatsoever to its implementation. Because the proposal was never embodied in the CBA, the promise has remained just that, a promise, the implementation of which cannot be validly demanded under the law. DISPOSITIVE: NLRC decision affirmed. 4. Suspension of Bargaining Collegio de San Juan de Letran v. Association of Employees, 340 SCRA 587 (2000) supra D. BARGAINABLE ISSUES – 252 1. Issues Philippine American Management Co., Inc. v. Philippine American Management Employees Association, 51 SCRA 98 (1973) Manila Fashions Inc. v. NLRC, 264 SCRA 104 (1996) On Feb 4, 1992, the union Nagkakaisang Manggagawa ng Manila Fashions, Inc., and the company entered into a CBA where one of the stipulations expresses the willingness of the union to condone the implementation of the P12 wage increase mandated by Wage Order No NCR-02 to forestall the closure of the company. On March 1993, the union through its president, on behalf of the employees filed a complaint before the Labor Arbiter a complaint for non-compliance with the wage order resulting to underpayment of complainants' basic pay, 13th month pay, service incentive leave pay, legal holiday pay, night shift differential and overtime pay. The complainants admitted the existence of the aforementioned provision in the CBA; however they denied the validity thereof inasmuch as it was not reached after due consultation with the members. Petitioner maintains that the condonation is valid as the agreement was voluntarily entered into and represents a reasonable settlement it is binding on the parties and may not be disowned simply because of a change of mind. Labor Arbiter held the stipulation invalid for being contrary to law; NLRC affirmed. Issue: WON the condonation of the implementation of Wage Order No. NCR-02 and 02-A contained in Sec. 3, Art. VIII, of the CBA valid Held: No. As in all other contracts, the parties in a CBA may establish such stipulations, clauses, terms and conditions as they may deem convenient provided they are not contrary to law, morals, good customs, public order or public policy. 6 Section 3, Art. VIII, of the CBA is a void provision because by agreeing to condone the implementation of the Wage Order the parties thereby contravened its mandate on wage increase of P12.00 effective 8 January 1991. Also, as stated by the Labor Arbiter, it is only the Tripartite Wage Productivity Board of the DOLE that could approve exemption of an establishment from coverage of a Wage Order. If petitioner is a financially distressed company then it should have applied for a wage exemption so that it could meet its labor costs without endangering its viability or its very existence upon which both management and labor depend for a living. The Office of the Solicitor General emphasizes the point that parties to a CBA may not by themselves, set a wage lower than the minimum wage. To do so would render nugatory the purpose of a wage exemption, not to mention the possibility that employees may be unwittingly put in a position to accept a lower wage. Republic Savings Bank v. CIR, 21 SCRA 226 (1967) supra 123 | P a g e

Nestle Philippines, Inc. v. NLRC, 193 SCRA 504 (1991) On June 30, 1987, the CBA between Nestle and the employees in Alabang/Cabuyao, Makati, Cagayan de Oro, and Cebu/Davao expired. Thereafter, UNion of Filipro Employees was certified as the sole and exclusive bargaining agent for all regular rank-and-file employees at the petitioner's Cagayan de Oro factory and Cebu/Davao Sales Office. While the parties are negotiating, employees at Cabuyao resorted to a "slowdown" and walk-outs prompting the petitioner to shut down the factory, after which, marathon negotiation ensued. UFE declared bargaining deadlock, thereafter SOLE assumed jurisdiction and issues return to work order. Despite the order, the union struck without notice which prompted the company to dismiss the union officers and negotiating panel who participated in the strike. UFE then filed a notice of strike on the ground of CBA deadlock and ULP. Subsequent to this, the company was able to conclude a CBA with the union at the Cebu and CDO offices. Union assailed validity of the CBA and filed a case of unfair labor practice against the company on November 16, 1988. The dispute was ceritifed to NLRC, which issued a resolution increasing the emoluments of the company's Retirement Plan. Both parties filed MR, which NLRC denied. Nestle filed certiorari. alleging that since its retirement plan is non-contributory, it (Nestlé) has the sole and exclusive prerogative to define the terms of the plan "because the workers have no vested and demandable rights thereunder, the grant thereof being not a contractual obligation but merely gratuitous. At most the company can only be directed to maintain the same but not to change its terms. Issue: WON the Retirement Plan is a collective bargaining issue. Held: Yes. The Retirement Plan was "a collective bargaining issue right from the start" for the improvement of the existing Retirement Plan was one of the original CBA proposals submitted by the UFE on May 8, 1987 to Arthur Gilmour, president of Nestlé Philippines. The fact that the retirement plan is non-contributory, i.e., that the employees contribute nothing to the operation of the plan, does not make it a non-issue in the CBA negotiations. As a matter of fact, almost all of the benefits that the petitioner has granted to its employees under the CBA — salary increases, rice allowances, mid-year bonuses, 13th and 14th month pay, seniority pay, medical and hospitalization plans, health and dental services, vacation, sick & other leaves with pay — are non-contributory benefits. Since the retirement plan has been an integral part of the CBA since 1972, the Union's demand to increase the benefits due the employees under said plan, is a valid CBA issue. The deadlock between the company and the union on this issue was resolvable by the Secretary of Labor, or the NLRC, after the Secretary had assumed jurisdiction over the labor dispute. The petitioner's contention, that employees have no vested or demandable right to a non-contributory retirement plan, has no merit for employees do have a vested and demandable right over existing benefits voluntarily granted to them by their employer. The latter may not unilaterally withdraw, eliminate or diminish such benefits. Samahang Manggagawa sa Top Form Manufacturing United Workers of the Philippines v. NLRC, 295 SCRA 171 (1998) supra Standard Chartered Bank Employees Union v. Confessor, 432 SCRA 304 (2004) supra READ: Bernard D. Meltzer, “The Subjects of Mandatory Bargaining”, (Library File) E. COLLECTIVE BARGAINING AGREEMENT 1. Definition Faculty Association of MIT v. CA, 523 SCRA 709 (2007) Facts: • MIT hired Arthur Andersen to develop a faculty ranking and compensation system. In the 5 th CBA negotiation meeting, MIT presented the new faculty ranking instrument to FAMIT, who agreed to the adoption and implementation of the instrument, with the reservation that there should be no diminution in rank and pay of the faculty members. • FAMIT and MIT entered into a new CBA effective 1 June 2001, which incorporated the new ranking for the college faculty. The faculty ranking sheet (Annex B) was annexed to the CBA, as well as the college faculty rates sheet (Annex C) for permanent faculty which included the point ranges and corresponding pay rates per faculty level. • After a month, MIT requested for an amendment of Annexes B, C and D (HS faculty rates for permanent faculty only), claiming that C and D contained data under the heading “TOTAL POINTS” that were not germane to the two other columns in both annexes, and that B was inadvertently not attached to the CBA. • FAMIT rejected the proposal, saying that these changes would constitute a violation of the ratified 2001 CBA and result in the diminution of rank and benefits of FAMIT college faculty. (the proposed amendment in the ranking system for the college faculty expands the 19 faculty ranks to 23) • MIT instituted some changes in the curriculum for the school year 2000-2001 which resulted in changes in the number of hours for certain subjects. Thus, MIT adopted a new formula for determining the pay rates of the HS faculty: [Rate/Load x Total Teaching Load = Salary] where total teaching load equals number of classes multiplied by hours of service per week divided by 3 hours (as practiced, one unit subject is equal to 3 hours of service.) • FAMIT opposed the formula, saying that MIT has not been implementing the relevant provisions of the 2001 CBA (25% increase in per rate/load for all high school faculty members effective Nov 2000; 10% increase in per rate/load for all permanent high school faculty members effective June 2001) • Together with the issue viz. college faculty, FAMIT brought the matter to the National Conciliation and Mediation Board for mediation. The case was submitted to the Panel of Voluntary Arbitrators for resolution. They ruled for FAMIT regarding both issues. 124 | P a g e

• The CA reversed the PVA ruling. FAMIT appealed to the SC. Issues: − Is MIT’s new proposal, regarding faculty ranking and evaluation, lawful and consistent with the ratified CBA? − Is MIT’s development of a new pay formula for the high school department, without the knowledge of FAMIT, lawful and consistent with the ratified CBA? Held: − No. In the light of the existing CBA, the new point range system proposed by MIT is an unauthorized modification of Annex "C" of the 2001 CBA. It is made up of a faculty classification that is substantially different from the one originally incorporated in the current CBA between the parties. Thus, the proposed system contravenes the existing provisions of the CBA, hence, violative of the law between the parties. − Until a new CBA is executed by and between the parties, they are duty-bound to keep the status quo and to continue in full force and effect the terms and conditions of the existing agreement. The law does not provide for any exception nor qualification on which economic provisions of the existing agreement are to retain its force and effect. Therefore, it must be understood as encompassing all the terms and conditions in the said agreement. The CBA is the norm of conduct between petitioner and private respondent and compliance therewith is mandated by the express policy of the law. − No. MIT cannot adopt its unilateral interpretation of terms in the CBA. It is clear from the provisions of the 2001 CBA that the salary of a high school faculty member is based on a rate per load and not on a rate per hour basis. PETITION GRANTED. TSPI Corporation v. TSPI Employers Union, 545 SCRA 216 (2008) Facts: − TSPIC is engaged in the business of designing, manufacturing and marketing integrated circuits to serve the communication, automotive, data processing and aerospace industries. TSPIC Employees Union is the registered bargaining agent of the rank-and-file employees of TSPIC. − TSPIC and the Union entered into a CBA for the years 2000 to 2004, which included a provision on yearly salary increases starting January 2000 until January 2002 for regular employees within the bargaining unit as such: 10% on 1 January 2000, 12% 1 January 2001 and 11% on 1 January 2002. − The same provision also states that the wage/salary increases for the years 2001 and 2002 shall be deemed inclusive of the mandated minimum wage increases under future Wage Orders that may be issued after Wage Order No. 7, and shall be considered as correction of any wage distortion that may have been brought about by the said future Wage Orders. − The CBA also provided that employees who acquire regular employment status within the year but after the effectivity of a particular salary increase shall receive a proportionate part of the increase: (1st quarter: 100%; 2nd quarter: 75%; 3rd quarter: 50%; 4th quarter: 25%) − 1 January 2000 – all regular rank-and-file employees received a 10% increase in their salary, including nine of the respondents. − 6 October 2000 – Wage Order No. 8 was issued raising the daily minimum wage from 223.50 to 250 effective 1 November 2000. − The wages of 17 probationary employees were increased according to the WO. On various dates during the last quarter of 2000, these 17 employees attained regular employment and received 25% of 10% of their salaries. − January 2001 – TSPIC implemented the CBA-mandated salary increase. As a result, nine regular employees (respondents) received less wages. − TSPIC's HR Department notified 24 employees that due to an error in the automated payroll system, they were overpaid and the overpayment would be deducted from their salaries in a staggered basis, starting February 2001. TSPIC explained that the correction of the erroneous computation was based on the crediting provision of the CBA. − The Union asserted that this constituted diminution of pay. − TSPIC and the Union agreed to undergo voluntary arbitration on the solitary issue of whether or not the acts of the management in making deductions from the salaries of the affected employees constituted diminution of pay. − Arbitrator Jimenez held that the unilateral deduction made by TSPIC violated Art. 100 of the Labor Code. The CA affirmed this decision. Issue: − Does the TSPIC's decision to deduct the alleged overpayment from the salaries of the affected members of the Union constitute diminution of benefits in violation of the Labor Code? Held: − No. The CBA is the law between the parties and they are obliged to comply with its provisions. If the terms of a contract, as in a CBA, are clear and leave no doubt upon the intention of the contracting parties, the literal meaning of their stipulations shall control.

125 | P a g e

− − − −

Sometimes, though the provisions of the CBA seem clear and unambiguous, the parties sometimes arrive at conflicting interpretations. Here, TSPIC wants to credit the increase granted by WO No. 8 to the increase granted under the CBA. The Union, on the other hand, insists that the “crediting” provision finds no application in the present case, since at the time WO No. 8 was issued, the probationary employees were not yet covered by the CBA, particularly by its crediting provision. The CBA states the specific condition that the wage/salary increases for the years 2001 and 2002 shall be deemed inclusive of the mandated minimum wage increases under future wage orders that may be issued after WO No. 7, and shall be considered as correction of the wage distortions that may be brought about by the said future wage orders. Thus, the wage/salary increases in 2001 and 2002 shall be deemed as compliance to future wage orders after WO No. 7. The intention of the parties is clear: As long as an employee is qualified to receive the 12% increase in salary, the employee shall be granted the 12% increase; and as long as an employee is granted the 12% increase, the amount shall be credited against any wage order issued after WO No. 7. Respondents should not be allowed to receive benefits from the CBA while avoiding the counterpart crediting provision.

Honda Philippines, Inc. v. Samahan ng Malayang Manggagawa sa Honda, 460 SCRA 186 (2005) University of the Immaculate Concepcion, Inc. v. Secretary of Labor, 374 SCRA 471 (2002) Nature: Appeal via Certiorari Facts - University of the Immaculate Concepcion, Inc. is a non-stock, non-profit educational institution, who, on 2 occasions, met with the ICTNE Union-FFW, through the auspices of the National Conciliation and Mediation Board (NCMB), to negotiate a CBA. - The Union filed with the NCMB a Notice of Strike, the first in a series of 3 notices of strike, alleging deadlock in the CBA negotiations and unfair labor practices on the part of the petition in the form of "mass termination of teaching and non-teaching employees, interference with union activities, discrimination, and harassments ." - The Univ. of IC denied the allegations in its Motion to Strike Out Notice of Strike. - During the parties' conciliation conference before the NCMB, petitioner and the Union reached an agreement on some issues (ie. ECONOMIC ISSUE~ pertaining to increase in amt of salary per increase in tuition fees; NON-ECONOMIC ISSUES~ union recognition and security, working schedule) - The panel of voluntary arbitrators rendered a decision excluding the secretaries, registrars, cashiers, guidance counselors and the chief of the accounting department of the petitioner from the coverage of the bargaining unit. - The University presented to the Union a draft of the CBA. After a study thereof, the Union rejected the draft on the ground that the manner of computing the net incremental proceeds has yet to be agreed upon by the parties. - The University wrote the Union insisting that the Union was bound to comply with the terms contained in the draft-CBA since said draft allegedly embodies all the items agreed upon by the parties during the conciliation sessions held by the NCMB. - the Union filed its Second Notice of Strike with the NCMB, therein alleging bargaining deadlock on "allocation of 5% (CBA) and distribution/computation of 70% incremental proceeds (RA6728)", and unfair labor practice by the petitioner in the form of "harassments, union busting and correct implementation of COLA," - After the Union's filing of its Second Notice of Strike, the University terminated the employment of 2 union members who later filed their complaints for illegal dismissal before the Regional Arbitration Branch No. XI of the NLRC in Davao City. - In the same venue, the University filed a complaint against the Union and its officers for unfair labor practices based (ie refusing to answer in writing, and within 10 days required by law, the cba proposals, refusing to bargain in good faith, by declaring a deadlock in the cba negotiations after just two days of negotiations, even if there were so many issues unresolved and still to be discussed at the bargaining table, etc) - The case was elevated to the Secretary of Labor. The Union also pushed through with the strike. - The Secretary of Labor issued an order assuming jurisdiction over the labor dispute and directed all workers to return to work within 24 hours upon receipt of the Order and for management to accept them back under the same terms and conditions prior to the strike The parties were further directed to cease and desist from committing any or all acts that might exacerbate the situation. - Eventually, the Sec of Labor found that the strike undertaken by the Union was a valid exercise of the workers' rights under the Labor Code. The Union observed the mandatory requirements/procedures for a valid strike and the issues raised in the Notice of Strike i.e., bargaining deadlock and ULP are strikeable issues specifically provided under Article 263 (c) of the Labor Code and then directed the University and the Union to execute a CBA embodying the dispositions contained herein as well as all items agreed upon by the parties. The CBA shall be effective for five (5) years starting SY 1995-96, subject to renegotiation of the economic provisions for the last two (2) years. - CA affirmed the decision of the Sec of Labor. Hence, this appeal. ISSUE: WON the CA erred in affirming the orders of the Secretary of Labor and Employment. HELD: NO. REASONING: - CA did not err in finding that there was still no new collective bargaining agreement because the parties had not reached a meeting of the minds. - A collective bargaining agreement (CBA) refers to the negotiated contract between a legitimate labor organization and the employer concerning wages, hours of work and all other terms and conditions of employment in a bargaining unit, including mandatory provisions for grievances and 126 | P a g e

arbitration machineries. (Manila Fashions v. NLRC) As in all other contracts, there must be clear indications that the parties reached a meeting of the minds. - In this case, no CBA could be concluded because of what the union perceived as illegal deductions from the 70% employees' share in the tuition fee increase from which the salary increases shall be charged. Also, the manner of computing the net incremental proceeds was yet to be agreed upon by the parties. - Petitioner insisted that a new collective bargaining agreement was concluded through the conciliation proceeding before the NCMB on all issues specified in the notice of strike. Although it is true that the university and the union may have reached an agreement on the issues raised during the collective bargaining negotiations, still no agreement was concluded by them because, among other reasons, the DOLE Secretary, who assumed jurisdiction on January 23, 1995 only was set to resolve the distribution of the salary increase of the covered employees. The Court of Appeals found that "there are many items in the draft-CBA that were not even mentioned in the minutes of the July 20, 1994 conference." - Considering the parties failed to reach an agreement regarding certain items of the CBA, they still have the duty to negotiate a new collective bargaining agreement in good faith, pursuant to the applicable provisions of the Labor Code. DISPOSITIVE: Petition Denied. The parties are enjoined to comply with the directive of the Secretary of Labor and Employment to negotiate a collective bargaining agreement in good faith. National Federation of Labor v. CA, 440 SCRA 604 (2004) NATURE Petition for review of the decision of the CA FACTS - American Rubber Company, Inc. (ARCI) is a domestic corporation existing in and incorporated under the laws of the Philippines. It owns a rubber plantation in Latuan, Isabella. It entered into a Farm Management Agreement (FMA) with Sime Darby Pilipinas, Inc. (SDPI), where SDPI was given the right to manage, administer, develop, cultivate, and improve the rubber plantations as an agro-industrial development project, specifically for planting rubber trees, processing of and marketing of its products and providing technical expertise for a period of twenty-five years, or up to the year 2011. - National Federation of Labor (NFL) was the duly registered bargaining agent of the daily-and-monthly-paid rank-and-file employees of SDPI in the Latuan rubber plantation. SDPI and NFL executed a collective bargaining agreement (CBA) in which they agreed that in case of permanent or temporary lay-off, workers affected would be entitled to termination pay as provided by the Labor Code. The 150 petitioners were daily-and-monthly-paid employees of SDPI in the Latuan plantation and were, likewise, members of NFL. - In 1988, RA 6657, aka Comprehensive Agrarian Reform Law took effect. - SDPI decided to cease its operations in certain plants, including the one in Latuan. - The employees were given ½ month pay for every year of service as separation pay, pursuant to the CBA. They were made to sign quitclaim, which they said they entered into voluntarily. - A few months later, they filed a complaint for illegal dismissal and insufficiency of separation pay. - Labor arbiter, NLRC, and CA all found that there was no illegal dismissal and that the employees were properly paid their separation pay. ISSUE WON the employees’ separation pay was insufficient. HELD NO. - The employees argue that they should have gotten 1 month per year of service, pursuant to company policy. The precedents they site are not applicable. - A collective bargaining agreement refers to the negotiated contract between the legitimate labor organization and the employer concerning wages, hours of work and all other terms and conditions of employment in the bargaining unit. During the negotiations, the parties, management and union meet and convene promptly and expeditiously in good faith for the purpose of negotiating an agreement. Had the daily-paid rank-and-file employees deemed the same to be a diminution of their benefits, they should have rejected the CBA. The petitioners never assailed the CBA as prejudicial to them or for having been in violation of Article 100 of the Labor Code. Unless annulled, the CBA, as a contract governing the employer and the employees respecting the terms of employment, should prevail. - If the separation pay was supposed to be 1 month per year of service, why wasn’t it specified in the CBA? Instead, the CBA says as provided by the LC. Art. 283 provides that in case of closure or cessation of operations, the separation pay shall be equivalent to 1 month pay or to at least ½ month per year of service, whichever is higher. In this case, the latter is higher, so that’s what the company gave them. Disposition Petition DENIED. Rivera v. Espiritu, 374 SCRA 351 (2002) supra 2. Contents a. See Sample Collective Bargaining Agreement b. Effect Sub-Standard Contract – 239 (f) Art. 239. Grounds for cancellation of union registration. The following shall constitute grounds for cancellation of union registration: f. Entering into collective bargaining agreements which provide terms and conditions of employment below minimum standards established by law; 127 | P a g e

c. Duration and Re-negotiation – 253-A Art. 253-A. Terms of a collective bargaining agreement. Any Collective Bargaining Agreement that the parties may enter into shall, insofar as the representation aspect is concerned, be for a term of five (5) years. No petition questioning the majority status of the incumbent bargaining agent shall be entertained and no certification election shall be conducted by the Department of Labor and Employment outside of the sixty-day period immediately before the date of expiry of such fiveyear term of the Collective Bargaining Agreement. All other provisions of the Collective Bargaining Agreement shall be renegotiated not later than three (3) years after its execution. Any agreement on such other provisions of the Collective Bargaining Agreement entered into within six (6) months from the date of expiry of the term of such other provisions as fixed in such Collective Bargaining Agreement, shall retroact to the day immediately following such date. If any such agreement is entered into beyond six months, the parties shall agree on the duration of retroactivity thereof. In case of a deadlock in the renegotiation of the Collective Bargaining Agreement, the parties may exercise their rights under this Code. (As amended by Section 21, Republic Act No. 6715, March 21, 1989) d. Grievance Procedure – 260 Art. 260. Grievance machinery and voluntary arbitration. The parties to a Collective Bargaining Agreement shall include therein provisions that will ensure the mutual observance of its terms and conditions. They shall establish a machinery for the adjustment and resolution of grievances arising from the interpretation or implementation of their Collective Bargaining Agreement and those arising from the interpretation or enforcement of company personnel policies. All grievances submitted to the grievance machinery which are not settled within seven (7) calendar days from the date of its submission shall automatically be referred to voluntary arbitration prescribed in the Collective Bargaining Agreement. For this purpose, parties to a Collective Bargaining Agreement shall name and designate in advance a Voluntary Arbitrator or panel of Voluntary Arbitrators, or include in the agreement a procedure for the selection of such Voluntary Arbitrator or panel of Voluntary Arbitrators, preferably from the listing of qualified Voluntary Arbitrators duly accredited by the Board. In case the parties fail to select a Voluntary Arbitrator or panel of Voluntary Arbitrators, the Board shall designate the Voluntary Arbitrator or panel of Voluntary Arbitrators, as may be necessary, pursuant to the selection procedure agreed upon in the Collective Bargaining Agreement, which shall act with the same force and effect as if the Arbitrator or panel of Arbitrators has been selected by the parties as described above. San Miguel Foods, Inc. v. San Miguel Corporation Employees Union, PTGWU, 535 SCRA 133 (2007) United Kimberly-Clark Employees Union v. Kimberly-Clark Philippines, Inc. 484 SCRA 187 (2006) NATURE Petition for Review.Certiorari FACTS - Petitioner is the labor union representing rank and file employees of respondent. Way back in 1980, the parties agreed to include in their CBA a provision which states that the company agrees to employ immediate relative of an employee who had retired, resigned, died provided that the employee had rendered at least ten years service. There were no other standards set with regard the acceptance of the said recommendees and as a matter of fact, even high school graduates were accepted. - In 1991, a case was filed against the company for refusing to employ a nephew of a retiring employee (Kimberly Clark vs Lorredo) as apparently the retiring employee had children who he did not recommend and the company was questioning this. In any case, the company lost in this case but as part of the ruling of the Court, it was stated that Kimberly was not obliged to unconditionally accept the recommendee since the latter must still meet the required employment standard theretofore set by it. Even a qualified recommendee would be hired only on a “probationary status.” As such, KCPI was not left without its own safeguards under the agreement. - In 1995, the company issued the now questioned guidelines which among others required that such recommendees must be at least 18 years of age but not more than 30 years old at the time of the hiring, and (b) have completed, after graduating from high school, at least a two-year technical/vocational course or a third year level of college education. Moreover, where both husband and wife are employees of the company, they shall be treated as one family; hence, only one of the spouses would be allowed to avail of the benefit. - The Union and the company agreed to postpone the implementation of said guidelines until January 1, 1997 but only with respect to the educational qualification. And the guidelines were in fact implemented in the second half of 1998. A voluntary arbitrator ruled on the controversy saying that the company cannot upgrade the educational qualification as this is contrary to what has been in existence and what had been a practice. - Appeal was filed with the CA which reversed the resolution of the voluntary arbitrator with regard the upgrade of the qualification of the recommendee. Hence this appeal. ISSUE/S WON the CA erred in ruling that, under Article XX, Section 1 of the 1997 CBA, respondent is required to hire only those recommendees of retired/resigned, deceased or disabled members of petitioner who had completed at least a two-year technical/vocational course or a third-year level of college education HELD NO. In the present case, the parties are in agreement that, on its face, Article XX, Section 1 of their 1997 CBA does not contain any provision relative to the employment qualification standards of recommendees of retired/resigned, deceased or disabled employees of respondent who are members of petitioner. However, in 128 | P a g e

determining the employment qualification standards for said recommendees, the VA should have relied on the November 7, 1995 Guidelines issued by respondent, which reads: D. Definition of the phrase “immediate member of the family of an employee” 1. The phrase “immediate member of the family of an employee” shall refer to the employee’s legitimate children and in default thereof to the employee’s collateral relatives within the third civil degree. 2. A resigned/retired employee may be allowed to recommend a collateral relative within the third civil degree (e.g., brother, sister, nephew or niece) as his/her replacement only in the following cases: a. Where the retired/resigned employee is single or if married has no legitimate children. b. Where the retired/resigned employee’s children are still minors (below 18 years old) at the time of his/her separation from the company. (Emphasis added) E. General Provisions 1. The privilege to recommend a replacement can be exercised by the employee concerned only once. Thus, in the following cases, a recommendee who has been hired on probationary status can no longer be substituted with another recommendee. a. where the recommendee fails to pass in his performance evaluation. b. where the recommendee resigns without completing his probationary period. c. where the recommendee is dismissed for cause. d. where the recommendee dies during his probationary period. 1[48] Respondent issued said Guidelines in light of the ruling of this Court in Kimberly Clark Philippines v. Lorredo. Respondent saw it imperative to do away with its practice of accommodating recommendees who were mere high school graduates, and to require higher employment standards for them. By agreement of the parties, the implementation of the Guidelines was deferred until January 1, 1997, unless revoked or amended by the 1997 CBA. Petitioner proposed that the practice of hiring recommendees of retired/resigned, deceased or disabled employees who were union members, who were at least high school graduates, be included in their CBA, but respondent did not agree. Hence, Article XX, Section 1 of the 1997 CBA of the parties remained intact. There was thus no more legal bar for respondent to implement the November 7, 1995 Guidelines. By executing the 1997 CBA, in its present form, petitioner is bound by the terms and conditions therein set forth. The Court has recognized in numerous instances the undoubted right of the employer to regulate, according to his own discretion and best judgment, all aspects of employment, including but not limited to, work assignments and supervision, working methods and regulations, time, place and manner of work, processes to be followed, and hiring, supervision, transfer, discipline, lay off, dismissal and recall of workers. Encompassing though it could be, the exercise of this right is not absolute. Management prerogative must be exercised in good faith for the advancement of the employer’s interest and not for the purpose of defeating or circumventing the rights of the employees under special laws, valid agreements such as the individual contract of employment and the collective bargaining agreement, and general principles of justice and fair play. 2[49] In this case, the Court finds that respondent acted in accord with the CBA and the November 7, 1995 Guidelines, which, by agreement of the parties, may be implemented by respondent after January 1, 1997. Disposition Petition is denied. Luzon Development Bank v. Association of Development Bank Employees, 249 SCRA 162 (1995) NATURE Petition for certiorari and prohibition seeking to set aside the decision of the Voluntary Arbitrator and to prohibit her from enforcing the same FACTS -Luzon Development Bank (LDB) and the Association of Luzon Development Bank Employees (ALDBE) submitted to arbitration to resolve WON the company has violated the Collective Bargaining Agreement provision and the Memorandum of Agreement dated April 1994, on promotion -The parties agreed to submit their respective Position Papers on December 1-15, 1994. -Atty. Ester S. Garcia, in her capacity as Voluntary Arbitrator, received ALDBE's Position Paper on January 18, 1995. -LDB, on the other hand, failed to submit its Position Paper -On May 24, 1995, without LDB's Position Paper, the Voluntary Arbitrator rendered a decision finding that the Bank has not adhered to the Collective Bargaining Agreement provision nor the Memorandum of Agreement on promotion. -Hence, this petition ISSUE WON the Voluntary Arbitrator erred in finding that the Bank has not adhered to the Collective Bargaining Agreement provision nor the Memorandum of Agreement on promotion

1 2
129 | P a g e

(the Court referred the case to the CA so the issue wasn’t resolved…it said that elevating a decision or award of a voluntary arbitrator to the Supreme Court on a petition for certiorari is in effect equating the voluntary arbitrator with the NLRC or the Court of Appeals, which in its view is illogical and imposes an unnecessary burden upon it) HELD (only obiter… pertaining to topic) -In labor law context, arbitration is the reference of a labor dispute to an impartial third person for determination on the basis of evidence and arguments presented by such parties who have bound themselves to accept the decision of the arbitrator as final and binding. -Arbitration may either be compulsory or voluntary. -Compulsory arbitration is a system whereby the parties to a dispute are compelled by the government to forego their right to strike and are compelled to accept the resolution of their dispute through arbitration by a third party. -Under voluntary arbitration, on the other hand, referral of a dispute by the parties is made, pursuant to a voluntary arbitration clause in their collective agreement, to an impartial third person for a final and binding resolution . Ideally, arbitration awards are supposed to be complied with by both parties without delay, such that once an award has been rendered by an arbitrator, nothing is left to be done by both parties but to comply with the same. After all, they are presumed to have freely chosen arbitration as the mode of settlement for that particular dispute. Pursuant thereto, they have chosen a mutually acceptable arbitrator who shall hear and decide their case. Above all, they have mutually agreed to de bound by said arbitrator's decision. -In the Philippine context, the parties to a Collective Bargaining Agreement (CBA) are required to include therein provisions for a machinery for the resolution of grievances arising from the interpretation or implementation of the CBA or company personnel policies . -For this purpose, parties to a CBA shall name and designate therein a voluntary arbitrator or a panel of arbitrators, or include a procedure for their selection, preferably from those accredited by the National Conciliation and Mediation Board (NCMB). Disposition The Court resolved to REFER this case to the Court of Appeals Navaro III v. Damaso, 246 SCRA 260 (1995) Petitioner molested a female coworker. After investigating the incident, it was recommended that he be dismissed. The case was submitted for decision by the voluntary arbiter who agreed with the recommendation of dismissal. He was dismissed by the company for violating the Company Code of Conduct. Petitioner contends that the grievance procedure provided for in the CBA was not followed; hence, the Voluntary Arbitrator exceeded his authority when he took cognizance of the labor case. Section 2, Article X of the CBA specifies the instances when the grievance machinery may be availed of, thus: Any protest or misunderstanding concerning any ruling, practice or working conditions in the Company, or any dispute arising as to the meaning, application or claim of violation of any provision of this Agreement or any complaint that any employee may have against the COMPANY shall constitute a grievance The instant case is not a grievance that must be submitted to the grievance machinery. What are subject of the grievance procedure for adjustment and resolution are grievances arising from the interpretation or implementation of the collective bargaining agreement. The acts of petitioner involved a violation of the Code of Employee Discipline, particularly the provision penalizing the immoral conduct of employees. Consequently, there was no justification for petitioner to invoke the grievance machinery provisions of the Collective Bargaining Agreement. However as the case of the petitioner was voluntarily submitted for voluntary arbitration by the union and the employer with the petitioners consent, the decision of the voluntary arbitrator was sustained. San Miguel Corporation v. Confesor, 262 SCRA 81 (1996) San Miguel Corporation was originally one company composed of four operating divisions namely: 1. beer, 2. packaging, 3. magnolia, 4. feeds and livestock. San Miguel Corporation employees for ALL DIVSIONS were represented by San Miguel Corp employees union-PTGWO. San Miguel underwent a restructuring. As a consequence of this Magnolia and Feeds & livestock Division were spun-off and became two separate and distinct corporation. ISSUE: WON the employees of the two new corporations (Magnolia Corp & San Miguel Foods) should still remain in the same bargaining unit and be included I the old bargaining unit of the old SMC? HELD: No. Employees in the Magnolia Corporation and San Miguel Foods may form a separate bargaining unit. 1. The transformation of the companies was a management prerogative and business judgment which the courts cannot look into unless it is contrary to law, public policy or morals. Neither can we impute any bad faith on the part of SMC to justify the application of the doctrine of piercing the corporate veil. 2. Each of the companies are run by, supervised and controlled by different management terms including separate human resource/personnel managers. Each company enforces its own administrative and operational rules. Each entity maintains separate financial statements and are audited separately from each other. 3. No mutuality of interest anymore between corporations: Considering the spin-offs, the companies would consequently have their respective and distinctive concerns in terms of the nature of work, wages, hours of work and other conditions of employment. Interests of employees in the different companies perforce differ. SMC is engaged in the business of beer manufacturing. Magnolia is involved in the manufacturing and processing of dairy products while SMFI is involved in the production of feeds and processing chicken. 130 | P a g e

4. The nature of their products and scales of business may require different skills which must necessarily be commensurate by different compensation packages. The different companies may have different volumes of work and different working conditions. For such reason, the employees of the different companies see the need to group themselves together and organized themselves into distinctive and different groups. General Milling v. CA, 422 SCRA 514 (2004) Facts: Corporation and union entered in CBA. Corp received collective and individual letters from workers stating they had withdrawn from their union membership, on grounds of religious affiliation and personal differences. Relying on this, GMC did not send any counter-proposal in the CBA. Union disclaimed disaffiliation. Union filed complaint alleging unfair labor practice against GMC for: (1) refusal to bargain collectively; (2) interference with the right to self-organization; and (3) discrimination. Issue: WON the corporation refused to bargain collectively Held: Yes Ratio: The union lived up to this obligation when it presented proposals for a new CBA to within three (3) years from the effectivity of the original CBA. But GMC failed in its duty under Article 252 to make a counter-proposal. Union devised a flimsy excuse by questioning the existence of the union and the status of its membership to prevent any negotiation. Corporation’s failure to make a timely reply to the proposals presented by the union is indicative of its utter lack of interest in bargaining with the union. Its excuse that it felt the union no longer represented the workers, was mainly dilatory as it turned out to be utterly baseless. This is an indication of bad faith. Where the employer did not even bother to submit an answer to the bargaining proposals of the union, there is a clear evasion of the duty to bargain collectively. Since it was GMC which violated the duty to bargain collectively (based on Kiok Loy and Divine Word University of Tacloban) it had lost its statutory right to negotiate or renegotiate the terms and conditions of the draft CBA proposed by the union. 3. Registration – Period, Requirements, and Actions – 231 Art. 231. Registry of unions and file of collective bargaining agreements. The Bureau shall keep a registry of legitimate labor organizations. The Bureau shall also maintain a file of all collective bargaining agreements and other related agreements and records of settlement of labor disputes and copies of orders and decisions of voluntary arbitrators. The file shall be open and accessible to interested parties under conditions prescribed by the Secretary of Labor and Employment, provided that no specific information submitted in confidence shall be disclosed unless authorized by the Secretary, or when it is at issue in any judicial litigation, or when public interest or national security so requires. Within thirty (30) days from the execution of a Collective Bargaining Agreement, the parties shall submit copies of the same directly to the Bureau or the Regional Offices of the Department of Labor and Employment for registration, accompanied with verified proofs of its posting in two conspicuous places in the place of work and ratification by the majority of all the workers in the bargaining unit. The Bureau or Regional Offices shall act upon the application for registration of such Collective Bargaining Agreement within five (5) calendar days from receipt thereof. The Regional Offices shall furnish the Bureau with a copy of the Collective Bargaining Agreement within five (5) days from its submission. The Bureau or Regional Office shall assess the employer for every Collective Bargaining Agreement a registration fee of not less than one thousand pesos (P1,000.00) or in any other amount as may be deemed appropriate and necessary by the Secretary of Labor and Employment for the effective and efficient administration of the Voluntary Arbitration Program. Any amount collected under this provision shall accrue to the Special Voluntary Arbitration Fund. The Bureau shall also maintain a file and shall undertake or assist in the publication of all final decisions, orders and awards of the Secretary of Labor and Employment, Regional Directors and the Commission. (As amended by Section 15, Republic Act No. 6715, March 21, 1989) 4. Contract Beneficiaries – 255; 212 (f) Art. 255. Exclusive bargaining representation and workers’ participation in policy and decision-making. The labor organization designated or selected by the majority of the employees in an appropriate collective bargaining unit shall be the exclusive representative of the employees in such unit for the purpose of collective bargaining. However, an individual employee or group of employees shall have the right at any time to present grievances to their employer. Any provision of law to the contrary notwithstanding, workers shall have the right, subject to such rules and regulations as the Secretary of Labor and Employment may promulgate, to participate in policy and decision-making processes of the establishment where they are employed insofar as said processes will directly affect their rights, benefits and welfare. For this purpose, workers and employers may form labor-management councils: Provided, That the representatives of the workers in such labor-management councils shall be elected by at least the majority of all employees in said establishment. (As amended by Section 22, Republic Act No. 6715, March 21, 1989) Art. 212. Definitions. f. "Employee" includes any person in the employ of an employer. The term shall not be limited to the employees of a particular employer, unless the Code so explicitly states. It shall include any individual whose work has ceased as a result of or in connection with any current labor dispute or because of any unfair labor practice if he has not obtained any other substantially equivalent and regular employment.

131 | P a g e

Beneficiaries PAL v. PAL Employees Association, 548 SCRA 117 (2008) Facts: Union filed complaint against PAL for ULP for alleged non-payment of 13 th month pay to employees who had not been regularized as of April 30, 1988, which is in contravention of the CBA. PAL’s position: CBA does not apply to non-regular employees and PAL’s company practice not to give mid-year bonus to non-regular employees. Issue: WON non-regular regular employees may receive 13th month pay or mid-year bonus under CBA. Held: Yes Ratio: 1) Under the CBA, the benefits extend to ALL employees in collective bargaining unit, including those who do not belong to the chosen bargaining labor organization, hence, including non-regular employees. 2) Employees must only be members of a bargaining unit but not necessarily of the labor organization designated as the bargaining unit."Bargaining unit"- comprised of all or less than all of the entire body of employees, which the collective interest of all the employees, with equity to the employer, indicates to be the best suited to serve the reciprocal rights and duties of parties. Non-regular employees are members of “bargaining unit.” 3) Memorandum Order No. 28 removed the salary ceiling, making all employees entitled to 13 th month pay. 4) Parties to a CBA may establish such stipulations, clauses, terms and conditions as they may deem convenient, provided these are not contrary to law, good customs, public order or public policy. New Pacific Timber and Supply Co., Inc. v. NLRC, 328 SCRA 404 (2000) FACTS: The National Federation of Labor (NFL) was certified as the sole and exclusive bargaining representative of all the regular rank-and-file employees of petitioner Company. As such, NFL started to negotiate for better terms and conditions of employment but the same was allegedly rejected by Petitioner Company, so that the former was prompted to file a complaint for ULP. The LA issued an order declaring (a) petitioner Company guilty of ULP; and (b) the CBA proposals submitted by the NFL as the CBA. Petitioner’s appeal—and later certiorari—were both dismissed. Petitioner Company complied with the LA’s order; and, the case was considered closed following NFL's manifestation that it will no longer appeal said order. However, notwithstanding such manifestation, a "Petition for Relief" was filed in behalf of 186 of the private respondents who claimed that they were wrongfully excluded from enjoying said benefits since the agreement with NFL and petitioner Company limited the CBA's implementation to only the 142 rank-and-file employees enumerated. They claimed that NFL's misrepresentations had precluded them from appealing their exclusion. NLRC issued a resolution declaring that the 186 excluded employees as part of the existing rank-and-file bargaining unit and were, therefore, entitled to the benefits under the CBA. Meanwhile, the private respondents, including the original 186 filed individual money claims but LA Villena dismissed these cases. The NLRC set aside the dismissal orders for lack of legal basis. It sustained the earlier NLRC resolution IFO the respondents. Hence the instant petition. ISSUES: 1) Procedural – WON the Petition for Relief is proper (even if treated as an appeal, WON it’s proper for being filed several months after allowable period). 2) A) Substantive - May the term of a CBA as to its economic provisions be extended beyond the term expressly stipulated therein, and, in the absence of a new CBA, even beyond the three-year period provided by law? B) Are employees hired after the stipulated term of a CBA entitled to the benefits provided thereunder? HELD: 1) YES. No grave abuse of discretion on the part of the NLRC, when it entertained the petition for relief. A careful scrutiny of the facts and circumstances of the instant case warrants liberality in the application of technical rules and procedure. 2) A) YES. It is clear from Article 253 that until a new CBA has been executed, the parties are duty-bound to keep the status quo and to continue in full force and effect the terms and conditions of the existing agreement. In the case at bar, the existing CBA in its entirety, continued to have legal effect. The automatic renewal clause provided for by the law, which is deemed incorporated in all CBA's, provides the reason why the new CBA can only be given a prospective effect. To rule otherwise would be to create a gap during which no agreement would govern, from the time the old contract expired to the time a new agreement shall have been entered into. Consequently, the employees from the year 1985 onwards would be deprived of a substantial amount of monetary benefits which runs contrary to the very intent and purpose of Articles 253 and 253-A of the Labor Code which is to curb labor unrest and to promote industrial peace, B) YES. In a long line of cases, this Court has held that when a collective bargaining contract is entered into by the union and the employer, even the non-member employees are entitled to the benefits of the contract. In the same vein, the benefits under the CBA in the instant case should be extended to those employees who only became such after 1984. To exclude them would constitute undue discrimination and deprive them of monetary benefits they would otherwise be entitled to under a new CBA to which they would have been parties. Since in this particular case, no new agreement had been entered into after the CBA's stipulated term, it is only fair and just that the employees hired thereafter be included in the existing CBA. 5. Contract Administration and Enforcement 1. Nature of Contract and Contract Interpretation 132 | P a g e

Almario v. PAL, Inc., 532 SCRA 614 (2007) supra P.I. Manufacturing Inc. v. P.I. Manufacturing, etc. 543 SCRA 614 (2008) P.I. MANUFACTURING, INC v. P.I. MANUFACTURING SUPERVISORS AND FOREMAN ASSOCIATION and the NATIONAL LABOR UNION The Court has always promoted the policy of encouraging employers to grant wage and allowance increases to their employees higher than the minimum rates of increases prescribed by statute or administrative regulation. Consistent with this, the Court also adopts the policy that requires recognition and validation of wage increases given by employers either unilaterally or as a result of collective bargaining negotiations in an effort to correct wage distortions.[1] FACTS: Petitioner Company, Inc is engaged in the manufacture and sale of household appliances while respondent PIMASUFA is an organization of petitioner’s supervisors and foremen, joined in this case by its federation, NLU. On December 10, 1987, the President signed into law R.A. No. 6640 providing an increase in the statutory minimum wage. Section 2 provides:The statutory minimum wage rates of workers and employees in the private sector, whether agricultural or non-agricultural, shall be increased by P10 per day, except non-agricultural workers and employees outside Metro Manila who shall receive an increase of P11 per day: Provided, That those already receiving above the minimum wage up to P100 shall receive an increase of P10.00 per day. Thereafter, on December 18, petitioner and respondent PIMASUFA entered into a new CBA (1987 CBA) whereby the supervisors were granted an increase of P625 per month and the foremen, P475 per month. The increases were made retroactive to May 12, 1987, or prior to the passage of R.A. No. 6640, and every year thereafter until July 26, 1989. PIMASUFA and NLU filed a complaint charging petitioner with violation of RA 6640. The LA ordered the Company to give the members of respondent PIMASUFA wage increases equivalent to 13.5% of their basic pay they were receiving prior to December 14, 1987. It held: “Employees cannot waive future benefits, much less those mandated by law. That is against public policy. Thus, the waiver in the CBA does not bar the union from claiming adjustments in pay as a result of distortion of wages brought about by the implementation of R.A. 6640.” The CA affirmed with modification in this wise: The increase of 13.5% in the supervisors and foremen’s basic salary must further be increased to 18.5% in order to correct the wage distortion brought about by the implementation of RA 6640. ISSUE: 1) WON the implementation of R.A. No. 6640 resulted in a wage distortion and whether such distortion was cured or remedied by the 1987 CBA. HELD: Petition is granted. 1) YES. Wage Distortion is a situation where an increase in prescribed wage rates results in the elimination or severe contraction of intentional quantitative differences in wage or salary rates between and among employee groups in an establishment as to effectively obliterate the distinctions embodied in such wage structure based on skills, length of service, or other logical bases of differentiation. In this case, the Court of Appeals correctly ruled that a wage distortion occurred. Notably, the implementation of R.A. No. 6640 resulted in the increase of P10.00 in the wage rates of Alcantara, supervisor, and Morales and Salvo, both foremen. They are petitioner’s lowest paid supervisor and foremen. As a consequence, the increased wage rates of foremen Morales and Salvo exceeded that of supervisor Buencuchillo. Also, the increased wage rate of supervisor Alcantara exceeded those of supervisors Buencuchillo and Del Prado. However, the same was cured or remedied when respondent PIMASUFA entered into the 1987 CBA with petitioner after the effectivity of R.A. No. 6640. The 1987 CBA increased the monthly salaries of the supervisors by P625.00 and the foremen, by P475.00, effective May 12, 1987. These increases re-established and broadened the pay gaps. The provisions of the CBA should be read in harmony with the wage orders, whose benefits should be given only to those employees covered thereby. Requiring petitioner to pay all the members of PIMASUFA a wage increase of 18.5%, over and above the negotiated wage increases provided under the 1987 CBA , is highly unfair and oppressive to the former. Obviously, it was not the intention of R.A. No. 6640 to grant an across-the-board increase in pay. Only 3 members of respondent PIMASUFA are receiving wage rates below P100.00, thus, only they are entitled to such increase. Now, to direct petitioner to grant an across-the-board increase to all of them, regardless of the amount of wages they are already receiving, would be harsh and unfair to the former. A CBA constitutes the law between the parties when freely and voluntarily entered into. Here, it has not been shown that respondent PIMASUFA was coerced or forced by petitioner to sign the 1987 CBA. Respondents cannot invoke the beneficial provisions of the 1987 CBA but disregard the concessions it voluntary extended to petitioner. TSPIC Co. v. TSPIC Employees Union, 545 SCRA 215 (2008) Bobcock v. Bobcock, 453 SCRA 156 (2005) NATURE Petition for review on certiorari under Rule 45. FACTS Some employees of Babcock-Hitachi Phils were transferred from Makati to Bauan Batnagas, in line with the company’s plan to transfer its Design Department in order to improve the operating efficiency and coordination among its various departments.

133 | P a g e

- The said employees demanded relocation allowance as provided for in their CBA. However, the company refused saying that under Policy Statement No. BHPI-G044A, they are not entitled to it considering that they are residents of Bauan or its adjacent towns. ISSUE WON union members are entitled to relocation allowance in light of the CBA between the parties. HELD YES. Ratio Any doubt or ambiguity in the contract between management and the union members should be resolved in favor of the latter (Article 1702 of the Civil Code) Contracts which are not ambiguous are to be interpreted according to their literal meaning and not beyond their obvious intendment. “the terms and conditions of a collective bargaining contract constitute the law between the parties. Those who are entitled to its benefits can invoke its provisions. In the event that an obligation therein imposed is not fulfilled, the aggrieved party has the right to go to court for redress.” (Mactan Workers Union vs. Aboitiz) DISPOSITION Petition is DENIED. Manalang v. Artex Development Co., Inc. 21 SCRA 561 (1967) 2. Grievance Procedure – 260, 255 – Dispute Settlement: Issues and Individual Grievance Art. 260. Grievance machinery and voluntary arbitration. The parties to a Collective Bargaining Agreement shall include therein provisions that will ensure the mutual observance of its terms and conditions. They shall establish a machinery for the adjustment and resolution of grievances arising from the interpretation or implementation of their Collective Bargaining Agreement and those arising from the interpretation or enforcement of company personnel policies. All grievances submitted to the grievance machinery which are not settled within seven (7) calendar days from the date of its submission shall automatically be referred to voluntary arbitration prescribed in the Collective Bargaining Agreement. For this purpose, parties to a Collective Bargaining Agreement shall name and designate in advance a Voluntary Arbitrator or panel of Voluntary Arbitrators, or include in the agreement a procedure for the selection of such Voluntary Arbitrator or panel of Voluntary Arbitrators, preferably from the listing of qualified Voluntary Arbitrators duly accredited by the Board. In case the parties fail to select a Voluntary Arbitrator or panel of Voluntary Arbitrators, the Board shall designate the Voluntary Arbitrator or panel of Voluntary Arbitrators, as may be necessary, pursuant to the selection procedure agreed upon in the Collective Bargaining Agreement, which shall act with the same force and effect as if the Arbitrator or panel of Arbitrators has been selected by the parties as described above. Art. 255. Exclusive bargaining representation and workers’ participation in policy and decision-making. The labor organization designated or selected by the majority of the employees in an appropriate collective bargaining unit shall be the exclusive representative of the employees in such unit for the purpose of collective bargaining. However, an individual employee or group of employees shall have the right at any time to present grievances to their employer. Any provision of law to the contrary notwithstanding, workers shall have the right, subject to such rules and regulations as the Secretary of Labor and Employment may promulgate, to participate in policy and decision-making processes of the establishment where they are employed insofar as said processes will directly affect their rights, benefits and welfare. For this purpose, workers and employers may form labor-management councils: Provided, That the representatives of the workers in such labor-management councils shall be elected by at least the majority of all employees in said establishment. (As amended by Section 22, Republic Act No. 6715, March 21, 1989) Master Iron Labor Union v. NLRC, 219 SCRA 47 (1993) NATURE Petition for review on certiorari to annul and set aside decision of NLRC FACTS - Master Iron (MI) and its labor union (MILU) entered into a CBA in Feb 1987. It provided that “there shall be no strike and no lookout, stoppage or shutdown of work, or any other interference with any of the operation of MI during the term of this CBA, unless allowed and permitted by law.” - Right after signing the CBA, MI subcontracted outside workers to do the usual jobs done by its regular workers including those done outside of the company plant. Thus, regular workers were scheduled by management to work on a rotation basis allegedly to prevent financial losses thereby allowing the workers only 10 working days/month. MILU requested implementation of the grievance procedure which had also been agreed upon in the CBA, but MI ignored the request. - MILU filed a notice of strike, and upon intervention of DOLE, it was agreed upon that MI would give back the usual work to its regular employees who were members of MILU - Notwithstanding the agreement, MI continued the practice of hiring outside workers. MILU again went on a strike. MI sought to have the strike declared illegal while MILU filed complaint for unfair labor practice. - LA and NLRC decided for MI (strike was illegal for failure to exhaust the provision in the CBA on grievance procedure), hence this petition ISSUE 1. WON the strike was illegal for failure to exhaust grievance procedure HELD 1. NO Ratio MI’s refusal to heed MILU’s request to undergo the grievance procedure clearly demonstrated its lack of intent to abide by the terms of the CBA, thus committing an unfair labor practice Reasoning MI’s failure to traverse MILU’s allegation that NLRC abused its discretion in holding that the provision on grievance procedure had not been exhausted clearly sustains such allegation and upholds the MILU’s contention that MI refused to undergo said procedure. It should be remembered that a grievance procedure is 134 | P a g e

part of the continuous process of collective bargaining. It is intended to promote a friendly dialogue between labor and management as a means of maintaining industrial peace. On nature of the strike: MILU contend that notwithstanding the CBA’s non-strike provision, the strike was legal because the reasons therefor are non-economic in nature. On the other hand, in holding that the strike was illegal, NLRC relied solely on the no-strike no-lockout provision of the CBA. As the SC has held, a no-strike clause in a CBA is applicable only to economic strikes. Corollarily, if the strike is founded on an unfair labor practice of the employer, a strike declared by the union cannot be considered a violation of the no-strike clause. - An economic strike is defined as one which is to force wage or other concessions from the employer which he is not required by law to grant. Here, MILU enumerated in their notice of strike the ff grounds: violation of CBA or MI’s practice of subcontracting workers; discrimination; coercion of employees; unreasonable suspension of union officials, and unreasonable refusal to entertain grievance. MILU is not asking for an economic benefit not already agreed upon, but is merely asking for the implementation of the same. Prof. Perfecto Fernandez: economic strike involves issues relating to demands for higher wages, higher pension or overtime rates, pensions, profit sharing, shorter working hours, fewer work days for the same pay, elimination of night work, lower retirement age, more healthful working conditions, better health services, sanitation and more safety appliances. The demands were within the power of MI to grant and therefore the strike was not an economic strike. Disposition Petition is granted. Philippine Airlines, Inc. v. Santos, 218 SCRA 415 (1993) Facts Respondents are Port Stewards of Catering Sub-Department, Passenger Services Department of PAL who have the following duties and responsibilities:Prepare meal orders and check-lists, setting up standard equipment in accordance with the requirements of the type of service for each flight; skiing, binning and inventorying of Commissary supplies and equipment. On various occasions, several deductions were made from their salary representing losses of inventoried items charged to them for mishandling of company properties. Respondents, represented by the union, made a formal notice regarding the deductions to petitioner thru Mr. Reynaldo Abad, Manager for Catering. PAL did not act on it thus respondents filed a formal grievance pursuant to the grievance machinery Step I of the CBA regarding the illegal/questionable salary deductions and inventory of bonded goods and merchandise being done by catering service personnel which they believed should not be their duty. The grievance was submitted to the office of Mr. Reynaldo Abad who at the time was on vacation leave. The grievants thru the shop steward wrote a letter on December 5, 1984 addressed to the office of Mr. Abad, who was still on leave at the time, that inasmuch as no reply was made to their grievance which 'was duly received by your secretary' and considering that petitioner had only five days to resolve the grievance as provided for in the CBA, Section 2, Article IV of the PAL-PALEA Collective Bargaining Agreement (hereinafter, CBA), to wit: "Section 2-Processing of Grievances xxx STEP 1-Any employee who believes that he has a justifiable grievance shall take the matter up with his shop steward. If the shop steward feels there is justification for taking the matter up with the Company, be shall record the grievance on the grievance form heretofore agreed upon by the parties. Two (2) copies of the grievance form properly filled, accepted, and signed shall then be presented to and discussed by the shop steward with the division head. The division head shall answer the grievance within five (5) days from the date of presentation by inserting his decision on the grievance form, signing and dating same, and returning one copy to the shop steward. If the division head fails to act within the five (5)-day regl(e)mentary period, the grievance must be resolved in favor of the aggrieved party. If the division head's decision is not appealed to Step 11, the grievance shall be considered settled on the basis of the decision made, and shall not be eligible for further appeal -said grievance as believed by them was deemed resolved in their favor.When Mr Abad returned he immediately scheduled a meeting with the grievants. Thereafter, the individual respondents refused to conduct inventory works. Mr. Abad resolved the grievance by denying the petition of individual respondents and adopted the position that inventory of bonded good is part of their duty as catering service personnel and that it was only proper that employees are charged for the amount due to mishandling of company property which resulted to losses. They were also suspended for not conducting inventory work. Held It is clear that the grievance was filed with Mr. Abad's secretary during his absence. Under Section 2 of the CBA aforequoted, the division head shall act on the grievance within five (5) days from the date of presentation thereof, otherwise "the grievance must be resolved in favor of the aggrieved party." It is not disputed that the grievants knew that division head Reynaldo Abad was then "on leave" when they filed their grievance which was received by Abad's secretary.This knowledge, however, should not prevent the application of the CBA. On this score, respondent NLRC aptly ruled: "x x x Based on the facts heretofore narrated, division head Reynaldo Abad had to act on the grievance of complainants within five days from 21 November 1984. Therefore, when Reynaldo Abad failed to act within the reglementary period, complainants, believing in good faith that the effect of the CBA had already set in, cannot be blamed if they did not conduct ramp inventory for the days thereafter xxx it is hard to believe that everything under Abad's authority would have to stand still during his absence from office. To be sure, it is to be expected that someone has to be left to attend to Abad's duties. 135 | P a g e

As respondent NLRC has pointed out, Abad's failure to act on the matter may have been due to petitioner's inadvertance, but it is clearly too much of an injustice if the employees be made to bear the dire effects thereof. Much as the latter were willing to discuss their grievance with their employer, the latter closed the door to this possibility by not assigning someone else to look into "the matter during Abad's absence. Thus, private respondents should not be faulted for believing that the effect of the C13A in their favor had already stepped into the controversy. Disposition petition denied, assailed decision of NLRC is affirmed Caltex Refinery Employees Association v. Brillantes, 279 SCRA 218 (1997) FACTS • Anticipating the expiration of their Collective Bargaining Agreement on July 31, 1995, petitioner CALTEX EMPLOYEES ASSOC and CALTEX PHIL. negotiated the terms and conditions of employment to be contained in a new CBA. The negotiation between the two parties was participated in by the National Conciliation and Mediation Board (NCMB) and the Office of the Secretary of Labor and Employment. Some items in the new CBA were amicably arrived at and agreed upon, but others were unresolved. • To settle the unresolved issues, eight meetings between the parties were conducted. Because the parties failed to reach any significant progress in these meetings, petitioner declared a deadlock. On July 24, 1995, petitioner filed a notice of strike. Six (6) conciliation meetings conducted by the NCMB failed to settle the parties' differences. Then, the parties held marathon meetings at the plant level, but this remedy proved also unavailing. • During a strike vote on August 16, 1995, the members of petitioner opted for a walkout. Private respondent then filed with the Department of Labor and Employment (DOLE) a petition for assumption of jurisdiction in accordance with Article 263 (g) of the Labor Code. Sec. Brillantes assumed jurisdiction. He enjoined any strike or lockout, whether actual or intended. The parties were further directed to cease and desist from committing any and all acts which might exacerbate the situation. • Several issues on benefits were raised. One of the issues which stood out however was the grievance procedure of the parties under the CBA. The Secretary ordered that the periods to process/resolve grievances based on existing practice be reduced from (45) days to (30) days at the first step and (10) days to seven (7) days at the second step which is the level of the VP for Manufacturing. The Secretary further reviewed the steps through which a grievance may be processed and in line with the principle to expedite the early resolution of grievances, and found that the establishment of a joint Council as an additional step in the grievance procedure, may only serve to protract the proceeding and, therefore, no longer necessary. Instead, the unresolved grievance, if, not settled within (7) days at the level of the VP for Manufacturing, shall automatically be referred by both parties to voluntary arbitration in accordance with R.A. 6715. • As to the number of Arbitrators for which the Union proposes to employ only one instead of a panel of three Arbitrators, the Secretary deemed it best to leave the matter to the agreement of both parties. Finally, the Secretary advised the parties that the list of accredited voluntary arbitrators is now being maintained and disseminated by the National Conciliation and Mediation Board and no longer by the Bureau of Labor Relations. ISSUE WON the Honorable Secretary committed grave abuse of discretion in modifying the grievance machinery HELD NO. • No particular setup for a grievance machinery is mandated by law. Rather, Article 260 of the Labor Code, as incorporated by RA 6715, provides for only a single grievance machinery in the company to settle problems arising from "interpretation or implementation of their collective bargaining agreement and those arising from the interpretation or enforcement of company personnel policies." • Article 260, as amended, reads: Grievance Machinery and Voluntary Arbitration. The parties to a Collective Bargaining Agreement shall include therein provisions that will ensure the mutual observance of its terms and conditions. They shall establish a machinery for the adjustment and resolution of grievances arising from the interpretation or implementation of their Collective Bargaining Agreement and those arising from the interpretation or enforcement of company personnel policies. All grievances submitted to the grievance machinery which are not settled within seven (7) calendar days from the date of its submission shall automatically be referred to voluntary arbitration prescribed in the Collective Bargaining Agreement. For this purpose, parties to a Collective Bargaining Agreement shall name and designate in advance a Voluntary Arbitrator or panel of Voluntary Arbitrators, or include in the agreement a procedure for the selection of such Voluntary Arbitrator or panel of Voluntary Arbitrators, preferably from the listing of qualified Voluntary Arbitrators duly accredited by the Board. In case the parties fail to select a Voluntary Arbitrator or panel of Voluntary Arbitrators, the Board shall designate the Voluntary Arbitrator or panel of Voluntary Arbitrators, as may be necessary, pursuant to the selection procedure agreed upon in the Collective Bargaining Agreement, which shall act with same force and effect as if the Arbitrator or panel of Arbitrators has been selected by the parties as described above. We believe that the procedure described by public respondent sufficiently complies with the minimum requirement of the law. Public respondent even provided for two steps in hearing grievances prior to their referral to arbitration. The parties will decide on the number of arbitrators who may hear a dispute only when the need for it arises. Even the law itself does not specify the number of arbitrators. Their alternatives whether to have one or three arbitrators have their respective advantages and disadvantages. In this matter, cost is not the only consideration; full deliberation on the issues is another, and it is best accomplished in a hearing conducted by three arbitrators. In effect, the parties are afforded the latitude to decide for themselves the composition of the grievance machinery as they find appropriate to a particular situation. At bottom, we cannot really impute grave abuse of discretion to public respondent on this issue. 3. Individual Worker and Contract J.L. Case Co. 321 US 332, 64 Sup. Ct. 576 99 L. Ed. (1944) Library File 136 | P a g e

FACTS: • JI offered its employees individual contracts of employment which were uniform for all and for a term of one year. The terms and conditions of employment were predetermined in such contracts. 75% of the employees accepted these agreements. • The execution of the contracts was not a condition for employment. Moreover, these contracts were voluntarily entered into by the employees. • While these contracts were in effect CIO (union) filed a petition for certification as the exclusive bargaining agent of the production and maintenance employees. • JI opposed on the ground that the individual contracts constitute a bar to representation proceedings. • The Board directed the holding of an election which was won by the CIO. • CIO asked JI to bargain. JI agreed to bargain only as to matters not already touched upon by the contracts. It said it would bargain as to all matters only after the expiration of the contracts. • The Board held that JI violated the National Labor Relations Act when it refused to bargain. • JI appealed. During the pendency of the appeal, the contracts expired. The court, however, declared that the case was not yet moot. ISSUE: WON the individual contracts bar the employer from bargaining/negotiating the terms of employment? HELD/RATIO: NO. 1. Difference between individual and collective agreement The negotiation between the management and the union DOES NOT result in a contract of employment BUT in a trade agreement. After the collective agreement is made, those who will be benefited thereby shall be determined by the individual hirings. [hence, it is distinct from the employment contract]. The employee, in a Collective Trade Agreement [CBA], becomes a third party beneficiary to all the benefits of the collective trade agreement even if he, on his own, would have yielded to less favorable terms. 2. The individual contract does not waive the benefits under the CBA The CBA being entered into to serve the purpose of the National Labor Relations Act [analogous to the Labor Code], individual contracts cannot be effective as waiver of any benefit to which the employee would otherwise be entitled to under the Agreement. The benefits and advantages are open to every employee of the represented unit, whatever type or terms of the pre=existing contract of employment. Individual hiring contracts do not waive the benefits under the CBA, the former being subsidiary to the latter. Individual contracts may not be availed of to defeat or delay the procedures prescribed by the National Labor Relations Act looking to collective bargaining, nor to exclude the contracting employee from a duly ascertained collective bargaining unit, nor to forestall bargaining or to limit or condition the terms of the collective agreement. While it is true that individual contracts may be more advantageous than the terms in the Agreement, however, advantages to individuals may prove as disruptive to industrial peace as disadvantages. These may be considered interference with the organization and choice of representatives. Eg. Increased compensation given to certain individuals is often earned at the cost of breaking down some other standards thought to be for the welfare of the group. It is also true that there may be benefits provided in the individual contracts which are not covered in the statutory scope of the CBA, and the CBA does not prevent the individual from entering into such agreements. However, in so doing the employer cannot diminish his obligation as to matters under the CBA. 4. Contract Infirmity Associated Labor Union v. Calleja, 173 SCRA 179 (1989) FACTS: • May 7, 1986 – Associated Labor Union (ALU) informed GAW Trading that the majority of its employees have authorized ALU to be their sole and exclusive bargaining representative. It likewise requested for the execution of a CBA.. • May 9, 1986 – Southern Phils. Federation of Labor (SPFL) and Nagkahiusang Mamumuo sa GAW (NAMGAW) undertook a strike when GAW did not accede to their own demands. • May 12, 1986 – GAW recognized ALU as exclusive bargaining representative. A conference for the execution of CBA was thereafter set on the same date. • May 13, 1986 – ALU presented copies of the proposed CBA for comment or signing. • May 15 1986 – the CBA was signed by ALU and GAW • May 19, 1986 – GAW Lumad Labor Union (GALLU) filed a Petition for Certification Election which was dismissed for non-compliance with the subscription requirement. • May 27, 1986 – CBA between ALU and GAW was filed with the Ministry of Labor. • May 28, 1986 – SPFL filed a Petition for Direct Recognition where GALLU likewise participated as an intervenor. • Notwithstanding, the Med Arbiter ordered the holding of a Certification Election. ALU appealed. BLR Director Trajano reversed the Med Arbiter on the ground that the CBA had been effective and the contract bar rule was applicable. 137 | P a g e

BLR Director Calleja, who succeeded to the position, reversed and ordered the holding of a Certification Election on the following grounds: (1) Contract Bar Rule does not apply since the CBA was defective; (2) No proof that CBA had been posted in at least 2 conspicuous places in the establishment at least five days before its ratification and that it had been ratified by the majority of the employees in the bargaining unit. ISSUE: WON the CBA was valid such that it would constitute the bar to the holding of a Certification Election under the Contract Bar Rule? HELD/RATIO: No. It suffers from infirmities: 1. There was failure to post the CBA in at least 2 conspicuous places places in the establishment at least five days before its ratification. Petitioner's rationalization that the failure was due to the illegal strike staged by SPFL in all the stores of GAW Trading, Inc. which made it impossible to comply with the posting requirement in so far as the realization of its purpose is concerned as there were no impartial members of the unit who could be apprised of the CBA's contents is untenable. The posting of the CBA is the employer’s responsibility which requires a mere mechanical act. That there were no impartial members is immaterial because precisely the purpose of posting is to inform the employees of the contents of the CBA such that they can intelligently decide whether or not to ratify it, regardless of whether they already have their own opinion regarding the same. The assembly of the members of ALU wherein the agreement in question was allegedly explained does not cure the defect. The contract is intended for all the employees and not only for the members of the purported representative alone. It may even be said that the need to inform the non-members of the terms thereof is more exigent and compelling since, in all likelihood, their contact with the persons who are supposed to represent them is limited. This requirement becomes all the more important in this case where there was an apparent and suspicious hurry in the formulation and finalization of said CBA. In the aforementioned letter where GAW required petitioner union to present proof of its support by the employees, the company already suggested that petitioner ALU at the same time submit the proposals that it intended to embody in the projected agreement. This was on May 12, 1986, and promptly on the following day the negotiating panel furnished respondent company final copies of the desired agreement which, with equal dispatch, was signed on May 15, 1986. 2. Another potent reason for annulling the disputed collective bargaining agreement is the finding of respondent director that one hundred eighty-one (181) of the two hundred eighty-one (281) workers who "ratified" the same now "strongly and vehemently deny and/or repudiate the alleged negotiation and ratification of the CBA.” 5. Contract Duration and Renewals – 253-A Art. 253-A. Terms of a collective bargaining agreement. Any Collective Bargaining Agreement that the parties may enter into shall, insofar as the representation aspect is concerned, be for a term of five (5) years. No petition questioning the majority status of the incumbent bargaining agent shall be entertained and no certification election shall be conducted by the Department of Labor and Employment outside of the sixty-day period immediately before the date of expiry of such fiveyear term of the Collective Bargaining Agreement. All other provisions of the Collective Bargaining Agreement shall be renegotiated not later than three (3) years after its execution. Any agreement on such other provisions of the Collective Bargaining Agreement entered into within six (6) months from the date of expiry of the term of such other provisions as fixed in such Collective Bargaining Agreement, shall retroact to the day immediately following such date. If any such agreement is entered into beyond six months, the parties shall agree on the duration of retroactivity thereof. In case of a deadlock in the renegotiation of the Collective Bargaining Agreement, the parties may exercise their rights under this Code. (As amended by Section 21, Republic Act No. 6715, March 21, 1989) Manila Electric Co. v. Quisumbing, 302 SCRA 173 (1999) Facts:  An arbitral award has been granted by the Secretary of Labor to the MERALCO Union. Petition had its origin in the renegotiation of the parties' 1992-1997 CBA, insofar as the last two-year period thereof is concerned. A letter from MERALCO’s Chairman of the Board and its President addressed to their stockholders, which states that the CBA "for the rank-and-file employees covering the period December 1, 1995 to November 30, 1997 is still with the Supreme Court," as indicative of petitioner's recognition that the CBA award covers the said period. Issue: The parties dispute the reckoning period when retroaction of the arbitral awards by the Secretary of Labor should commence. Held: The period is herein set at two (2) years from 01 December 1995 to 30 November 1997. The arbitral award can be considered as an approximation of a CBA which would otherwise have been entered into by the parties. The terms or periods set forth in Article 253-A pertains explicitly to a CBA. But there is nothing that would prevent its application by analogy to an arbitral award by the Secretary considering the absence of an applicable law. Despite the silence of the law, the Court ruled that CBA arbitral awards granted after six months from the expiration of the last CBA shall retroact to such time agreed upon by both employer and the employees or their union. If there is no such agreement, the award shall retroact to the first day after the six months following the last day of the CBA. In the absence of a CBA, the Secretary’s determination of the retroactivity date shall control.   138 | P a g e



Rule RE CBA retroactivity: (Art. 253-A) If a CBA is renewed within 6 months from its expiry, it will be retroactive from the day immediately following the expiry of the original CBA. For instance, a CBA expires on December 31 and its renegotiation is finished within 6 months, then the renewed CBA dates back to January 01. If there is no new CBA concluded within 6 months, then there will be no automatic retroaction; both the retroaction (if any) and the effectivity date of the new CBA will be left to the parties to agree on. Manila Central Line Corporation v. Manila Central Line Free Workers Union, 290 SCRA 690 (2000) Facts:  Bargaining agreement had expired on March 15, 1989. As the parties failed to reach a new agreement, the Union sought the aid of the National Conciliation and Mediation Board on October 30, 1989, but the deadlock remained unresolved.  Parties submitted the case for compulsory arbitration. Issues: 1. 2. Held: 1. WON the Labor Arbiter had authority to act as voluntary arbitrator. Disputed effectivity date of the new CBA.

Manila Central Line Corporation must be deemed to be estopped from questioning the authority of Labor Arbiter Donato G. Quinto, Jr. to act as voluntary arbitrator and render a decision in this case. Petitioner agreed, together with the union, to refer their dispute for arbitration to him. 2. The CBA in this case is part of an arbitral award. As such, it may be made retroactive to the date of expiration of the previous agreement. In the absence of a specific provision of law prohibiting retroactivity of the effectivity of arbitral awards issued by the Secretary of Labor pursuant to Article 263(g) of the Labor Code, public respondent is deemed vested with plenary and discretionary powers to determine the effectivity thereof. Rule RE CBA retroactivity: (Art. 253-A) If a CBA is renewed within 6 months from its expiry, it will be retroactive from the day immediately following the expiry of the original CBA. For instance, a CBA expires on December 31 and its renegotiation is finished within 6 months, then the renewed CBA dates back to January 01. If there is no new CBA concluded within 6 months, then there will be no automatic retroaction; both the retroaction (if any) and the effectivity date of the new CBA will be left to the parties to agree on. Rivera v. Espiritu, 374 SCRA 351 (2002) supra 6. CBA and 3rd Party Applicability Sundowner Development Corporation v. Drilon, 180 SCRA 14 (1989) NATURE Petition for certiorari to review the orders of the Secretary of the Department of Labor and Employment FACTS - Hotel Mabuhay, Inc. leased the premises belonging to Santiago Syjuco, Inc. in Ermita, Manila. - Due to non-payment of rentals, a case for ejectment was filed by Syjuco against Mabuhay in the Metropolitan Trial Court of Manila. Mabuhay offered to amicably settle the case by surrendering the premises to Syjuco and to sell its assets and personal property to any interested party. - Syjuco offered the said premises for lease to Sundowner. - April 16, 1987 – The lease agreement was finalized and was agreed to commence on May 1, 1987 and to expire on April 30, 1992. - May 4, 1987 - National Union of Workers in Hotel, Restaurant and Allied Services (NUWHRAIN for short) picketed the leased premises, barricaded the entrance to the leased premises and denied petitioner's officers, employees and guests free access to and egress from said premises. This prompted Sundowner to write a letter of complaint to Syjuco. - May 7, 1987 - A complaint for damages with preliminary injunction and/or temporary restraining order was filed by Sundowner. The Executive Judge of the court issued a restraining order against respondent NUWHRAIN and its officers and members. NUWHRAIN nevertheless maintained their strike but filed an answer to the complaint. - May 14, 1987 - An order was issued by public respondent Secretary of Labor assuming jurisdiction over the labor dispute pursuant to Article 263(g) of the Labor Code. It required the 91 striking employees to return to work and for Mabuhay to accept all returning employees pending final determination of the issue of the absorption of the former employees of Mabuhay. - Mabuhay submitted its position paper alleging: - That it had sold all its assets and personal properties to Sundowner and that there was no sale or transfer of its shares whatsoever. - Mabuhay completely ceased operation effective April 28,1987 and surrendered the premises to Sundowner so that there exists a legal and physical impossibility on its part to comply with the return to work order specifically on absorption.

139 | P a g e

- June 26, 1987 - In order to commence its operation, Sundowner signed a tripartite agreement so the workers may lift their strike. In this agreement among Sundowner, NUWHRAIN and Mabuhay, the latter paid to respondent NUWHRAIN the sum of P638,000.00 in addition to the first payment in the sum of P386,447.11, for which reason respondent NUWHRAIN agreed to lift the picket. - July 13, 1987 - NUWHRAIN filed its position paper alleging connivance between Mabuhay and Sundowner in selling the assets and closing the hotel to escape its obligations to the employees of Mabuhay. NUWHRAIN prays that petitioner accept the workforce of Mabuhay and pay backwages from April 16, 1986 to April 28, 1987, the day Mabuhay stopped operation. - January 20, 1988 – Drilon, as DOLE secretary, issued an order requiring Sundowner to absorb the members of the union and to pay backwages from the time it started operations up to the date of the order. - January 27, 1988 – Sundowner filed a motion for reconsideration of the order, alleging that the theory of implied acceptance and assumption of statutory wrong does not apply in the instant case and that there is no law requiring bona fide purchasers of the assets of an on-going concern to absorb in its employ the employees of the latter. - Drilon denied the MFR. ISSUE WON the purchaser of the assets of an employer corporation can be considered a successor employer of the latter's employees HELD NO Ratio The rule is that unless expressly assumed, labor contracts such as employment contracts and collective bargaining agreements are not enforceable against a transferee of an enterprise, labor contracts being in personam, thus binding only between the parties. Reasoning - As a general rule, there is no law requiring a bona fide purchaser of assets of an on-going concern to absorb in its employ the employees of the latter. - However, although the purchaser of the assets or enterprise is not legally bound to absorb in its employ the employers of the seller of such assets or enterprise, the parties are liable to the employees if the transaction between the parties is colored or clothed with bad faith. - In the case at bar, contrary to the claim of the public respondent that the transaction between petitioner and Mabuhay was attended with bad faith, the court finds no cogent basis for such contention. Thus, the absorption of the employees of Mabuhay may not be imposed on petitioner. - It is undisputed that when Mabuhay surrendered the leased premises to Syjuco and asked Syjuco to offer same to other lessees, it was Syjuco who found petitioner and persuaded petitioner to lease said premises. Mabuhay had nothing to do with the negotiation and consummation of the lease contract between petitioner and Syjuco. - In the tri-partite agreement that was entered into by petitioner with respondents NUWHRAIN and Mabuhay, it is clearly stipulated that immediately after the execution of the agreement, Mabuhay shall give a list of its members to Sundowner that it desires to recommend for employment so that the latter can consider them for employment, with no commitment whatsoever on the part of Sundowner to hire them in the business that it will operate in the premises formerly occupied by the Hotel Mabuhay. - There can be no implied acceptance of the employees of Mabuhay by petitioner and acceptance of statutory wrong as it is expressly provided in the agreement that petitioner has no commitment or duty to absorb them. - The court does not subscribe to the theory of Drilon that petitioner should have informed NUWHRAIN of its lease of the premises and its purchase of the assets and personal properties of Mabuhay so that said employees could have taken steps to protect their interest. The court finds no such duty on the part of petitioner and its failure to notify said employees cannot be an indicium of bad faith. - While it is true that petitioner is using the leased property for the same type of business as that of respondent Mabuhay, there can be no continuity of the business operations of the predecessor employer by the successor employer as respondent Mabuhay had not retained control of the business. Disposition Petition granted. Orders reversed and set aside. Manlimos v. NLRC, 242 SCRA 145 (1995) FACTS - The petitioners were among the regular employees of the Super Mahogany Plywood Corporation - A new owner/management group acquired complete ownership of the corporation. The petitioners were advised of such change of ownership; however, the petitioners continued to work for the new owner and were considered terminated, with their conformity, only when they received their separation pay, 13th month pay, and all other benefits due them. - Each of them then executed a Release and Waiver which they acknowledged before the Hearing Officer of the Butuan City District Office of the DOLE - The new owner caused the publication of a notice for the hiring of workers, indicating therein who of the separated employees could be accepted on probationary basis. The petitioners then filed their applications for employment. Except for Rosario Cuarto, they were hired on probationary basis for six months as patchers or tapers, but were compensated on piece-rate or task basis. - For their alleged absence without leave, Perla Cumpay and Virginia Etic were considered, to have abandoned their work. The rest were dismissed because they allegedly committed acts prejudicial to the interest of the new management. - Petitioners then filed against the respondent a complaint. - The petitioners maintained that they remained regular employees regardless of the change of management and their execution of the Release and Waiver. 140 | P a g e

- Respondent contended that the petitioners were deemed legally terminated from their previous employment; that the new owner was well within its legal right or prerogative in considering as terminated the petitioners' probationary/temporary appointment; and that the petitioners were not illegally dismissed; hence, they are not entitled to the reliefs prayed for. - Labor Arbiter ruled for the petitioners. The Labor Arbiter, however, ruled that there was no "cessation of operations which would lead to the dismissal of the employees." - Respondent appealed to the NLRC which reversed the judgment of the Labor Arbiter, subject, however, to recomputation based on the actual services of the petitioners under the new owner up to the actual date of their separation from the service. It found that the change of ownership in this case was made in good faith since there was no evidence on record that "the former owners conspired with the new owners to insulate the former management of any liability to its workers." - Their motion to reconsider the resolution having been denied by the NLRC, the petitioners filed a special civil action for certiorari. ISSUE WON the NLRC acted with grave abuse of discretion when it reversed the decision of the Labor Arbiter. HELD NO. - There was only a change of ownership of Super Mahogany Plywood Corporation which resulted in a change of ownership. In short, the corporation itself, as a distinct and separate juridical entity, continues to exist. The issue of whether there was a closing or cessation of business operations which could have operated as just cause for the termination of employment was not material. - The change in ownership of the management was done bona fide and the petitioners did not for any moment before the filing of their complaints raise any doubt on the motive for the change. On the contrary, upon being informed thereof and of their eventual termination from employment, they freely and voluntarily accepted their separation pay and other benefits and individually executed the Release or Waiver which they acknowledged before no less than a hearing officer of the DOLE. - A change of ownership in a business concern is not proscribed by law. - Central Azaucarera del Danao vs. CA: It is a principle well-recognized, that it is within the employer's legitimate sphere of management control of the business to adopt economic policies or make some changes or adjustments in their organization or operations that would insure profit to itself or protect the investment of its stockholders. As in the exercise of such management prerogative, the employer may merge or consolidate its business with another, or sell or dispose all or substantially all of its assets and properties which may bring about the dismissal or termination of its employees in the process. Such dismissal or termination should not however be interpreted in such a manner as to permit the employer to escape payment of termination pay. For such a situation is not envisioned in the law. It strikes at the very concept of social justice. - In a number of cases on this point, the rule has been laid down that the sale or disposition must be motivated by good faith as an element of exemption from liability. - Indeed, an innocent transferee of a business establishment has no liability to the employees of the transferor to continue employing them. Nor is the transferee liable for past unfair labor practices of the previous owner, except, when the liability therefor is assumed by the new employer under the contract of sale, or when liability arises because of the new owner's participation in thwarting or defeating the rights of the employees. - Where such transfer of ownership is in good faith, the transferee is under no legal duty to absorb the transferor's employees as there is no law compelling such absorption. The most that the transferee may do, for reasons of public policy and social justice, is to give preference to the qualified separated employees in the filling of vacancies in the facilities of the purchaser. - Since the petitioners were effectively separated from work due to a bona fide change of ownership and they were accordingly paid their separation pay, which they freely and voluntarily accepted, the private respondent corporation was under no obligation to employ them; it may, however, give them preference in the hiring. The private respondent in fact hired, but on probationary basis, all the petitioners, except Rosario Cuarto. The non-hiring of Cuarto was legally permissible. Disposition Petition waspartly GRANTED. The challenged resolutions of NLRC were MODIFIED; respondent was ordered to pay petitioners Perla Cumpay and Virginia Etic their backwages up to the expiration of their probationary employment contracts. 7. CBA and Disaffiliation – Substitution Doctrine Eliso-Elirol Labor Union v. Noriel, 80 SCRA 682 (1977) Facts: • In Feburary 1974, Elisco-Elirol Labor Union (Union) negotiated and concluded a CBA with Elizalde Steel Consolidated (Company). • Since the Union was not yet registered with the BLR, the Union members resolved to register their union to preserve the integrity of the CBA in its meeting on May 20, 1975. A Certificate of Registration was issued in favour of the Union on May 28, 1975. • The Union then sought the implementation of the CBA. On June 10, 1975, the Union members decided to disaffiliate from NAFLU. The Union president then wrote to the Company informing it of its disaffiliation from NAFLU and demanded that it be recognized as the sole and exclusive bargaining representative of the Company’s employees however the Company refused. • Thus, a petition was filed with the BLR to cite the Company as guilty of ULP and to prevent NAFLU from representing itself as the sole and exclusive bargaining representative of the employees of the Company. The Med-Arbiter dismissed the petition for lack of merit and was affirmed by the BLR Director. Issue: WON the Union is the sole and exclusive bargaining representative of the employees of the Company. Held: YES • The respondent Director erred when he ruled that when the Union members disaffiliated from NAFLU, their employment status with the Company was terminated. 141 | P a g e

• • • • •

The employees did not form a new union but merely registered the local union which was their right under the law. In a CBA, the local union is the principal party to the agreement while a “mother union” is merely an agent of the local union which remained the basic unit of association to act for the interests of the employees which includes the freedom to affiliate or disaffiliate. Such ruling was enunciated in the case of Liberty Cotton Mills Workers Union v. Liberty Cotton Mills, Inc. Further, the substitutionary doctrine fully supports the petitioner’s stand. Petitioner union to whom the employees owe their allegiance has from the beginning expressly avowed that it "does not intend to change and/or amend the provisions of the present collective bargaining agreement but only to be given the chance to enforce the same since there is a shift of allegiance in the majority of the employees at respondent company. Even during the effectivity of a collective bargaining agreement executed between employer and employees thru their agent, the employees can change said agent but the contract continues to bind then up to its expiration date. They may bargain however for the shortening of said expiration date. In formulating the "substitutionary" doctrine, the only consideration involved as the employees' interest in the existing bargaining agreement. The agent's interest never entered the picture.

8. CBA Assistance – DOLE – 267-268 Art. 267. Assistance by the Department of Labor. The Department of Labor, at the initiative of the Secretary of Labor, shall extend special assistance to the organization, for purposes of collective bargaining, of the most underprivileged workers who, for reasons of occupation, organizational structure or insufficient incomes, are not normally covered by major labor organizations or federations. Art. 268. Assistance by the Institute of Labor and Manpower Studies. The Institute of Labor and Manpower Studies shall render technical and other forms of assistance to labor organizations and employer organizations in the field of labor education, especially pertaining to collective bargaining, arbitration, labor standards and the Labor Code of the Philippines in general. 9. Effect Expiry – 253 Art. 253. Duty to bargain collectively when there exists a collective bargaining agreement. When there is a collective bargaining agreement, the duty to bargain collectively shall also mean that neither party shall terminate nor modify such agreement during its lifetime. However, either party can serve a written notice to terminate or modify the agreement at least sixty (60) days prior to its expiration date. It shall be the duty of both parties to keep the status quo and to continue in full force and effect the terms and conditions of the existing agreement during the 60-day period and/or until a new agreement is reached by the parties. Faculty Association of MIT v. CA, 523 SCRA 709 (2007) supra New Pacific Timber and Supply Co., Inc. v. NLRC, 328 SCRA 404 (2000) Facts: • National Federation of Labor (NFL) was the certified sole and exclusive bargaining representative of all rank and file employees of New Pacific Timber Supply (Company). They had an existing CBA which was to expire after 1984. • When NFL began to negotiate better terms for its members, the Company refused to negotiate prompting NFL to file a case against it for ULP. • The Executive Labor Arbiter Abdulwahid cited the Company as guilty of ULP in an Order dated March 31, 1987. He also declared the CBA proposals of NFL as the CBA between the parties. The Order of the Executive Labor Arbiter was upheld by the SC on appeal. • On October 18, 1993, Labor Arbiter Villena issued an Order implementing the Order of Executive Labor Arbiter Abdulwahid. However, its benefits were only extended to 142 employees of the Company who were enumerated in the Order. The case was deemed closed when NFL manifested that it would no longer appeal the Order. • On May 12, 1994, a Petition for Relief was filed by other rank and file employees of the Company demanding that they be accorded the benefits of the CBA. • The NLRC treated the Petition for Relief as an appeal of the Order of the Labor Arbiter. It ruled that the other employees were entitled to the benefits of the CBA. • The Company filed a petition with the SC claiming that the NLRC acted with grave abuse of discretion in treating the petition for relief as an appeal. It also claimed that the other employees were not entitled to the benefits under the CBA because employees hired after the term of the CBA are not parties ot the agreement and therefore cannot claim benefits thereunder. Issues: (1) WON the NLRC acted with grave abuse of discretion in treating the petition for relief as an appeal; (2) WON the term of an existing CBA can be extended beyond the period stipulated therein; and (3) WON the other rank and file employees are entitled to the benefits of the CBA. Held: 1st Issue: NO • We find no grave abuse of discretion on the part of the NLRC, when it entertained the petition for relief filed by the private respondents and treated it as an appeal. even if it was filed beyond the reglementary period for filing an appeal. • Ordinarily, once a judgment has become final and executory, it can no longer be disturbed, altered or modified. However, a careful scrutiny of the facts and circumstances of the instant case warrants liberality in the application of technical rules and procedure. private respondents, in their petition for relief, claimed that they were wrongfully excluded from the list of those entitled to the CBA benefits by their union, NFL, without their knowledge; and, because they were under the impression that they were ably represented, they were not able to appeal their case on time. 142 | P a g e

2nd Issue: YES • It is clear from Article 253 of the Labor Code that until a new Collective Bargaining Agreement has been executed by and between the parties, they are duty-bound to keep the status quo and to continue in full force and effect the terms and conditions of the existing agreement. The law does not provide for any exception nor qualification as to which of the economic provisions of the existing agreement are to retain force and effect; therefore, it must be understood as encompassing all the terms and conditions in the said agreement. • Articles 253 and 253-A mandate the parties to keep the status quo and to continue in full force and effect the terms and conditions of the existing agreement during the 60-day period prior to the expiration of the old CBA and/or until a new agreement is reached by the parties. Consequently, the automatic renewal clause provided for by the law, which is deemed incorporated in all CBA's, provides the reason why the new CBA can only be given a prospective effect. 3rd Issue: YES • When a collective bargaining contract is entered into by the union representing the employees and the employer, even the non-member employees are entitled to the benefits of the contract. To accord its benefits only to members of the union without any valid reason would constitute undue discrimination against nonmembers • In the same vein, the benefits under the CBA in the instant case should be extended to those employees who only became such after the year 1984. To exclude them would constitute undue discrimination and deprive them of monetary benefits they would otherwise be entitled to under a new collective bargaining contract to which they would have been parties. • Since in this particular case, no new agreement had been entered into after the CBA's stipulated term, it is only fair and just that the employees hired thereafter be included in the existing CBA. This is in consonance with our ruling that the terms and conditions of a collective bargaining agreement continue to have force and effect beyond the stipulated term when no new agreement is executed by and between the parties to avoid or prevent the situation where no collective bargaining agreement at all would govern between the employer company and its employees. Manila Electric v. Quisumbing, 302 SCRA 173 (1999) supra Citizens Labor Union v. CIR, 18 SCRA 624 (1966) FACTS: • Malayang Manggagawa sa Esso (MME) filed a petition for certificaton election with the CIR to determine which union will administer the contract during the remainder of the term of the contract. • The Citizens Labor Union (CLU) and Esso Standard Eastern, Inc. (ESSO) opposed the said petition, one of the grounds is that the existing collective bargaining contract is a bar to the holding of a certification election. • The CIR denied the motion to dismiss the petition, holding that the existing collective bargaining contract is no bar to a certification election, and requested the Department of Labor to conduct the necessary election. • The results of the election shows that the MME obtained votes of more than one-half of the rank and file employees and laborers of the ESSO Pandacan Terminal unit eligible to vote. • Thereafter, CLU and ESSO filed separate motions to annul the certification election on the ground that the election was held illegally and irregularly as it was conducted on a holiday, and that it was had without participation of the CLU therein. • The motions were denied and MME was certified as the sole and exclusive bargaining agent of all the non-supervisory employees of the ESSO at its Pandacan Terminal unit. • CLU filed an urgent petition praying for issuance of a writ of preliminary injunction and a petition for review. • Pending the adjudication of these cases, a "Motion for Preliminary Injunction" was granted, refraining CLU and ESSO from negotiating and concluding a new CBA. ON EFFECT EXPIRY: No breaks. It is a sound and unassailable labor practice for labor and management to conclude a new contract before the expiry date of any CBA in order to avoid a hiatus in management-labor relations. The Industrial Peace Act was designed primarily to promote industrial peace through encouragement of collective bargaining. Any undue delay in the selection of a bargaining representative can hardly be said to contribute to that end . PART SIX EMPLOYER-UNION ACTS OF UNFAIR LABOR PRACTICES-ACTS VIOLATIVE OF EMPLOYEE RIGHTS TO SELF-ORGANIZATION AND COLLECTIVE BARGAINING I. INTRODUCTORY CONCEPTS A. Definition and General Concept – 212 (k); 247; ILO Convention 98, Art. 1,2 143 | P a g e

Art. 212. Definitions. "Unfair labor practice" means any unfair labor practice as expressly defined by the Code. Art. 247. Concept of unfair labor practice and procedure for prosecution thereof. Unfair labor practices violate the constitutional right of workers and employees to self-organization, are inimical to the legitimate interests of both labor and management, including their right to bargain collectively and otherwise deal with each other in an atmosphere of freedom and mutual respect, disrupt industrial peace and hinder the promotion of healthy and stable labor-management relations. Consequently, unfair labor practices are not only violations of the civil rights of both labor and management but are also criminal offenses against the State which shall be subject to prosecution and punishment as herein provided. Subject to the exercise by the President or by the Secretary of Labor and Employment of the powers vested in them by Articles 263 and 264 of this Code, the civil aspects of all cases involving unfair labor practices, which may include claims for actual, moral, exemplary and other forms of damages, attorney’s fees and other affirmative relief, shall be under the jurisdiction of the Labor Arbiters. The Labor Arbiters shall give utmost priority to the hearing and resolution of all cases involving unfair labor practices. They shall resolve such cases within thirty (30) calendar days from the time they are submitted for decision. Recovery of civil liability in the administrative proceedings shall bar recovery under the Civil Code. No criminal prosecution under this Title may be instituted without a final judgment finding that an unfair labor practice was committed, having been first obtained in the preceding paragraph. During the pendency of such administrative proceeding, the running of the period of prescription of the criminal offense herein penalized shall be considered interrupted: Provided, however, that the final judgment in the administrative proceedings shall not be binding in the criminal case nor be considered as evidence of guilt but merely as proof of compliance of the requirements therein set forth. (As amended by Batas Pambansa Bilang 70, May 1, 1980 and later further amended by Section 19, Republic Act No. 6715, March 21, 1989) ILO Convention 98, Article 1 1. (1) Workers shall enjoy adequate protection against acts of anti-union discrimination in respect of their employment. 1. (2) Such protection shall apply more particularly in respect of acts calculated to(a) make the employment of a worker subject to the condition that he shall not join a union or shall relinquish trade union membership; (b) cause the dismissal of or otherwise prejudice a worker by reason of union membership or because of participation in union activities outside working hours or, with the consent of the employer, within working hours. Article 2 2. (1) Workers' and employers' organisations shall enjoy adequate protection against any acts of interference by each other or each other's agents or members in their establishment, functioning or administration. 2. (2) In particular, acts which are designed to promote the establishment of workers' organisations under the domination of employers or employers' organisations, or to support workers' organisations by financial or other means, with the object of placing such organisations under the control of employers or employers' organisations, shall be deemed to constitute acts of interference within the meaning of this Article. Definition Sterling Products International, Inc. v. Sol, 7 SCRA 446 (1963) Facts: • Loreta Sol charged Sterling Products International and its Radio Director V. San Pedro with having committed unfair labor practice in a complaint, alleging that she has been a regular Radio Monitor for them, and that she filed a complaint in January of 1960, against the firm for underpayment, money equivalent of her vacation leave from 1952 to 1959, and Christmas bonus for 1959, and that the complaint resulted in her dismissal without just cause in December 1960. • Sterling alleged that Sol is an independent contractor, and that they no longer required her services, for which they gave her a notice of termination. • Judge Tabigne of the Court of Industrial Relations ruled that complainant is not an employee but only an independent contractor, and that respondent firm was justified in dismissing her due to economic reasons. • Sol filed a motion for reconsideration, and the CIR reversed the earlier decision and ruled that Sol was an employee, ordered her reinstatement with back wages, and further ruled that Sterling was guilty of unfair labor practice, due to the following circumstances: (1) Complainant was given an identification card stating that "Bearer Loreta C. Sol is a bona fide employee of this Company;" (2) when she applied for purchase of a lot from the PHHC, she was given a certificate to show that she was indeed an employee of the respondent company for the last five years or six years; and (3) as such employee, she enjoyed the privilege of borrowing money from the Employees Loan Association of the firm. • The lower court found that the company's control over Sol's work is shown by the fact that can not listen to broadcasts other than those that were contained in the schedule given to her by the company, and that supervision and control of her work could be done by checking or verifying the contents of her reports on said broadcasts. • Sterling brought the case to the SC for certiorari Issues: − W/N Sol was an employee − W/N Sterling is guilty of unfair labor practice Held: − Sol was an employee. 144 | P a g e







She was directed to listen to certain broadcasts, directing her, in the instructions given her, when to listen and what to listen, petitioners herein naming the stations to be listened to, the hours of broadcasts, and the days when listening was to be done. Respondent Sol had to follow these directions. The mere fact that while performing the duties assigned to her she was not under the supervision of the petitioners does not render her a contractor, because what she has to do, the hours that she has to work and the report that she has to submit all — these are according to instructions given by the employer. It is not correct to say, therefore, that she was an independent contractor, for an independent contractor is one who does not receive instructions as to what to do, how to do, without specific instructions. − The very act of respondent Sol in demanding vacation leave, Christmas bonus and additional wages shows that she considered herself an employee. A contractor is not entitled to a vacation leave or to a bonus nor to a minimum wage. This act of hers in demanding these privileges are inconsistent with the claim that she was an independent contractor. No, Sterling did not commit unfair labor practice. − The term unfair labor practice has been defined as any of those acts listed in See. 4 of the Industrial Peace Act. The respondent Sol has never been found to commit any of the acts mentioned in paragraph (a) of Sec. 4. Respondent Sol was not connected with any labor organization, nor has she ever attempted to join a labor organization, or to assist, or contribute to a labor organization. The company cannot, therefore, be considered as having committed an unfair labor practice. The contract between the petitioners and the respondent Sol providing that the respondent Sol can be dismissed upon fifteen days' notice is null and void. Inasmuch as respondent Sol was employed since the year 1952 and was in the employment of the petitioners from that time up to 1959, or a period of seven years, she is entitled to three and one-half months pay in accordance with the above quoted section 1 of the Act.

Philcom Employees Union v. Philippine Global Communications, 495 SCRA 214 (2006) Nature Petition for Review of the decision of the CA Facts -Upon the expiration of the Collective Bargaining Agreement (CBA) between petitioner Philcom Employees Union (PEU or union) and private respondent Philippine Global Communications, Inc. (Philcom, Inc.) the parties started negotiations for the renewal of their CBA in July 1997. While negotiations were ongoing, PEU filed on October 21, 1997 with the National Conciliation and Mediation Board (NCMB) – National Capital Region, a Notice of Strike due to perceived unfair labor practice committed by the company. Company suspended negotiations on the CBA which moved the union to file for another Notice of Strike on the ground of bargaining deadlock. -At a conciliation conference held at the NCMB-NCR office, the parties agreed to consolidate the two (2) Notices of Strike filed by the union and to maintain the status quo during the pendency of the proceedings. While the union and the company officers and representatives were meeting, the remaining union officers and members staged a strike at the company premises, barricading the entrances and egresses thereof and setting up a stationary picket at the main entrance of the building. The following day, the company immediately filed a petition for the Secretary of Labor and Employment to assume jurisdiction over the labor dispute in accordance with Article 263(g) of the Labor Code. Acting Labor Secretary Cresenciano B. Trajano issued an Order assuming jurisdiction over the dispute, enjoining any strike or lockout, whether threatened or actual, directing the parties to cease and desist from committing any act that may exacerbate the situation, directing the striking workers to return to work within twenty-four (24) hours from receipt of the Secretary’s Order and for management to resume normal operations, as well as accept the workers back under the same terms and conditions prior to the strike. The parties were likewise required to submit their respective position papers and evidence within ten (10) days from receipt of said order. A second order was issued reiterating the previous directive to all striking employees to return to work immediately. The union filed a Motion for Reconsideration assailing, among others, the authority of then Acting Secretary Trajano to assume jurisdiction over the labor dispute. Said motion was denied in an Order dated January 7, 1998. -The Secretary denied both motions for reconsideration of Philcom and PEU. PEU filed with the SC a petition for certiorari and prohibition under Rule 65 of the Rules of Court assailing the Secretary’s Orders. SC referred the case to the CA. CA denied petition. The Court of Appeals ruled that, contrary to PEU’s view, the Secretary could take cognizance of an issue, even only incidental to the labor dispute, provided the issue must be involved in the labor dispute itself or otherwise submitted to him for resolution. Issue: WON the Labor Secretary could take cognizance of the case and WON PEU is liable for the illegal strike SC: Yes to both. The Secretary properly took cognizance of the issue on the legality of the strike. The very reason of the Secretary’s assumption of jurisdiction was PEU’s declaration of the strike, any issue regarding the strike is not merely incidental to, but is essentially involved in, the labor dispute itself. Article 263(g) of the Labor Code applies. The powers granted to the Secretary under Article 263(g) of the Labor Code have been characterized as an exercise of the police power of the State, with the aim of promoting public good. When the Secretary exercises these powers, he is granted “great breadth of discretion” in order to find a solution to a labor dispute. The most obvious of these powers is the automatic enjoining of an impending strike or lockout or its lifting if one has already taken place. The authority of the Secretary to assume jurisdiction over a labor dispute causing or likely to cause a strike or lockout in an industry indispensable to national interest includes and extends to all questions and controversies arising from such labor dispute. The power is plenary and discretionary in nature to enable him to effectively and efficiently dispose of the dispute. 145 | P a g e

-Unfair labor practice refers to acts that violate the workers’ right to organize. The prohibited acts are related to the workers’ right to selforganization and to the observance of a CBA. Without that element, the acts, no matter how unfair, are not unfair labor practices. The only exception is Article 248(f), which in any case is not one of the acts specified in PEU’s charge of unfair labor practice. -A review of the acts complained of as unfair labor practices of Philcom convinces us that they do not fall under any of the prohibited acts defined and enumerated in Article 248 of the Labor Code. The issues of misimplementation or non-implementation of employee benefits, non-payment of overtime and other monetary claims, inadequate transportation allowance, water, and other facilities, are all a matter of implementation or interpretation of the economic provisions of the CBA between Philcom and PEU subject to the grievance procedure. -PEU could not have validly anchored its defiance to the return-to-work orders on the motion for reconsideration that it had filed on the assumption of jurisdiction order. A return-to-work order is immediately effective and executory despite the filing of a motion for reconsideration. It must be strictly complied with even during the pendency of any petition questioning its validity. Philcom published in the Philippine Daily Inquirer a notice to striking employees to return to work. These employees did not report back to work but continued their mass action. A return-to-work order imposes a duty that must be discharged more than it confers a right that may be waived. While the workers may choose not to obey, they do so at the risk of severing their relationship with their employer The following provision of the Labor Code governs the effects of defying a return-to-work order: ART. 264. Prohibited activities. - (a) x x x x No strike or lockout shall be declared after assumption of jurisdiction by the President or the Minister or after certification or submission of the dispute to compulsory or voluntary arbitration or during the pendency of cases involving the same grounds for the strike or lockout x x x x Any union officer who knowingly participates in illegal strike and any worker or union officer who knowingly participates in the commission of illegal acts during a strike may be declared to have lost his employment status: Provided, That mere participation of a worker in a lawful strike, shall not constitute sufficient ground for termination of his employment, even if a replacement had been hired by the employer during such lawful strike. A strike undertaken despite the Secretary’s issuance of an assumption or certification order becomes a prohibited activity, and thus, illegal, under Article 264(a) of the Labor Code. The union officers who knowingly participate in the illegal strike are deemed to have lost their employment status. The union members, including union officers, who commit specific illegal acts or who knowingly defy a return-to-work order are also deemed to have lost their employment status. Otherwise, the workers will simply refuse to return to their work and cause a standstill in the company operations while retaining the positions they refuse to discharge and preventing management to fill up their positions. -Failure of PEU’s officers and members to comply immediately with the return-to-work orders dated 19 November and 28 November 1997 cannot be condoned. Defiance of the return-to-work orders of the Secretary constitutes a valid ground for dismissal. The right to strike, while constitutionally recognized, is not without legal constrictions. Article 264(e) of the Labor Code, on prohibited activities, provides: No person engaged in picketing shall commit any act of violence, coercion or intimidation or obstruct the free ingress to or egress from the employer’s premises for lawful purposes, or obstruct public thoroughfares. The Labor Code is emphatic against the use of violence, coercion, and intimidation during a strike and to this end prohibits the obstruction of free passage to and from the employer’s premises for lawful purposes. A picketing labor union has no right to prevent employees of another company from getting in and out of its rented premises, otherwise, it will be held liable for damages for its acts against an innocent by-stander. The sanction provided in Article 264(a) is so severe that any worker or union officer who knowingly participates in the commission of illegal acts during a strike may be declared to have lost his employment status. By insisting on staging the prohibited strike and defiantly picketing Philcom’s premises to prevent the resumption of company operations, the striking employees have forfeited their right to be readmitted. Section 6. Conciliation. - x x x xDuring the proceedings, the parties shall not do any act which may disrupt or impede the early settlement of dispute. They are obliged, as part of their duty, to bargain collectively in good faith, to participate fully and promptly in the conciliation meetings called by the regional branch of the Board. x x x x Article 264(a) of the Labor Code also considers it a prohibited activity to declare a strike “during the pendency of cases involving the same grounds for the same strike. Lamentably, PEU defiantly proceeded with their strike during the pendency of the conciliation proceedings. PEU staged the strike in utter disregard of the grievance procedure established in the CBA. PEU should have immediately resorted to the grievance machinery provided for in the CBA. In disregarding this procedure, the union leaders who knowingly participated in the strike have acted unreasonably. The law cannot interpose its hand to protect them from the consequences of their illegal acts. A strike declared on the basis of grievances which have not been submitted to the grievance committee as stipulated in the CBA of the parties is premature and illegal. Disposition Petition DISMISSED; CA’s decision is AFFIRMED with the MODIFICATION that the Secretary of Labor is directed to determine who among the Philcom Employees Union officers participated in the illegal strike, and who among the union members committed illegal acts or defied the return-to-work orders. Galaxie Steel Workers Union v. NLRC, 504 SCRA 692 (2006) 146 | P a g e

CARPIO MORALES; October 17, 2006 NATURE Petition for Review FACTS - Respondent Galaxie Steel Corporation (Galaxie) is a corporation engaged in the business of manufacturing and sale of re-bars and steel billets which are used primarily in the construction of high-rise buildings. On account of serious business losses which occurred in 1997 up to mid-1999 totaling around P127M, Galaxie decided to close down its business operations. - Galaxie filed a written notice with DOLE informing DOLE of its intended closure and the consequent termination of its employees effective August 31, 1999. And it posted the notice of closure on the corporate bulletin board. Petitioners Galaxie Steel Workers Union and Galaxie employees filed a complaint for illegal dismissal, unfair labor practice, and money claims against Galaxie. - The Labor Arbiter, declared valid Galaxie’s closure of business but nevertheless ordered it to pay petitioner-employees separation pay, pro-rata 13 th month pay, and vacation and sick leave credits. - NLRC upheld the LA’s decision but reversed the award of pro-rata 13 th month pay and vacation and sick leave credits, the same not being among petitioners’ causes of action as in fact they were not even mentioned in their pleadings. It reversed too the award for separation pay, the closure of Galaxie’s business being due to serious business losses. Nevertheless, the NLRC directed Galaxie to grant petitioners, by way of financial assistance, the same amount given to the employees who had executed quitclaims. MFR denied. - CA upheld the NLRC decision and accordingly denied petitioners’ petition for certiorari as it did their MFR. - Petitioners contend that (1) CA erred in not finding that Galaxie’s closure of business operations was motivated not by serious business losses but by their anti-union stance (2) Galaxie did not serve written notices of the closure of business operations upon its employees, it having merely posted a notice on the company bulletin board, hence, Galaxie should be liable for backwages; and (3) CA’s upholding of the deletion by the NLRC of separation pay is contrary to the ruling in Banco Filipino Savings and Mortgage Bank v. NLRC which held that separation pay is proper in cases where closure or cessation of business operations is due to serious business losses or financial reverses. ISSUES 1 WON Galaxie is guilty of unfair labor practice in closing its business operations shortly after petitioner union filed for certification election. 2 WON petitioners are entitled to separation pay. 3 WON the written notice posted by Galaxie on the company bulletin board sufficiently complies with the notice requirement under A283 of LC. HELD 1 NO - The Court is not a trier of facts, a rule which applies with greater force in labor cases where the findings of fact of NLRC are accorded respect and even finality, as long as they are supported by substantial evidence from which an independent evaluation of the facts may be made. The Labor Arbiter, the NLRC, and CA were unanimous in ruling that Galaxie’s closure or cessation of business operations was due to serious business losses or financial reverses, and NOT because of any alleged anti-union position. 2 NO - First. Galaxie’s documentary evidence shows that it had been experiencing serious financial losses at the time it closed business operation. It is supported by substantial evidence consisting of the audited financial statements - Second. (IMPT. for our purposes) Petitioners failed to present concrete evidence supporting their claim of unfair labor practice. Unfair labor practice refers to acts that violate the workers’ right to organize, and are defined in A248 and A261 of LC. The prohibited acts relate to the workers’ right to self-organization and to the observance of CBA without which relation the acts, no matter how unfair, are not deemed unfair labor practices. - In North Davao Mining Corp v. NLRC , it was held that A283 governs the grant of separation benefits "in case of closures or cessation of operation" of business establishments "NOT due to serious business losses or financial reverses" Where, the closure then is due to serious business losses, LC does not impose any obligation upon the employer to pay separation benefits. Thus, denial of petitioners’ claim for separation pay was proper. 3 NO - The mere posting on the company bulletin board does not, however, meet the requirement under A283 of "serving a written notice on the workers." The purpose of the written notice is to inform the employees of the specific date of termination or closure of business operations, and must be served upon them at least one month before the date of effectivity to give them sufficient time to make the necessary arrangements. In order to meet the foregoing purpose, service of the written notice must be made individually upon each and every employee of the company. This was not met. Dispositive CA decision AFFIRMED with the MODIFICATION. Galaxie Steel Corp is ORDERED to PAY each of the individual petitioners the amount of P20K as nominal damages for non-compliance with statutory due process. B. Requisite Relationship Required Relationship Phelps-Dodge Corporation v. NLRB, 313 U.S. 177 (1941) Library File

147 | P a g e

FACTS: The source of the controversy was a strike, begun on June 10, 1935, by the International Union of Mine, Mill and Smelter Workers at Phelps Dodge's Copper Queen Mine, Bisbee, Arizona. Picketing of the mine continued until August 24, 1935, when the strike terminated. During the strike, the National Labor Relations Act 1935 came into force. The basis of the Board's conclusion that the Corporation had committed unfair labor practices in violation of § 8(3) of the Act was a finding, not challenged here, that a number of men had been refused employment because of their affiliations with the Union. Of these men, two, Curtis and Daugherty, had ceased to be in the Corporation's employ before the strike, but sought employment after its close. The others, thirty-eight in number, were strikers. To "effectuate the policies" of the Act, § 10(c), the Board ordered the Corporation to offer Curtis and Daugherty jobs and to make them whole for the loss of pay resulting from the refusal to hire them, and it ordered thirty-seven of the strikers reinstated with back pay, and the other striker made whole for loss in wages up to the time he became unemployable. Save for a modification presently to be discussed, the Circuit Court of Appeals enforced the order affecting the strikers, but struck down the provisions relating to Curtis and Daugherty. ISSUE: Whether an employer subject to the National Labor Relations Act may refuse to hire employees solely because of their affiliations with a labor union. Subsidiarily, what are the powers open to the Board if it finds the discrimination in hiring as a ULP. RULING: NO, employer committed ULP when it discriminated in hiring employees with union affiliation. First. The denial of jobs to men because of union affiliations is an old and familiar aspect of American industrial relations. The Act’s purpose is to eliminate the causes of certain substantial obstructions to the free flows of commerce. This vital national purpose was to be accomplished "by encouraging the practice and procedure of collective bargaining and by protecting the exercise by workers of full freedom of association." § 1. Only thus could workers ensure themselves economic standards consonant with national wellbeing. Protection of the workers' right to self-organization does not curtail the appropriate sphere of managerial freedom; it furthers the wholesome conduct of business enterprise. "The Act does not interfere with the normal exercise of the right of the employer to select its employees or to discharge them." But "under cover of that right," the employer may not "intimidate or coerce its employees with respect to their self-organization and representation." "When employers freely recognize the right of their employees to their own organizations and their unrestricted right of representation, there will be much less occasion for controversy in respect to the free and appropriate exercise of the right of selection and discharge." Discrimination against union men at the time of hiring violated its declared policy that "The right of workers to organize in trade unions and to bargain collectively . . . shall not be denied, abridged, or interfered with by the employers in any manner whatsoever. Such a policy is an inevitable corollary of the principle of freedom of organization. The effect of such discrimination is not confined to the actual denial of employment; it inevitably operates against the whole idea of the legitimacy of organization. In a word, it undermines the principle which is recognized as basic to the attainment of industrial peace. We have seen the close link between a bar to employment because of union affiliation and the opportunities of labor organizations to exist and to prosper. Such an embargo against employment of union labor was notoriously one of the chief obstructions to collective bargaining through self-organization. Judge Learned Hand, in his opinion stated that there is "no greater limitation in denying him [the employer] the power to discriminate in hiring than in discharging." Second. Since the refusal to hire Curtis and Daugherty solely because of their affiliation with the Union was an unfair labor practice under § 8(3), the remedial authority of the Board under § 10(c) became operative. It could issue an order "to cease and desist from such unfair labor practice" in the future. Board may order employer to undo the wrong by offering the discriminated individuals opportunity for employment. Reinstatement is the conventional correction for discriminatory discharges. Third. The lower court is correct that the record warrants the Board's finding that the strikers were denied reemployment because of their union activities. Board can order their reinstatement. Fourth. There remain for consideration the limitations upon the Board's power to undo the effects of discrimination. Specifically, we have the question of the Board's power to order employment in cases where the men discriminated against had obtained "substantially equivalent employment." The Board, as a matter of fact, found that no such employment had been obtained, but alternatively concluded that, in any event, the men should be offered employment. "The term 'employee' shall include any employee, and shall not be limited to the employees of a particular employer but shall include any individual whose work has ceased as a consequence of, or in connection with, any current labor dispute or because of any unfair labor practice, and who has not obtained any other regular and substantially equivalent employment." To deny the Board power to neutralize discrimination merely because workers have obtained compensatory employment would confine the "policies of this Act" to the correction of private injuries. The Board was not devised for such a limited function. It is the agency of Congress for translating into concreteness the purpose of safeguarding and encouraging the right of self-organization. The Board, we have held very recently, does not exist for the "adjudication of private rights"; To be sure, reinstatement is not needed to repair the economic loss of a worker who, after discrimination, has obtained an equally profitable job. But to limit the significance of discrimination merely to questions of monetary loss to workers would thwart the central purpose of the Act, directed as that is toward the achievement and maintenance of workers' self-organization. To deny the Board power to wipe out the prior discrimination by ordering the employment of such workers would sanction a most effective way of defeating the right of self-organization. Therefore, the mere fact that the victim of discrimination has obtained equivalent employment does not itself preclude the Board from undoing the discrimination and requiring employment. In the exercise of its informed discretion, the Board may find that effectuation of the Act's policies may or may not require reinstatement. Reinstatement may be the effective assurance of the right of self-organization. Again, without such a remedy, industrial peace might be endangered because workers would be resentful of their inability to return to jobs to which they may have been attached and from which they were wrongfully discharged.

148 | P a g e

Fifth. As part of its remedial action against the unfair labor practices, the Board ordered that workers who had been denied employment be made whole for their loss of pay. In specific terms, the Board ordered payment to the men of a sum equal to what they normally would have earned from the date of the discrimination to the time of employment less their earnings during this period. Since only actual losses should be made good, it seems fair that deductions should be made not only for actual earnings by the worker, but also for losses which he willfully incurred. C. Construction Caltex Filipino Managers and Supervisors Association v. CIR, 44 SCRA 350 (1972) 1) It is well known that the scheme in Republic Act No. 875 for achieving industrial peace rests essentially on a FREE AND PRIVATE AGREEMENT between the employer and his employees as to the terms and conditions under which the employer is to give work and the employees are to furnish labor, unhampered as far as possible by judicial or administrative intervention. On this premise the lawmaking body has virtually prohibited the issuance of injunctive relief involving or growing out of labor disputes. 2) The prohibition to issue labor injunctions is designed to give labor a comparable bargaining power with capital and must be liberally construed to that end. RATIONALE for prohibition: 1) It is well known that the scheme in Republic Act No. 875 for achieving industrial peace rests essentially on a FREE AND PRIVATE AGREEMENT between the employer and his employees as to the terms and conditions under which the employer is to give work and the employees are to furnish labor, unhampered as far as possible by judicial or administrative intervention. On this premise the lawmaking body has virtually prohibited the issuance of injunctive relief involving or growing out of labor disputes. 2) The prohibition to issue labor injunctions is designed to give labor a comparable bargaining power with capital and must be liberally construed to that end. GENERAL RULE: there can be no injunction issued against any strike. EXCEPT in only one instance, that is, when a labor dispute arises in an INDUSTRY INDISPENSABLE TO THE NATIONAL INTEREST and such dispute is CERTIFIED BY THE PRESIDENT of the Philippines to the Court of Industrial Relations in compliance with Sec. 10 of Republic Act No. 875. Purpose of an injunction in an UNCERTIFIED case: As a corollary to this, an injunction in an uncertified case must be based on the strict requirements of Sec. 9(d) of Republic Act No. 875; the purpose of such an injunction is not to enjoin the strike itself, but only unlawful activities. In this case Judge Tabigne cautioned the parties to “maintain the status quo”; he specifically advised the employees NOT TO GO ON strike. Will disobedience to the judge’s advice constitute contempt of court? No. According to SC what Judge Tabigne stated during said hearing should be construed what actually it was — an advice. To say that it was an order would be to concede that respondent court could validly enjoin a strike, especially one which is not certified in accordance with Sec. 10 of Republic Act No. 875. To adopt the view of respondent court would not only set at naught the policy of the law as embodied in the said statute against issuance of injunctions, but also remove from the hands of labor unions and aggrieved employees an effective lawful weapon to either secure favorable action on their economic demands or to stop unfair labor practices on the part of their employer. Hongkong and Shanghai Bank Corporation Employees Union v. NLRC, 281 SCRA 509 (1997) Necessarily, a determination of the validity of the Batik's unilateral Implementation of the JE Program or the Union's act of engaging in concerted activities involves all appraisal of their motives. In cases of this nature, motivations are seldom expressly avowed, and avowals are not always candid. There must thus he a measure of reliance in the administrative agency. It was incumbent upon the labor arbiter, in the first instance, to weigh such expressed motives in determining the effect of an otherwise equivocal act. The Labor Code does not undertake the impossible task of specifying in precise and unmistakable language each incident which constitute, an unfair labor practice. Rather, it leaves to the court the work of applying the law's general prohibitory language in light of infinite combinations of events which may be charged as violative of its terms. D. Estoppel Standard Chartered Bank v. Confesor, 432 SCRA 308 (2004) FACTS: - the Bank and the Union signed a five-year collective bargaining agreement (CBA) with a provision to renegotiate the terms thereof on the third year. Prior to the expiration of the three-year period but within the sixty-day freedom period, the Union initiated the negotiation; - the Union, through its President, Eddie L. Divinagracia, sent a letter containing its proposals covering political provisions and thirty-four (34) economic provisions. Included therein was a list of the names of the members of the Union’s negotiating panel; - the Bank, through its Country Manager Peter H. Harris, took note of the Union’s proposals and likewise, listed the members of its negotiating panel.The parties agreed to set meetings to settle their differences on the proposed CBA; - before the commencement of the negotiation, the Union suggested to the Bank’s Human Resource Manager and head of the negotiating panel, Cielito Diokno, that the bank lawyers should be excluded from the negotiating team. The Bank acceded. Meanwhile, Diokno suggested to Divinagracia that Jose P. Umali, Jr., the President of the National Union of Bank Employees (NUBE), the federation to which the Union was affiliated, be excluded from the Union’s negotiating panel.12 However, Umali was retained as a member thereof; 149 | P a g e

- the proposed non-economic provisions of the CBA were discussed first. There were still provisions on which the Union and the Bank could not agree. Eventually, parties agreed to place the notation "DEFERRED/DEADLOCKED; - then came the negotiation for economic provisions. Except for the provisions on signing bonus and uniforms, the Union and the Bank failed to agree on the remaining economic provisions of the CBA. The Union declared a deadlock and filed a Notice of Strike before the National Conciliation and Mediation Board (NCMB); - the Bank, on the other hand, filed a complaint for Unfair Labor Practice (ULP) and Damages before the Arbitration Branch of the National Labor Relations Commission (NLRC) in Manila. Sec. of Labor Confesor: ordered the Bank and the Union to execute a CBA; dismissed the charges of ULP of both the Union and the Bank; denied Motions for Reconsideration of both parties. - the Bank and the Union signed the CBA. Immediately thereafter, the wage increase was effected and the signing bonuses based on the increased wage were distributed to the employees covered by the CBA. ISSUE: whether or not the petitioner union is estopped from filing the instant action HELD, RATIO: NO. The approval of the CBA and the release of signing bonus do not necessarily mean that the Union waived its ULP claim against the Bank during the past negotiations. After all, the conclusion of the CBA was included in the order of the SOLE, while the signing bonus was included in the CBA itself. Moreover, the Union twice filed a motion for reconsideration respecting its ULP charges against the Bank before the SOLE. SC: affirmed the Order and Resolutions of Sec. Confesor; dismissed petition for certiorari. E. Law Nomenclature and Inter-relations of Acts of ULP – 212 (k); 248-249 Art. 212. Definitions. "Unfair labor practice" means any unfair labor practice as expressly defined by the Code. Art. 248. Unfair labor practices of employers. It shall be unlawful for an employer to commit any of the following unfair labor practice: a. To interfere with, restrain or coerce employees in the exercise of their right to self-organization; b. To require as a condition of employment that a person or an employee shall not join a labor organization or shall withdraw from one to which he belongs; c. To contract out services or functions being performed by union members when such will interfere with, restrain or coerce employees in the exercise of their rights to self-organization; d. To initiate, dominate, assist or otherwise interfere with the formation or administration of any labor organization, including the giving of financial or other support to it or its organizers or supporters; e. To discriminate in regard to wages, hours of work and other terms and conditions of employment in order to encourage or discourage membership in any labor organization. Nothing in this Code or in any other law shall stop the parties from requiring membership in a recognized collective bargaining agent as a condition for employment, except those employees who are already members of another union at the time of the signing of the collective bargaining agreement. Employees of an appropriate bargaining unit who are not members of the recognized collective bargaining agent may be assessed a reasonable fee equivalent to the dues and other fees paid by members of the recognized collective bargaining agent, if such non-union members accept the benefits under the collective bargaining agreement: Provided, that the individual authorization required under Article 242, paragraph (o) of this Code shall not apply to the non-members of the recognized collective bargaining agent; f. To dismiss, discharge or otherwise prejudice or discriminate against an employee for having given or being about to give testimony under this Code; g. To violate the duty to bargain collectively as prescribed by this Code; h. To pay negotiation or attorney’s fees to the union or its officers or agents as part of the settlement of any issue in collective bargaining or any other dispute; or i. To violate a collective bargaining agreement. The provisions of the preceding paragraph notwithstanding, only the officers and agents of corporations, associations or partnerships who have actually participated in, authorized or ratified unfair labor practices shall be held criminally liable. (As amended by Batas Pambansa Bilang 130, August 21, 1981) Art. 249. Unfair labor practices of labor organizations. It shall be unfair labor practice for a labor organization, its officers, agents or representatives: a. To restrain or coerce employees in the exercise of their right to self-organization. However, a labor organization shall have the right to prescribe its own rules with respect to the acquisition or retention of membership; b. To cause or attempt to cause an employer to discriminate against an employee, including discrimination against an employee with respect to whom membership in such organization has been denied or to terminate an employee on any ground other than the usual terms and conditions under which membership or continuation of membership is made available to other members; c. To violate the duty, or refuse to bargain collectively with the employer, provided it is the representative of the employees; d. To cause or attempt to cause an employer to pay or deliver or agree to pay or deliver any money or other things of value, in the nature of an exaction, for services which are not performed or not to be performed, including the demand for fee for union negotiations; e. To ask for or accept negotiation or attorney’s fees from employers as part of the settlement of any issue in collective bargaining or any other dispute; or f. To violate a collective bargaining agreement.

150 | P a g e

The provisions of the preceding paragraph notwithstanding, only the officers, members of governing boards, representatives or agents or members of labor associations or organizations who have actually participated in, authorized or ratified unfair labor practices shall be held criminally liable. (As amended by Batas Pambansa Bilang 130, August 21, 1981) Hongkong Shanghai Banking Corporation v. NLRC, 281 SCRA 509 (1997) Necessarily, a determination of the validity of the Batik's unilateral Implementation of the JE Program or the Union's act of engaging in concerted activities involves all appraisal of their motives. In cases of this nature, motivations are seldom expressly avowed, and avowals are not always candid. There must thus he a measure of reliance in the administrative agency. It was incumbent upon the labor arbiter, in the first instance, to weigh such expressed motives in determining the effect of an otherwise equivocal act. The Labor Code does not undertake the impossible task of specifying in precise and unmistakable language each incident which constitute, an unfair labor practice. Rather, it leaves to the court the work of applying the law's general prohibitory language in light of infinite combinations of events which may be charged as violative of its terms. Republic Savings Bank v. CIR, 21 SCRA 226 (1967) Fernando J., concurring opinion, 240-241 It is for the Court of Industrial Relations, in the first instance, to make the determination, "to weigh the employer's expressed motive in determining the effect on the employees of management's otherwise equivocal act" (NLRB vs. Stowe Spinning Co., 336 U.S. 226). For the Industrial Peace Act does not undertake the impossible task of specifying in precise and unmistakable language each incident which constitutes an unfair labor practice, rather, it leaves to the court the work of applying the Act's general prohibitory language in the light of infinite combinations of events which may be charged as violative of its terms. F. ULP and Management Functions Royal Inter-Ocean, Lines v. CIR, 109 P 900 (1960) Great Pacific Life Employees Union v. Great Pacific Life Assurance Corporation, 303 SCRA 113 (1999) CBA between Union and company resulted in deadlock. Union staged strike. Union officers were terminated. Union and company executed Memorandum of Agreement stating that except for officers Domingo and dela Rosa, employees/members are given amnesty and reinstated. Domingo and dela Rosa filed resignation letter but stated it is not a waiver of any of their rights under contracts and labor legislations. The resignation letters were not acted upon but they were dismissed. Afterwards, they filed case for illegal dismissal, ULP and damges. Issue: WON company was guilty of ULP Held: No. Ratio: Under the MOA, although the resignation letter was never acted upon, the termination was valid because of the Domingo and dela Rosa’s earlier dismissal. The decision of the employer to consider top union officers of union as unfit for reinstatement is not essentially discriminatory and constitutive of ULP – not a form of discrimination under the Labor Code. Dismissal of undesirable labor leaders (committed prohibited activities during strike) upheld in pursuit of the objective of Labor Code to ensure a stable but dynamic and just industrial peace. Recall of workers is valid management prerogative as long as it is done in good faith. Reasoning: While an act or decision of an employer may be unfair, certainly not every unfair act or decision constitutes unfair labor practice (ULP) as defined and enumerated under Art. 248 of the Labor Code.18 There should be no dispute that all the prohibited acts constituting unfair labor practice in essence relate to the workers' right to self-organization. Thus, an employer may be held liable under this provision if his conduct affects in whatever manner the right of an employee to self-organize. The decision of respondent GREPALIFE to consider the top officers of petitioner UNION as unfit for reinstatement is not essentially discriminatory and constitutive of an unlawful labor practice of employers under the above-cited provision. Discriminating in the context of the Code involves either encouraging membership in any labor organization or is made on account of the employee's having given or being about to give testimony under the Labor Code. These have not been proved in the case at bar. Philcom Employees Union v. Philippine Global Communications, 495 SCRA 214 (2006), supra Republic Savings Bank v. CIR, 21 SCRA 226 (2007), supra G. Nature of Act Allied Banking Corporation v. CA, 416 SCRA 65 (2003) II. UNFAIR LABOR PRACTICE: EMPLOYER AND LABOR ORGANIZATION ACTS VIOLATING RIGHT OF SELF-ORGANIZATION A. Specific Acts 151 | P a g e

1. Interference, Restraint and Coercion – 248 (a); 255, 277 (g) (h); 249 (a) Art. 248. Unfair labor practices of employers. It shall be unlawful for an employer to commit any of the following unfair labor practice: a. To interfere with, restrain or coerce employees in the exercise of their right to self-organization; Art. 255. Exclusive bargaining representation and workers’ participation in policy and decision-making. The labor organization designated or selected by the majority of the employees in an appropriate collective bargaining unit shall be the exclusive representative of the employees in such unit for the purpose of collective bargaining. However, an individual employee or group of employees shall have the right at any time to present grievances to their employer. Any provision of law to the contrary notwithstanding, workers shall have the right, subject to such rules and regulations as the Secretary of Labor and Employment may promulgate, to participate in policy and decision-making processes of the establishment where they are employed insofar as said processes will directly affect their rights, benefits and welfare. For this purpose, workers and employers may form labor-management councils: Provided, That the representatives of the workers in such labor-management councils shall be elected by at least the majority of all employees in said establishment. (As amended by Section 22, Republic Act No. 6715, March 21, 1989) Art. 277. Miscellaneous provisions. g. The Ministry shall help promote and gradually develop, with the agreement of labor organizations and employers, labormanagement cooperation programs at appropriate levels of the enterprise based on the shared responsibility and mutual respect in order to ensure industrial peace and improvement in productivity, working conditions and the quality of working life. (Incorporated by Batas Pambansa Bilang 130, August 21, 1981) h. In establishments where no legitimate labor organization exists, labor-management committees may be formed voluntarily by workers and employers for the purpose of promoting industrial peace. The Department of Labor and Employment shall endeavor to enlighten and educate the workers and employers on their rights and responsibilities through labor education with emphasis on the policy thrusts of this Code. (As amended by Section 33, Republic Act No. 6715, March 21, 1989) ILO Convention No. 98, Art. 1-2 pls. refer to Part I.A 2. Non-Union Membership or Withdrawal from Membership as Condition Employment – 248 (b) Art. 248. Unfair labor practices of employers. It shall be unlawful for an employer to commit any of the following unfair labor practice: b. To require as a condition of employment that a person or an employee shall not join a labor organization or shall withdraw from one to which he belongs; 3. Company Domination Union – 248 (d); 212 (i) Art. 248. Unfair labor practices of employers. It shall be unlawful for an employer to commit any of the following unfair labor practice: d. To initiate, dominate, assist or otherwise interfere with the formation or administration of any labor organization, including the giving of financial or other support to it or its organizers or supporters; Art. 212. Definitions. "Company union" means any labor organization whose formation, function or administration has been assisted by any act defined as unfair labor practice by this Code. 4. Discrimination Encourage Discourage Unionism – 248 (e); 249 (b) Art. 248. Unfair labor practices of employers. It shall be unlawful for an employer to commit any of the following unfair labor practice: e. To discriminate in regard to wages, hours of work and other terms and conditions of employment in order to encourage or discourage membership in any labor organization. Nothing in this Code or in any other law shall stop the parties from requiring membership in a recognized collective bargaining agent as a condition for employment, except those employees who are already members of another union at the time of the signing of the collective bargaining agreement. Employees of an appropriate bargaining unit who are not members of the recognized collective bargaining agent may be assessed a reasonable fee equivalent to the dues and other fees paid by members of the recognized collective bargaining agent, if such non-union members accept the benefits under the collective bargaining agreement: Provided, that the individual authorization required under Article 242, paragraph (o) of this Code shall not apply to the non-members of the recognized collective bargaining agent; Art. 249. Unfair labor practices of labor organizations. It shall be unfair labor practice for a labor organization, its officers, agents or representatives: b. To cause or attempt to cause an employer to discriminate against an employee, including discrimination against an employee with respect to whom membership in such organization has been denied or to terminate an employee on any ground other than the usual terms and conditions under which membership or continuation of membership is made available to other members; a) Retaliation Testimony against Employer – 248 (f) Art. 248. Unfair labor practices of employers. It shall be unlawful for an employer to commit any of the following unfair labor practice: f. To dismiss, discharge or otherwise prejudice or discriminate against an employee for having given or being about to give testimony under this Code; b) Exaction – Featherbedding – 249 (b) Art. 248. Unfair labor practices of employers. It shall be unlawful for an employer to commit any of the following unfair labor practice: b. To require as a condition of employment that a person or an employee shall not join a labor organization or shall withdraw from one to which he belongs; 152 | P a g e

c) Contracting out to Discourage Unionism – 248 (c); Art. 248. Unfair labor practices of employers. It shall be unlawful for an employer to commit any of the following unfair labor practice: c. To contract out services or functions being performed by union members when such will interfere with, restrain or coerce employees in the exercise of their rights to self-organization; a. Interrogation Scoty’s Department Store v. Micaller, 99 P 762 (1956) Philippine Steam Navigation Co. v. Philippine Marine Officers Guild, 15 SCRA 174 (1965) b. Polling Operating Engineers Local 49 v. NLRB 353 F 2nd 852 (65) Library File The General Counsel's complaint charged that Struksnes Construction Co., Inc. of Minot, North Dakota (herein, Employer), had violated section 8(a) (1), 29 U.S.C. § 158(a) (1), by conducting an open, signed poll of its employees to determine whether they wished the Employer to bargain with the Union, and section 8(a) (5), 29 U.S.C. § 158(a) (5), by refusing to bargain with the Union. Reversing the Trial Examiner, a panel of the Board, by a divided vote, entered its Decision and Order of September 28, 1964, dismissing the complaint. The Union's motion for reconsideration or rehearing was denied by order of March 25, 1965, reaffirming the original decision. In the latter part of July 1963, the Union's area representative, McPherson, commenced organizational activity at a road construction job for which the Employer had been awarded a contract by the North Dakota Highway Department. The Employer's president, Mr. Struksnes, was away much of the time attending to other highway projects. The record did not show when Struksnes was expected to return to Minot, but it is clear that McPherson on August 7, 1963 consulted the Employer's attorney, Van Sickle, and asked him to arrange a meeting with Struksnes. McPherson testified that he "felt" he "had the majority of the people and felt that we should sit down and talk about a contract." The record is silent as to details of developments following the "Bruce" and "Bob" correspondence until as of August 16, 1963, Van Sickle wrote to "Dear Bob" that "Christ Struksnes has now informed me that a majority of his men have advised him that they do not want him to negotiate with you with reference to a contract. I feel that I should advise you of this immediately * * *" (Emphasis added.) It thus would seem that at least as of August 16th, and probably earlier, Struksnes had returned and had conferred with Van Sickle. We may assume that the latter must have informed Struksnes of the contents of McPherson's letter of August 12. Also, we do not doubt that by the 16th, Van Sickle definitely represented the Employer, and thus felt impelled "in good faith" to apprise McPherson that a majority of the men in the appropriate unit did not want Struksnes "to negotiate with you with reference to a contract." McPherson testified that the Employer had shown no bias in its employment, whether with respect to men who had previously been members of the Union or to those who had been organized in late July or early August as a result of McPherson's activities. McPherson testified further that neither the Employer nor his representatives had interfered with his solicitation of the employees, either before August 7, before August 12th or thereafter, even at the job site when Struksnes himself was on the job and was aware of McPherson's activities. In that setting, even when considered with an episode we next will reach, there was ample basis for the Board's conclusion that the General Counsel had failed to establish the allegations with respect to the claimed section 8(a) (5) violation. The record as a whole substantially supports that conclusion. There had been no strike, no evidence of anti-union animus as the Board found, no discrimination and no firing because of union activity.6 There had been no substantial evidence of coercion of the employees or of actual restraint of their exercising their section 7 rights. But with respect to the section 8(a) (1) charge, the Employer had taken one step which we deem fateful. We are by no means satisfied with the Board's ad hoc acquiescence in, if not approval of, the manner in which the Employer polled his men. Commencing some time after the McPherson letter of August 12, text supra, p. 853, Struksnes "went up and asked the guys. All I did was ask them. I told them what was up here, and I asked them to sign yes or no, and it wouldn't make any difference. It was up to those guys." What "was up here" on Exhibit No. 7 was the following question addressed to "THE MEN OF STRUCKSNES [sic] CONSTRUCTION CO. "DO YOU WANT ME TO BARGAIN WITH AND SIGN A CONTRACT WITH OPERATING ENGINEERS LOCAL 49? "PLEASE SIGN YOUR NAME AND ANSWER YES OR NO." Twenty-four men signed their names upon that statement, nine voting "yes," fifteen "no," with one refusing to sign at all. The Employer had called no meeting of the men at which he might explain his purpose to ascertain whether or not the men desired a union contract on this particular job, and assure them of no reprisals. Struksnes personally at the end of one shift presented the statement to each of the employees then available for his approach. His two foremen reached the remaining employees in like manner. How each man voted was known to the Employer or his foremen, not only at the time each man signed, but as a matter of record thereafter, just as the General Counsel's Exhibit 7 made manifest. We can understand an inference that the men, members of the Union or not, might actually have voted in good faith not to have a union contract on this job; they might have wanted all the work they could get, even up to 9 P.M. on the second shift, before the "freeze-up" and the darkness of winter set in, precluding further employment. The question on the Exhibit is equivocal to be sure, and seems not to have been directed to an ascertainment of union affiliation of the employees or of their activities, pro or con. On the other hand, as the Board itself has observed, 153 | P a g e

"[A]ny employer who engages in interrogation does so with notice that he risks a finding of unfair labor practices if the circumstances are such that his interrogation restrains or interferes with employees in the exercise of their rights under the Act." But how can he know when he will be deemed to have acted at his peril? The Board at page 775 defined the purposes of an employer poll which have been held to be legitimate and pronounced "specific safeguards designed to minimize the coercive impact of such employer interrogation." One such was that the questioning "must occur in a context free from employer hostility to union organization and must not be itself coercive in nature." There the company attorney had interviewed the employees, explaining his status and his purpose in investigating a charge which the union had filed against the company. He assured each employee that his rights were protected by law and would be respected. Each employee had been asked to identify his signature on the photostat of his union authorization card, and other details were developed. There, said the Board, was an unfair labor practice; no, said the Court of Appeals. The Board here simply dismisses, sub silentio, the development of a permanent record of the votes of each employee set against his signature after interrogation had been conducted in personal approach by the employer and his foremen, and otherwise under the circumstances we outlined. Although a majority of the employees were members of the union, had they succumbed to coercion when they voted in the negative upon being queried as to whether or not they desired their employer to enter into a contract with their union? The Board's Decision and Order discloses no treatment of the possibly inherent restraint resulting from such contacts. There was no attempt at reconciliation of the Board's assessment of the situation here with the section 8(a) (1) violations as found by the Board in Lorben, supra note 10, and in Johnnie's Poultry Co., supra note 11. But the Act makes no exception for conditions thus varying, for it leaves treatment to the Board as a matter of policy. The rights of the employees must be safeguarded nonetheless. So it is we do not understand how the Board could approve the tactics here utilized, the technique employed, or the form in which ascertainment of the attitude of the men was sought. We think the Board should come to grips with this constantly recurring problem for the protection of the employees as to their section 7 rights and for that of an employer acting in good faith. It would seem that the Board could, in the exercise of its expertise, develop appropriate policy considerations and outline at least minimal standards to govern the ascertainment of union status, or even in given permissible situations, the desire of the employees respecting a contract with the Union. We will set aside the Board's order and remand this case for further consideration not inconsistent with this opinion. Reversed and remanded. c. Speech The Insular Life Assurance Co. Ltd. Employees Association-NATU v. Insular Life Assurance Co. Ltd., 37 SCRA 244 (1971) – Aika The letters, exhibits A and B, should not be considered by themselves alone but should be read in the light of the preceding and subsequent circumstances surrounding them. The letters should be interpreted according to the "totality of conduct doctrine, "...whereby the culpability of an employer's remarks were to be evaluated not only on the basis of their implicit implications, but were to be appraised against the background of and in conjunction with collateral circumstances. Under this 'doctrine' expressions of opinion by an employer which, though innocent in themselves, frequently were held to be culpable because of the circumstances under which they were uttered, the history of the particular employer's labor relations or anti-union bias or because of their connection with an established collateral plan of coercion or interference." (Rothenberg on Relations, p. 374, and cases cited therein.) Effect of Failure of the Act Interference constituting unfair labor practice will not cease to be such simply because it was susceptible of being thwarted or resisted, or that it did not proximately cause the result intended. For success of purpose is not, and should not, be the criterion in determining whether or not a prohibited act constitutes unfair labor practice. "The test of whether an employer has interfered with and coerced employees within the meaning of subsection (a) (1) is o whether the employer has engaged in conduct which it may reasonably be said tends to interfere with the free exercise of employees' rights under section 3 of the Act, o and it is not necessary that there be direct evidence that any employee was in fact intimidated or coerced by statements of threats of the employer if there is a reasonable inference that anti-union conduct of the employer does have an adverse effect on self-organization and collective bargaining." (Francisco, Labor Laws 1956, Vol. II, p. 323, citing NLRB v. Ford, C.A., 1948, 170 F2d 735). International Association of Machinists v. NLRB, 311 U.S. 72 (40) Library File FACTS: Fouts, Shock, Dininger, Bolander, Byroad, and Baker were all employees of the toolroom. Four of these -- Fouts, Shock, Byroad, and Bolander -- were old and trusted employees. Fouts was "more or less an assistant foreman," having certain employees under him. Shock was in charge of the toolroom during the absence of the foreman. Dininger and Bolander were in charge of the second and third shifts, respectively, working at night. Prior to mid-July, 1937, they had been actively engaged on behalf of the company union. When it became apparent at that time that the efforts to build up that union were not successful, Fouts, Shock, Byroad, and Bolander suddenly shifted their support from the company union to petitioner and moved into the forefront in enlisting the support of the employees for petitioner. The general manager told Shock that he would close the plant rather than deal with UAW. The superintendent and Shock reported to toolroom employees that the employer would 154 | P a g e

not recognize the CIO. The superintendent let it be known that the employer would deal with an A.F. of L. union. At the same time, the superintendent also stated to one of the employees that some of the "foremen don't like the CIO," and added, with prophetic vision, that there was "going to be quite a layoff around here, and these fellows that don't like the CIO are going to lay those fellows off first." During working hours, Byroad conducted a straw vote among the employees, and, under the direction of Fouts and Shock, left the plant to seek out an organizer for petitioner. Fouts solicited among workmen in the toolroom, stating that his purpose was to "beat" the UAW. For a week preceding August 13, Shock spent much time, as did Byroad, going "from one bench to another soliciting" for petitioner. Baker likewise solicited. Dininger offered an employee a "good rating" if he would join petitioner. Not less than a week before August 13, the personnel director advised two employees to "join the A.F. of L." Byroad spent considerable time during working hours soliciting employees, threatening loss of employment to those who did not sign up with petitioner and representing that he was acting in line with the desires of the toolroom foreman, McCoy. This active solicitation for petitioner was on company time, and was made openly in the shop. Much of it was made in the presence of the toolroom foreman, McCoy, who clearly knew what was being done. Yet the freedom allowed solicitors for petitioner was apparently denied solicitors for UAW. The plant manager warned some of the latter to check out their time for a conference with him on UAW, and questioned their right to discuss UAW matters on company property. The inference is justified that UAW solicitors were closely watched, while those acting for petitioner were allowed more leeway. Five UAW officials had been discharged in June, 1937, because of their union activities. The known antagonism of the employer to UAW before petitioner's drive for membership started made it patent that the employees were not free to choose UAW as their bargaining representative. Petitioner started its drive for membership late in July, 1937, and its closed shop contract was signed August 11, 1937. On August 10, 1937, the UAW, having a clear majority of all the employees, presented to the employer a proposed written contract for collective bargaining. This was refused. On August 13, 1937, all toolroom employees who refused membership in petitioner, some 20 in number, were discharged. On August 15, 1937, the management circulated among the employees a statement which, as found by the Board, was a thinly veiled attack on the UAW and a firm declaration that the employer would not enter into any agreement with it. Petitioner insists that the employer's hostility to UAW cannot be translated into assistance to the petitioner, and that none of the acts of the employees above mentioned, who were soliciting for petitioner, can be attributed to the employer. The Board found, in proceedings duly had under § 10 of the Act, that the employer, Serrick Corporation, had engaged in unfair labor practices within the meaning of the Act. It ordered the employer to cease and desist from those practices and to take certain affirmative action. More specifically, it directed the employer to cease giving effect to a closed shop contract with petitioner covering the tool room employees; to deal with UAW, an industrial unit, as the exclusive bargaining agent of its employees, including the toolroom men; to desist from various discriminatory practices in favor of petitioner and against UAW, and to reinstate and make whole certain employees who had been improperly discharged. The employer has complied with the Board's order. But petitioner, an intervener in the proceedings before the Board, filed a petition in the court below to review and set aside those portions of the order which direct the employer to cease and desist from giving effect to its closed shop contract with petitioner and to bargain exclusively with UAW The court below affirmed the order of the Board. BASIC ISSUE: It is whether or not the employer "assisted" the petitioner in enrolling its majority. ISSUES: (1) whether, on the facts of this case, the National Labor Relations Board was without authority in finding that an industrial unit was appropriate for collective bargaining purposes to the exclusion of a craft unit, and (2) whether the Board had authority to require the employer to bargain with that industrial unit, despite a claim submitted to the Board by the craft unit before the order issued that the latter then had been designated by a majority of all the employees. RULING: We agree with the court below that the toolroom episode was but an integral part of a long plant controversy. What happened during the relatively brief period from late July to August 11, 1937, cannot properly be divorced from the events immediately preceding and following. The active opposition of the employer to UAW throughout the whole controversy has a direct bearing on the events during that intermediate period. Known hostility to one union and clear discrimination against it may indeed make seemingly trivial intimations of preference for another union powerful assistance for it. Slight suggestions as to the employer's choice between unions may have telling effect among men who know the consequences of incurring that employer's strong displeasure. The freedom of activity permitted one group and the close surveillance given another may be more powerful support for the former than campaign utterances. To be sure, it does not appear that the employer instigated the introduction of petitioner into the plant. But the Board was wholly justified in finding that the employer "assisted" it in its organizational drive. Silent approval of or acquiescence in that drive for membership and close surveillance of the competitor; the intimations of the employer's choice made by superiors; the fact that the employee-solicitors had been closely identified with the company union until their quick shift to petitioner; the rank and position of those employee solicitors; the ready acceptance of petitioner's contract and the contemporaneous rejection of the contract tendered by UAW; the employer's known prejudice against the UAW, were all proper elements for it to take into consideration in weighing the evidence and drawing its inferences. To say that the Board must disregard what preceded and what followed the membership drive would be to require it to shut its eyes to potent imponderables permeating this entire record. The detection and appraisal of such imponderables are indeed one of the essential functions of an expert administrative agency. Petitioner asserts that it had obtained its majority of toolroom employees by July 28, 1938, and that there was no finding by the Board that that majority was maintained between then and the date of execution of the closed shop contract by unfair labor practices. In this case, however, that is an irrelevant refinement. The existence of unfair labor practices throughout this whole period permits the inference that the employees did not have that freedom of choice which is the essence of collective bargaining. And the finding of the Board that petitioner did not represent an uncoerced majority of toolroom employees when the closed shop contract was executed is adequate to support the conclusion that the maintenance, as well as the acquisition, of the alleged majority was contaminated by the employer's aid.

155 | P a g e

1. The National Labor Relations Board, finding that a labor organization, having a closed shop contract with an employer, had been "assisted" in its organizational drive by unfair labor practices of the employer, was authorized to order the employer to cease and desist from giving effect to the contract. 2. The finding of the National Labor Relations Board in this case that a labor organization had been "assisted" by unfair labor practices of the employer is supported by substantial evidence. 4. In determining, upon the record of this case, whether a labor organization was "assisted" by unfair labor practices of the employer, the Board could properly consider not only the employer's activities during the organization's membership drive, but also previous and subsequent activities. 5. That, in respect of the period between the time as of which a labor organization claims to have obtained a majority of the workers in an appropriate unit and the date of the execution of a closed shop contract between it and the employer, the Board made no finding that the claimed majority was maintained by unfair labor practices is not material in this case. The finding of the Board that the labor organization did not represent an uncoerced majority of the employees in such unit when the closed shop contract was executed is adequate to support the conclusion that the maintenance, as well as the acquisition, of the alleged majority was wrongfully achieved. 6. An employer may be found under the National Labor Relations Act to have "assisted" a labor organization by unfair labor practices even though the employees through whose activities the employer is regarded as having so assisted were not employed in a "supervisory" capacity, and even though their acts were not expressly authorized or were not such a might constitute a basis of employer liability under the doctrine of respondeat superior. 7. Where, as here, there is ample evidence to support an inference that the employees believed that certain solicitors, though bona fide members of a labor organization and professedly acting therefor, were in fact acting for and on behalf of the employer, the Board may justifiably find that the employees did not have the complete and unhampered freedom of choice which the Act contemplates. 8. Where, in a proceeding under § 10 of the National Labor Relations Act, the Board finds that a labor organization has been assisted by unfair labor practices of the employer, it may order the employer to deal exclusively, for purpose of collective bargaining, with a rival labor organization, and the Board may properly refuse to act upon a notice received from the first labor organization, prior to the issuance of its order, that that organization has obtained a majority of the employees in an appropriate bargaining unit. NLRB v. Virginia Electric and Power Co., 314 U.S. 469, (41) Library File d. Espionage The Insular Life Assurance Co. Ltd. Employees Association-NATU v. Insular Life Assurance Co. Ltd., 37 SCRA 244 (1971), supra e. Economic Coercion and Inducement The Insular Life Assurance Co. Ltd. Employees Association-NATU v. Insular Life Assurance Co. Ltd., 37 SCRA 244 (1971), supra NLRB v. Exchange Parts Co., 375 U.S. 405 (64) Library File f. Union Solicitation and Distribution of Literature and Materials Lechmere, Inc. v. NLRB, 117 L. Ed. 2d 79 (92) Library File Republic Aviation Corporation v. NLRB, 324 U.S. 793 (45) Library File NLRB v. Babcock and Wilcox Co., 351 U.S. 105 (56) Library File NLRB v. United Steel Workers of America, 357 U.S. 357 (58) Library File g. Discrimination Wise and Co., Inc. v. Wise and Co., Inc. Employees Union-NATU, 178 SCRA 536 (1989) – Vonn Facts:  On 03 April 1987, petitioner Wise & Co. Inc. (Company) introduced a profit-sharing scheme for its managers and supervisors.  The respondent Union requested to be included in this scheme but was denied by the Company, saying that it had to adhere strictly to the CBA.  On 30 March 1988, the Company distributed the profit sharing benefit not only to managers and supervisors but also to all other rank and file employees not covered by the CBA.  The Union filed a notice of strike alleging that petitioner was guilty of unfair labor practice because the union members were discriminated against in the grant of the profit sharing benefits.  The voluntary arbitrator ordered that the benefits of the 1987 profit sharing scheme be extended to the members of the Union. Issue: WON the grant by management of profit sharing benefits to its non-union member employees is discriminatory against its workers who are union members. Held: Not discriminatory. There can be no discrimination since the situation of the union employees is different and distinct from the non-union employees. Indeed, discrimination per se is not unlawful. There can be no discrimination where the employees concerned are not similarly situated.

156 | P a g e

There is a clause in the CBA where the employees are classified into those who are members of the union and those who are not. In the case of the union members, they derive their benefits from the terms and conditions of the CBA contract which constitute the law between the contracting parties. The non-union employees are not covered by the CBA. They do not derive and enjoy the benefits under the CBA. The Court holds that it is the prerogative of management to regulate, according to its discretion and judgment, all aspects of employment… [provided that it is] not exercised in a malicious, harsh, oppressive, vindictive or wanton manner or out of malice or spite. Me-Shurn Corporation v. Me-Shurn Workers Union, 448 SCRA 41 (2005) Petitioners: ME-SHURN Corporation, Sammy Chou Respondents: ME-SHURN Workers Union-FSM, Rosalina Cruz FACTS: On June 7, 1998, the regular rank-and-file employees of Me-Shurn Corporation organized the Me-Shurn Workers Union-FM, an affiliate of the February 6 Movement (FSM). 10 days later, the Corporation started to place on forced leave all the rank and file employees who were members of the Union. The Union filed a petition for certification election but instead of answering the petition, the company stated that it would temporarily lay off employees and cease operations on the basis of its alleged inability to meet the export quota required by the Board of Investment. 184 Union members retracted their membership during the petition which forced the med-arbiter to dismiss the petition. DOLE, however, ordered a certification election which accredited the Union. On August 4, 1998, the Union filed a Notice of Strike against the Corporation on the ground of illegal lockout and union busting. On August 31,1998, Sammy Chou and the Company’s administrative manager, Raquel Lamayra, imposed a precondition for the resumption of operation and the rehiring of laid off workers. It required the remaining union officers to sign an Agreement containing a guarantee that upon their return to work, no union or labor organization would be organized and the union officers will serve as mediators between labor and management. When the Agreement was signed, the operations of the Corporation resumed in September 1998. On November 1998, the Union reorganized and elected a new set of officers which elected Cruz, the other respondent, as President. The Union filed two complaints at the NLRC for ULP, illegal dismissal, underpayment of wages, and deficiency in separation pay. The Corporation countered by saying that because of economic reverses it was compelled to close and cease its operations to prevent serious business losses, that under Art 283, it had the right to do so, that in August 1998, it had paid its 342 laid off employees separation pay and benefits and that because of this, the case is already moot and academic. It also claimed that it had resumed operations in September 1998 which was announced at the Bataan Export Processing Zone and that some of the former employees had reapplied. LA ruled in favor of the Corporation stating that actual and expected losses justified the closure and dismissal of the company’s employees and the employees’ acceptance of separation pay by the workers precluded them from questioning the validity of their dismissal and thus their claim for separation pay is without any factual basis. NLRC reversed the LA’s decision and found the company guilty of ULP because the closure of the Corporation shortly after the Union’s organization and the dismissal of the employees have been made under false pretenses since the true reason for doing so is to bar the formation of the Union. CA affirmed the NLRC’s decision and added that it is the employer who has the burden of proving that the dismissal was for a valid or authorized cause and the Company has failed to submit sufficient proof of business losses. ISSUE: WON the dismissal of the employees was based on an authorized cause RULING: NO, the dismissal was not for an authorized cause. Despite the Corporation’s claims that the 1998 financial crisis and their supposed difficulty in obtaining an export quota, it never presented any report on the financial operations of the corporation during the period before its shutdown and was never able to submit credible evidence to substantiate their allegation of business losses. Since the employer has the burden of showing the authorized cause for the dismissal, the Corporation should present convincing evidence of imminent economic or business reversals as a form of affirmative defense both at the LA proceedings and even on appeal to the NLRC and CA. No evidence was submitted and the Corporation belatedly submitted the corporation’s income tax returns from 1996 to 1999 as proof of alleged continued losses during those years only at the SC. Matters regarding the financial condition of a company – those justifying the closing of its business and show the losses in its operations – are questions of facts that must be proven in the lower courts. Their failure to present convincing evidence of losses at the early stages of the case clearly belies the credibility of their claim at the SC. But even if the additional pieces of evidence are accepted by the SC, the Corporation’s claim would still be untenable. It claimed that it was compelled to close down to prevent further losses but it resumed operations barely a month after. It also claimed that its loss is mainly because of its failure to obtain an export quota from the Garment and Textile Export Board but when it resumed its business, it did not first obtain an export quota with the Board and these quotas only pertain to businesses with companies in the US and do not preclude the Corporation from exporting its products to other countries. Also, the Statement of Income and Deficit showed that the company had a deficit of Php2M in 1996 but the closure was only effected in mid 1998 shortly after the formation of the Union. Besides, the Corporation was more profitable in 1997. Also, the Union officers was made to sign an Agreement which was a precondition for rehiring employees, this is contrary to law. The Corporation also hastily formed a new union and signed a CBA with it when it learned that a Union was about to be formed. It also did not present evidence to prove its claim that some of the employees’ contracts had expired even before the cessation of operations and did not send the proper written notices of the closure to DOLE and the employees at least one month before the effectivity date of the termination. Notice to DOLE is mandatory to enable the proper authorities to ascertain whether the closure and/or dismissals were made in good faith and not just as a pretext for evading compliance with the employer’s just obligations to the affected employees. This requirement is intended to protect the workers’ right to security of tenure and absence of it taints the dismissal. All these point to the fact that the real reason behind the shutdown of the corporation was the formation of their Union. Concededly, the determination to cease operations is a management prerogative and no business can be required to continue operating at a loss simply to maintain the workers in employment because it would be a taking of property without due process of law. But if the closure is motivated by a desire to discourage workers from organizing themselves into a Union for more effective negotiations with management, the State is bound to intervene. To justify the closure of a business and the termination of the services of the concerned employees, the 157 | P a g e

law requires the employer to prove that it suffered substantial actual losses. The cessation of a company’s operations shortly after the organization of a labor union, as well as the resumption of business barely a month after, gives credence to the employee’s claim that the closure was meant to discourage union membership and to interfere in union activities and thus constitutes unfair labor practice. Petition denied. h. Run-away Shop – IR-I(s) IRR, R1(s) "Strike Area" means the establishment, warehouse, depots, plants or offices, including the sites or premises used as run-away shops, of the employer struck against, as well as the immediate vicinity actually used by picketing strikers in moving to and fro before all points of entrance to and exits from said establishment. Complex Electronics Employees Association v. NLRC, 310 SCRA 403 (1999) Facts: Complex was actually a subcontractor of electronic products where its customers gave their job orders, sent their own materials and consigned their equipment to it. The rank and file workers of Complex were organized into a union known as the Complex Electronics Employees Association (UNION) Complex received a facsimile message from Lite-On Philippines Electronics Co ., requiring it to lower its price by 10%. Complex regretfully informed the employees that it was left with no alternativae but to close down the operations of the Lite-On Line. The company, however, promised that proper compensation shall be given. Union, on the other hand, pushed for a retrenchment pay equivalent to one (1) month salary for every year of service, which Complex refused. Complex were pulled-out from the company premises and transferred to the premises of Ionics. Union filed a notice of strike. ULP case filed where Ionics was impleaded as a party defendant. Contention of Ionics: Ionics contended that it was an entity separate and distinct from Complex. Like Complex, it was also engaged in the semi-conductor business where the machinery, equipment and materials were consigned to them by their customers. The transfer of the machinery, equipment and materials from Complex was the decision of the owners who were common customers of Complex and Ionics. Contention of Union: President of both companies - Lawrence Qua. Ease in transferring of assets. ISO Certification for both companies published in a newspaper. Complex was the majority shareholder of Ionics. LA = Ionex. All parties appealed. All were denied; hence, this consolidated case. Issue: Won Complex and Ionics are separate and distinct; hence, Ionics was just a runaway shop. SC: Yes. Ionics was not just a runaway shop. A "runaway shop" is defined as an industrial plant moved by its owners from one location to another to escape union labor regulations or state laws, but the term is also used to describe a plant removed to a new location in order to discriminate against employees at the old plant because of their union activities. A "runaway shop" in this sense, is a relocation motivated by anti-union animus rather than for business reasons. In this case, however, Ionics was not set up merely for the purpose of transferring the business of Complex. At the time the labor dispute arose at Complex, Ionics was already existing as an independent company. As earlier mentioned, it has been in existence since July 5, 1984. It cannot, therefore, be said that the temporary closure in Complex and its subsequent transfer of business to Ionics was for anti-union purposes. The mere fact that one or more corporations are owned or controlled by the same or single stockholder is not a sufficient ground for disregarding separate corporate personalities justify the piercing of the corporate veil. This fiction of corporate entity can only be disregarded in certain cases such as when it is used to defeat public convenience, justify wrong, protect fraud, or defend crime. The photo/newspaper clipping merely showed that some plants of Ionics were recertified to ISO 9002 and does not show that there is a relation between Complex and Ionics except for the fact that Lawrence Qua was also the president of Ionics. However, as we have stated above, the mere fact that both of the corporations have the same president is not in itself sufficient to pierce the veil of corporate fiction of the two corporations. III. UNFAIR LABOR PRACTICE: EMPLOYER AND LABOR ORGANIZATION ACTS VIOLATIVE OF RIGHT TO COLLECTIVE BARGAINING A. Violate Duty of Bargain – 248 (g); 249 (c); 250-253 Art. 248. Unfair labor practices of employers. It shall be unlawful for an employer to commit any of the following unfair labor practice: g. To violate the duty to bargain collectively as prescribed by this Code; Art. 249. Unfair labor practices of labor organizations. It shall be unfair labor practice for a labor organization, its officers, agents or representatives: c. To violate the duty, or refuse to bargain collectively with the employer, provided it is the representative of the employees; Art. 250. Procedure in collective bargaining. The following procedures shall be observed in collective bargaining: 1. When a party desires to negotiate an agreement, it shall serve a written notice upon the other party with a statement of its proposals. The other party shall make a reply thereto not later than ten (10) calendar days from receipt of such notice;

158 | P a g e

2. Should differences arise on the basis of such notice and reply, either party may request for a conference which shall begin not later than ten (10) calendar days from the date of request. 3. If the dispute is not settled, the Board shall intervene upon request of either or both parties or at its own initiative and immediately call the parties to conciliation meetings. The Board shall have the power to issue subpoenas requiring the attendance of the parties to such meetings. It shall be the duty of the parties to participate fully and promptly in the conciliation meetings the Board may call; 4. During the conciliation proceedings in the Board, the parties are prohibited from doing any act which may disrupt or impede the early settlement of the disputes; and 5. The Board shall exert all efforts to settle disputes amicably and encourage the parties to submit their case to a voluntary arbitrator. (As amended by Section 20, Republic Act No. 6715, March 21, 1989) Art. 251. Duty to bargain collectively in the absence of collective bargaining agreements. In the absence of an agreement or other voluntary arrangement providing for a more expeditious manner of collective bargaining, it shall be the duty of employer and the representatives of the employees to bargain collectively in accordance with the provisions of this Code. Art. 252. Meaning of duty to bargain collectively. The duty to bargain collectively means the performance of a mutual obligation to meet and convene promptly and expeditiously in good faith for the purpose of negotiating an agreement with respect to wages, hours of work and all other terms and conditions of employment including proposals for adjusting any grievances or questions arising under such agreement and executing a contract incorporating such agreements if requested by either party but such duty does not compel any party to agree to a proposal or to make any concession. Art. 253. Duty to bargain collectively when there exists a collective bargaining agreement. When there is a collective bargaining agreement, the duty to bargain collectively shall also mean that neither party shall terminate nor modify such agreement during its lifetime. However, either party can serve a written notice to terminate or modify the agreement at least sixty (60) days prior to its expiration date. It shall be the duty of both parties to keep the status quo and to continue in full force and effect the terms and conditions of the existing agreement during the 60-day period and/or until a new agreement is reached by the parties. NLRB v. General Electric Co., 418 F 2d 736 (1969) Library File Collegio de San Juan de Letran v. Association of Employees and Faculty of Letran, 340 SCRA 587 (2000) Facts: Salvador Abtria, then President of respondent union, Association of Employees and Faculty of Letran, initiated the renegotiation of its Collective Bargaining Agreement with petitioner Colegio de San Juan de Letran for the last two (2) years of the CBA's five (5) year lifetime from 1989-1994. In the same year, the union elected a new set of officers wherein private respondent Eleanor Ambas emerged as the newly elected President. On January 18, 1996, the parties agreed to disregard the unsigned CBA and to start negotiation on a new five-year CBA starting 1994-1999. On February 7, 1996, the union submitted its proposals to petitioner, which notified the union six days later, or on February 13, 1996, that the same had been submitted to its Board of Trustees. In the meantime, Ambas was informed through a letter dated February 15, 1996 from her superior that her work schedule was being changed from Monday to Friday to Tuesday to Saturday. Ambas protested and requested management to submit the issue to a grievance machinery under the old CBA. On March 29, 1996, the union received petitioner's letter dismissing Ambas for alleged insubordination. Hence, the union amended its notice of strike to include Ambas' dismissal. On April 20, 1996, both parties again discussed the ground rules for the CBA renegotiation. However, petitioner stopped the negotiations after it purportedly received information that a new group of employees had filed a petition for certification election. On June 18, 1996, the union finally struck. On July 2, 1996, public respondent Secretary of Labor and Employment assumed jurisdiction and ordered all striking employees, including the union president, to return to work, and for petitioner to accept them back under the same terms and conditions before the actual strike. Petitioner readmitted the striking members except Ambas. Public respondent issued an order declaring petitioner guilty of unfair labor practice on two counts and directing the reinstatement of private respondent Ambas with backwages. Petitioner filed a motion for reconsideration which was denied. Petitioner sought a review of the order of the Secretary of Labor and Employment before the Court of Appeals. The appellate court dismissed the petition and affirmed the findings of the Secretary of Labor and Employment. Hence, the present petition. Issue: (1) WON petitioner is guilty of unfair labor practice by refusing to bargain with the union when it unilaterally suspended the ongoing negotiations for a new Collective Bargaining Agreement (CBA) upon mere information that a petition for certification has been filed by another legitimate labor organization (2) WON the termination of the union president amounts to an interference of the employees' right to self-organization. Held: The Supreme Court denied the petition. The petitioner's actuation showed a lack of sincere desire to negotiate rendering it guilty of unfair labor practice. Petitioner's utter lack of interest in bargaining with the union is obvious in its failure to make a timely reply to the proposals presented by the latter. More than a month after the proposals were submitted by the union, petitioner still had not made any counter-proposals. Said inaction on the part of petitioner prompted the union to file its second notice of strike on March 13, 1996. Petitioner could only offer a feeble explanation that the Board of Trustees had not yet convened to discuss the matter as its excuse for failing to file its reply. Petitioner clearly violated Article 250 of the Labor Code governing the procedure in collective bargaining. Concerning the issue on the validity of the termination of the union president, the Court ruled that the dismissal was effected in violation of the employees' right to self-organization. The factual backdrop of the termination of Ms. Ambas led to no other conclusion that she was dismissed in order to strip the union of a leader who would fight for the right of her co-workers at the bargaining table. The record showed that it was when Ms. Ambas was the union president and during the period of tense and difficult negotiations when her work schedule was altered from Mondays to Fridays to Tuesdays to Saturdays. When she did not budge, although her schedule was changed, she was outrightly dismissed for alleged insubordination. EIAHcC 159 | P a g e

The mere filing of a petition for certification election does not ipso facto justify the suspension of negotiation by the employer. The petition must first comply with the provisions of the Labor Code and its Implementing Rules. No petition for certification election for any representation issue may be filed after the lapse of the sixty-day freedom period. The old CBA is extended until a new one is signed. The rule is that despite the lapse of the formal effectivity of the CBA the law still considers the same as continuing in force and effect until a new CBA shall have been validly executed. Hence, the contract bar rule still applies. The purpose is to ensure stability in the relationship of the workers and the company by preventing frequent modifications of any CBA earlier entered into by them in good faith and for the stipulated original period. General Milling Corporation v. CA, 422 SCRA 514 (2004) General Milling Corporation (GMC) had a three-year CBA with its Union, effective from December 1, 1988 to November 30, 1991. A day before its expiration, the Union sent to it a proposed CBA, with a request that counter-proposals be sent to them within 10 days. As early as October 1991, however, GMC had been receiving letters from its employees stating that they had withdrawn from the Union. GMC relayed this to the Union officers through a letter, and said that there was no basis to negotiate with a union that no longer existed. The officers disclaimed this. A union member was dismissed, sparking protests from the Union which requested that the matter be submitted to the grievance procedure as outlined in the CBA. GMC advised them to refer to their earlier letter. The Union filed a complaint against GMC, alleging ULP for 1) refusal to bargain collectively, 2) interference with the right to self-organization, and 3) discrimination. The labor arbiter dismissed, with a recommendation that a certification election be held. The Union appealed to the NLRC, which set aside the LA's decision and ordered GMC to abide by the Union's proposed CBA for a period of two years. GMC moved for reconsideration. NLRC set aside its own decision, finding that GMC's doubts as to the Union's status were legitimate. The Union appealed to the CA, which ruled for it. After its Motion for Reconsideration was denied, GMC elevated the case to the SC. Issues: 1. WON GMC was guilty of ULP for refusal to bargain collectively and for interference with the workers' right to self-organization, 2. WON the draft CBA proposed by the Union should be imposed on GMC. Held: 1. Yes. Art 253-A mandates that the representation provision of a CBA should last for five years. The Union was still the certified collective bargaining agent of the workers because it sought renegotiation of the CBA within five years from the date of effectivity of their first CBA. The Union's proposal was also submitted within the prescribed three-year period of the CBA. Hence, the GMC had no reason to refuse to negotiate in good faith with the Union. The letters presented by GMC signifying the intention of the workers to resign from the Union were received during the pendency of the case before the LA. That showed that GMC exerted pressure on its employees, which amounted to interference. 2. Yes. Citing Kiok Loy and Divine Word University of Tacloban, the SC ruled that GMC had lost the right to bargain collectively when it acted in bad faith. Therefore, the Union's proposed CBA should be imposed on them. B. Negotiation or Attorneys Fees – 248 (h); 249 (e) Art. 248. Unfair labor practices of employers. It shall be unlawful for an employer to commit any of the following unfair labor practice: h. To pay negotiation or attorney’s fees to the union or its officers or agents as part of the settlement of any issue in collective bargaining or any other dispute; o Art. 249. Unfair labor practices of labor organizations. It shall be unfair labor practice for a labor organization, its officers, agents or representatives: e. To ask for or accept negotiation or attorney’s fees from employers as part of the settlement of any issue in collective bargaining or any other dispute; or C. Violate Collective Bargaining Agreement – 248 (j); 249 (f); 261 Art. 248. Unfair labor practices of employers. It shall be unlawful for an employer to commit any of the following unfair labor practice: i. To violate a collective bargaining agreement. Art. 249. Unfair labor practices of labor organizations. It shall be unfair labor practice for a labor organization, its officers, agents or representatives: f. To violate a collective bargaining agreement Art. 261. Jurisdiction of Voluntary Arbitrators or panel of Voluntary Arbitrators. The Voluntary Arbitrator or panel of Voluntary Arbitrators shall have original and exclusive jurisdiction to hear and decide all unresolved grievances arising from the interpretation or implementation of the Collective Bargaining Agreement and those arising from the interpretation or enforcement of company personnel policies referred to in the immediately preceding article. Accordingly, violations of a Collective Bargaining Agreement, except those which are gross in character, shall no longer be treated as unfair labor practice and shall be resolved as grievances under the Collective Bargaining Agreement. For purposes of this article, gross violations of Collective Bargaining Agreement shall mean flagrant and/or malicious refusal to comply with the economic provisions of such agreement. The Commission, its Regional Offices and the Regional Directors of the Department of Labor and Employment shall not entertain disputes, grievances or matters under the exclusive and original jurisdiction of the Voluntary Arbitrator or panel of Voluntary Arbitrators and shall immediately dispose and refer the same to the Grievance Machinery or Voluntary Arbitration provided in the Collective Bargaining Agreement. 160 | P a g e

Flight Attendants, etc. v. PAL, 559 SCRA 252 (2008) FACTS • FASAP is the certified CB Representative of PAL flight attendants and stewards (cabin crew personnel).

• •

June 1998 – PAL was placed under Corporate Rehabilitation Plan June 15, 1998 – PAL retrenched[1] some 5000 of its employees including 1400 of the cabin crew personnel to take effect on July 15, 1998. Prior to implementation of retrenchment, FASAP and PAL had consultations and agreed that the retrenchment shall be in accordance with the CBA, that is, based on: efficiency rating and seniority. While the consultations were ongoing, PAL terminated 140 probationary cabin attendants. However, they were subsequently rehired and even made permanent. June 22, 1998 – FASAP filed a complaint for illegal retrenchment and unfair labor practice. July 6, 1998 – LA ordered the reinstatement of the retrenched employees. NLRC reversed and ordered the payment of separation benefits. September 4, 1998 – Lucio Tan made an offer to transfer shares of stock to its employees. September 23, 1998 – PAL ceased operations and terminated its employees September 27, 1998 –PALEA made a proposal to cushion the effect of the financial difficulties (this included the 10 year suspension of the CBA) November 1998 – PAL recalled to service those it previously retrenched December 1998 – PAL filed a stand alone rehabilitation plan with the SEC which provided for the infusion of 200M capital. Said plan was approved WON the retrenchment was justified? WON there was ULP? NO NO

• • • • • • •
ISSUE 1. 2. HELD: 1. 2. RATIO: 1.

Retrenchment is a management prerogative. But it must be a measure of last resort. The management has to prove the existence of some standards to justify the retrenchment: (1) reasonably necessary and likely to prevent business losses which, if already incurred, are not merely de minimis, but substantial, serious, actual and real, or if only expected, are reasonably imminent as perceived objectively and in good faith by the employer; (2) written notice (3) payment of separation benefits (4) good faith (5) use of reasonable criteria in determining who should be retrenched. PAL failed to meet the 1st , 4th and 5th standards. 1st standard - In the instant case, PAL failed to substantiate the retrenchment. There were no audited financial statements submitted for the purpose of proving substantial losses. In fact, even PAL itself seems to have not been aware of its financial status as can be gleaned from the fact that initially it planned to use the Plan 14 scheme but eventually used Plan 22. Moreover, PAL’s act of proposing a stand alone rehabilitation plan negates the assertion that it was suffering from losses. 4th standard – PAL presented no reason for the abandonment of the Plan 14 scheme. The retrenchment then subsequent hiring of new employees or rehiring of retrenched employees is a badge of bad faith. More so in this case where the probationary employees who were initially retrenched were recalled and given permanent statuses while the permanent employees were not recalled. 5th standard – PAL used “other reasons” for retrenchment which were not set out in detail. 2. NO Anent the claim of unfair labor practices committed against petitioner, we find the same to be without basis. Article 261 of the Labor Code provides that violations of a CBA, except those which are gross in character, shall no longer be treated as unfair labor practice and shall be resolved as grievances under the parties' CBA. Moreover, "gross violations of CBA" under the same Article referred to flagrant and/or malicious refusal to comply with the economic provisions of such agreement, which is not the issue in the instant case. Also, we fail to see any specific instance of union busting, oppression or harassment and similar acts of FASAP's officers. The fact that majority of FASAP's officers were either retrenched or demoted does not prove restraint or coercion in their right to organize. Instead, we see a simple retrenchment scheme gone wrong for failure to abide by the stringent rules prescribed by law, and a failure to discharge the employer's burden of proof in such cases.

161 | P a g e

[1] The retrenchment was due to the losses in the airline industry due to the Asian Financial Crisis. PAL alleged that at the time, its asset amounted only to 85B while liabilities amounted to 90B. The retrenchment scheme was called Plan 14 where the fleet of aircrafts would be reduced from 54 to 14 and would thus require only the service of 654 personnel. Such allegedly resulted in 24M/month savings. Philcom Employees Union v. Philippine Global Communications, 495 SCRA 214 (2006), supra Philippine Airlines, Inc. v. NLRC, 283 SCRA 324 (1997) FACTS: In 1971, PAL and PALEA agreed in writing that their current CBA would be extended for another year. • PALEA consented to the extension apparently because PAL then undertook to conduct a Job Evaluation Program (JEP) which would be the basis of a new pay scale to replace that which was previously negotiated • PAL then drew up a pay scale which, being acceptable to PALEA, was immediately implemented. This pay scale categorized employees as supervisory and nonsupervisory, the latter further classified and divided into job grades each with a minimum and maximum salary rate. • Thereafter, a series of enactments increasing the minimum wage came into effect • In 1981, the parties concluded negotiations for a new CBA wherein they agreed that there would be across-the-board pay increases and that PAL shall revise the existing payscale • In 1982, PALEA president Mario Santos addressed a letter to PAL for its failure to make any move or any action to inform the union of its position on the provisions of the CBA." Khan, the VP on HR, replied that PAL was ten in its final process to ensure that possible pay istortions be minimized if not totally eliminated. • PALEA sent its “Counter-Payscale” to PAL with the request that the negotiating panel be convened. However, PAL did not act on the said request. • Hence, PALEA filed a complaint accusing PAL and VP Khan of ULP in reneging on the obligation to consult the union regarding implementation of the pay scale • The case was, however, held in abeyance by agreement of the parties, for the reason that negotiations were then going on for a new CBA to cover the period from 1983 to 1986. • After the execution of the new CBA, the ULP against PAL was resumed, which presented a supplemental complaint alleging additionally that PAL had also violated Wage Orders which were previously effected. • LA rendered an Order declaring that there exist a wage distortion and directed the parties to seek a solution to this problem. • On appeal, NLRC affirmed the LA's decision. ISSUE: WON the Labor Arbiter and the NLRC acted within their jurisdiction in taking cognizance of an action for correction of wage distortion. PAL's contention: while the complaint purports to charge PAL with unfair labor practice, it actually prays for correction of wage distortions, the correction for which is either (1) the prescribed grievance machinery or negotiation between employer and union, and (2) voluntary arbitration, in case the requisite correction cannot thereby be accomplished. PALEA's contention: − its complaint essentially accuses PAL of violating its duty to bargain collectively — because it implemented the new payscale without consulting it, and refused to convene the negotiation panel to resolve their differences thereon — and is thus really one for unfair labor practice under Article 248 (g) of the Labor Code falling within the original and exclusive jurisdiction of Labor Arbiters in accordance with Article 217 of the same Code. − PAL is in estoppel to challenge the jurisdiction of the Labor Arbiter and the NLRC since it "never questioned their jurisdiction in both the original and appellate proceedings . . (and) even attended conciliation conferences with the end in view of amicably settling the case," it having "by its actions, . . willingly agreed to the arbitration by the NLRC." HELD: Yes, LA and NLRC acted within their juridiction. NLRC decision is affirmed. RATIO: − The LA under Art 217 has original and exclusive jurisdiction of ULP cases and money claims but changes were effected by RA 6715 which took effect on March 21, 1989. The Act provides that cases arising from interpretation or implementation of the CBA and those arising from the interpretation of company policies shall be disposed of by the LA by referring the same to the grievance machinery and voluntary arbitration. − This exclusionary proviso is reflected in Art 261 in LC. Hence as of March 21, 1989, violations of CBA were no longer deemed ULP except those gross in character and were considered mere grievances resolvable through the appropriate grievance machinery or voluntary arbitration provided in the CBA. Jurisdiction over such case was withdrawn from the LA. This law was not yet in effect when the complaint was filed and it was only after PAL’s MFR that said law was effective. Under the circumstances, no estoppel can be ascribed to PAL. − The breach of agreement on PAL’s part may not also be considered so gross a character as to constitute an exception to the rule making the case cognizable by the LA. But court did not dismiss the proceeding on the ground of lack of jurisdiction because both parties have extensively and exhaustively ventilated the issue before LA and NLRC. It would serve no useful purpose to have same evidence and arguments adduced before another arbitrator considering that the proceedings were had before the law was effective and that the parties are directed to convene in order to correct the wage distortion with the intervention of 162 | P a g e

the Office of Socio Economic Analyst as well as the willingness of PAL to correct any wage distortion which may exist. This appears to the court the most expedient option. IV. MOTIVE, CONDUCT AND PROOF A. Employer Motive and Proof Visayan Bicycle Manufacturing Co., Inc. v. National Labor Union, 14 SCRA 5 (1965) The findings of the CIR to the foregoing effect are supported by substantial evidence. No reason obtains to alter the conclusion that Besana and Rodiel were in reality dismissed because of their union activities and not because of their violation of a company rule against fight in the premises or during working hours. Furthermore, the so-called violation of company rules having been brought about by the company itself, thru the recent employment of Saturnino Reyes and Silvestre Pacia who provoked the fight as above indicated, the same cannot be regarded as a ground to punish the aforementioned employees. Such being the case, the dismissal of Besana and Rodiel constituted unfair labor practice under Section 4(a) (1) and (4) of Republic Act 875: Sec. 4. Unfair Labor Practices.(a) It shall be unfair labor practice for an employer: 1) To interfere with, restrain ,or coerce employees in the exercise of their rights guaranteed in section three;" 4) To discriminate in regard to hire or tenure of employment or any term or condition of employment to encourage or discourage membership in any labor organization: . . ." Rothenberg has this to say: ". . . it can be established that the true and basic inspiration for the employer's act is derived from the employee's union affiliations or activities, the assignment by the employer of another reason, whatever its semblance or validity, is unavailing. Thus, it has been held that the facts disclosed that the employer's acts in discharging employees were actually prompted by the employer's improper interest in the affected employee's union affiliations and activities, even though the employer urged that his acts were predicated on economic necessity, desire to give employment to more needy persons, lack of work, cessation of operations, refusal to work overtime, refusal of non-union employees to work with union employees, seasonal lay-off, libelous remarks against management, violation of company rules." Since the only reason or basis for Besana and Rodiel's dismissal was in fact their actuation as officers of VIBEMWU, the dismissal is clearly discriminatory. It is this inconsiderate act of power that makes a subordinate a rebel, it is this malicious tactic that forces labor to dislike management; this unjustifiable conduct that creates a gap between management and labor; and this attitude that makes the laborer hate the officials of the company to the detriment of all efforts to harmonize management and labor for the benefit of both as envisioned by the Industrial Peace Act. So plain from the record is the bad faith that attended the company's deliberate and calculated act of unfair labor practice that we find in the present appeal an obvious attempt to delay and carry on a pretense which this Court can ill afford to let go without stern disapproval. Me-Shurn Corporation v. Me-Shurn Workers Union, 448 SCRA 41 (2005), supra B. Totality of Conduct Rule and Effect of Failure of Act The Insular Life Assurance Co. Ltd. Employees Association-NATU v. Insular Life Assurance Co. Ltd., 37 SCRA 244 (1971), supra V. ENFORCEMENT, REMEDIES, AND SANCTIONS A. Parties Against Whom ULP Committed – 248; 249; 212 (e) (f) (g) Art. 248. Unfair labor practices of employers. It shall be unlawful for an employer to commit any of the following unfair labor practice: a. To interfere with, restrain or coerce employees in the exercise of their right to self-organization; b. To require as a condition of employment that a person or an employee shall not join a labor organization or shall withdraw from one to which he belongs; c. To contract out services or functions being performed by union members when such will interfere with, restrain or coerce employees in the exercise of their rights to self-organization; d. To initiate, dominate, assist or otherwise interfere with the formation or administration of any labor organization, including the giving of financial or other support to it or its organizers or supporters; e. To discriminate in regard to wages, hours of work and other terms and conditions of employment in order to encourage or discourage membership in any labor organization. Nothing in this Code or in any other law shall stop the parties from requiring membership in a recognized collective bargaining agent as a condition for employment, except those employees who are already members of another union at the time of the signing of the collective bargaining agreement. Employees of an appropriate bargaining unit who are not members of the recognized collective bargaining agent may be assessed a reasonable fee equivalent to the dues and other fees paid by members of the recognized collective bargaining agent, if such non-union members accept the benefits under the collective bargaining agreement: Provided, that the individual authorization required under Article 242, paragraph (o) of this Code shall not apply to the non-members of the recognized collective bargaining agent; 163 | P a g e

f. To dismiss, discharge or otherwise prejudice or discriminate against an employee for having given or being about to give testimony under this Code; g. To violate the duty to bargain collectively as prescribed by this Code; h. To pay negotiation or attorney’s fees to the union or its officers or agents as part of the settlement of any issue in collective bargaining or any other dispute; or i. To violate a collective bargaining agreement. The provisions of the preceding paragraph notwithstanding, only the officers and agents of corporations, associations or partnerships who have actually participated in, authorized or ratified unfair labor practices shall be held criminally liable. (As amended by Batas Pambansa Bilang 130, August 21, 1981) Art. 249. Unfair labor practices of labor organizations. It shall be unfair labor practice for a labor organization, its officers, agents or representatives: a. To restrain or coerce employees in the exercise of their right to self-organization. However, a labor organization shall have the right to prescribe its own rules with respect to the acquisition or retention of membership; b. To cause or attempt to cause an employer to discriminate against an employee, including discrimination against an employee with respect to whom membership in such organization has been denied or to terminate an employee on any ground other than the usual terms and conditions under which membership or continuation of membership is made available to other members; c. To violate the duty, or refuse to bargain collectively with the employer, provided it is the representative of the employees; d. To cause or attempt to cause an employer to pay or deliver or agree to pay or deliver any money or other things of value, in the nature of an exaction, for services which are not performed or not to be performed, including the demand for fee for union negotiations; e. To ask for or accept negotiation or attorney’s fees from employers as part of the settlement of any issue in collective bargaining or any other dispute; or f. To violate a collective bargaining agreement. The provisions of the preceding paragraph notwithstanding, only the officers, members of governing boards, representatives or agents or members of labor associations or organizations who have actually participated in, authorized or ratified unfair labor practices shall be held criminally liable. (As amended by Batas Pambansa Bilang 130, August 21, 1981) Art. 212. Definitions. e. "Employer" includes any person acting in the interest of an employer, directly or indirectly. The term shall not include any labor organization or any of its officers or agents except when acting as employer. f. "Employee" includes any person in the employ of an employer. The term shall not be limited to the employees of a particular employer, unless the Code so explicitly states. It shall include any individual whose work has ceased as a result of or in connection with any current labor dispute or because of any unfair labor practice if he has not obtained any other substantially equivalent and regular employment. g. "Labor organization" means any union or association of employees which exists in whole or in part for the purpose of collective bargaining or of dealing with employers concerning terms and conditions of employment. Phelps-Dodge Corporation v. NLRC, 313 U.S. 177 (1941) Library File, supra B. Parties Liable for Acts 1) Employer – 248, last paragraph; 212 (e); 288-290 Art. 248. Unfair labor practices of employers. It shall be unlawful for an employer to commit any of the following unfair labor practice: The provisions of the preceding paragraph notwithstanding, only the officers and agents of corporations, associations or partnerships who have actually participated in, authorized or ratified unfair labor practices shall be held criminally liable. (As amended by Batas Pambansa Bilang 130, August 21, 1981) Art. 288. Penalties. Except as otherwise provided in this Code, or unless the acts complained of hinge on a question of interpretation or implementation of ambiguous provisions of an existing collective bargaining agreement, any violation of the provisions of this Code declared to be unlawful or penal in nature shall be punished with a fine of not less than One Thousand Pesos (P1,000.00) nor more than Ten Thousand Pesos (P10,000.00) or imprisonment of not less than three months nor more than three years, or both such fine and imprisonment at the discretion of the court. In addition to such penalty, any alien found guilty shall be summarily deported upon completion of service of sentence. Any provision of law to the contrary notwithstanding, any criminal offense punished in this Code, shall be under the concurrent jurisdiction of the Municipal or City Courts and the Courts of First Instance. (As amended by Section 3, Batas Pambansa Bilang 70) Art. 289. Who are liable when committed by other than natural person. If the offense is committed by a corporation, trust, firm, partnership, association or any other entity, the penalty shall be imposed upon the guilty officer or officers of such corporation, trust, firm, partnership, association or entity. Art. 290. Offenses. Offenses penalized under this Code and the rules and regulations issued pursuant thereto shall prescribe in three (3) years. All unfair labor practice arising from Book V shall be filed with the appropriate agency within one (1) year from accrual of such unfair labor practice; otherwise, they shall be forever barred. 2) Labor Organization – 249, last paragraph; 288-290, same Art. 249. Unfair labor practices of labor organizations. It shall be unfair labor practice for a labor organization, its officers, agents or representatives:

164 | P a g e

The provisions of the preceding paragraph notwithstanding, only the officers, members of governing boards, representatives or agents or members of labor associations or organizations who have actually participated in, authorized or ratified unfair labor practices shall be held criminally liable. (As amended by Batas Pambansa Bilang 130, August 21, 1981) C. Prosecution and Prescriptive Period 1) Civil Aspect – 247; paragraphs 2-4; 290 Art. 247. Concept of unfair labor practice and procedure for prosecution thereof. Consequently, unfair labor practices are not only violations of the civil rights of both labor and management but are also criminal offenses against the State which shall be subject to prosecution and punishment as herein provided. Subject to the exercise by the President or by the Secretary of Labor and Employment of the powers vested in them by Articles 263 and 264 of this Code, the civil aspects of all cases involving unfair labor practices, which may include claims for actual, moral, exemplary and other forms of damages, attorney’s fees and other affirmative relief, shall be under the jurisdiction of the Labor Arbiters. The Labor Arbiters shall give utmost priority to the hearing and resolution of all cases involving unfair labor practices. They shall resolve such cases within thirty (30) calendar days from the time they are submitted for decision. Recovery of civil liability in the administrative proceedings shall bar recovery under the Civil Code. No criminal prosecution under this Title may be instituted without a final judgment finding that an unfair labor practice was committed, having been first obtained in the preceding paragraph. During the pendency of such administrative proceeding, the running of the period of prescription of the criminal offense herein penalized shall be considered interrupted: Provided, however, that the final judgment in the administrative proceedings shall not be binding in the criminal case nor be considered as evidence of guilt but merely as proof of compliance of the requirements therein set forth. (As amended by Batas Pambansa Bilang 70, May 1, 1980 and later further amended by Section 19, Republic Act No. 6715, March 21, 1989) Art. 290. Offenses. Offenses penalized under this Code and the rules and regulations issued pursuant thereto shall prescribe in three (3) years. All unfair labor practice arising from Book V shall be filed with the appropriate agency within one (1) year from accrual of such unfair labor practice; otherwise, they shall be forever barred. 2) Criminal Aspect – 247, last paragraph; 290 Art. 247. Concept of unfair labor practice and procedure for prosecution thereof. No criminal prosecution under this Title may be instituted without a final judgment finding that an unfair labor practice was committed, having been first obtained in the preceding paragraph. During the pendency of such administrative proceeding, the running of the period of prescription of the criminal offense herein penalized shall be considered interrupted: Provided, however, that the final judgment in the administrative proceedings shall not be binding in the criminal case nor be considered as evidence of guilt but merely as proof of compliance of the requirements therein set forth. (As amended by Batas Pambansa Bilang 70, May 1, 1980 and later further amended by Section 19, Republic Act No. 6715, March 21, 1989) Art. 290. Offenses. Offenses penalized under this Code and the rules and regulations issued pursuant thereto shall prescribe in three (3) years. All unfair labor practice arising from Book V shall be filed with the appropriate agency within one (1) year from accrual of such unfair labor practice; otherwise, they shall be forever barred. Cochangco Workers Union v. NLRC, 161 SCRA 655 (1988) D. Compromise AFP Mutual Benefit Association, Inc. v. AFP MBAI-EU, 97 SCRA 715 (1980) Facts: A case for ULP was filed by 34 members of the union for illegal and discriminatory dismissals. The complaint alleged that they were dismissed due to active campaigning of some members of the union by sending economic demands to the Association. The Association alleged that the dismissal was due to lawful and justified causes – the alleged financial loss due to the order by the Insurance Commissioner to cease operation of its four major loan operations and the implementation of the new Minimum Wage Law. The Association filed a Motion to Dismiss, insofar as 19 complainants were concerned on the ground that they voluntarily withdrew from this case by freely executing under oath quitclaim papers. The CIR declared the Association guilty of ULP and ordered reinstatement and payment of backwages. However, the CIR ordered the dismissal of the case insofar as the 19 complainants who executed quitclaims are concerned. Both parties filed separate Motions for Reconsideration of the CIR order. Complainants allege that receipt of separation pay and quitclaims cannot absolve the Assoc. from the consequences of ULP. The Assoc. maintained that the complainants are not entitled to reinstatement nor backwages. Ruling: On the 19 complainants whose complaints were dismissed on the basis of the quitclaim voluntarily entered into: 165 | P a g e

· It is well-established that quitclaims and/or complete releases executed by the employees do not estop them from pursuing their claims arising from the unfair labor practice of the employer. · The basic reason for this is that such quitclaims and/or complete releases are against public policy and, therefore, null and void. · The acceptance of termination pay does not divest a laborer of the right to prosecute his employer for unfair labor practice acts.[1] · The Association's argument that it is not the receipt of separation pay and the execution of quitclaim documents by the individual petitioners which compelled the Court of Industrial Relations to dismiss the complaint insofar as they are concerned but rather their voluntary desistance and withdrawal from the case as party litigants which gave the Court of Industrial Relations no other alternative but to dismiss the complaint, is untenable. · In the first place, the finding of the Court of Industrial Relations that the Association was guilty of unfair labor practice for having dismissed 34 individual complainants from their employment by reason of their Union activities is correct, as well as its Order for the reinstatement of the fifteen (15) complainants who did not execute quitclaims and/or complete releases, which We affirm. · But the dismissal of the complaint insofar as the other nineteen (19) complainants are concerned on the ground that they have voluntarily desisted and withdrawn from the case is not warranted because their desistance or withdrawal is not only voluntary but also illegal, being contrary to public policy. And since the dismissal of the employees constitutes an unfair labor practice, it is immaterial whether some have executed quitclaims and releases or not. · Unfair labor practice acts are beyond and outside the sphere of compromises such as quitclaims, release and settlements. On the ULP Issue: Held: The termination of the complainants constitute ULP · Legal basis: The Supreme Court in a number of cases has recognized and affirmed the right of the employer to lay-off or dismiss employees because of lack of work caused by a considerable reduction in its business, or that their continued employment will only result in further losses in the operation of its business [2], due to lack of work[3], and considerable reduction in the volume of his business[4]. We have held that such acts of dismissal do not constitute unfair labor practice. Under the law, an employer may close his business provided the same is done in good faith and is due to causes beyond his control. To rule otherwise would be oppressive and inhuman.[5] · In the Report of the Chief of the Examining Division of the Court showed that the current or working capital ration of the Assoc. is more than the standard ratio. · The alleged financial loss as a consequence of the implementation of the New Minimum Wage Law and the cessation of the 4 major operations are belied by increases in the salaries of the officers and other personnel. · While a CBA was entered into between the Assoc. and the Union, there is evidence that the Assoc. did not have the honest intention of complying with its provisions. o The Board of Directors of the Assoc. knew, before the signing of the CBA, that the Insurance Commissioner would order stoppage of the 4 major loan operations as being unlawful pursuant to the Revised Administrative Code. o The feasibility study submitted by the General Manager to the BOD, before the signing of the CBA, recommended “Retain 70 personnel; provided that 26 personnel out of the 70 personnel retained will be phased out within a period of two years." o The fact that the Association actually terminated the services of the individual complainants, only 12 days after the acknowledgment of the contract by the parties without referring the matter of phasing out or lay-off to the proper labor management committee, as well as the fact that the supposed guidelines containing the criteria in the selection of those who were to be terminated was not presented to the court despite requests therefor. [1] (Cariño vs. ACCFA, L-19808, Sept. 29, 1966, 18 SCRA 183; Philippine Sugar Institute, vs. CIR, L-13475, Sept. 29, 1960, 109 Phil. 452; Mercury Drug Co. vs. CIR, L23357, April 30, 1974, 56 SCRA 694, 704). [2] Phil. American Embroideries Inc. vs. Embroidery & Garment Workers Union, 26 SCRA 634, 643; Northern Luzon Transportation Co. vs. CIR, 73 Phil. 41 [3] Union of Philippine Education Employees vs. Phil. Education Co., Inc. L-7161, May 19, 1955, 97 Phil. 953 [4] (Gregorio Araneta Employees Union vs. Arsenio Roldan, 97 Phil. 304). [5] Tio Kinh vs. CIR, 90 Phil. 564, 568; LVN Pictures Employees and Workers Association (NLU) vs. LVN Pictures, Inc., L-23495, Sept. 30, 1970; LVN Pictures Checkers' Union (NLU) vs. LVN Pictures, Inc., L-26432, September 30, 1970, 35 SCRA 147 CCLG G.E. Gochangco Workers Union, et. al. v. NLRC, 161 SCRA 656 (1988) Facts: · Petitioner union is a local chapter of the Central Luzon Labor Congress (CLLC), a legitimate labor federation duly registered with the Ministry of Labor and Employment (MOLE), · Respondent is a corporation engaged in packing and crating, general hauling, warehousing, sea van and freight forwarding, · Majority of the rank and file employees of respondent firm organized the union as an affiliate of the CLLC. The union filed a petition for certification election which was granted by the MOLE. · On February 7,1980, the CLLC national president wrote the general manager of respondent firm informing him of the organization of the union and requesting for a labor management conference to normalize employer-employee relations. 166 | P a g e

· On February 26,1980, the, union sent a written notice to respondent firm requesting permission for certain member officers and members of the union to attend the hearing of the petition for certification election. The management refused to acknowledge receipt of said notice. · On February 28, 1980, private respondent preventively suspended the union officers and members who attended the hearing The common ground alleged by private respondent for its action was "abandonment of work on February 27, 1980." Their gate passes were confiscated by the base guard. · Union and its members filed a complaint for constructive lockout and unfair labor practice against private respondent · On March 12, 1980, the private respondent filed an application for clearance to dismiss other members and officers of the union. · On April 30, 1980, nine (9) more union members were terminated by the employer. · On May 9, 1980, private respondent filed with MOLE, Region III, a Notice of Termination of Contract together with a list of employees affected by the expiration of the contract, among them, the 39 individual petitioners herein. · All complaints were assigned to one labor arbiter. The labor arbiter granted the complaint of the workers and ordered their reinstatement with full backwages. The NLRC reversed. Issue: WON the employer was guilty of ULP Held: YES · We are convinced that the respondent company is indeed guilty of an unfair labor practice. It is no coincidence that at the time said respondent issued its suspension and termination orders, the petitioners were in the midst of a certification election preliminary to a labor management conference, purportedly, "to normalize employer-employee relations." · In this connection, the respondent company deserves our strongest condemnation for ignoring the petitioners' request for permission for some time out to attend to the hearing of their petition before the med-arbiter. It is not only an act of arrogance, but a brazen interference as well with the employees right to selforganization. · As if to add insult to injury, the company suspended the petitioners on the ground of "abandonment of work" on February 27, 1980, the date on which, apparently, the pre-election conference had been scheduled. · As a consequence of such a suspension, the Clark Air Base guards confiscated the employees' gate passes, and banned them from the base premises. We cannot be befooled by the company's pretenses that "[t]he subsequent confiscation by the Americans of the complainants' passes is beyond the powers of management." To start with, those passes would not have been confiscated had not management ordered the suspension. · In finding the petitioners' suspension illegal, with more reason do we hold their subsequent dismissal to be illegal. We are not persuaded by the respondent firm's argument that final termination should be effected as the contract has expired." We adopt the Solicitor General's submission that the petitioners were regular employees and as such, their tenure did not end with the expiration of the contract. As regular employees, the petitioners' tenure is secure, and their dismissal must be premised on a just cause. · But even if dismissal were warranted, the same nonetheless faces our disapproval in the absence of a proper clearance then required under the Labor Code. It is true that efforts were undertaken to seek such a clearance, yet there is no showing that it was issued. That still taints the dismissal with the vice of illegality. · On the matter of alleged waiver by the employees concerned of their economic demands, we have held that unfair labor practice cases are not, in view of the public interest involved, subject to compromises. · Before Batas Blg. 70 was enacted into law, unfair labor practices were considered administrative offenses, and have been held akin to tort, wherein damages are payable. We therefore not only order herein the reinstatement of the petitioners and the payment of backwages (including cost-of-living allowances) to them, but impose as well moral and exemplary damages. Reformists Union of R.B. Liner, Inc. v. NLRC, 266 SCRA 713 (1997) E. Remedies and Sanctions 1) Civil Remedies – 247 – 2nd-3rd paragraph 223; 279 Art. 247. Concept of unfair labor practice and procedure for prosecution thereof. Consequently, unfair labor practices are not only violations of the civil rights of both labor and management but are also criminal offenses against the State which shall be subject to prosecution and punishment as herein provided. Subject to the exercise by the President or by the Secretary of Labor and Employment of the powers vested in them by Articles 263 and 264 of this Code, the civil aspects of all cases involving unfair labor practices, which may include claims for actual, moral, exemplary and other forms of damages, attorney’s fees and other affirmative relief, shall be under the jurisdiction of the Labor Arbiters. The Labor Arbiters shall give utmost priority to the hearing and resolution of all cases involving unfair labor practices. They shall resolve such cases within thirty (30) calendar days from the time they are submitted for decision. Art. 279. Security of tenure. In cases of regular employment, the employer shall not terminate the services of an employee except for a just cause or when authorized by this Title. An employee who is unjustly dismissed from work shall be entitled to reinstatement without loss of seniority rights and other privileges and to his full backwages, inclusive of allowances, and to his other benefits or their monetary equivalent computed from the time his compensation was withheld from him up to the time of his actual reinstatement. (As amended by Section 34, Republic Act No. 6715, March 21, 1989) 167 | P a g e

Nueva Ecija Electric Cooperative, Inc. v. NLRC, 323 SCRA 86 (2000) - Salma 2) Penal Remedies – 247; 2nd paragraph 288-290 Art. 247. Concept of unfair labor practice and procedure for prosecution thereof. Consequently, unfair labor practices are not only violations of the civil rights of both labor and management but are also criminal offenses against the State which shall be subject to prosecution and punishment as herein provided. Art. 288. Penalties. Except as otherwise provided in this Code, or unless the acts complained of hinge on a question of interpretation or implementation of ambiguous provisions of an existing collective bargaining agreement, any violation of the provisions of this Code declared to be unlawful or penal in nature shall be punished with a fine of not less than One Thousand Pesos (P1,000.00) nor more than Ten Thousand Pesos (P10,000.00) or imprisonment of not less than three months nor more than three years, or both such fine and imprisonment at the discretion of the court. In addition to such penalty, any alien found guilty shall be summarily deported upon completion of service of sentence. Any provision of law to the contrary notwithstanding, any criminal offense punished in this Code, shall be under the concurrent jurisdiction of the Municipal or City Courts and the Courts of First Instance. (As amended by Section 3, Batas Pambansa Bilang 70) Art. 289. Who are liable when committed by other than natural person. If the offense is committed by a corporation, trust, firm, partnership, association or any other entity, the penalty shall be imposed upon the guilty officer or officers of such corporation, trust, firm, partnership, association or entity. Art. 290. Offenses. Offenses penalized under this Code and the rules and regulations issued pursuant thereto shall prescribe in three (3) years. All unfair labor practice arising from Book V shall be filed with the appropriate agency within one (1) year from accrual of such unfair labor practice; otherwise, they shall be forever barred.

168 | P a g e

Sponsor Documents

Or use your account on DocShare.tips

Hide

Forgot your password?

Or register your new account on DocShare.tips

Hide

Lost your password? Please enter your email address. You will receive a link to create a new password.

Back to log-in

Close